Решите уравнение х 3 7х 6 0: Х^3-7х+6=0 Как решать??? — Школьные Знания.com

Решение уравнений 3 и 4 степени

1. Исследовательская работа по теме: «Решение уравнений 3-ей и 4-ой степени»

Выполнил:
ученик 9 класса
Кравченко Виталий
Руководитель:
учитель математики
Нечаева
Елена Николаевна
© Фокина Лидия Петровна

2. Основные методы решения уравнений высших порядков

1. Метод разложения на множители
левой части уравнения.
2.Метод введения новой переменной.
3.Функционально-графический метод
© Фокина Лидия Петровна

3. Уравнения вида ax3 + bx2 + cx + d = 0, где a ≠ 0, называются уравнениями 3-ей степени

Уравнение вида
x 3 + px + q = 0
называется приведённым
кубическим уравнением
Известные формулы Кардано для решения уравнений этого типа
очень сложны и почти не применяются на практике.
© Фокина Лидия Петровна

4. Решу уравнение х3 -7х+6=0 разными способами

1. Разложение на множители
х3 -7х + 6 =0
х3 — х2 + х2 – х — 6х + 6=0
х2 (х-1)+ х(х-1)-6(х-1)=0
(х-1)(х2 + х — 6) = 0
х-1=0 или х2 + х – 6 = 0
х1 =1
х2 =-3 х3 = 2
Ответ: 1; 2; -3
© Фокина Лидия Петровна

5. 2.Метод деления на многочлен

х3 -7х+6 = 0 делители 6: ±1; ±2; ±3; ±6
1³-7+6=0
3-7х+6 =(х-1)(х2 +х-6)=0
х
x³-0х2-7x+6 x-1
2 +х-6=0
х-1=0
или
х
x³-x²
x²+x-6
х1 =1
х2 =-3 х3 = 2
x²-7x
x²-x
-6x+6
-6x+6
0
© Фокина Лидия Петровна
Ответ: 1; 2; -3

6. 3.Функционально-графический метод х3 -7х+6 = 0

у = х3 и у = 7х-6
Ответ:1;2;-3
© Фокина Лидия Петровна

7. Уравнение четвертой степени общего вида ax4 + bx3 + cx2 + dx + e = 0 где а ≠ 0 

Уравнение четвертой степени общего вида
ax4 + bx3 + cx2 + dx + e = 0 где а ≠ 0
1.Разложение на множители
x4 + 2×3 + 5×2 + 4x – 12 = 0
x4 + 2×3 + 5×2 + 10x – 6x – 12 = 0
(x4 + 2×3) + (5×2 + 10x) – (6x + 12 ) = 0
x3 (x+2) +5х (х+2) – 6 (х+2) =0
(x + 2) (x3 + 5x – 6) = 0
(x + 2)(x – 1)(x2 + x + 6) = 0
x1 = -2, x2 = 1.
Ответ: -2 ; 1
© Фокина Лидия Петровна

8. 2.Деление на многочлен Х4 — Х3-13 Х -15=0 -1 делитель числа -15 (1+1+13-15=0) Х4 — Х3-13 Х -15 = (Х+1)(Х-3)(Х2 +Х +5) = 0 Х+1

=0 или Х-3=0 или Х2 +Х +5 =0 (Д
Х1=-1 Х 2=3
Ответ: -1; 3
© Фокина Лидия Петровна

9. Биквадратное уравнение вида ax4 + bx2 + с = 0. 3.Метод: введение новой переменной

Биквадратное уравнение вида ax4 + bx2 + с = 0.
3.Метод: введение новой переменной
x4 + 5×2 – 36 = 0.
Замена y = x2.
У2+ 5У-36=0
У1*У2 =-36= -9*4
У1=-9
У1 + У1 =-5= -9+4
У2 =4
X2 =-9
x2 =4
Корней нет
х1 =2 х2 =-2
Ответ: 2; -2
© Фокина Лидия Петровна

10. Задание:Решите уравнение Х3+2Х2- 5Х — 6 = 0

Делители -6: ±1; ±2; ±3; ±6
-1 корень уравнения (-1+2+5-6=0)
Х3+2Х2- 5Х — 6 = (Х+1)(Х2+Х -6) = 0
Х+1= 0 или Х2+Х -6=0
Х1 =-1
Х2 =-3 Х3 = 2
Ответ: -1; -3; 2
© Фокина Лидия Петровна

10.5. Нахождение рациональных корней многочлена с целыми коэффициентами.

10.5. НАХОЖДЕНИЕ РАЦИОНАЛЬНЫХ КОРНЕЙ МНОГОЧЛЕНА С ЦЕЛЫМИ КОЭФФИЦИЕНТАМИ

Теорема 4. Если многочлен с целыми коэффициентами f (x) = anxn + an-1xn-1 + … + a1x+a0  имеет рациональный корень x=p/q (q ≠ 0, дробь p/q  несократимая), то р является делителем свободного члена (a0), а q — делителем коэффициента при стар­шем члене аn.

     Если p/q является корнем многочлена f (х), то f(p/q) = 0. Подставляем p/q вместо х в f(x) и из последнего равенства имеем

an * pn/qn + an-1 * pn-1/qn-1 + … + a1 * p/q + a0 = 0.

(1)

            Умножим обе части равенства (1) на  (q ≠ 0). Получаем

аnрn + an-1pn-1q + … + a1pqn-1 + a0qn = 0.

(2)

В равенстве (2) все слагаемые, кроме последнего, делятся на р. Поэтому

a0qn = -(аnрn + an-1pn-1q + … + a1pqn-1) делится на р.

Но когда мы записываем рациональное число в виде p/q, то эта дробь счи­тается несократимой, то есть р и q не имеют общих делителей. Произве­дение a0qn может делиться на р (если р и q — взаимно простые числа) только тогда, когда a0 делится на р. Таким образом, р — делитель свобод­ного члена a0.

Аналогично все слагаемые равенства (2), кроме первого, делятся на q. Тогда

anpn = -(an-1pn-1q + … + a1pq-1 + a0qn) делится на q. Поскольку р и q — взаимно простые числа, то an делится на q, следовательно, q — де­литель коэффициента при старшем члене.

Отметим два следствия из этой теоремы. Если взять q = 1, то корнем многочлена будет целое число р — делитель a0. Таким образом, имеет место:

Следствие 1. Любой целый корень многочлена с целыми коэффи­циентами является делителем его свободного члена.

Если в заданном многочлене f (х) коэффициент аn = 1, то делителями аn могут быть только числа ±1, то есть q =±1, и имеет место:

Следствие 2. Если коэффициент при старшем члене уравнения с целыми коэффициентами равен 1, то все рациональные корни этого уравнения (если они существуют) — целые числа.

Задача 1 Найдите рациональные корни многочлена 2х3х2 + 12х – 6.

Пусть несократимая дробь p/q является корнем многочлена. Тогда р не­обходимо искать среди делителей свободного члена, то есть среди чисел ±1, ±2, ±3, ±6, а q — среди делителей старшего коэффициента: ±1, ±2.

Таким образом, рациональные корни многочлена необходимо искать сре­ди чисел ±1/2, ±1, +±3/2, ±2, ±3, ±6. Проверять, является ли данное число корнем многочлена, целесообразно с помощью схемы Горнера. При x = 1/2 имеем следующую таблицу.

Кроме того, по схеме Горнера мож­но записать, что

3 – х2 + 12х – 6 = (x 1/2) (2x2 + 12).

Многочлен 2 + 12 не имеет действительных корней (а тем более рацио­нальных), поэтому заданный многочлен имеет единственный рациональ­ный корень x =1/2.

Задача 2 Разложите многочлен Р (х) = 2х4 + 3х3 – 2х2х – 2 на множители.

Ищем целые корни многочлена среди делителей свободного члена: ±1, ±2. Подходит 1. Делим Р (х) на х – 1 с помощью схемы Горнера.

Тогда Р (х) = (х – 1)(2х3 + 5х2 + 3х + 2). Ищем целые корни кубического многочлена 3 + 5х2 + 3х + 2 среди делителей его свободного члена: ±1, ±2. Подходит (–2). Делим на х + 2

Имеем  Р (х) = (х – 1)(х + 2)(2х2 + х +1).

Квадратный трехчлен 2х2 + х +1 не имеет действительных корней и на линейные множители не расклады­вается.

Ответ: Р (х) = (х – 1)(х + 2)(2х2 + х +1).

Отметим, что во множестве действительных чисел не всегда можно найти все корни многочлена (например, квадратный трехчлен х2 + х + 1 не имеет действительных корней). Таким образом, многочлен n-й степени не всегда можно разложить на линейные множители. В курсах высшей алгебры дока­зывается, что многочлен нечетной степени всегда можно разложить на ли­нейные и квадратные множители, а многочлен четной степени представить в виде произведения квадратных трехчленов.

Например, многочлен четвертой степени раскладывается в произведение двух квадратных трехчленов. Для нахождения коэффициентов этого раз­ложения иногда можно применить метод неопределенных коэффициентов.

Задача 3 Разложите на множители многочлен х4 + х3 + 3х2 + х + 6.

Попытка найти рациональные корни ничего не дает: многочлен не имеет рациональных (целых) корней.

Попытаемся разложить этот многочлен в произведение двух квадратных трехчленов. Поскольку старший коэффициент многочлена равен 1, то и у квадратных трехчленов возьмем старшие коэффициенты равными 1. То есть будем искать разложение нашего многочлена в виде:

х4 + х3 + 3х2 + х + 6 = (х2 + ах + b)(х2 + сх + d),

(3)

где а, b, с и d — неопределенные (пока что) коэффициенты. Многочлены, стоящие в левой и правой частях этого равенства, тождественно равны, поэтому и коэффициенты при одинаковых степенях х у них равны. Рас­кроем скобки в правой части равенства и приравняем соответствующие коэффициенты. Это удобно записать так:

х4+ х3+ 3х2 + х + 6 = x4+ cx3+ dx2+

                                                      + ax3+ acx2+ adx +

                                                                    + bx2+ bcx + bd.

Получаем систему

(4)

Попытка решить эту систему методом подстановки приводит к уравне­нию 4-й степени, поэтому попробуем решить систему (4) в целых числах. Из последнего равенства системы (4) получаем, что b и d могут быть толь­ко делителями числа 6. Все возможные варианты запишем в таблицу.

Коэффициенты b и d в равенстве (3) равноправны, поэтому мы не рас­сматриваем случаи b = 6 и d = 1 или b = –6 и d = –1 и т. д.

Для каждой пары значений b и d из третьего равенства системы (4) най­дем ас = 3 – (b + d), а из второго равенства имеем а + с = 1.

Зная а + с и ас, по теореме, обратной теореме Виета, находим а и с как корни квадратного уравнения. Найденные таким образом значения а, b, с, d подставим в четвертое равенство системы (4) + ad = 1, чтобы выбрать те числа, которые являются решениями системы (4). Удобно эти рассуждения оформить в виде таблицы:

Как видим, системе (4) удовлетворяет набор целых чисел а = –1, b = 2, с = 2, d = 3. Тогда равенство (3) имеет вид

x4 + х3 + 3х2 + х + 6 = (х2х + 2)(х2 + 2х + 3).

(5)

Поскольку квадратные трехчлены х2х + 2 и х2 + 2х + 3 не имеют не только рациональных, но и действительных корней, то равенство (5) дает окончательный ответ.

Упражнения

  1. Найдите целые корни многочлена:

1) х3 – 5х + 4;

2) 2x3 + x2 – 13x + 6;

3) 5х3 + 18х2 – 10х – 8;

4) 4х4 – 11х2 + 9х – 2.

  1. Найдите рациональные корни уравнения:

1) х3 – 3х2 + 2 = 0;

2) 2х3 – 5х2х + 1 = 0;

3) 3х4 + 5х3х2 – 5х – 2 = 0;

4) 3х4 – 8х3 – 2х2 + 7х – 2 = 0.

  1. Разложите многочлен на множители:

1) 2х3х2 – 5х – 2;

2) х3 + 9х2 + 23х +15;

3) х4 – 2х3 + 2х – 1;

4) х4 – 2х3 – 24х2 + 50х – 25.

  1. Найдите действительные корни уравнения:

1) х3 + х2 – 4х + 2 = 0;

2) х3 – 7х – 6 = 0;

3) 2х4 – 5х3 + 5х2 – 2 = 0;

4) 2х3 – 5х2 + 1 = 0.

5*. Разложите многочлен на множители методом неопределенных коэффи­циентов:

1) х4 + х3 – 5х2 + 13х – 6;

2) х4 – 4х3 – 20х2 + 13х – 2.

6*. Разложите многочлен на множители, заранее записав его с помощью ме­тода неопределенных коэффициентов в виде (х2 + + с)2 – (+ n)2: :

1) х4+ 4х – 1;

2) х4 – 4х3 – 1;

3) х4 + 4а3х а4.

Решить уравнение y 0. Решение квадратных уравнений

2x 4 + 5x 3 — 11x 2 — 20x + 12 = 0

Для начала нужно методом подбора найти один корень. Обычно он является делителем свободного члена. В данном случае делителями числа 12 являются ±1, ±2, ±3, ±4, ±6, ±12. Начнем их подставлять по-очереди:

1: 2 + 5 — 11 — 20 + 12 = -12 ⇒ число 1

-1: 2 — 5 — 11 + 20 + 12 = 18 ⇒ число -1 не является корнем многочлена

2: 2 ∙ 16 + 5 ∙ 8 — 11 ∙ 4 — 20 ∙ 2 + 12 = 0 ⇒ число 2 является корнем многочлена

Мы нашли 1 из корней многочлена. Корнем многочлена является 2, а значит исходный многочлен должен делиться на x — 2 . Для того, чтобы выполнить деление многочленов, воспользуемся схемой Горнера:

В верхней строке выставляются коэффициенты исходного многочлена. В первой ячейке второй строки ставится найденный нами корень 2. Во второй строке пишутся коэффициенты многочлена, который получится в результате деления. Они считаются так:

Во вторую ячейку второй строки запишем число 2, просто перенеся его из соответствующей ячейки первой строки.
2 ∙ 2 + 5 = 9
2 ∙ 9 — 11 = 7
25-11-2012
2297-6
2 ∙ 7 — 20 = -6
25-11-2012
2297-60
2 ∙ (-6) + 12 = 0

Последнее число — это остаток от деления. Если он равен 0, значит мы все верно посчитали.

2x 4 + 5x 3 — 11x 2 — 20x + 12 = (x — 2)(2x 3 + 9x 2 + 7x — 6)

Но это еще не конец. Можно попробовать разложить таким же способом многочлен 2x 3 + 9x 2 + 7x — 6.

Опять ищем корень среди делителей свободного члена. Делителями числа -6 являются ±1, ±2, ±3, ±6.

1: 2 + 9 + 7 — 6 = 12 ⇒ число 1 не является корнем многочлена

-1: -2 + 9 — 7 — 6 = -6 ⇒ число -1 не является корнем многочлена

2: 2 ∙ 8 + 9 ∙ 4 + 7 ∙ 2 — 6 = 60 ⇒ число 2 не является корнем многочлена

-2: 2 ∙ (-8) + 9 ∙ 4 + 7 ∙ (-2) — 6 = 0 ⇒ число -2 является корнем многочлена

Напишем найденный корень в нашу схему Горнера и начнем заполнять пустые ячейки:

25-11-2012
2297-60
-22
Во вторую ячейку третьей строки запишем число 2, просто перенеся его из соответствующей ячейки второй строки.
25-11-2012
2297-60
-225
-2 ∙ 2 + 9 = 5
25-11-2012
2297-60
-225-3
-2 ∙ 5 + 7 = -3
25-11-2012
2297-60
-225-30
-2 ∙ (-3) — 6 = 0

Таким образом мы исходный многочлен разложили на множители:

2x 4 + 5x 3 — 11x 2 — 20x + 12 = (x — 2)(x + 2)(2x 2 + 5x — 3)

Многочлен 2x 2 + 5x — 3 тоже можно разложить на множители. Для этого можно решить квадратное уравнение через дискриминант , а можно поискать корень среди делителей числа -3. Так или иначе, мы придем к выводу, что корнем этого многочлена является число -3

25-11-2012
2297-60
-225-30
-32
Во вторую ячейку четвертой строки запишем число 2, просто перенеся его из соответствующей ячейки третьей строки.
25-11-2012
2297-60
-225-30
-32-1
-3 ∙ 2 + 5 = -1
25-11-2012
2297-60
-225-30
-32-10
-3 ∙ (-1) — 3 = 0

Таким образом мы исходный многочлен разложили на линейные множители:

2x 4 + 5x 3 — 11x 2 — 20x + 12 = (x — 2)(x + 2)(x + 3)(2x — 1)

А корнями уравнения являются.

I. Линейные уравнения

II. Квадратные уравнения

ax 2 + bx + c = 0, a ≠ 0, иначе уравнение становится линейным

Корни квадратного уравнения можно вычислять различными способами, например:

Мы хорошо умеем решать квадратные уравнения. Многие уравнения более высоких степеней можно привести к квадратным.

III. Уравнения, приводимые к квадратным.

замена переменной: а) биквадратное уравнение ax 2n + bx n + c = 0, a ≠ 0, n ≥ 2

2) симметрическое уравнение 3 степени – уравнение вида

3) симметрическое уравнение 4 степени – уравнение вида

ax 4 + bx 3 + cx 2 + bx + a = 0, a ≠ 0, коэффициенты a b c b a или

ax 4 + bx 3 + cx 2 – bx + a = 0, a ≠ 0, коэффициенты a b c (–b) a

Т.к. x = 0 не является корнем уравнения, то возможно деление обеих частей уравнения на x 2 , тогда получаем: .

Произведя замену решаем квадратное уравнение a (t 2 – 2) + bt + c = 0

Например, решим уравнение x 4 – 2x 3 – x 2 – 2x + 1 = 0, делим обе части на x 2 ,

, после замены получаем уравнение t 2 – 2t – 3 = 0

– уравнение не имеет корней.

4) Уравнение вида (x – a )(x – b )(x – c )(x – d ) = Ax 2 , коэффициенты ab = cd

Например, (x + 2 )(x +3 )(x + 8 )(x + 12 ) = 4x 2 . Перемножив 1–4 и 2–3 скобки, получим (x 2 + 14x + 24)(x 2 +11x + 24) = 4x 2 , разделим обе части уравнения на x 2 , получим:

Имеем (t + 14)(t + 11) = 4.

5) Однородное уравнение 2 степени – уравнение вида Р(х,у) = 0, где Р(х,у) – многочлен, каждое слагаемое которого имеет степень 2.

Ответ: -2; -0,5; 0

IV. Все приведенные уравнения узнаваемы и типичны, а как быть с уравнениями произвольного вида?

Пусть дан многочлен P n (x ) = a n x n + a n-1 x n-1 + …+a 1 x + a 0 , где a n ≠ 0

Рассмотрим метод понижения степени уравнения.

Известно, что, если коэффициенты a являются целыми числами и a n = 1 , то целые корни уравнения P n (x ) = 0 находятся среди делителей свободного члена a 0 . Например, x 4 + 2x 3 – 2x 2 – 6x + 5 = 0, делителями числа 5 являются числа 5; –5; 1; –1. Тогда P 4 (1) = 0, т.е. x = 1 является корнем уравнения. Понизим степень уравнения P 4 (x ) = 0 с помощью деления “уголком” многочлена на множитель х –1, получаем

P 4 (x ) = (x – 1)(x 3 + 3x 2 + x – 5).

Аналогично, P 3 (1) = 0, тогда P 4 (x ) = (x – 1)(x – 1)(x 2 + 4x +5), т.е. уравнение P 4 (x) = 0 имеет корни x 1 = x 2 = 1. Покажем более короткое решение этого уравнения (с помощью схемы Горнера).

12–2–65
1131–50
11450

значит, x 1 = 1 значит, x 2 = 1.

Итак, (x – 1) 2 (x 2 + 4x + 5) = 0

Что мы делали? Понижали степень уравнения.

V. Рассмотрим симметрические уравнения 3 и 5 степени.

а) ax 3 + bx 2 + bx + a = 0, очевидно, x = –1 корень уравнения, далее понижаем степень уравнения до двух.

б) ax 5 + bx 4 + cx 3 + cx 2 + bx + a = 0, очевидно, x = –1 корень уравнения, далее понижаем степень уравнения до двух.

Например, покажем решение уравнения 2x 5 + 3x 4 – 5x 3 – 5x 2 + 3x + = 0

23–5–532
–121–6120
123–3–20
12520

x = –1

Получаем (x – 1) 2 (x + 1)(2x 2 + 5x + 2) = 0. Значит, корни уравнения: 1; 1; –1; –2; –0,5.

VI. Приведем список различных уравнений для решения в классе и дома.

Предлагаю читателю самому решить уравнения 1–7 и получить ответы…

Квадратные уравнения изучают в 8 классе, поэтому ничего сложного здесь нет. Умение решать их совершенно необходимо.

Квадратное уравнение — это уравнение вида ax 2 + bx + c = 0, где коэффициенты a , b и c — произвольные числа, причем a ≠ 0.

Прежде, чем изучать конкретные методы решения, заметим, что все квадратные уравнения можно условно разделить на три класса:

  1. Не имеют корней;
  2. Имеют ровно один корень;
  3. Имеют два различных корня.

В этом состоит важное отличие квадратных уравнений от линейных, где корень всегда существует и единственен. Как определить, сколько корней имеет уравнение? Для этого существует замечательная вещь — дискриминант .

Дискриминант

Пусть дано квадратное уравнение ax 2 + bx + c = 0. Тогда дискриминант — это просто число D = b 2 − 4ac .

Эту формулу надо знать наизусть. Откуда она берется — сейчас неважно. Важно другое: по знаку дискриминанта можно определить, сколько корней имеет квадратное уравнение. А именно:

  1. Если D
  2. Если D = 0, есть ровно один корень;
  3. Если D > 0, корней будет два.

Обратите внимание: дискриминант указывает на количество корней, а вовсе не на их знаки, как почему-то многие считают. Взгляните на примеры — и сами все поймете:

Задача. Сколько корней имеют квадратные уравнения:

  1. x 2 − 8x + 12 = 0;
  2. 5x 2 + 3x + 7 = 0;
  3. x 2 − 6x + 9 = 0.

Выпишем коэффициенты для первого уравнения и найдем дискриминант:
a = 1, b = −8, c = 12;
D = (−8) 2 − 4 · 1 · 12 = 64 − 48 = 16

Итак, дискриминант положительный, поэтому уравнение имеет два различных корня. Аналогично разбираем второе уравнение:
a = 5; b = 3; c = 7;
D = 3 2 − 4 · 5 · 7 = 9 − 140 = −131.

Дискриминант отрицательный, корней нет. Осталось последнее уравнение:
a = 1; b = −6; c = 9;
D = (−6) 2 − 4 · 1 · 9 = 36 − 36 = 0.

Дискриминант равен нулю — корень будет один.

Обратите внимание, что для каждого уравнения были выписаны коэффициенты. Да, это долго, да, это нудно — зато вы не перепутаете коэффициенты и не допустите глупых ошибок. Выбирайте сами: скорость или качество.

Кстати, если «набить руку», через некоторое время уже не потребуется выписывать все коэффициенты. Такие операции вы будете выполнять в голове. Большинство людей начинают делать так где-то после 50-70 решенных уравнений — в общем, не так и много.

Корни квадратного уравнения

Теперь перейдем, собственно, к решению. Если дискриминант D > 0, корни можно найти по формулам:

Основная формула корней квадратного уравнения

Когда D = 0, можно использовать любую из этих формул — получится одно и то же число, которое и будет ответом. Наконец, если D

  1. x 2 − 2x − 3 = 0;
  2. 15 − 2x − x 2 = 0;
  3. x 2 + 12x + 36 = 0.

Первое уравнение:
x 2 − 2x − 3 = 0 ⇒ a = 1; b = −2; c = −3;
D = (−2) 2 − 4 · 1 · (−3) = 16.

D > 0 ⇒ уравнение имеет два корня. Найдем их:

Второе уравнение:
15 − 2x − x 2 = 0 ⇒ a = −1; b = −2; c = 15;
D = (−2) 2 − 4 · (−1) · 15 = 64.

D > 0 ⇒ уравнение снова имеет два корня. Найдем их

\[\begin{align} & {{x}_{1}}=\frac{2+\sqrt{64}}{2\cdot \left(-1 \right)}=-5; \\ & {{x}_{2}}=\frac{2-\sqrt{64}}{2\cdot \left(-1 \right)}=3. \\ \end{align}\]

Наконец, третье уравнение:
x 2 + 12x + 36 = 0 ⇒ a = 1; b = 12; c = 36;
D = 12 2 − 4 · 1 · 36 = 0.

D = 0 ⇒ уравнение имеет один корень. Можно использовать любую формулу. Например, первую:

Как видно из примеров, все очень просто. Если знать формулы и уметь считать, проблем не будет. Чаще всего ошибки возникают при подстановке в формулу отрицательных коэффициентов. Здесь опять же поможет прием, описанный выше: смотрите на формулу буквально, расписывайте каждый шаг — и очень скоро избавитесь от ошибок.

Неполные квадратные уравнения

Бывает, что квадратное уравнение несколько отличается от того, что дано в определении. Например:

  1. x 2 + 9x = 0;
  2. x 2 − 16 = 0.

Несложно заметить, что в этих уравнениях отсутствует одно из слагаемых. Такие квадратные уравнения решаются даже легче, чем стандартные: в них даже не потребуется считать дискриминант. Итак, введем новое понятие:

Уравнение ax 2 + bx + c = 0 называется неполным квадратным уравнением, если b = 0 или c = 0, т.е. коэффициент при переменной x или свободный элемент равен нулю.

Разумеется, возможен совсем тяжелый случай, когда оба этих коэффициента равны нулю: b = c = 0. В этом случае уравнение принимает вид ax 2 = 0. Очевидно, такое уравнение имеет единственный корень: x = 0.

Рассмотрим остальные случаи. Пусть b = 0, тогда получим неполное квадратное уравнение вида ax 2 + c = 0. Немного преобразуем его:

Поскольку арифметический квадратный корень существует только из неотрицательного числа, последнее равенство имеет смысл исключительно при (−c /a ) ≥ 0. Вывод:

  1. Если в неполном квадратном уравнении вида ax 2 + c = 0 выполнено неравенство (−c /a ) ≥ 0, корней будет два. Формула дана выше;
  2. Если же (−c /a )

Как видите, дискриминант не потребовался — в неполных квадратных уравнениях вообще нет сложных вычислений. На самом деле даже необязательно помнить неравенство (−c /a ) ≥ 0. Достаточно выразить величину x 2 и посмотреть, что стоит с другой стороны от знака равенства. Если там положительное число — корней будет два. Если отрицательное — корней не будет вообще.

Теперь разберемся с уравнениями вида ax 2 + bx = 0, в которых свободный элемент равен нулю. Тут все просто: корней всегда будет два. Достаточно разложить многочлен на множители:

Вынесение общего множителя за скобку

Произведение равно нулю, когда хотя бы один из множителей равен нулю. Отсюда находятся корни. В заключение разберем несколько таких уравнений:

Задача. Решить квадратные уравнения:

  1. x 2 − 7x = 0;
  2. 5x 2 + 30 = 0;
  3. 4x 2 − 9 = 0.

x 2 − 7x = 0 ⇒ x · (x − 7) = 0 ⇒ x 1 = 0; x 2 = −(−7)/1 = 7.

5x 2 + 30 = 0 ⇒ 5x 2 = −30 ⇒ x 2 = −6. Корней нет, т.к. квадрат не может быть равен отрицательному числу.

4x 2 − 9 = 0 ⇒ 4x 2 = 9 ⇒ x 2 = 9/4 ⇒ x 1 = 3/2 = 1,5; x 2 = −1,5.

Цели:

  1. Систематизировать и обобщить знания и умения по теме: Решения уравнений третьей и четвертой степени.
  2. Углубить знания, выполнив ряд заданий, часть из которых не знакома или по своему типу, или способу решения.
  3. Формирование интереса к математике через изучение новых глав математики, воспитание графической культуры через построение графиков уравнений.

Тип урока : комбинированный.

Оборудование: графопроектор.

Наглядность: таблица «Теорема Виета».

Ход урока

1. Устный счет

а) Чему равен остаток от деления многочлена р n (х) = а n х n + а n-1 х n-1 + … + а 1 х 1 + a 0 на двучлен х-а?

б) Сколько корней может иметь кубическое уравнение?

в) С помощью чего мы решаем уравнение третьей и четвертой степени?

г) Если b четное число в квадратном уравнение, то чему равен Д и х 1 ;х 2

2. Самостоятельная работа (в группах)

Составить уравнение, если известны корни (ответы к заданиям закодированы) Используется «Теорема Виета»

1 группа

Корни: х 1 = 1; х 2 = -2; х 3 = -3; х 4 = 6

Составить уравнение:

B=1 -2-3+6=2; b=-2

с=-2-3+6+6-12-18= -23; с= -23

d=6-12+36-18=12; d= -12

е=1(-2)(-3)6=36

х 4 — 2 х 3 — 23х 2 — 12 х + 36 = 0 (это уравнение решает потом 2 группа на доске)

Решение . Целые корни ищем среди делителей числа 36.

р = ±1;±2;±3;±4;±6…

р 4 (1)=1-2-23-12+36=0 Число 1 удовлетворяет уравнению, следовательно, =1 корень уравнения. По схеме Горнера

р 3 (x) = х 3 -х 2 -24x -36

р 3 (-2) = -8 -4 +48 -36=0, х 2 =-2

р 2 (x) = х 2 -3х -18=0

х 3 =-3, х 4 =6

Ответ: 1;-2;-3;6 сумма корней 2 (П)

2 группа

Корни: х 1 = -1; х 2 = х 3 =2; х 4 =5

Составить уравнение:

B=-1+2+2+5-8; b= -8

с=2(-1)+4+10-2-5+10=15; с=15

D=-4-10+20-10= -4; d=4

е=2(-1)2*5=-20;е=-20

8+15+4х-20=0 (это уравнение решает на доске 3 группа)

р = ±1;±2;±4;±5;±10;±20.

р 4 (1)=1-8+15+4-20=-8

р 4 (-1)=1+8+15-4-20=0

р 3 (x) = х 3 -9х 2 +24x -20

р 3 (2) = 8 -36+48 -20=0

р 2 (x) = х 2 -7х +10=0 х 1 =2; х 2 =5

Ответ: -1;2;2;5 сумма корней 8(Р)

3 группа

Корни: х 1 = -1; х 2 =1; х 3 =-2; х 4 =3

Составить уравнение:

В=-1+1-2+3=1;в=-1

с=-1+2-3-2+3-6=-7;с=-7

D=2+6-3-6=-1; d=1

е=-1*1*(-2)*3=6

х 4 — х 3 — 7х 2 + х + 6 = 0 (это уравнение решает потом на доске 4 группа)

Решение. Целые корни ищем среди делителей числа 6.

р = ±1;±2;±3;±6

р 4 (1)=1-1-7+1+6=0

р 3 (x) = х 3 — 7x -6

р 3 (-1) = -1+7-6=0

р 2 (x) = х 2 -х -6=0; х 1 =-2; х 2 =3

Ответ:-1;1;-2;3 Сумма корней 1(О)

4 группа

Корни: х 1 = -2; х 2 =-2; х 3 =-3; х 4 =-3

Составить уравнение:

B=-2-2-3+3=-4; b=4

с=4+6-6+6-6-9=-5; с=-5

D=-12+12+18+18=36; d=-36

е=-2*(-2)*(-3)*3=-36;е=-36

х 4 + 4х 3 – 5х 2 – 36х -36 = 0 (это уравнение решает потом 5 группа на доске)

Решение. Целые корни ищем среди делителей числа -36

р = ±1;±2;±3…

р(1)= 1 + 4-5-36-36 = -72

р 4 (-2) = 16 -32 -20 + 72 -36 = 0

р 3 (х) = х 3 +2х 2 -9х-18 = 0

р 3 (-2)= -8 + 8 + 18-18 = 0

р 2 (х) = х 2 -9 = 0; x=±3

Ответ: -2; -2; -3; 3 Сумма корней-4 (Ф)

5 группа

Корни: х 1 = -1; х 2 =-2; х 3 =-3; х 4 =-4

Составить уравнение

х 4 + 10х 3 + 35х 2 + 50х + 24 = 0 (это уравнение решает потом 6группа на доске)

Решение . Целые корни ищем среди делителей числа 24.

р = ±1;±2;±3

р 4 (-1) = 1 -10 + 35 -50 + 24 = 0

р 3 (х) = x- 3 + 9х 2 + 26x+ 24 = 0

p 3 (-2) = -8 + 36-52 + 24 = О

р 2 (х) = x 2 + 7x+ 12 = 0

Ответ:-1;-2;-3;-4 сумма-10 (И)

6 группа

Корни: х 1 = 1; х 2 = 1; х 3 = -3; х 4 = 8

Составить уравнение

B=1+1-3+8=7;b=-7

с=1 -3+8-3+8-24= -13

D=-3-24+8-24= -43; d=43

х 4 — 7х 3 — 13х 2 + 43 x — 24 = 0 (это уравнение решает потом 1 группа на доске)

Решение . Целые корни ищем среди делителей числа -24.

р 4 (1)=1-7-13+43-24=0

р 3 (1)=1-6-19+24=0

р 2 (x)= х 2 -5x — 24 = 0

х 3 =-3, х 4 =8

Ответ: 1;1;-3;8 сумма 7 (Л)

3. Решение уравнений с параметром

1. Решить уравнение х 3 + 3х 2 + mх — 15 = 0; если один из корней равен (-1)

Ответ записать в порядке возрастания

R=Р 3 (-1)=-1+3-m-15=0

х 3 + 3х 2 -13х — 15 = 0; -1+3+13-15=0

По условию х 1 = — 1; Д=1+15=16

Р 2 (х) = х 2 +2х-15 = 0

х 2 =-1-4 = -5;

х 3 =-1 + 4 = 3;

Ответ:- 1;-5; 3

В порядке возрастания: -5;-1;3. (Ь Н Ы)

2. Найти все корни многочлена х 3 — 3х 2 + ах — 2а + 6, если остатки от его деления на двучлены х-1 и х +2 равны.

Решение: R=Р 3 (1) = Р 3 (-2)

Р 3 (1) = 1-3 + а- 2а + 6 = 4-а

Р 3 (-2) = -8-12-2а-2а + 6 = -14-4а

x 3 -Зх 2 -6х + 12 + 6 = х 3 -Зх 2 -6х + 18

x 2 (x-3)-6(x-3) = 0

(х-3)(х 2 -6) = 0

Произведение двух множителей равно нулю тогда и только тогда, когда хотя бы один из этих множителей равен нулю, а другой при этом имеет смысл.

2 группа . Корни: -3; -2; 1; 2;

3 группа . Корни: -1; 2; 6; 10;

4 группа . Корни: -3; 2; 2; 5;

5 группа . Корни: -5; -2; 2; 4;

6 группа . Корни: -8; -2; 6; 7.

Примеры ОГЭ по математике | Геометрия

Примеры ОГЭ по математике | Геометрия — просто!
Добрый день, друзья!

Сегодня мы будем решать примеры ОГЭ по математике —
четвёртое задание, а именно уравнения.
В этом задании представлены  разные виды уравнений: линейные, квадратные, дробные, а также системы уравнений.
Для каждого из видов есть свои методы решения, о чём мы и поговорим сегодня.
Задача 1. Решить уравнение (х-9)² = (х-3)²

Решение: Не смотря на квадраты, это уравнение линейное, и его можно решать двумя способами.
Способ 1. х² — 18х + 81 = х² — 6х + 9
18х — 6х = 81 — 9
12х = 72
х = 6
Способ 2. (х-9)² — (х-3)² = 0
(х-9+х-3)(х-9-х+3) = 0
(2х-12)(-6) = 0
(-12)(х-6) = 0
х = 6
Ответ: 6
Задача 2. Решить уравнение (х+3)² + (х-7)² = 2х²

Решение: Это уравнение также приводится к линейному уравнению:
х²+6х+9+х²-14х+49=2х²
14х — 6х = 9 + 49
8х = 58
х = 7,25
Ответ: 7,25
Задача 3. Решите уравнение  х² — 4х + 35 = -9х² + 11х + 45

Решение: Уравнение квадратное.
Приводим его к каноническому виду и проверяем дискриминант.
10х² — 15х — 10 = 0     Делим правую и левую часть уравнения на 5
2х² — 3х — 2 = 0
Поскольку свободный член имеет знак -,
то уравнение ВСЕГДА будет иметь 2 корня.
_3±√(9 + 4•2•2)_  =    _3±5_
2•2                                    4
х1 = 2 х2 = -0,5
Ответ: -0,5; 2
Задача 4. Решите уравнение  х²/2 +3х + 4 = 0

Решение: В первую очередь освобождаемся от дроби в левой части уравнения.
Для этого правую и левую часть уравнения умножаем на 2.
х² + 6х + 8 = 0 Это приведённое квадратное уравнение.
И проще всего его решать по теореме Виета.
Сумма корней приведённого квадратного уравнения равна второму коэффициенту с противоположным знаком, а произведение корней равно свободному члену.
Другими словами:
х1 + х2 = -6
х1•х2 = 8
Начинаем с произведения. Раскладываем число 8 на множители и подбираем их таким образом, чтобы в сумме они дали -6.
Это очевидно -2  и -4.
Произведение двух отрицательных чисел даёт нам положительное число,
а их сумма также отрицательна.
Ответ: -4; -2
Задача 5.  Решите уравнение   х² + 3,5х = 2

Решение: Это уравнение по теореме Виета будет решить сложнее,
чем предыдущее, потому что второй множитель — дробный.
В таком случае домножим правую и левую часть уравнения на 2.
2х² + 7х = 4
2х² + 7х — 4 = 0    Придётся решать это уравнение как полное.
_-7±√(49 + 4•2•4)_  =    _-7±9_
2•2                                     4
х1 = -4 х2 = 0,5
Ответ: -4; 0,5
Задача 6. Решить уравнение   х² — 6(х — 4) — 4х + 1  = 0 

Решение: х² — 6(х — 4) — 4х + 1  = х² — 6х + 24 — 4х + 1  = х² — 10х  + 25  = 0
Если внимательно присмотреться, то можно увидеть
формулу сокращённого умножения — квадрат разности двух чисел х и 5.
х² — 10х  + 25  = (х — 5)²
Поэтому корень данного уравнения будет один.
Ответ: 5
Задача 7. Решить уравнение  -2х² + 7х  = 9

Решение: Перенесём число 9 в левую часть уравнения,
а затем умножим правую и левую часть уравнения на (-1).
2х² — 7х  + 9 = 0    Имеем полное квадратное уравнение.
_7±√(49 — 4•2•9)_  =    _7±√(-23)_    Дискриминант получился отрицательный. 
  2•2                                      4                                      Корней нет.
Ответ: нет решений.
Задача 8. Решите уравнение  2(х² — 40) = -х² + 6(х + 4) + 1

Решение: Раскрываем скобки, приводим подобные
2х² — 80 = -х² + 6х + 24 + 1 2х² — 80  + х² — 6х — 24 — 1 = 0
3х² —  6х — 105 = 0    Делим правую и левую часть уравнения на 3
х² — 2х — 35 = 0   И опять теорема Виета.
Множители числа 35:  7 и 5.
Свободный член отрицательный, значит корни имеют разные знаки.
Теперь надо их правильно расставить.
Сумма равна +2, значит положительное число по модулю больше,
чем отрицательное.
А теперь уже просто.
х1 = 7;  х2 = -5
Ответ: -5; 7
Задача 9.  Решите уравнение  х²/2 — 1/2 = х•(х+5)/6

Решение:  Как в предыдущих примерах с дробями, избавляемся от дробей путём умножения правой и левой части уравнения на 6.
3х² — 3 = х(х+5)
3х² — 3 = х² + 5х
2х² — 5х —  3 = 0
_5±√(25 + 4•2•3)_  =    _5±7_
            2•2                         4               
х1 = 3,   х2 = -0,5
Ответ: -0,5;  3
На сегодня всё. Успехов и до новых задач!

Вам так же будет интересно:

Оставить комментарий

3. Решение задач.

Решить уравнения:

а) ;

б) ;

в) ;

г) .

4. Итоги урока. Какие методы можно применять при решении дробно-рациональных уравнений?

5. Домашнее задание

Решить уравнения:

а)

б)

в)

г)

Творческое задание: решить уравнение:

Самостоятельные работы

Самостоятельная работа 1

Вариант 1

1. Преобразовать в многочлен:

а) (2а2 – 3в)3,

б) (а + 2)6.

2. Разложить на множители:

а) 27х3 + 108х2 +144х + 64,

б) 64х6 – у6.

3. Разделить многочлен на многочлен:

а) (х3 + 8х2 + 11х – 20) : (х + 5),

в) (х3 + 2х2 – 7х – 14) : (х + 2),

с) (2х4 + 4х3 – 11х2 – 10х +15) : (2х2 – 5).

Вариант 2

1. Преобразовать в многочлен:

а) (3а4 + 2в)3,

б) (х – 4)5.

2. Разложить на множители:

а) 8х3 – 60х2 +150х – 125,

б) 243х5 – у5.

3. Разделить многочлен на многочлен:

а) (х3 — 7х2 + 14х – 8) : (х – 2),

в) (х3 + 4х2 – 5х – 20) : (х + 4),

с) (2х4 + 6х3 – 9х2 – 21х +7) : (2х2 – 7).

Самостоятельная работа 2

Вариант 1

1. Сократить дробь: .

2. Выделить целую часть: а) ; в) .

3. Решить уравнения с помощью теоремы Безу:

а) х3 – 6х2 + 11х – 6 = 0,

в) х3 – 5х2 – 2х + 24 = 0,

с) х4 + 3х3 – 13х2 – 17х + 26 = 0.

Вариант 2

х3 + 9х2 + 27х + 27

1. Сократить дробь: х3 + 27 .

х4 + 5х – 2 х5 + 4

2. Выделить целую часть: а) х – 3 ; в) х3 – 2х + 1 .

3. Решить уравнения с помощью теоремы Безу:

а) х3 – 7х2 + 14х – 8 = 0,

в) х3 – х2 – 14 х + 24 = 0,

с) х4 + 4х3 – 9х2 – 16х + 20 = 0.

Самостоятельная работа 3

Вариант 1

1. Решить уравнения:

а) ,

б) ,

в) .

г) (х +2) (х + 4) (х + 6) (х + 8) = –15,

д) .

Вариант 2

1. Решить уравнения:

а) ,

б) .

в) (х2 +3х – 4) (х2 +3х –7 ) = 18,

г) (х – 2) (х – 4) (х – 6) (х – 8) = –15,

д) .

Самостоятельная работа № 4

Вариант 1

  1. Решить возвратные уравнения:

    1. 3 – 5х2 – 5х + 4 = 0,

    2. 4 + 5х3 – + 5х + 3 = 0.

  2. Решить однородные уравнения:

    1. 3(х2 – 5)2 + 4(х2 – 5) (х + 7) – 7 (х + 7)2 = 0,

    2. (х – 2)4 + 5(х + 2)4 = 6(х2 – 4)2.

Вариант 2

  1. Решить возвратные уравнения:

    1. 3 – 4х2 – 4х + 5 = 0,

    2. 4 – 5х3 + 4х2 – 5х + 2 = 0.

  2. Решить однородные уравнения:

  1. 3(х2 + 5)2 + 4(х2 + 5) (х – 7) – 7 (х – 7)2 = 0,

  2. (х-3)4 + 4(х + 3)4 = 5(х2 – 9)2.

Самостоятельная работа № 5

Вариант 1

  1. Решить дробно-рациональные уравнения:

    а) ,

    б) .

  2. Решить уравнения:

    1. |х — 5| + |х + 2| = 7,

    2. |2х – 3| + |2х – 5| = 2,

    3. 5|х|2 – 3|х| = 2.

Вариант 2

      1. Решить дробно-рациональные уравнения:

а) ,

б) .

      1. Решить уравнения:

        1. |х + 4| + |х — 7| = 11,

        2. |2х + 3| + |2х – 5| = 8,

        3. 7|х|2 – 4|х| = 3.

Самостоятельная работа № 6

Вариант 1

  1. Решить уравнения:

    1. ,

    2. sin 2x – 3cos 4x = 4,

    3. sin x = х2 + 4х + 5.

  2. Найти значения а, при которых уравнение

3sin x – 7 cos x = a

имеет корни.

Вариант 2

  1. Решить уравнения:

    1. .

    2. sin 2x – 4 cos 4x = 5,

    3. cos x = х2 + 6х + 10.

  1. Найти значения а, при которых уравнение

4 sin x – 5 cos x = a

имеет корни.

Самостоятельная работа № 7

Вариант 1

  1. Решить систему уравнений:

а ) х2 + ху + у2 = 37,

х + у = 7;

б ) 5х2 – 7ху + 2 у2 = 0,

2 + у2 = 4;

в) х2 + у2 + 3ху = 31,

ху = 6.

Вариант 2

  1. Решить систему уравнений:

а ) х2 — ху + у2 = 21,

х + у = 9;

б) 4х2 – 9ху + 5у2 = 0,

2 + 2у2 = 7;

в ) х2 + у2 + 5ху = 60,

ху = 8. .

Самостоятельная работа 8

Вариант 1

Решить систему уравнений:

а ) ,

;

x + y + 3z = 1,

б) 2x – y + 2z = 5,

–x + 2y – 5z = –4;

в) ах +2у = 6,

3ах — у = 2.

Вариант 2

Решить систему уравнений:

а ) ,

;

x + 2y – 4z = –1,

б) 2x – y + 3z = 9,

–x + 4y – 2z = –5;

в ) х – 3ау = 4,

3х + ау = 7.

Самостоятельная работа 9

Вариант 1

1. Решить неравенства:

а) ,

б)  5.

2. Изобразить на плоскости множество решений неравенства:

а) 2х – 5у + 10  0,

б) ху  –6.

Вариант 2

1. Решить неравенства:

а) ,

б)  4.

2. Изобразить на плоскости множество решений неравенства:

а) 3х + 2у – 8  0,

б) ху  –8.

Зачет № 1

по теме «Алгебраические уравнения»

Вариант 1.

  1. Теорема Безу.

Решить уравнение: х3 – 8х2 + 19х – 12 = 0.

  1. Какое уравнение называется следствием из другого уравнения?

Какое из данных уравнений является следствием другого уравнения:

2(х + 3) + х (х – 4) = (х – 4) (х + 3 )

или + = 1 ?

  1. Какое уравнение называется однородным? Привести пример уравнения с двумя переменными. Решить уравнение:

2(х2 – 1)2 – 5(х2 – 1) (х2 + 4х) + 2 (х2 + 4х)2 = 0.

Решить квадратное уравнение онлайн. Уравнения с двумя переменными Решение уравнений с параметром

Цели:

  1. Систематизировать и обобщить знания и умения по теме: Решения уравнений третьей и четвертой степени.
  2. Углубить знания, выполнив ряд заданий, часть из которых не знакома или по своему типу, или способу решения.
  3. Формирование интереса к математике через изучение новых глав математики, воспитание графической культуры через построение графиков уравнений.

Тип урока : комбинированный.

Оборудование: графопроектор.

Наглядность: таблица «Теорема Виета».

Ход урока

1. Устный счет

а) Чему равен остаток от деления многочлена р n (х) = а n х n + а n-1 х n-1 + … + а 1 х 1 + a 0 на двучлен х-а?

б) Сколько корней может иметь кубическое уравнение?

в) С помощью чего мы решаем уравнение третьей и четвертой степени?

г) Если b четное число в квадратном уравнение, то чему равен Д и х 1 ;х 2

2. Самостоятельная работа (в группах)

Составить уравнение, если известны корни (ответы к заданиям закодированы) Используется «Теорема Виета»

1 группа

Корни: х 1 = 1; х 2 = -2; х 3 = -3; х 4 = 6

Составить уравнение:

B=1 -2-3+6=2; b=-2

с=-2-3+6+6-12-18= -23; с= -23

d=6-12+36-18=12; d= -12

е=1(-2)(-3)6=36

х 4 — 2 х 3 — 23х 2 — 12 х + 36 = 0 (это уравнение решает потом 2 группа на доске)

Решение . Целые корни ищем среди делителей числа 36.

р = ±1;±2;±3;±4;±6…

р 4 (1)=1-2-23-12+36=0 Число 1 удовлетворяет уравнению, следовательно, =1 корень уравнения. По схеме Горнера

р 3 (x) = х 3 -х 2 -24x -36

р 3 (-2) = -8 -4 +48 -36=0, х 2 =-2

р 2 (x) = х 2 -3х -18=0

х 3 =-3, х 4 =6

Ответ: 1;-2;-3;6 сумма корней 2 (П)

2 группа

Корни: х 1 = -1; х 2 = х 3 =2; х 4 =5

Составить уравнение:

B=-1+2+2+5-8; b= -8

с=2(-1)+4+10-2-5+10=15; с=15

D=-4-10+20-10= -4; d=4

е=2(-1)2*5=-20;е=-20

8+15+4х-20=0 (это уравнение решает на доске 3 группа)

р = ±1;±2;±4;±5;±10;±20.

р 4 (1)=1-8+15+4-20=-8

р 4 (-1)=1+8+15-4-20=0

р 3 (x) = х 3 -9х 2 +24x -20

р 3 (2) = 8 -36+48 -20=0

р 2 (x) = х 2 -7х +10=0 х 1 =2; х 2 =5

Ответ: -1;2;2;5 сумма корней 8(Р)

3 группа

Корни: х 1 = -1; х 2 =1; х 3 =-2; х 4 =3

Составить уравнение:

В=-1+1-2+3=1;в=-1

с=-1+2-3-2+3-6=-7;с=-7

D=2+6-3-6=-1; d=1

е=-1*1*(-2)*3=6

х 4 — х 3 — 7х 2 + х + 6 = 0 (это уравнение решает потом на доске 4 группа)

Решение. Целые корни ищем среди делителей числа 6.

р = ±1;±2;±3;±6

р 4 (1)=1-1-7+1+6=0

р 3 (x) = х 3 — 7x -6

р 3 (-1) = -1+7-6=0

р 2 (x) = х 2 -х -6=0; х 1 =-2; х 2 =3

Ответ:-1;1;-2;3 Сумма корней 1(О)

4 группа

Корни: х 1 = -2; х 2 =-2; х 3 =-3; х 4 =-3

Составить уравнение:

B=-2-2-3+3=-4; b=4

с=4+6-6+6-6-9=-5; с=-5

D=-12+12+18+18=36; d=-36

е=-2*(-2)*(-3)*3=-36;е=-36

х 4 + 4х 3 – 5х 2 – 36х -36 = 0 (это уравнение решает потом 5 группа на доске)

Решение. Целые корни ищем среди делителей числа -36

р = ±1;±2;±3…

р(1)= 1 + 4-5-36-36 = -72

р 4 (-2) = 16 -32 -20 + 72 -36 = 0

р 3 (х) = х 3 +2х 2 -9х-18 = 0

р 3 (-2)= -8 + 8 + 18-18 = 0

р 2 (х) = х 2 -9 = 0; x=±3

Ответ: -2; -2; -3; 3 Сумма корней-4 (Ф)

5 группа

Корни: х 1 = -1; х 2 =-2; х 3 =-3; х 4 =-4

Составить уравнение

х 4 + 10х 3 + 35х 2 + 50х + 24 = 0 (это уравнение решает потом 6группа на доске)

Решение . Целые корни ищем среди делителей числа 24.

р = ±1;±2;±3

р 4 (-1) = 1 -10 + 35 -50 + 24 = 0

р 3 (х) = x- 3 + 9х 2 + 26x+ 24 = 0

p 3 (-2) = -8 + 36-52 + 24 = О

р 2 (х) = x 2 + 7x+ 12 = 0

Ответ:-1;-2;-3;-4 сумма-10 (И)

6 группа

Корни: х 1 = 1; х 2 = 1; х 3 = -3; х 4 = 8

Составить уравнение

B=1+1-3+8=7;b=-7

с=1 -3+8-3+8-24= -13

D=-3-24+8-24= -43; d=43

х 4 — 7х 3 — 13х 2 + 43 x — 24 = 0 (это уравнение решает потом 1 группа на доске)

Решение . Целые корни ищем среди делителей числа -24.

р 4 (1)=1-7-13+43-24=0

р 3 (1)=1-6-19+24=0

р 2 (x)= х 2 -5x — 24 = 0

х 3 =-3, х 4 =8

Ответ: 1;1;-3;8 сумма 7 (Л)

3. Решение уравнений с параметром

1. Решить уравнение х 3 + 3х 2 + mх — 15 = 0; если один из корней равен (-1)

Ответ записать в порядке возрастания

R=Р 3 (-1)=-1+3-m-15=0

х 3 + 3х 2 -13х — 15 = 0; -1+3+13-15=0

По условию х 1 = — 1; Д=1+15=16

Р 2 (х) = х 2 +2х-15 = 0

х 2 =-1-4 = -5;

х 3 =-1 + 4 = 3;

Ответ:- 1;-5; 3

В порядке возрастания: -5;-1;3. (Ь Н Ы)

2. Найти все корни многочлена х 3 — 3х 2 + ах — 2а + 6, если остатки от его деления на двучлены х-1 и х +2 равны.

Решение: R=Р 3 (1) = Р 3 (-2)

Р 3 (1) = 1-3 + а- 2а + 6 = 4-а

Р 3 (-2) = -8-12-2а-2а + 6 = -14-4а

x 3 -Зх 2 -6х + 12 + 6 = х 3 -Зх 2 -6х + 18

x 2 (x-3)-6(x-3) = 0

(х-3)(х 2 -6) = 0

3) а=0, х 2 -0*х 2 +0 = 0; х 2 =0; х 4 =0

а=0; х=0; х=1

а>0; х=1; х=а ± √а

2. Составить уравнение

1 группа . Корни: -4; -2; 1; 7;

2 группа . Корни: -3; -2; 1; 2;

3 группа . Корни: -1; 2; 6; 10;

4 группа . Корни: -3; 2; 2; 5;

5 группа . Корни: -5; -2; 2; 4;

6 группа . Корни: -8; -2; 6; 7.

Предлагаем вам удобный бесплатный онлайн калькулятор для решения квадратных уравнений. Вы сможете быстро получить и разобраться, как они решаются, на понятных примерах.
Чтобы произвести решение квадратного уравнения онлайн , вначале приведите уравнение к общему виду:
ax 2 + bx + c = 0
Заполните соответственно поля формы:

Как решить квадратное уравнение

Как решить квадратное уравнение: Виды корней:
1. Привести квадратное уравнение к общему виду:
Общий вид Аx 2 +Bx+C=0
Пример: 3х — 2х 2 +1=-1 Приводим к -2х 2 +3х+2=0

2. Находим дискриминант D.
D=B 2 -4*A*C .
Для нашего примера D= 9-(4*(-2)*2)=9+16=25.

3. Находим корни уравнения.
x1=(-В+D 1/2)/2А.
Для нашего случая x1=(-3+5)/(-4)=-0,5
x2=(-В-D 1/2)/2А.
Для нашего примера x2=(-3-5)/(-4)=2
Если В — четное число, то дискриманант и корни удобнее считать по формулам:
D=К 2 -ac
x1=(-K+D 1/2)/А
x2=(-K-D 1/2)/А,
Где K=B/2

1. Действительные корни. Причем. x1 не равно x2
Ситуация возникает, когда D>0 и A не равно 0.

2. Действительные корни совпадают. x1 равно x2
Ситуация возникает, когда D=0. Однако при этом, ни А, ни В, ни С не должны быть равны 0.

3. Два комплексных корня. x1=d+ei, x2=d-ei, где i=-(1) 1/2
Ситуация возникает, когда D
4. Уравнение имеет одно решение.
A=0, B и C нулю не равны. Уравнение становиться линейным.

5. Уравнение имеет бесчисленное множество решений.
A=0, B=0, C=0.

6. Уравнение решений не имеет.
A=0, B=0, C не равно 0.


Для закрепления алгоритма, вот еще несколько показательных примеров решений квадратных уравнений .

Пример 1. Решение обычного квадратного уравнения с разными действительными корнями.
x 2 + 3x -10 = 0
В этом уравнении
А=1, B = 3, С=-10
D=B 2 -4*A*C = 9-4*1*(-10) = 9+40 = 49
квадратный корень будем обозначать, как число 1/2 !
x1=(-В+D 1/2)/2А = (-3+7)/2 = 2
x2=(-В-D 1/2)/2А = (-3-7)/2 = -5

Для проверки подставим:
(x-2)*(x+5) = x2 -2x +5x – 10 = x2 + 3x -10

Пример 2. Решение квадратного уравнения с совпадением действительных корней.
х 2 – 8x + 16 = 0
А=1, B = -8, С=16
D = k 2 – AC = 16 – 16 = 0
X = -k/A = 4

Подставим
(x-4)*(x-4) = (x-4)2 = X 2 – 8x + 16

Пример 3. Решение квадратного уравнения с комплексными корнями.
13х 2 – 4x + 1 = 0
А=1, B = -4, С=9
D = b 2 – 4AC = 16 – 4*13*1 = 16 — 52 = -36
Дискриминант отрицательный – корни комплексные.

X1=(-В+D 1/2)/2А = (4+6i)/(2*13) = 2/13+3i/13
x2=(-В-D 1/2)/2А = (4-6i)/(2*13) = 2/13-3i/13
, где I – это квадратный корень из -1

Вот собственно все возможные случаи решения квадратных уравнений.
Надеемся, что наш онлайн калькулятор окажется весьма полезным для вас.
Если материал был полезен, вы можете

Представление об уравнениях с двумя переменными впервые формируется в курсе математики за 7 класс. Рассматриваются конкретные задачи, процесс решения которых приводит к такому виду уравнений.

При этом они изучаются довольно поверхностно. В программе главный акцент делается на системах уравнений с двумя неизвестными.

Это стало причиной того, что задачи, в которых на коэффициенты уравнения накладываются определенные ограничения, практически не рассматриваются. Недостаточно внимания уделено методам решения заданий типа «Решить уравнение в натуральных или целых числах». Известно, что материалы ЕГЭ и билеты вступительных экзаменов часто содержат такие упражнения.

Какие именно уравнения определяются как уравнения с двумя переменными?

ху = 8, 7х + 3у = 13 или х 2 + у = 7 – примеры уравнений с двумя переменными.

Рассмотрим уравнение х – 4у = 16. Если х = 4, а у = -3, оно будет правильным равенством. Значит, эта пара значений – решение данного уравнения.

Решение любого уравнения с двумя переменными – множество пар чисел (х; у), которые удовлетворяют это уравнение (превращают его в верное равенство).

Часто уравнение преобразовывают так, чтобы из него можно было получить систему для нахождения неизвестных.

Примеры

Решить уравнение: ху – 4 = 4х – у.

В данном примере можно воспользоваться методом разложения на множители. Для этого нужно сгруппировать слагаемые и вынести общий множитель за скобки:

ху – 4 = 4х – у;

ху – 4 – 4х + у = 0;

(ху + у) – (4х + 4) = 0;

у(х + 1) – 4(х + 1) = 0;

(х + 1)(у — 4) = 0.

Ответ: Все пары (х; 4), где х – любое рациональное число и (-1; у), где у – любое рациональное число.

Решить уравнение: 4х 2 + у 2 + 2 = 2(2х — у).

Первый шаг – группирование.

4х 2 + у 2 + 2 = 4х – 2у;

4х 2 + у 2 + 1 — 4х + 2у + 1 = 0;

(4х 2 – 4х +1) + (у 2 + 2у + 1) = 0.

Применив формулу квадрата разности, получим:

(2х — 1) 2 + (у + 1) 2 = 0.

При суммировании двух неотрицательных выражений ноль получится только в том случае, если 2х – 1 = 0 и у + 1 = 0. Отсюда следует: х = ½ и у = -1.

Ответ: (1/2; -1).

Решить уравнение (х 2 – 6х + 10)(у 2 + 10у + 29) = 4.

Рационально применить оценочный метод, выделив полные квадраты в скобках.

((х — 3) 2 + 1)((у + 5) 2 + 4) = 4.

При этом (х — 3) 2 + 1 ≥ 1, а (у + 5) 2 + 4 ≥ 4. Тогда левая часть уравнения всегда не меньше 4. Равенство возможно в случае

(х — 3) 2 + 1 = 1 и (у + 5) 2 + 4 = 4. Следовательно, х = 3, у = -5.

Ответ: (3; -5).

Решить уравнение в целых числах: х 2 + 10у 2 = 15х + 3.

Можно записать это уравнение в таком виде:

х 2 = -10у 2 + 15х + 3. Если правую часть равенства делить на 5, то 3 – остаток. Из этого следует, что х 2 не делится на 5. Известно, что квадрат числа, которое не делится на 5, должен дать в остатке или 1, или 4. Значит, уравнение корней не имеет.

Ответ: Решений нет.

Не стоит расстраиваться из-за трудностей в поиске верного решения для уравнения с двумя переменными. Упорство и практика обязательно принесут свои плоды.

В этой статье мы будем учиться решать биквадратные уравнения.

Итак, уравнения какого вида называются биквадратными?
Все уравнения вида ах 4 + bx 2 + c = 0 , гдеа ≠ 0 , являющиеся квадратными относительно х 2 , и называются биквадратными уравнениями. Как видите, эта запись очень похожа на запись квадратного уравнения, поэтому и решать биквадратные уравнения будем используя формулы, которые мы применяли при решении квадратного уравнения.

Только нам необходимо будет ввести новую переменную, то есть обозначим х 2 другой переменной, например, у или t (или же любой другой буквой латинского алфавита).

Например, решим уравнение х 4 + 4х 2 ‒ 5 = 0.

Обозначим х 2 через у (х 2 = у ) и получим уравнение у 2 + 4у – 5 = 0.
Как видите, такие уравнения вы уже умеете решать.

Решаем полученное уравнение:

D = 4 2 – 4 (‒ 5) = 16 + 20 = 36, √D = √36 = 6.

у 1 = (‒ 4 – 6)/2= ‒ 10 /2 = ‒ 5,

у 2 = (‒ 4 + 6)/2= 2 /2 = 1.

Вернемся к нашей переменной х.

Получили, что х 2 = ‒ 5 и х 2 = 1.

Замечаем, что первое уравнение решений не имеет, а второе дает два решения: х 1 = 1 и х 2 = ‒1. Будьте внимательны, не потеряйте отрицательный корень (чаще всего получают ответ х = 1, а это не правильно).

Ответ: — 1 и 1.

Для лучшего усвоения темы разберем несколько примеров.

Пример 1. Решите уравнение 2х 4 ‒ 5 х 2 + 3 = 0.

Пусть х 2 = у, тогда 2у 2 ‒ 5у + 3 =0.

D = (‒ 5) 2 – 4· 2 · 3 = 25 ‒ 24 = 1, √D = √1 = 1.

у 1 = (5 – 1)/(2· 2) = 4 /4 =1, у 2 = (5 + 1)/(2· 2) = 6 /4 =1,5.

Тогда х 2 = 1 и х 2 = 1,5.

Получаем х 1 = ‒1, х 2 = 1, х 3 = ‒ √1,5 , х 4 = √1,5.

Ответ: ‒1; 1; ‒ √1,5; √1,5.

Пример 2. Решите уравнение 2х 4 + 5 х 2 + 2 = 0.

2у 2 + 5у + 2 =0.

D = 5 2 – 4 · 2 · 2 = 25 ‒ 16 = 9, √D = √9 = 3.

у 1 = (‒ 5 – 3)/(2 · 2) = ‒ 8 /4 = ‒2, у 2 = (‒5 + 3)/(2 · 2) = ‒ 2 /4 = ‒ 0,5.

Тогда х 2 = ‒ 2 и х 2 = ‒ 0,5. Обратите внимание, ни одно из этих уравнений не имеет решения.

Ответ: решений нет.

Неполные биквадратные уравнения — это когда b = 0 (ах 4 + c = 0) или же c = 0

(ах 4 + bx 2 = 0) решают как и неполные квадратные уравнения.


Пример 3. Решить уравнение х 4 ‒ 25х 2 = 0

Разложим на множители, вынесем х 2 за скобки и тогда х 2 (х 2 ‒ 25) = 0.

Получим х 2 = 0 или х 2 ‒ 25 = 0, х 2 = 25.

Тогда имеем корни 0; 5 и – 5.

Ответ: 0; 5; – 5.

Пример 4. Решить уравнение 5х 4 ‒ 45 = 0 .

х 2 = ‒ √9 (решений не имеет)

х 2 = √9, х 1 = ‒ 3, х 2 = 3.

Как видите, умея решать квадратные уравнения, вы сможете справиться и с биквадратными.

Если же у вас остались вопросы, записывайтесь на мои уроки. Репетиор Валентина Галиневская.

сайт, при полном или частичном копировании материала ссылка на первоисточник обязательна.

Решение квадратных уравнений. Поурочные планы, Поурочные планы по алгебре 8 класс, Поурочные планы по алгебре и математике

Тема: Решение квадратных уравнений.

«Науками, которые предпочитаются другим
вследствие глубины доказательств, являются науки,
подобные математике.»

Цели урока: повторение, обобщение и систематизация методов решения квадратных уравнений;
формировать умение самоанализа и контроля;
развивать умение применять формулы при решении уравнений, умение пошаговой реализации алгоритма при решении уравнения;
развивать умение анализировать и делать выводы;
содействовать нравственному воспитанию учащихся.
Оборудование: карточки, таблички, таблицы с ответами, компьютер.
Тип урока: Повторительно – обобщающий
Ход урока.

Этап. Организационный момент.
Приветствие.
Деление на группы: 1,2, 3
Ознакомление учащихся с темой урока.
Проверяем домашнее задание
2. Этап. Актуализация знаний
1 задание. Мозговой штурм
Заполни пропуски…(по группам)
1. Равенство содержащее переменную, называется ….(уравнением)
2. Конем уравнения называется значение переменной, при котором уравнение обращается в верное …( числовое равенство)
3. Решить уравнение, это значит найти все его … (корни или доказать что корней нет)
4. Виды уравнений … (Неполные квадратные уравнения, квадратное-2 степени, …)
Устный счет (Выполнить по цепочке)
Выполнить своё и проверить предыдущее
5. По какому признаку можно объединить следующие уравнения
5х2-35х=0 5х2=0 5х2-35=0
(Неполные квадратные уравнения)

6. Назвать коэффициенты квадратного уравнения
5х2-7х+2=0 (5,-7,2)

7. Найти корни квадратного уравнения х2+5х-6=0 (-6 и 1)
8. Составить квадратное уравнение имеющее коэффициенты
а=3, в=5, с=2 (3х2+5х+2=0)
9. Составить квадратное уравнение имеющее корни
2 и 7 (х2-9х+14)=0

10. Исключить лишнее:
х(х-2)=0
х3+2х2+5=0
-5х-4=4х2
х2+2х+1=0

11. Найти корни уравнения:
У2-у=0 (0,1)

12. Исключить лишнее:
3х2+6х-9=0
х2-4х+4=0
х2-х+1=0
х2+6х-9=0

ІІІ. 3. Этап.
Решение уравнений
Работа в группах
Решив данные уравнения мы «вычислим» автора высказывания, являющегося эпиграфом нашего урока
«Науками, которые предпочитаются другим вследствие глубины доказательств, являются науки, подобные математике»

1 А 4х-5,5=5х-3(2х-1,5)
2 И 25-100х2=0
3 А 3х2+7х-6=0
4 Р 3х2-4х+1=0
5 Б 3х4-13х2+4=0
6 Ф 9х2+45х=0
Ответ:
Ф А Р А Б И
-5 и 0 -3;2/3 1/3;1 2 -2; – 1/3; 1/3; 2 -0,5;0,5

Это высказывание принадлежит великому ученому средневековья- «Аристотелю Востока»- Аль-Фараби. Он обладал широким кругозором, знал астрономию, медицину, социологию, этику, музыку, риторику- то есть был разносторонне одаренным человеком. Эти качества актуальны и в наше время – вам юношеству, можно порекомендовать брать пример с таких людей, как Аль-Фараби.
Имя ученого не забыто в нашей стране – о нём написано в казахстанских учебниках; его именем названы улицы, университеты.
Физминутка для глаз
4. Этап.

Дифференцированная самостоятельная разноуровневая работа:

Уровень А
1. Составить квадратное уравнение, где а=3, в=7,с=1.
2. Сколько корней имеет квадратное уравнение х2+2х+1=0.
3. Решить уравнение х2-3х=0.
4. Решить уравнение х2+25=0.
5. Решить уравнение 2х2-4х+3=0.

Уровень В
1.Решить уравнение х2-7х=0
2. Решить уравнение 4х2+36=0.
3. Решить уравнение 2х2-3х+1=0.
4. Найти сумму и произведение корней квадратного уравнения
х2-6х+8=0.
5. Составить квадратное уравнение имеющего корни х1=3 и х2=5.

Уровень С.
1. Найти сумму и произведение корней квадратного уравнения
3х2+4х+1=0.
2. Найти подбором корни уравнения х2+х-56=0.
3. Составить квадратное уравнение имеющего корни х1=-7 и х2=9.
4. Решить уравнение х3 +5х2+6х=0.
5. Решить уравнение Х4-7х2+6=0.
Ответы:

Уровень А
Уровень В
Уровень С

1 3х2+7х+1=0 0:7 х1+х2=-4/3
х1*х2=1/3

2 1 Нет корней -8 и 7
3 0; 3 0,5: 1 х2-2х-63=0
4 Нет корней х1+х2=6
х1*х2=8
-3; -2 и 0
5 1 и 3 х2-8х+15=0 -√6;-1;1; √6

6. Этап.
1. Найти наиболее рациональным способом корни уравнения:
а+в+с=о х1=1; х2=с/а; а-в+с=о х1=-1; х2=-с/а;

уравнения корни
2001х2-2008х+7=0
303 х2+27х-330=0
х2+2х-3=0
х2-5х+4=0
х2-10х+9=0

2. Мини исследование. Ученик решил уравнение:
Х2-|5х|-6=0
Х2-5х-6=0
Х2+5х-6=0
Получил корни {-6;-1;1;6} Прав ли ученик.
7. Этап.
Итоги урока
Мы рассмотрели различные методы решения квадратных уравнений
1. Метод коэффициентов.
2. По теореме Виета.
3. Если в- чётное.
4. По общей формуле.
6. Этап.
Домашнее задание.
Составить 4 уравнения, решить их. Составить кроссворд по теоретическому материалу.
8. Этап. Рефлексия
Оценочный лист
Ф.И.О. ………………………………………………………
№ Задание Оценка
1 Устная работа
2 Работа в группах

-5 и 0 -3;2/3 1/3;1 2 -2; – 1/3; 1/3; 2 -0,5;0,5

3 Уровневые задания
Уровень ответ
1 2 3 4 5

4 Найти ошибку
5 Мини исследование

6 Найти наиболее рациональным способом корни уравнения

7 Рефлексия. Итоги.

 

3−7x-6 = 0 Tiger Algebra Solver

Пошаговое решение:

Шаг 1:

Калькулятор полиномиальных корней:

1.1 Найдите корни (нули): F (x) = x 3 -7x- 6
Калькулятор полиномиальных корней — это набор методов, направленных на поиск значений x, для которых F (x) = 0

Rational Roots Test является одним из вышеупомянутых инструментов. Он может найти только рациональные корни, то есть числа x, которые можно выразить как частное двух целых чисел

Теорема рационального корня утверждает, что если полином обнуляется для рационального числа P / Q, то P является множителем конечной константы и Q является коэффициентом ведущего коэффициента

В этом случае ведущий коэффициент равен 1, а конечная константа — -6.

Фактор (ы):

ведущего коэффициента: 1
конечной постоянной: 1, 2, 3, 6

Давайте проверим ….

P Q P / Q F (P / Q) Делитель
-1 1 -1,00 0.00 x + 1
-2 1 -2,00 0,00 x + 2
-3 1 -3,00 -12,00
-6 1-6.00 -180,00
1 1 1,00 -12,00
2 1 2,00 -12,00
3 1 3.00 0,00 x-3
6 1 6,00 168,00


Теорема о факторах утверждает, что если P / Q является корнем многочлена, тогда этот многочлен можно разделить на q * xp.Обратите внимание, что q и p происходят от P / Q, уменьшенного до самого низкого значения

В нашем случае это означает, что
x 3 -7x-6
можно разделить на 3 разных полинома, в том числе на x-3

Полиномиальное деление в длину:

1.2 Полиномиальное длинное деление
Деление: x 3 -7x-6
(«Дивиденд»)
По: x-3 («Делитель»)

делимое x 3 7x 6
— делитель * x 2 x 3 3x 2
остаток 3x 2 7x 6
— делитель * 3x 1 3x 2 9x
остаток 2x 6
— делитель * 2x 0 2x 6
остаток 0

Частное: x 2 + 3x + 2 Остаток: 0

Попытка коэффициент путем разделения среднего члена

1.3 Факторинг x 2 + 3x + 2

Первый член равен x 2 , его коэффициент равен 1.
Средний член + 3x, его коэффициент равен 3.
Последний член, «константа», равен +2

Шаг-1: Умножьте коэффициент первого члена на константу 1 • 2 = 2

Шаг-2: Найдите два множителя 2, сумма которых равна коэффициенту среднего члена, который равен 3.

-2 +-1 =-3
-1 +-2 =-3
1 + 2 = 3 Вот и все


Шаг 3: Перепишите полином, разделяющий средний член, используя два фактора, найденные на шаге 2 выше: 1 и 2
x 2 + 1x + 2x + 2

Шаг 4: сложите первые 2 члена, вычитая одинаковые множители:
x • (x + 1)
Складываем последние 2 члена, вычитая общие множители :
2 • (x + 1)
Шаг 5: Сложите четыре члена шага 4:
(x + 2) • (x + 1)
Какая желаемая факторизация

Уравнение в конце шага 1:
 (x + 2) • (x + 1) • (x - 3) = 0
 

Шаг 2:

Теория — Корни продукта:

2.1 Произведение нескольких членов равно нулю.

Если произведение двух или более членов равно нулю, то хотя бы один из членов должен быть равен нулю.

Теперь мы решим каждый член = 0 отдельно

Другими словами, мы собираемся решить столько уравнений, сколько членов содержится в произведении

Любое решение для члена = 0 также решает продукт = 0.

Решение уравнения с одной переменной:

2.2 Решите: x + 2 = 0

Вычтите 2 из обеих частей уравнения:
x = -2

Решение уравнения с одной переменной:

2.3 Решите: x + 1 = 0

Вычтите 1 из обеих частей уравнения:
x = -1

Решение уравнения с одной переменной:

2.4 Решите: x-3 = 0

Добавьте 3 к обеим сторонам уравнения:
x = 3

Приложение: Непосредственное решение квадратного уравнения

 Непосредственное решение x  2  + 3x + 2 = 0 

Ранее мы разложили этот многочлен на множители, разделив средний член.давайте теперь решим уравнение, заполнив квадрат и используя квадратичную формулу

Парабола, найдя вершину:

3.1 Найдите вершину y = x 2 + 3x + 2

Параболы имеют наибольшее или наименьшее значение. точка называется Вершиной. Наша парабола открывается и, соответственно, имеет самую низкую точку (также известную как абсолютный минимум). Мы знаем это даже до того, как нанесли «y», потому что коэффициент первого члена, 1, положительный (больше нуля).

Каждая парабола имеет вертикальную линию симметрии, проходящую через ее вершину.Из-за этой симметрии линия симметрии, например, будет проходить через середину двух x-точек пересечения (корней или решений) параболы. То есть, если парабола действительно имеет два реальных решения.

Параболы могут моделировать множество реальных жизненных ситуаций, например высоту над землей объекта, брошенного вверх через некоторый промежуток времени. Вершина параболы может предоставить нам информацию, например, максимальную высоту, которую может достичь объект, брошенный вверх. По этой причине мы хотим иметь возможность найти координаты вершины.

Для любой параболы Ax 2 + Bx + C координата x вершины задается как -B / (2A). В нашем случае координата x равна -1,5000

Подставив в формулу параболы -1,5000 для x, мы можем вычислить координату y:
y = 1,0 * -1,50 * -1,50 + 3,0 * -1,50 + 2,0
или y = — 0,250

Парабола, графическая вершина и пересечения по оси X:

Корневой график для: y = x 2 + 3x + 2
Ось симметрии (пунктирная линия) {x} = {- 1,50}
Вершина в точке {x, y } = {-1.50, -0,25}
x -Пересечения (корни):
Корень 1 при {x, y} = {-2,00, 0,00}
Корень 2 при {x, y} = {-1,00, 0,00}

Решите квадратное уравнение путем заполнения квадрата

3.2 Решение x 2 + 3x + 2 = 0 путем заполнения квадрата.

Вычтем 2 из обеих частей уравнения:
x 2 + 3x = -2

Теперь умный бит: возьмите коэффициент при x, равный 3, разделите его на два, получив 3/2, и, наконец, возведите в квадрат. это дает 9/4

Добавьте 9/4 к обеим частям уравнения:
В правой части мы имеем:
-2 + 9/4 или, (-2/1) + (9/4)
. общий знаменатель этих двух дробей равен 4. Сложение (-8/4) + (9/4) дает 1/4
Таким образом, сложив обе части, мы в итоге получаем:
x 2 + 3x + (9/4) = 1 / 4

Добавление 9/4 завершило левую часть в полный квадрат:
x 2 + 3x + (9/4) =
(x + (3/2)) • (x + (3/2)) =
(x + (3/2)) 2
Вещи, которые равны одному и тому же, также равны друг другу.Поскольку
x 2 + 3x + (9/4) = 1/4 и
x 2 + 3x + (9/4) = (x + (3/2)) 2
, то по закону транзитивность,
(x + (3/2)) 2 = 1/4

Мы будем называть это уравнение уравнением. # 3.2.1

Принцип квадратного корня гласит, что когда две вещи равны, их квадратные корни равны.

Обратите внимание, что квадратный корень из
(x + (3/2)) 2 равен
(x + (3/2)) 2/2 =
(x + (3/2)) 1 =
x + (3/2)

Теперь, применяя принцип квадратного корня к уравнению.# 3.2.1 получаем:
x + (3/2) = √ 1/4

Вычтем 3/2 с обеих сторон, чтобы получить:
x = -3/2 + √ 1/4

Поскольку квадратный корень имеет два значения, одно положительное, а другое отрицательное
x 2 + 3x + 2 = 0
имеет два решения:
x = -3/2 + √ 1/4
или
x = -3/2 — √ 1 / 4

Обратите внимание, что √ 1/4 можно записать как
√ 1 / √ 4, что равно 1/2

Решите квадратное уравнение, используя квадратичную формулу

3.3 Решение x 2 + 3x + 2 = 0 по квадратичной формуле.

Согласно квадратичной формуле, x, решение для Ax 2 + Bx + C = 0, где A, B и C — числа, часто называемые коэффициентами, дается как:

— B ± √ B 2 -4AC
x = ————————
2A

В нашем случае A = 1
B = 3
C = 2

Соответственно B 2 — 4AC =
9-8 =
1

Применяя квадратную формулу:

-3 ± √ 1
x = —————
2

Итак, теперь мы смотрим на:
x = (-3 ± 1) / 2

Два Реальные решения:

x = (- 3 + √1) / 2 = -1.3-7x-6 = 0 | Wyzant Спросите эксперта

К сожалению, не существует простого правила разложения кубического многочлена на множители. Единственное, что мы можем сказать для любого полинома : если x = r является корнем полиномиального выражения, одним из его множителей будет (x-r). Когда мы имеем дело с кубическим уравнением, мы ожидаем трех корней — назовем их r 1 , r 2 и r 3 .

Это может показаться произвольным делом (потому что это так), но один из самых быстрых способов найти корни — это просто угадать и проверить простые значения.Я попробую 0, 1, 2 и 3:

f (x) = x 3 -7x-6

f (0) = 0 3 -7 * 0-6 = 0-0-6 = -6 x = 0 не является корнем, поэтому (x-0) = x не является множителем.

f (1) = 1 3 -7 * 1-6 = 1-7-6 = -12 x = 1 не является корнем, поэтому (x-1) не является множителем.

f (2) = 2 3 -7 * 2-6 = 8-14-6 = -12 x = 2 не является корнем, поэтому (x-2) не является множителем.

f (3) = 3 3 -7 * 3-6 = 27-21-6 = 0 x = 3 — корень, поэтому (x-3) — множитель.

Мы знаем, что x 3 -7x-6 превращается в нечто похожее на (x-3) (……..). Отсюда у нас есть два варианта:

  1. Сделайте полиномиальное деление, чтобы выяснить, что осталось: (…….) = (x 3 -7x-6) / (x-3). Тем не менее, деление полиномов беспорядочно, и если бы мы могли его избежать, это было бы неплохо.
  2. Продолжайте гадать, чтобы увидеть больше корней!

Мы нашли корни, соответствующие положительному значению x, поэтому давайте посмотрим с другой стороны:

f (-1) = (-1) 3 -7 * (- 1) -6 = -1 + 7-6 = 0 x = -1 — корень.(x + 1) — фактор.

f (-2) = (-2) 3 -7 * (- 2) -6 = -8 + 14-6 = 0 x = -2 — корень. (x + 2) — фактор.

Нам повезло, и мы довольно быстро нашли корни наугад и проверке. Делаем вывод, что x 3 -7x-6 = (x-3) (x + 2) (x + 1) .

Просто ради этого, умножьте разложенный на множители многочлен еще раз, чтобы убедиться, что мы получили приемлемый ответ.

(х-3) (х + 2) (х + 1)

2 + 2x-3x-6) (x + 1)

(x 2 -x-6) (x + 1)

(x 2 -x-6) x + (x 2 -x-6) 1

x 3 -x 2 -6x + x 2 -x-6

x 3 -7x-6

x3 — 7x + 6 = 0 имеет три решения: x = 1, 2, -3 / — The Beat The GMAT Forum

Привет.3 — 7x + 6, относительно легко увидеть, что f (1) = 0. Если x = 1 является корнем f (x), это означает, что (x — 1) является множителем f (x). (Это теорема о множителях.) Это означает, что мы можем разделить f (x) на (x — 1), чтобы получить квадратичную. Мы можем добиться этого либо путем деления полиномов в длину, либо путем синтетического деления. 2 + x — 6 = (x + 3) * (x — 2)

Это сразу дает два других корня, 2 и -3, так что три корня равны {1, 2, -3}

. Повторюсь, все это выходит далеко за рамки всего, что вам нужно знать для GMAT.(Вам жаль, что вы спросили в первую очередь?)

И снова GMAT может дать вам кубическую величину и попросить вас подставить числа — что из следующего является корнем — и тому подобное. Но на самом деле факторинг куба с нуля, без калькулятора — это лиги сверх того, что GMAT может разумно ожидать от тестируемых.

Надеюсь, все это поможет. Вот вопрос, который больше похож на GMAT:
https://gmat.magoosh.com/questions/114
Когда вы отправите свой ответ на этот вопрос, на следующей странице будет полное видео решение.В Magoosh у нас есть более 800 вопросов GMAT, каждый со своим видео-решением. У нас также есть более 200 видеоуроков, охватывающих весь контент и стратегии, которые вам понадобятся для GMAT. Из всех доступных высококачественных программ подготовки к GMAT Magoosh — самый доступный. В настоящее время у нас распродажа, которая заканчивается во вторник 3 апреля, так что сейчас самое подходящее время, чтобы проверить нас.

Пожалуйста, дайте мне знать, если у вас возникнут дополнительные вопросы, касающиеся материала GMAT или Precalculus.

Майк

Используйте формулу корней квадратного уравнения, чтобы решить уравнение.2 — 7x — 6 = 0 Покажи работу, пожалуйста.

Решение уравнения.

Дополнительные пояснения:

Уравнение со степенью 2 называется квадратным уравнением.

Общее квадратное уравнение можно записать как,

В приведенной выше формуле — действительные числа.

Корни квадратного уравнения могут быть найдены по правилу квадратов.

Здесь обозначает дискриминант.

Так как мы знаем, что в квадратном корне действительных чисел не существует отрицательного значения.

Следовательно, значение дискриминанта не может быть отрицательным.

Отрицательное значение в корне не определено для действительных чисел.

Дано:

Данное уравнение есть.

Пошаговое объяснение:

Шаг 1:

Данное уравнение является квадратным уравнением, поскольку его степень равна 2.

Сначала нам нужно найти значения коэффициентов и констант.

Теперь сравните данное квадратное уравнение с общим квадратным уравнением, чтобы получить значения коэффициентов и константы как,

Шаг 2:

Теперь используйте правило квадратичного уравнения, чтобы найти данное квадратное уравнение.

Теперь подставьте значение в формулу квадратного правила, чтобы получить решение уравнения.

Еще больше упростите приведенное выше уравнение.

Следовательно, корни уравнения.

Подробнее:

  1. Подробнее о функции в диаграммах ниже brainly.com/question/95

  2. Подробнее о симметрии функции brainly.com/question/1286775
  3. Узнай больше о средней точке сегмента мозгом.2-5x + 6 = 0

    x 4 -5 x 3 + 7 x 2 -5 x + 6 = 0

    Теорема о рациональном корне, если рациональное число в простейшей форме p / q является корнем полиномиального уравнения a n x n + a n 1 x n — 1 + … + a 1 x + a 0 = 0, тогда p является множителем a 0 и q является коэффициент, если a n.

    Если p / q является рациональным нулем, то p является множителем 6, а q является множителем 1.

    Возможные значения p : ± 1, ± 2, ± 3.

    Возможные значения для q : ± 1

    По теореме о рациональных корнях единственными возможными рациональными корнями являются, p / q = ± 1, ± 2, ± 3

    Составьте таблицу для синтетического деления и проверьте возможные действительные нули.

    p / q

    1

    -5

    7

    -5

    6

    1

    1

    -4

    3

    -1

    7

    -1

    1

    6

    14

    27

    60

    -2

    1

    -7

    21

    -47

    100

    2

    1

    -3

    1

    -3

    0

    Поскольку f (2) = 0, x = 2 является нулем.Пониженный полином равен x 3 — 3 x 2 + x — 3 = 0.

    Synthetic Division & Factoring

    Синтетика Дивизион и факторинг (стр. 4 из 4)

    Разделы: Введение, Наработанные примеры, поиск нули, Факторинговые многочлены


    • Используйте синтетический деление, чтобы определить, x 4 множитель:

      2 x 5 + 6 x 4 + 10 x 3 6 x 2 9 х + 4

    • Для x 4, чтобы быть фактором, у вас должно быть x = 4 как ноль.С использованием эта информация, я сделаю синтетическое деление с x = 4 в качестве тестового нуля слева:

      Поскольку остаток равен ноль, затем x = 4 действительно является нулем из 2 x 5 + 6 x 4 + 10 x 3 6 x 2 9 х + 4, итак:

        Да, x 4 — коэффициент 2 x 5 + 6 x 4 + 10 x 3 6 x 2 9 х + 4

    • Найдите все коэффициенты 15 x 4 + x 3 52 x 2 + 20 x + 16 с помощью синтетического деления.

      Помните, что если x = — ноль, затем x a — коэффициент. Так что используйте Rational Тест корней (и может быть, быстрый график), чтобы найти хорошее значение для проверки нуля ( x -перехват). Попробую х = 1:

      Этот раздел дает нулевой остаток, поэтому x = 1 должен быть нулем, что означает, что x 1 — фактор.Поскольку я разделил линейный коэффициент (а именно x 1) из оригинала полином, тогда мой результат должен быть кубическим: 15 x 3 + 16 x 2 36 x 16. Поэтому мне нужно найти еще один ноль, прежде чем я смогу применить квадратичный Формула. Больной попробуйте x = 2:

      Поскольку у меня нулевой остаток, затем x = 2 является нулем, поэтому х + 2 — фактор.Кроме того, теперь я скатился к квадратичной 15 x 2 14 х 8, что происходит с коэффициентом:

      Тогда полностью учтен форма исходного многочлена:

        15 x 4 + x 3 52 x 2 + 20 x + 16

    • Дано тот является нулем x 4 + 6 x 3 7 x 2 30 x + 10, полностью решить уравнение
      x 4 + 6 x 3 7 x 2 30 x + 10 = 0.

      Поскольку они дали мне один из нулей, я использую синтетическое деление, чтобы разделить его:

      (Возможно, вам понадобится использовать бумажные заметки для вычислений требуется при манипулировании корневым корнем.) Авторские права Элизабет Стапель 2002-2011 Все права защищены

      Так как вы получаете только эти квадратный корень ответы с использованием квадратичного Формула и так как части формулы квадратного корня предшествует «плюс-минус» знак, то эти квадратные корни ответы всегда должны быть парами.Таким образом, если является корнем, значит, тоже должно быть корнем. Итак, мой следующий шаг — разделить на:

      Я начал с полином четвертой степени. После первого дивизиона у меня остался кубическая (с очень мерзкие коэффициенты!). После второго деления я стал квадратичным ( x 2 + 0 х 5, или просто x 2 5), который я знаю как решить:

      Тогда полное решение это:


    Если вы изучали комплекс числа, то вы можете увидеть проблему следующего типа.

    • Дано что 2 я является нулем x 5 6 x 4 + 11 x 3 x 2 14 x + 5, полностью решить
      уравнение x 5 6 x 4 + 11 x 3 x 2 14 х + 5 = 0.

      Они дали нам ноль, поэтому я воспользуюсь синтетическим делением и разделю 2 и :

      (Возможно, вам понадобится использовать бумажные заметки для вычислений требуется при выполнении сложного деления.)

      Напомним, чтобы прибыть при нуле 2 и , они должен был использовать квадратичный Формула, которая всегда выдает сложные ответы парами.То есть вы получаете воображаемое часть (часть с « i «) от отрицательного значения внутри «плюс-минус квадратный корень из» часть Формулы. Это означает, что, поскольку 2 i — ноль, затем 2 + i также необходимо быть нулем. Так что я разделю на 2 + и :

      Это оставляет мне кубический, поэтому мне нужно будет найти еще один ноль самостоятельно.(То есть я не могу применить квадратичный Формулы пока нет.) Я могу использовать Rational Тест на корни помочь найти потенциальные нули и быстрый график x 3 2 x 2 2 x + 1 может помочь. Буду пробовать х = 1:

      Теперь я перехожу к квадратичной ( х 2 3 х + 1, что не влияет на множитель), поэтому я применяю квадратичный Формула получения:

      Тогда все нули x 5 6 x 4 + 11 x 3 x 2 14 х + 5 Выдают:


    Примеры выше неоднократно относятся к соотношению между коэффициентами и нулями.В других уроках (например, при решении полиномы), эти концепции будут уточнены. На данный момент имейте в виду, что проверка график (если у вас есть графический калькулятор) может быть очень полезным для поиска проверять нули для выполнения синтетического деления, и что нулевой остаток после синтетическое деление на x = a означает, что х a — коэффициент полинома.Если у вас нет доступа к графический калькулятор, который поможет вам найти нужные нули и попробовать, есть некоторые уловки вы можете использовать.


    Для объяснения причин синтетическое подразделение работает (и для информации о методе варианта, который будет работать для нелинейных делителей), посмотрите файл Adobe Acrobat под названием «Как Synthetic Division Works, или «Безумие, скрывающееся за методом», написанный Уолтером Кеховски из Глендейлского муниципального колледжа в Аризоне.

    << Предыдущая Вверх | 1 | 2 | 3 | 4 | Вернуться к индексу

    Цитируйте эту статью как:

    Стапель, Елизавета. «Синтетическое подразделение и факторинг». Пурпурная математика . Доступно по номеру
    https://www.purplemath.com/modules/synthdiv4.htm . Доступ [Дата] [Месяц] 2016 г.

    Решения и объяснения промежуточных вопросов по алгебре в примере 5

    1. Если f (x) = 4x 3 — 4x 2 + 10 , то f (-2) =
      Решение
      Заменить x на -2 в f (x) следующим образом
      f (-2) = 4 (-2) 3 -4 (-2) 2 + 10
      = 4 (-8) — 4 (4) + 10 = — 32 — 16 + 10 = — 38
    2. Какое из этих значений x удовлетворяет неравенству -7x + 6 ≤ -8

      Решение
      Решите неравенство
      -7x + 6 ≤ -8, учитывая
      -7x + 6-6 ≤ -8-6, прибавить — 6 к обеим сторонам
      -7x ≤ — 14, упростить
      -7x / -7 ≥ -14 / -7, разделить на — 7 и ИЗМЕНИТЬ символ неравенства
      x ≥ 2, набор решений
      Ответ на поставленный выше вопрос — D, поскольку 2.
    3. Область определения функции f (x) = √ (6 — 2x) задается выражением
      Решение
      f (x) вещественно, если выражение под радикалом положительно или равно нулю. Следовательно, чтобы найти область определения, необходимо решить следующее неравенство.
      (6 — 2x) ≥ 0
      x ≤ 3, область f
    4. Строки y = 2x и 2y = — x являются
      A. параллельно Б. перпендикулярный
      С.горизонтальный D. вертикальный

      Решение
      Горизонтальные линии имеют вид y = постоянный, а вертикальные линии имеют форму от x = постоянный, поэтому эти две линии не являются ни горизонтальными, ни вертикальными. Найдем наклоны двух заданных прямых
      y = 2x имеет наклон, равный 2
      2y = — x эквивалентно y = — (1/2) x, а его наклон равен — (1/2)
      Поскольку наклоны не равны, две линии не параллельны. Произведение двух наклонов дается выражением
      2 (-1/2) = — 1
      и, следовательно, две линии перпендикулярны.
    5. Уравнение | -2x — 5 | — 3 = k не имеет решения, если k =

      Решение
      Сначала перепишем данное уравнение в виде
      | -2x — 5 | = к + 3
      Термин | -2x — 5 | либо положительно, либо равно нулю. Следовательно, указанное выше уравнение не имеет решений, если выражение k + 3 отрицательно. Значения k, для которых указанное выше уравнение не имеет решений, являются решениями неравенства
      k + 3 <0 или k <- 3
      Ответ — A, поскольку — 5 меньше — 3.
    6. Неравенство, соответствующее утверждению: «цена не менее 100 долларов» есть
      Решение
      Если цена не меньше 100 долларов, то цена равна или больше 100 долларов.
      х ≥ 100
    7. Какое из этих отношений НЕ НЕ представляет функцию?
      A. {(2,3), (- 4,3), (7,3)} Б. {(0,0), (- 1, -1), (2,2)}
      С.{(2,3), (- 5,3), (2,7)} Д. {(-1,3), (- 5,3), (- 9,0)}

      Решение
      Для отношения в C, когда x = 2, есть два возможных значения y: 3 или 7, и поэтому отношение в C не является функцией.
    8. Какая из этих точек НЕ лежит на графике y = -x + 3 ?
      A. (9, — 6) Б. (3,0) С. (-2,5) Д. (2,2)

      Решение
      Подставьте координаты данных точек в данное уравнение и проверьте, какое из них дает ложное утверждение.
      Точка (9, — 6): — 6 = — (9) + 3, — 6 = — 6, истина, точка лежит на прямой
      Точка (3,0): 0 = — (3) + 3, 0 = 0, истина, точка лежит на прямой
      Точка (-2,5): 5 = — (-2) + 3, 5 = 5, истина, точка лежит на прямой
      Точка (2,2): 2 = — (2) + 3, 2 = 1, false, точка НЕ ​​лежит на линии
      Ответ D.
    9. Каков наклон прямой, перпендикулярной прямой y = -5x + 9 ?
      Решение
      Наклон заданной (в форме пересечения наклона) прямой равен — 5.Пусть m — наклон прямой, перпендикулярной данной прямой. Две прямые перпендикулярны, если произведение их угловых коэффициентов равно -1. Следовательно
      м * (- 5) = — 1
      Решить относительно m. Следовательно
      м = 1/5 — наклон прямой, перпендикулярной данной прямой.
    10. Какое свойство используется для записи: 3 (x y) = (3 x) y ?
      A. Коммутативное свойство умножения Б. Мультипликативное обратное свойство
      С.Распределительная собственность D. Ассоциативное свойство умножения

      Решение
      Мы можем использовать ассоциативное свойство умножения для записи
      3 (х у) = (3 х) у
    11. В каком квадранте пересекаются прямые x = 3 и y = — 4?
      Решение
      Две прямые пересекаются в точке (3, -4), которая находится в квадранте IV.
    12. Стоимость 2 — | — 2 | это
      Решение
      2 — | — 2 | = 2 — 2 , поскольку | — 2 | = 2
      = 1/2 2 , так как -n = 1 / a n
      = 1/4 = 0.25
    13. Если a и b — положительные действительные числа, то (a 0 — 3b 0 ) 5 =
      Решение
      Упростить.
      (a 0 — 3b 0 ) 5 = (1 — 3 * 1) 5 = (- 2) 5 = — 32
    14. Какое неравенство описывает ситуацию: «длина L не более 45 см» .
      Решение
      С
    15. Уравнение m x — 8 = 6-7 (x + 3) НЕ имеет решения, если m =
      Решение
      Решить относительно x.
      м x — 8 = 6-7 (x + 3)
      м x + 7x = 6 — 21 + 8
      х (м + 7) = -7
      х = — 7 / (м + 7)
      м не может быть равно -7, иначе знаменатель будет равен нулю.
    16. Уравнение — m x + 1 = 13-4 (x + 3) является тождеством, если m =

      Решение
      Разверните обе правые стороны.
      — м х + 1 = — 4 х + 1
      Вышеупомянутое является идентичностью, если m = 4.


    17. Что из следующего ВСЕГДА верно?
      Решение
      Каждая функция — это отношение
    18. Какое из этих неравенств НЕ имеет решения?
      Решение
      Абсолютное значение любого выражения положительно или равно нулю.Отсюда неравенство.
      | x + 3 | <-2
      не имеет решений
    19. Прямые y = (a — 5) x + 5 и y = -2x + 7 параллельны, если a =
      Решение
      Две прямые параллельны, если их наклоны равны. Следовательно
      а — 5 = — 2
      Решить для
      а = 3
    20. Прямые y = (a — 5) x + 5 и y = -2 x + 7 перпендикулярны, если a =
      Решение
      Две прямые перпендикулярны, если произведение их наклонов равно -1.Следовательно
      -2 (а — 5) = — 1
      Решить для
      а = 11/2
    Ответы на вышеперечисленные вопросы
    1. В
    2. D
    3. С
    4. B
    5. А
    6. B
    7. С
    8. D
    9. С
    10. D
    11. С
    12. B
    13. С
    14. D
    15. С
    16. А
    17. B
    18. С
    19. B
    20. А
    Алгебра Вопросы и задачи
    Дополнительная практика ACT, SAT и Compass
    .

Вычислите площадь фигуры ограниченной линиями y x 2 1: Найдите площадь фигуры ограниченной линиями y=x^2+1,x=2,x=0,y=0

2,y=…

Лучший ответ по мнению автора

11. 05.17
Лучший ответ по мнению автора

Евгений

Читать ответы

Михаил Александров

Читать ответы

Андрей Андреевич

Читать ответы

Посмотреть всех экспертов из раздела Учеба и наука

Похожие вопросы

Решено

Площадь треугольника ABC равна 31, DE — средняя линия, параллельная стороне AB. 2 x))/log_31 (корень из 2 *Cosx)

Решено

Последовательность задана условиями b1=-6,bn+1=-3×1/bn.найдите b3.

Пользуйтесь нашим приложением

Площадь фигуры ограниченной кривыми в прямоугольных координатах

Площадь фигуры между двумя кривыми в прямоугольных координатах определяется интегралом
от разницы кривых, где одна из них всегда принимает не меньшие значения чем другая , а также кривые непрерывны.
Пределы интегрирования — прямые x1=a, x2=b — ограничивают фигуру (a<b чаще всего это точки пересечения заданных кривых).
Данный цикл задач в первую очередь подойдет студентам мех-мата Львовского национального университета имени Ивана Франко для прохождения практикума из математического анализа.
Студенты других Вузов могут набираться практики на подобных интегралах, и изучать методику вычисления.
Первый номер в примерах отвечает номеру основного задания из сборника М. В. Заболоцький, Фединяк С.И., Филевич П.В. «Практикум из математического анализа» (рядом стоит номер из сборника Б. П. Демидовича). 

 

Пример 2.81 (2397). Найти площадь фигуры, ограниченной кривыми, заданными в прямоугольных координатах ax=y2, ay=x2,(a>0).

Вычисление: Построим графики функций, которые ограничивают искомую площадь фигуры:

На графике они будут иметь следующий вид

Площадь между кривыми и нужно найти. Как правило, Вам редко будет известно сам график, поэтому в заданиях где не заданы области на которой находить площадь в первую очередь необходимо найти точки пересечения кривых.
Найдем пределы интегрирования, то есть точки абсцисс пересечения заданных функций y1(x)=y2(x):

Как видите таким условием есть условие равенства функций.
Из последнего уравнения получим две точки x1=0, x2=a.
Дальше, когда Вы не видите графика функций необходимо установить какая из кривых принимает большие значения. Это нужно лишь для того, чтобы с первого раза получить положительное значение площади фигуры. Поскольку площадь всегда больше нуля, а интеграл может принимать произвольные значения, то без проверки следующего условия для нахождения площади интеграл нужно брать за модулем.
Выбираем произвольную точку из отрезка интегрирования [0;a] и убеждаемся в правильности неравенства , то есть проверяем которая из кривых принимает большее значения .
Как отмечалось выше, это нужно для того, чтобы после интегрирования получить положительную площадь фигуры между кривыми.
Вычисляем площадь фигуры, которая ограничена заданными кривыми интегрированиям:

Здесь мы имели достаточно простые функции, поэтому возведя их к табличным интегралам найти площадь достаточно легко. Следующие примеры будут содержать все более тяжелые функции, для интегрирования которых нужно применять знание практически всех формул интегрирования.
Следует заметить: значения площадей (во всех заданиях) измеряются в квадратных единицах (кв. од.), об этом Вы должны помнить, однако для экономии места и времени здесь будут приведены лишь значения определенных интегралов.

 

Пример 2.82 (2398) Вычислить площадь фигуры, ограниченной кривыми y=x2, x+y=2.
Вычисление: По методике записываем уравнение кривых, которые ограничивают площадь фигуры:
y1(x)=x2, y2(x)=2-x.
Здесь функции выразить достаточно просто.
Вычислим пределы интегрирования, приравняв между собой функции y1(x)=y2(x):
x2=2-x.
Переносим переменные по одну сторону от знака равенства и решаем квадратное уравнение
x2+x-2=0;
(x+2)(x-1)=0.
Следовательно, корни уравнения x1=-2, x2=1.
Сам график кривых и фигуры, площадь которой ищем, приведен на рисунку

Подстановкой любой точки из промежутка [-2;1], например x=0 в функции убеждаемся, что выполняется неравенство
, поэтому .
Площадь фигуры вычисляем интегрированием разницы кривых в найденных пределах:

Площадь равна S=4,5 квадратных единиц.
По физическому содержанию площадь фигуры равна разнице площадей двух криволинейных трапеций. Первая отвечает за верхний график y2(x), нижняя криволинейная трапеция за функцию, которая принимает меньшие значения y2(x). Разница заключается в том, что здесь еще нужно определять пределы интегрирования.

 

Пример 2.83 (2399) Найти площадь фигуры, ограниченной кривыми y=2x-x2, x+y=0.
Вычисление: Запишем уравнение кривых, которые ограничивают искомую фигуру:
y1(x)=-x, y2(x)=2x-x2.
Из условия равенства функций y1(x)=y2(x) найдем пределы интегрирования:
2x-x2=-x;
x2-3x=0;
x (x-3) =0.
Следовательно, x1=0, x2=3.
Подстановкой единицы видим, что на промежутке [0;3] исполняется неравенство
, то есть .

Находим площадь фигуры ограниченной заданными кривыми:

Под интегралом простая квадратичная функция, поэтому само интегрирование не сложно.
Следующие функции будут более сложными в плане интегрирования, однако используя табличные интегралы площадь найти удается.

 

Пример 2.84 (2400) Найти площадь фигуры, ограниченной кривыми y=2x, y=2, x=0.
Вычисление: Запишем подынтегральные функции:
y1(x)=2x, y2(x)=2, а также прямую x1=0 (ограничивает фигуру по оси абсцисс).
Найдем вторую границу интегрирования из условия равенства функций y1(x)=y2(x):
2x=2, 2x=21, отсюда имеем вторую точку x1=1.
На промежутке [0;1] исполняется неравенство , поэтому .
График  степенной функции и прямой приведен ниже.

Площадь фигуры, которая ограничена кривыми равна интегралу:

При интегрировании получим логарифм.
На калькуляторах можете проверить, что площадь положительна.

 

Пример 2.85 (2401) Вычислить площадь фигуры, ограниченной кривыми y=x, y=x+sin2x, .

Вычисление: Запишем уравнение кривых, которые ограничивают площадь фигуры:
y1(x)=x, y2(x)=x+sin2x.
Дальше пределы интегрирования:
x1=0, x2=Pi (это известно нам по условию).
На промежутке справедливо неравенство
, поэтому .

Если бы существовала дополнительная точка пересечения, то площадь была бы равна сумме двух интегралов.
Площадь фигуры вычисляем интегрированием: квадрат синуса под интегралом понижаем и выражаем с помощью косинуса двойного угла, а дальше за классической формулой интегрирования

Площадь равна Pi/2, что приблизительно равно 1,5708.

 

Пример 2.86 (2402) Найти площадь фигуры, ограниченной кривыми
Вычисление: Переписываем функции

Найдем пределы интегрирования, то есть точки абсцисс пересечения заданных функций из условия y1(x)=y2(x):
Поскольку функция парная

то найдем половину площади и результат умножим на двойку.
Из условия находим

что пределы равны плюс, минус бесконечности.
Чтобы легко представить, что мы интегрируем наведем график подынтегральных функций

Учитывая четность функции интегрировать будем от 0 к бесконечности , а полученное значение умножим на двойку.
Получим несвойственный интеграл первого рода (детальнее о нем в части ІІІ).
Площадь фигуры вычисляем через предел интеграла:

В результате интегрирования получим арктангенс, который в предельном случае стремится к Pi/2.
Конечная формула достаточно компактна и удобна для расчетов, хотя с таким типом интегралов Вы знакомитесь впервые.

 

Пример 2.87 (2403) Вычислить площадь фигуры, ограниченной кривыми

Вычисление: Все Вы должны знать, что такой формулой задается уравнение эллипса.
Так как оси эллипса в канонической системе координат являются его осями симметрии, то эти оси делят эллипс на 4 равные части. Поэтому будем рассматривать часть эллипса, который находится в первом квадранте канонической (прямоугольной) системы координат.
Выражаем уравнение функции, которая ограничивает искомую площадь (четверть эллипса):

Запишем пределы интегрирования: из аналитической геометрии известно, что четверть эллипса ограничена прямыми x1=0, x2=a.

Для вычисления площади эллипса в самом интеграле необходимо выполнить замену переменных, что в свою очередь ведет к изменению пределов интегрирование. При этом придем к квадрату косинуса, который понижаем через косинус двойного угла.
В конце манипуляций приходим к табличным интегралам, которые легко интегрируем и подставляем пределы:

Получили классическую формулу площади эллипса S=Pi*a*b .
Видим, если эллипс вырождается в круг при (a=b=R), тогда формула площади круга S=Pi*R2.

 

Пример 2.88 (2404) Вычислить площадь фигуры, ограниченной кривыми y2=x2(a2-x2).
Вычисление: Так как все переменные в заданном уравнении входят в квадратах, то оси прямоугольной системы координат являются осями симметрии фигуры, которая ограничена этой линией, потому эти оси делят заданную фигуру на 4 равных части. Достаточно рассмотреть часть фигуры, которая заходиться в первом квадранте прямоугольной системы координат.
Построим график функции, которая ограничивает искомую площадь четвертины фигуры:

График неизвестной фигуры подобен на крылья бабочки.

При y=0 имеем два корня уравнения x1=0 и x2=a.
Площадь фигуры равна 4 умножить на интеграл с найденными пределами.
Во время интегрирования выполняем замену переменных и пределов интегрирования

Это позволяет перейти к показательной функции, которая легко интегрируется.
Всегда помните, что замена переменных под интегралом ведет к изменению пределов интегрирования.

 

Пример 2.89 Найти площадь фигуры, ограниченную линиями

Вычисление: Запишем графику функций, которые ограничивают искомую площадь фигуры:

Определим пределы интегрирования из условия y1(x)=y2(x):
 отсюда x1=0 и x2=1.
Между функциями справедлива зависимость на [0;1], поэтому .
График функций, что анализируем следующий

Площадь фигуры через определенный интеграл равна 1/3 (сравните 2.81 при a=1) :

 

Пример 2.90 Вычислить площадь фигуры, ограниченной кривыми
Вычисление: Вычислим пределы интегрирования из условия равенства функций y1(x)=y2(x):

Из биквадратного уравнения получим значение точек пересечения:
 x1=-1 и x2=1.
Сами же функции в прямоугольных координатах будут иметь вид

Интегрированием находим площадь фигуры (смотри рисунок и образец 2.89) :

Первый интеграл даст арктангенс, запомните хорошо эту формулу.

 

Пример 2.91 Вычислить площадь фигуры, ограниченной кривыми y=ex, y=e-x,x=1.
Вычисление: Из условия, которое Вы из-за повторяемости должны выучить y1(x)=y2(x) находим точки пересечения кривых:
ex=e-x,x=-x, 2x=0, следовательно, x1=0.
x2=1 (известно за условием).
График функций следующий

Экспоненту интегрировать не трудно, а площадь фигуры выражается формулой (смотри рисунок и образец 2.84) :

 

Пример 2.92 Найти площадь фигуры, ограниченной кривыми y=ln(x), y=ln2(x).
Вычисление: Пределы интегрирования из условия равенства функций y=ln(x), y=ln2(x) равны x1=1 и x2=e.

Интегрированием логарифмов находим площадь фигуры (смотри рисунок):

Здесь надо проинтегрировать по частям, положив ln(x) =u, (ln2(x)=u) и dx=dv. Попробуйте промежуточные действия провести самостоятельно.

 

Пример 2.93 Вычислить площадь фигуры, ограниченной кривыми
y=ln(x), y=ln(a), y=ln(b), x=0, где 0<a<b.
Вычисление: Построим графики функций, которые ограничивают искомую площадь фигуры:
x (y) =ey (то есть обратная функция к заданной функции y(x)=ln(x)) .
Такой прием применяют, когда пределы интегрирования параллельны оси Оx, то есть y=const.
Запишем пределы интегрирования:
y1=ln(a), y2=ln(b) (берем из начального условия).
График искомой фигуры следующий

Площадь фигуры, которая ограничена заданными кривыми:

 

Пример 2.94 Найти площадь фигуры, ограниченной кривыми
Вычисление: Пределы интегрирования в формуле площади находим из условия y1(x)=y2(x):
ln(x)/(4x)=x*ln(x).
Упростив на логарифм (если он больше нуля), получим
1=4x2; 4x2-1=0, x1=1/2.
Из условия на логарифм (=0) получим
ln(x) =0; x2=1.
ОДЗ: x>0.
График фигуры в прямоугольных координатах следующий

Площадь фигуры между кривыми (на [0,5;1]) находим интегрированием:
для вычисления интегралов используем метод замены переменных

Вычисление не так просты, поэтому с превращениями попробуйте разобраться самостоятельно.

 

Пример 2.95 Вычислить площадь фигуры, ограниченной кривыми y=arcsin(x), y=arccos(x), y=0.
Вычисление: Находим точки пересечения кривых из равенства x1(y)=x2(y):
sin(x)=cos(y), отсюда y1=0 (известно за условием) и y1=Pi/4 (образец 2.93).
На графике это выглядит следующим образом

Учитывая справедливость неравенства вычисляем площадь фигуры:

Думаю, что с такими заданиями на экзамене или модулях Вы справитесь.

 

Пример 2.96 Найти площадь фигуры, ограниченной кривыми y=tg(x), y=2/3*cos(x), x=0.
Вычисление: Найдем пределы интегрирования, то есть абсциссы точек  пересечения заданных функций y1(x)=y2(x):
tg(x)=2/3*cos(x), отсюда
(вторая точка известна за условием).
Кривые на плоскости имеют вид

Площадь фигуры, которая ограничена заданными кривыми () равна интегралу:

 

Пример 2.97 (2400) Вычислить площадь фигуры, ограниченной кривыми y=|ln(x)|, y=0, x=0,1; x=10.
Вычисление: Выписываем пределы интегрирования x1=0,1; x2=10 из начального условия.
Как строить модуль от логарифма Вы, по-видимому, еще не забыли

Площадь фигуры равна сумме двух интегралов, причем первый берем со знаком минус ():

Во время интегрирования использовали интегрирование частями.

 

Пример 2.98 (2400) Найти площадь фигуры, ограниченной кривыми y=(x+1)2, x=sin(Pi*y), y=0 .
Вычисление: Построим график функций, которые ограничивают искомую площадь фигуры:
(здесь взяли обратную функцию к заданной y1(x)=(x+1)2), x2=sin(Pi*y).
Выпишем пределы интегрирования:
y1=0; y2=1 (известно за условием).
График функций приведен ниже

Неизвестную площадь фигуры вычисляем интегрированием ():

 

Пример 2.99 Вычислить площадь фигуры, ограниченной кривыми y=sin(x), y=cos(x), y=0
Вычисление: Из рисунку видно, что площадь S лучше разбить на две части: S=S1+S2.

Запишем уравнение функций, которые ограничивают искомую площадь фигуры:

Интегрируем синус и косинус функции и находим площадь.

Второй вариант заключается в интегрировании разницы обратных функций по y.

 

Пример 2407 Найти площадь фигуры, ограниченной кривыми (циссоида Диокла) x=2a (a>0).
Вычисление: Поскольку график функции симметричен относительно оси Ox, то будем рассматривать половину площади фигуры (над осью Ox) и результат умножим на 2.
В точке x=2a функция не определена, поэтому будем иметь интеграл второго рода (детальнее смотрите часть ІІІ), он совпадает и, следовательно, площадь будет выражена числом.
Запишем пределы интегрирования:
x1=0 (потому что ) x2=2a (за условием).
График функций следующий

Площадь фигуры, что ограниченна заданной кривой находится достаточно непростым интегрированием



Здесь пришлось трижды выполнять замену переменных, чтобы прийти к правильному ответу.
Еще раз внимательно разберите интеграл.

 

Пример 2408 Вычислить площадь фигуры, ограниченной кривыми (трактриса), y=0.
Вычисление: Трактриса — кривая, по которой двигается объект, когда его тянуть по горизонтальной плоскости за бечевку фиксированной длины, если направление движения тягача является ортогональным к начальному положению бечевки и скорость тягача бесконечно малая величина.
Очевидно, что (смотри рисунок).

Принимая к сведению, что положительному приросту x отвечает отрицательный прирост y, и что фигура не квадрируема (в общем понимании), допускаем

где дифференциал за x находим через производную

Площадь фигуры через определенный интеграл равна

Следующим идет материал из которого Вы научитесь находить площадь фигуры, ограниченной кривыми заданными параметрически.

Нахождение площади фигуры, ограниченной линиями y=f(x), x=g(y)

В предыдущем разделе, посвященном разбору геометрического смысла определенного интеграла, мы получили ряд формул для вычисления площади криволинейной трапеции:

S(G)=∫abf(x)dx  для непрерывной и неотрицательной функции y=f(x) на отрезке [a;b],

S(G)=-∫abf(x)dx  для непрерывной и неположительной функции y=f(x) на отрезке [a;b].

Эти формулы применимы для решения относительно простых задач. На деле же нам чаще придется работать с более сложными фигурами. В связи с этим, данный раздел мы посвятим разбору алгоритмов вычисления площади фигур, которые ограничены функциями в явном виде, т.е. как y=f(x) или x=g(y).

Формула для вычисления площади фигуры, ограниченной линиями y=f(x) или x=g(y)

Теорема

Пусть функции y=f1(x)  и y=f2(x) определены и непрерывны на отрезке [a;b], причем f1(x)≤f2(x) для любого значения x из [a;b]. Тогда формула для вычисления площади фигуры G, ограниченной линиями x=a, x=b, y=f1(x)  и y=f2(x) будет иметь вид S(G)=∫abf2(x)-f1(x)dx.

Похожая формула будет применима для площади фигуры, ограниченной линиями y=c, y=d, x=g1(y) и x=g2(y): S(G)=∫cd(g2(y)-g1(y)dy.

Доказательство

Разберем три случая, для которых формула будет справедлива.

В первом случае, учитывая свойство аддитивности площади, сумма площадей исходной фигуры G и криволинейной трапеции G1 равна площади фигуры G2. Это значит, что

Поэтому, S(G)=S(G2)-S(G1)=∫abf2(x)dx-∫abf1(x)dx=∫ab(f2(x)-f1(x))dx.

Выполнить последний переход мы можем с использованием третьего свойства определенного интеграла.

Во втором случае справедливо равенство: S(G)=S(G2)+S(G1)=∫abf2(x)dx+-∫abf1(x)dx=∫ab(f2(x)-f1(x))dx

Графическая иллюстрация будет иметь вид:

Если обе функции неположительные, получаем: S(G)=S(G2)-S(G1)=-∫abf2(x)dx—∫abf1(x)dx=∫ab(f2(x)-f1(x))dx . Графическая иллюстрация будет иметь вид:

Перейдем к рассмотрению общего случая, когда  y=f1(x)  и y=f2(x) пересекают ось Ox.

Точки пересечения мы обозначим как  xi, i=1, 2,…, n-1. Эти точки разбивают отрезок [a; b] на n частей xi-1; xi, i=1, 2,…, n, где α=x0<x1<x2<…<xn-1<xn=b. Фигуру G можно представить объединением фигур Gi, i=1, 2,…, n. Очевидно, что на своем интервале Gi попадает под один из трех рассмотренных ранее случаев, поэтому их площади находятся как S(Gi)=∫xi-1xi(f2(x)-f1(x))dx, i=1, 2,. .., n

Следовательно, 

S(G)=∑i=1nS(Gi)=∑i=1n∫xixif2(x)-f1(x))dx==∫x0xn(f2(x)-f(x))dx=∫abf2(x)-f1(x)dx

Последний переход мы можем осуществить с использованием пятого свойства определенного интеграла.

Проиллюстрируем на графике общий случай.

Формулу S(G)=∫abf2(x)-f1(x)dx можно считать доказанной.

А теперь перейдем к разбору примеров вычисления площади фигур, которые ограничены линиями y=f(x) и x=g(y).

Примеры вычисления площади фигуры, ограниченной линиями y=f(x) или x=g(y)

Рассмотрение любого из примеров мы будем начинать с построения графика. Изображение позволит нам представлять сложные фигуры как объединения более простых фигур. Если построение графиков и фигур на них вызывает у вас затруднения, можете изучить раздел об основных элементарных функциях, геометрическом преобразовании графиков функций, а также построению графиков во время исследования функции.

Пример 1

Необходимо определить площадь фигуры, которая ограничена параболой y=-x2+6x-5 и прямыми линиями y=-13x-12, x=1, x=4.

Решение

Изобразим линии на графике в декартовой системе координат.

На отрезке [1;4] график параболы y=-x2+6x-5 расположен выше прямой y=-13x-12. В связи с этим, для получения ответа используем формулу, полученную ранее, а также способ вычисления определенного интеграла по  формуле Ньютона-Лейбница:

S(G)=∫14-x2+6x-5—13x-12dx==∫14-x2+193x-92dx=-13×3+196×2-92×14==-13·43+196·42-92·4—13·13+196·12-92·1==-643+1523-18+13-196+92=13

Ответ: S(G)=13

Рассмотрим более сложный пример.

Пример 2

Необходимо вычислить площадь фигуры, которая ограничена линиями y=x+2, y=x, x=7.

Решение

В данном случае мы имеем только одну прямую линию, расположенную параллельно оси абсцисс. Это x=7. Это требует от нас найти второй предел интегрирования самостоятельно.

Построим график и нанесем на него линии, данные в условии задачи.

Имея график перед глазами, мы легко можем определить, что нижним пределом интегрирования будет абсцисса точки пересечения графика прямой y=x и полу параболы y=x+2. Для нахождения абсциссы используем равенства:

y=x+2ОДЗ: x≥-2×2=x+22×2-x-2=0D=(-1)2-4·1·(-2)=9×1=1+92=2∈ОДЗx2=1-92=-1∉ОДЗ

Получается, что абсциссой точки пересечения является x=2.

Обращаем ваше внимание на тот факт, что в общем примере на чертеже линии y=x+2 , y=x пересекаются в точке (2;2), поэтому такие подробные вычисления могут показаться излишними. Мы привели здесь такое подробное решение только потому, что в более сложных случаях решение может быть не таким очевидным. Это значит, что координаты пересечения линий лучше всегда вычислять аналитически.

На интервале [2;7] график функции y=x расположен выше графика функции y=x+2 . Применим формулу для вычисления площади:

S(G)=∫27(x-x+2)dx=x22-23·(x+2)3227==722-23·(7+2)32-222-23·2+232==492-18-2+163=596

Ответ: S(G)=596

Пример 3

Необходимо вычислить площадь фигуры, которая ограничена графиками функций y=1x и y=-x2+4x-2.

Решение

Нанесем линии на график.

Определимся с пределами интегрирования. Для этого определим координаты точек пересечения линий, приравняв выражения 1x  и -x2+4x-2. При условии, что x не равно нулю, равенство 1x=-x2+4x-2становится эквивалентным уравнению третьей степени -x3+4×2-2x-1=0 с целыми коэффициентами. Освежить в памяти алгоритм по решению таких уравнений мы можете, обратившись к разделу «Решение кубических уравнений».

Корнем этого уравнения является х=1: -13+4·12-2·1-1=0.

Разделив выражение -x3+4×2-2x-1 на двучлен x-1, получаем: -x3+4×2-2x-1⇔-(x-1)(x2-3x-1)=0

Оставшиеся корни мы можем найти из уравнения x2-3x-1=0:

x2-3x-1=0D=(-3)2-4·1·(-1)=13×1=3+132≈3.3 ; x2=3-132≈-0.3

Мы нашли интервал x∈1; 3+132, на котором фигура G заключена выше синей и ниже красной линии. Это помогает нам определить площадь фигуры:

S(G)=∫13+132-x2+4x-2-1xdx=-x33+2×2-2x-ln x13+132==-3+13233+2·3+1322-2·3+132-ln3+132—133+2·12-2·1-ln 1=7+133-ln3+132

Ответ: S(G)=7+133-ln3+132

Пример 4

Необходимо вычислить площадь фигуры, которая ограничена кривыми y=x3, y=-log2x+1 и осью абсцисс.

Решение

Нанесем все линии на график. Мы можем получить график функции y=-log2x+1 из графика y=log2x, если расположим его симметрично относительно оси абсцисс и поднимем на одну единицу вверх. Уравнение оси абсцисс у=0.

Обозначим точки пересечения линий.

Как видно из рисунка, графики функций y=x3 и y=0 пересекаются в точке (0;0). Так получается потому, что х=0 является единственным действительным корнем уравнения x3=0.

x=2 является единственным корнем уравнения -log2x+1=0, поэтому графики функций y=-log2x+1  и y=0 пересекаются в точке (2;0).

x=1 является единственным корнем уравнения x3=-log2x+1. В связи с этим графики функций y=x3 и y=-log2x+1 пересекаются в точке (1;1). Последнее утверждение может быть неочевидным, но уравнение x3=-log2x+1 не может иметь более одного корня, так как функция y=x3 является строго возрастающей, а функция y=-log2x+1 строго убывающей.

Дальнейшее решение предполагает несколько вариантов.

Вариант №1

Фигуру G мы можем представить как сумму двух криволинейных трапеций, расположенных выше оси абсцисс, первая из которых располагается ниже средней линии на отрезке x∈0; 1, а вторая ниже красной линии на отрезке x∈1;2. Это значит, что площадь будет равна S(G)=∫01x3dx+∫12(-log2x+1)dx.

Вариант №2

Фигуру G можно представить как разность двух фигур, первая из которых расположена выше оси абсцисс и ниже синей линии на отрезке x∈0; 2, а вторая между красной и синей линиями на отрезке x∈1; 2. Это позволяет нам найти площадь следующим образом:

S(G)=∫02x3dx-∫12×3-(-log2x+1)dx

В этом случае для нахождения площади придется использовать формулу вида S(G)=∫cd(g2(y)-g1(y))dy.  Фактически, линии, которые ограничивают фигуру, можно представить в виде функций от аргумента y.

Разрешим уравнения y=x3 и -log2x+1 относительно x: 

y=x3⇒x=y3y=-log2x+1⇒log2x=1-y⇒x=21-y

Получим искомую площадь:

S(G)=∫01(21-y-y3)dy=-21-yln 2-y4401==-21-1ln 2-144—21-0ln 2-044=-1ln 2-14+2ln 2=1ln 2-14

Ответ: S(G)=1ln 2-14

Пример 5

Необходимо вычислить площадь фигуры, которая ограничена линиями y=x, y=23x-3, y=-12x+4.

Решение

Красной линией нанесем на график линию, заданную функцией y=x. Синим цветом нанесем линию y=-12x+4, черным цветом обозначим линию y=23x-3.

Отметим точки пересечения.

Найдем точки пересечения графиков функций y=x и y=-12x+4 :

x=-12x+4ОДЗ: x≥0x=-12x+42⇒x=14×2-4x+16⇔x2-20x+64=0D=(-20)2-4·1·64=144×1=20+1442=16; x2=20-1442=4Проверка:x1=16=4, -12×1+4=-12·16+4=-4⇒x1=16 не является решением уравненияx2=4=2, -12×2+4=-12·4+4=2⇒x2=4 является решением уравниния ⇒(4; 2) точка пересечения y=x и y=-12x+4

Найдем точку пересечения графиков функций y=x  и y=23x-3:

x=23x-3ОДЗ: x≥0x=23x-32⇔x=49×2-4x+9⇔4×2-45x+81=0D=(-45)2-4·4·81=729×1=45+7298=9, x245-7298=94Проверка:x1=9=3, 23×1-3=23·9-3=3⇒x1=9 является решением уравнения ⇒(9; 3) точка пересечания y=x и y=23x-3×2=94=32, 23×1-3=23·94-3=-32⇒x2=94 не является решением уравнения

Найдем точку пересечения линий y=-12x+4  и y=23x-3:

-12x+4=23x-3⇔-3x+24=4x-18⇔7x=42⇔x=6-12·6+4=23·6-3=1⇒(6; 1) точка пересечения y=-12x+4 и y=23x-3

Дальше мы можем продолжить вычисления двумя способами.

Способ №1

Представим площадь искомой фигуры как сумму площадей отдельных фигур.

Тогда площадь фигуры равна:

S(G)=∫46x—12x+4dx+∫69x-23x-3dx==23×32+x24-4×46+23×32-x23+3×69==23·632+624-4·6-23·432+424-4·4++23·932-923+3·9-23·632-623+3·6==-253+46+-46+12=113

Способ №2

Площадь исходной фигуры можно представить как сумму двух других фигур.

Тогда решим уравнение линии относительно x, а только после этого применим формулу вычисления площади фигуры.

y=x⇒x=y2 красная линияy=23x-3⇒x=32y+92 черная линияy=-12x+4⇒x=-2y+8 синяя линия

Таким образом, площадь равна:

S(G)=∫1232y+92—2y+8dy+∫2332y+92-y2dy==∫1272y-72dy+∫2332y+92-y2dy==74y2-74y12+-y33+3y24+92y23=74·22-74·2-74·12-74·1++-333+3·324+92·3—233+3·224+92·2==74+2312=113

Как видите, значения совпадают.

Ответ: S(G)=113

Итоги

Для нахождения площади фигуры, которая ограничена заданными линиями нам необходимо построить линии на плоскости, найти точки их пересечения, применить формулу для нахождения площади. В данном разделе мы рассмотрели наиболее часто встречающиеся варианты задач.

Решение задач от 1 дня / от 150 р. Курсовая работа от 5 дней / от 1800 р. Реферат от 1 дня / от 700 р.

Автор: Ирина Мальцевская

Преподаватель математики и информатики. Кафедра бизнес-информатики Российского университета транспорта

вычисление площади фигуры ограниченной линиями

Вы искали вычисление площади фигуры ограниченной линиями? На нашем сайте вы можете получить ответ на любой математический вопрос здесь. Подробное решение с описанием и пояснениями поможет вам разобраться даже с самой сложной задачей и вычисление площади фигуры ограниченной линиями онлайн, не исключение. Мы поможем вам подготовиться к домашним работам, контрольным, олимпиадам, а так же к поступлению в вуз. И какой бы пример, какой бы запрос по математике вы не ввели — у нас уже есть решение. Например, «вычисление площади фигуры ограниченной линиями».

Применение различных математических задач, калькуляторов, уравнений и функций широко распространено в нашей жизни. Они используются во многих расчетах, строительстве сооружений и даже спорте. Математику человек использовал еще в древности и с тех пор их применение только возрастает. Однако сейчас наука не стоит на месте и мы можем наслаждаться плодами ее деятельности, такими, например, как онлайн-калькулятор, который может решить задачи, такие, как вычисление площади фигуры ограниченной линиями,вычисление площади фигуры ограниченной линиями онлайн,вычислите площадь фигуры,вычислите площадь фигуры ограниченной,вычислите площадь фигуры ограниченной линиями,вычислите площадь фигуры ограниченной линиями y,вычислите площадь фигуры ограниченной линиями y 0 x 1 y x,вычислите площадь фигуры ограниченной линиями онлайн,вычислите площадь фигуры ограниченной линиями онлайн решение,вычислите площадь фигуры ограниченной линиями у 1 x y 2 x 2,вычислить онлайн площадь ограниченную линиями,вычислить площади фигур ограниченных линиями,вычислить площадь ограниченную линиями,вычислить площадь ограниченную линиями онлайн,вычислить площадь плоской фигуры ограниченной заданными кривыми онлайн,вычислить площадь плоской фигуры ограниченной линиями,вычислить площадь плоской фигуры ограниченной линиями онлайн с решением,вычислить площадь фигур ограниченных линиями онлайн,вычислить площадь фигуры,вычислить площадь фигуры ограниченной,вычислить площадь фигуры ограниченной графиками функций,вычислить площадь фигуры ограниченной графиками функций онлайн,вычислить площадь фигуры ограниченной графиками функций онлайн решение,вычислить площадь фигуры ограниченной линиями,вычислить площадь фигуры ограниченной линиями y,вычислить площадь фигуры ограниченной линиями y x 2 1 y x 1,вычислить площадь фигуры ограниченной линиями y x 2 y 2 x,вычислить площадь фигуры ограниченной линиями y x 2 y x,вычислить площадь фигуры ограниченной линиями онлайн,вычислить площадь фигуры ограниченной линиями онлайн калькулятор,вычислить площадь фигуры ограниченной линиями онлайн калькулятор с графиком,вычислить площадь фигуры ограниченной линиями онлайн калькулятор с решением,вычислить площадь фигуры ограниченной линиями онлайн подробное решение,вычислить площадь фигуры ограниченной линиями онлайн с решением калькулятор,вычислить площадь фигуры ограниченной линиями примеры решения,вычислить площадь фигуры ограниченной указанными линиями сделать чертеж,вычислить площадь фигуры онлайн,заштрихуй фигуры ограниченные двумя линиями,заштрихуй фигуры ограниченные линиями,как найти площадь фигуры ограниченной графиками функций,как найти площадь фигуры ограниченной линиями,калькулятор вычислить площадь фигуры ограниченной линиями онлайн с решением,калькулятор онлайн площадь фигуры,найдите площадь плоской фигуры ограниченной линиями,найдите площадь фигуры ограниченной линиями,найдите площадь фигуры ограниченной линиями y 5 x 2 y 1,найдите площадь фигуры ограниченной линиями y x 2 1 y 1 x,найдите площадь фигуры ограниченной линиями онлайн,найдите площадь фигуры ограниченной линиями онлайн калькулятор,найдите площадь фигуры ограниченной указанными линиями,найти площадь криволинейной трапеции ограниченной линиями онлайн,найти площадь криволинейной трапеции онлайн,найти площадь области ограниченной линиями онлайн,найти площадь ограниченной фигуры,найти площадь ограниченную линиями,найти площадь ограниченную линиями онлайн калькулятор,найти площадь плоской фигуры ограниченной линиями,найти площадь плоской фигуры ограниченной линиями онлайн,найти площадь фигуры,найти площадь фигуры ограниченной,найти площадь фигуры ограниченной графиками функций,найти площадь фигуры ограниченной кривыми,найти площадь фигуры ограниченной линиями,найти площадь фигуры ограниченной линиями онлайн,найти площадь фигуры ограниченной линиями онлайн калькулятор,найти площадь фигуры ограниченной линиями онлайн калькулятор подробно,найти площадь фигуры ограниченной линиями онлайн решение,найти площадь фигуры ограниченной линиями онлайн с подробным решением,найти площадь фигуры ограниченной линиями примеры решения,найти площадь фигуры ограниченной линиями с помощью определенного интеграла сделать иллюстрацию,найти площадь фигуры ограниченной указанными линиями,найти площадь фигуры онлайн,нахождение площади фигуры ограниченной линиями,нахождение площади фигуры ограниченной линиями онлайн,онлайн вычисление площади фигуры ограниченной линиями,онлайн вычислить площадь фигуры ограниченной графиками функций онлайн,онлайн калькулятор площадь фигуры ограниченной линиями,онлайн нахождение площади фигуры ограниченной линиями,онлайн площадь фигуры,площадь криволинейной трапеции онлайн,площадь ограниченная линиями,площадь плоской фигуры ограниченной линиями онлайн,площадь под графиком,площадь фигуры ограниченной графиками функций,площадь фигуры ограниченной линиями,площадь фигуры ограниченной линиями онлайн,площадь фигуры ограниченной линиями онлайн калькулятор,площадь фигуры онлайн,построить фигуру ограниченную линиями онлайн,сделайте чертеж и вычислите площадь фигуры ограниченной данными линиями,фигуры ограниченные двумя линиями,фигуры ограниченные линиями. 2+x+6 и y=0 Вычислить площадь фигуры, ограниченной линиями — Учеба и наука

Ответы

26. 02.17

Евгений

Читать ответы

Михаил Александров

Читать ответы

Андрей Андреевич

Читать ответы

Посмотреть всех экспертов из раздела Учеба и наука > Математика

Похожие вопросы

Решено

В прямоугольном треугольнике АВС угол С равен 90 градусов, AB = 4, tg А=0. 75 . Найдите АС.

Имеется два сосуда, содержащие 30 кг и 20 кг раствора кислоты различной концентрации. Если их слить вместе, то получим раствор, содержащий 81%

Дано: геометрическая прогрессия (bn) задана условиями: b1=-2 , bn+1=3bn. Найдите b6. Объясните пожалуйста, как это решить?

Решено

в зоопарке живут крокодилы и страусы. В сумме у них 40 голов и 94 ноги. Сколько там крокодилов и страусов?

Решено

дана арифмитическая прогрессия (аn)в которой a9=-22,2,a23=-41,8 найдите разность прогрессии

Пользуйтесь нашим приложением

Как вычислить площадь фигуры ограниченной линиями ℹ️ примеры

Общие сведения

Вычислить площадь фигуры на плоскости считается довольно простой операцией. Для ее выполнения необходимо знать только формулу. Существенно усложняет задачу фигура, ограниченная прямыми.

Одной из них считается криволинейная трапеция. Ее площадь можно определить только при нахождении значений определенного интеграла.

Операция интегрирования считается довольно сложной, поскольку необходимо знать основные правила. Перед нахождением площади криволинейной трапеции специалисты рекомендуют внимательно изучить и освоить правила интегрирования основных функций.

Разбирается неопределенный интеграл, а затем осуществляется переход к более сложным операциям.

Информация об интегралах

С понятием интеграла связано много направлений научных отраслей. Обозначается он символом «∫». С помощью интеграла открываются большие возможности по быстрому и эффективному нахождению значений следующих величин: площади криволинейной трапеции, объема тела вращения, поверхности, пути при неравномерном движении, массы неоднородного физического тела и так далее.

Упрощенный вариант представления и определения интеграла — сумма бесконечно малых слагаемых. Интеграл бывает нескольких типов: одинарный, двойной, тройной, криволинейный и так далее. Для любого элемента он может быть двух типов:

  1. Неопределенный.
  2. Определенный.

Операция нахождения первого типа значительно проще второго. Это объясняется тем, что во втором случае следует не только найти первообразную, но и выполнить правильную подстановку значений.

Неопределенным интегралом функции вида f(х) называется такая первообразная функция F(х), производная которой равна подинтегральному выражению. Записывается это таким образом: ∫(f(x)) = F(х) + С.

Последняя величина является константой, поскольку при выполнении операции нахождения производной константа равна 0.

Для нахождения первообразной используется специальная таблица интегралов:

Рисунок 1. Таблица интегралов и их первообразные.

В таблице приведены простые функции. Для нахождения площади фигуры, которая ограничена линиями, достаточно значений первообразных на рисунке 1. Вычисление определенного интеграла заключается в получении первообразной и подстановке начального и конечного значений. Следует отметить, что константа при этом не берется. Существует способ, чтобы найти определенный интеграл. Формула Ньютона-Лейбница позволяет быстро и эффективно вычислить площадь фигуры. Для этого нужно подставить значения ее границ (a и b) в первообразные: F(x)|(a;b) = F(b) — F(a).

Криволинейные фигуры

Криволинейная фигура (трапеция) — класс плоских фигур, которые ограничены графиком неотрицательной и непрерывной функции, а также осью ОУ и прямыми (х = а, х = b). Она изображена на рисунке 2. Для нахождения ее площади следует использовать определенный интеграл.

Рисунок 2. Фигуры с криволинейными сторонами.

Интегрирование разбивает фигуру на прямоугольные части. Длина каждой из них равна ординате y = f(х) через промежутки, которые очень малы, по оси декартовой системы координат (есть еще и полярная) ОХ на отрезке [a;b]. Ширина является бесконечно малым значением. При интегрировании находятся площади прямоугольников и складываются. Для того чтобы не путаться в графиках, геометрическую фигуру следует заштриховать.

Криволинейная трапеция — геометрическая фигура с неровными сторонами, которые образовались в результате пересечения графика непрерывной функции с осями абсцисс и ординат.

Применение обыкновенных методов нахождения площади этой фигуры невозможно, поскольку она обладает одной или несколькими неровными сторонами (кривыми линиями).

Способы вычисления и рекомендации

Для расчетов площади криволинейной трапеции используется несколько методов. Их условно можно разделить на следующие: автоматизированные и ручные. Первый из них выполняется при помощи специализированного программного обеспечения (ПО). Примером является онлайн-калькулятор, который не только находит площадь заданной фигуры, но и изображает ее в декартовой системе координат.

Существует и другое ПО, которое является более «мощным». К нему можно отнести наиболее популярные среды: Maple и Matlab. Однако существует множество программ, написанных на языке программирования Python. Программы нужны также при освоении темы интегрирования. Если необходимо рассчитать множество интегралов и площадей криволинейных фигур, то без них не обойтись.

Новичку для автоматизированных вычислений рекомендуется применять различные онлайн-калькуляторы. Однако следует выделить неплохую программу, которая обладает довольно неплохими функциональными возможностями.

Она называется Integral calculator и представляет собой очень удобное приложение для Android-устройств. Кроме того, можно скачать подобное ПО для Linux, Mac и Windows.

Программа — это калькулятор, который используется для нахождения интегралов и производных, а также его можно применять для решения уравнений интегрального и дифференциального типов. Integral calculator обладает такими функциональными возможностями:

  1. Вычисление производных.
  2. Нахождения первообразных для определенных и неопределенных интегралов.
  3. Решение систем уравнений.
  4. Выполнения операций над матрицами и определителями.
  5. Построение графиков заданных функций в 2D и 3D.
  6. Расчет точек перегиба.
  7. Вычисление рядов Фурье.
  8. Решение дифференциальных уравнений линейного типа первого и второго порядков.

Однако специалисты не рекомендуют использовать приложения такого типа, поскольку нужно уметь решать подобные задачи самостоятельно. Любые математические операции развивают мышление, а злоупотребление ПО приводит к значительной деградации. Решать какие-либо задачи рекомендуется также людям, которые не имеют отношения к математической сфере.

Основной алгоритм

При нахождении площади криволинейной трапеции рекомендуется следовать определенному алгоритму. Он поможет избежать ошибок, поскольку задача разбивается на несколько простых подзадач, решение которых довольно просто контролировать. Алгоритм имеет следующий вид:

  1. Нужно прочитать и понять условие задачи.
  2. Начертить декартовую систему координат.
  3. Построить график заданной функции.
  4. Изобразить линии, ограничивающие фигуру.
  5. После определения границ нужно аккуратно заштриховать фигуру.
  6. Вычислить неопределенный интеграл функции, которая дана в условии.
  7. Посчитать площадь, подставив значения ограничивающих прямых в первообразную.
  8. Проверить решение задачи при помощи программы.

Первый пункт — внимательное чтение условия задачи. Этап считается очень важным, поскольку формирует дальнейший алгоритм. Необходимо выписать все известные данные, а затем подумать над дальнейшим решением задачи. Следует обратить особое внимание на график функции, который при возможности нужно упростить. Далее следует выписать линии, которые будут ограничивать фигуру.

Следующий пункт считается наиболее простым, поскольку нужно начертить обыкновенную систему координат. В условии должен быть указан ее тип. Если обозначена полярная система, то следует ее начертить. Во всех остальных случаях изображается декартовая система координат.

Третий пункт алгоритма — правильное построение графика функции. В этом случае нет необходимости составлять таблицу зависимости значения функции от аргумента. График должен быть схематичным. Например, если это парабола, то нужно ее изобразить. В этом случае необходимо ознакомиться с основными базовыми функциями и их графиками.

Следующим шагом является правильное изображение прямых. Если ее уравнение имеет следующий вид «x = 5» или что-то подобное, то она будет проходить параллельно оси ОУ. Например, при y = 10 прямая проходит параллельно оси ОХ. В других случаях нужно составить таблицу зависимостей значений уравнения прямой от переменной. Следует брать всего два значения аргумента, поскольку их достаточно для проведения прямой.

После всех операций образуется фигура, которая ограничена линиями. Ее необходимо заштриховать. После этого вычисляется неопределенный интеграл заданной функции. Необходимо воспользоваться табличными значениями первообразных на рисунке 2. Однако здесь есть небольшой нюанс: константу записывать нет необходимости. Она «уничтожается» при подстановке в формулу Ньютона-Лейбница.

В полученное значение следует подставить значения границ. 2) / 2) + (-1)] = 3 — 0,75 = 2,25 (кв. ед.).

Для определения значения площади криволинейной фигуры (трапеции) необходимо использовать определенные интегралы. При решении нужно внимательно следить за знаками и первообразными из таблицы на рисунке 1.


2-1|-3|x|+3)dx$$

Правильно? Помогите пожалуйста с правильным решением.

Не знаю, как вычислить $\int|x|dx.$

исчисление интегрирование области определенных интегралов

$\endgroup$

2

$\begingroup$

Прежде чем решать задачи такого типа, вы должны тщательно нарисовать графики двух заданных кривых, а для этого вам нужно найти точки, в которых они пересекаются. Давайте сделаем это 92-2)dx$$

$\endgroup$

6

Твой ответ

Зарегистрируйтесь или войдите в систему

Зарегистрируйтесь с помощью Google

Зарегистрироваться через Facebook

Зарегистрируйтесь, используя адрес электронной почты и пароль

Опубликовать как гость

Электронная почта

Обязательно, но не отображается

Опубликовать как гость

Электронная почта

Требуется, но не отображается

Нажимая «Опубликовать свой ответ», вы соглашаетесь с нашими условиями обслуживания, политикой конфиденциальности и политикой использования файлов cookie

6.

1: Области между кривыми — Mathematics LibreTexts
  1. Последнее обновление
  2. Сохранить как PDF
  • Идентификатор страницы
    2519
    • Гилберт Странг и Эдвин «Джед» Герман
    • OpenStax
    Цели обучения
    • Определить площадь области между двумя кривыми путем интегрирования по независимой переменной.
    • Найдите площадь составной области.
    • Определите площадь области между двумя кривыми путем интегрирования по зависимой переменной.

    В разделе «Введение в интегрирование» мы разработали концепцию определенного интеграла для вычисления площади под кривой на заданном интервале. В этом разделе мы расширим эту идею, чтобы вычислить площадь более сложных регионов. Начнем с нахождения площади между двумя кривыми, являющимися функциями \(\displaystyle x\), начиная с простого случая, когда значение одной функции всегда больше другого. Затем рассмотрим случаи, когда графики функций пересекаются. Наконец, мы рассмотрим, как вычислить площадь между двумя кривыми, которые являются функциями \(\displaystyle y\).

    Площадь области между двумя кривыми

    Пусть \(\displaystyle f(x)\) и \(\displaystyle g(x)\) — непрерывные функции на интервале \(\displaystyle [a,b]\), такие что \(\displaystyle f(x)≥g(x)\) на \(\displaystyle [a,b]\). Нам нужно найти площадь между графиками функций, как показано на рисунке \(\PageIndex{1}\).

    Рисунок \(\PageIndex{1}\): Площадь между графиками двух функций, \(\displaystyle f(x)\) и \(\displaystyle g(x)\), на интервале \( \displaystyle [а,б]\) 9b_a[f(x)−g(x)]dx. \nonumber \]

    Эти результаты резюмируются в следующей теореме.

    Нахождение площади между двумя кривыми

    Пусть \(\displaystyle f(x)\) и \(\displaystyle g(x)\) — непрерывные функции такие, что \(\displaystyle f(x)≥g(x) \) на интервале [\(\displaystyle a,b]\). Обозначим через R область, ограниченную сверху графиком \(\displaystyle f(x)\), снизу графиком \(\displaystyle g(x)\), а слева и справа линиями \(\ displaystyle x=a\) и \(\displaystyle x=b\) соответственно. Тогда площадь \(\textbf{R}\) равна 9b_a[f(x)−g(x)]dx. \nonumber \]

    Применим эту теорему в следующем примере.

    Пример \(\PageIndex{1}\): нахождение площади области между двумя кривыми I

    Если \(\textbf{R}\) — это область, ограниченная сверху графиком функции \(\displaystyle f(x)=x+4\) и ниже по графику функции \(\displaystyle g(x)=3−\dfrac{x}{2}\) на интервале \(\displaystyle [1,4 ]\), найдите площадь области \(\textbf{R}\).

    Раствор

    92\).

    Упражнение \(\PageIndex{1}\)

    Если \(\textbf{R}\) область, ограниченная графиками функций \(\displaystyle f(x)=\dfrac{x}{2 }+5\) и \(\displaystyle g(x)=x+\dfrac{1}{2}\) на интервале \(\displaystyle [1,5]\), найти площадь области \(\textbf {Р}\).

    Подсказка

    Нарисуйте графики функций, чтобы определить, график какой функции образует верхнюю границу, а график нижней границы, затем выполните процесс, описанный в примере.

    Ответить

    \(\displaystyle 12\) единиц 2

    В примере \(\PageIndex{1}\) мы определили интересующий интервал как часть условия задачи. Однако довольно часто мы хотим определить интересующий нас интервал на основе того, где пересекаются графики двух функций. Это показано в следующем примере.

    Пример \(\PageIndex{2}\): нахождение площади области между двумя кривыми II 92\) и ниже по графику функции \(\displaystyle g(x)=6−x\) найти площадь области \(\textbf{R}\).

    Решение

    Область изображена на следующем рисунке.

    Рисунок \(\PageIndex{4}\): на этом графике показана область ниже графика \(\displaystyle f(x)\) и выше графика \(\displaystyle g(x).\)

    Сначала мы необходимо вычислить, где пересекаются графики функций. Установив \(\displaystyle f(x)=g(x),\), получим

    \[ \begin{align*} \displaystyle f(x) =g(x) \\[4pt] 94\), найдите площадь области \(\textbf{R}\).

    Подсказка

    Используйте процесс из примера \(\PageIndex{2}\).

    Ответить

    \(\displaystyle \dfrac{3}{10}\) ед. 2

    Площади составных областей

    До сих пор нам требовалось \(\displaystyle f(x)≥g(x)\) на всем интересующем интервале, но что, если мы хотим посмотреть на области, ограниченные графами пересекающиеся друг с другом функции? В этом случае мы модифицируем процесс, который мы только что разработали, используя функцию абсолютного значения. 9b_a|f(x)−g(x)|dx. \nonumber \]

    На практике применение этой теоремы требует, чтобы мы разбивали интервал \(\displaystyle [a,b]\) и вычисляли несколько интегралов, в зависимости от того, какое из значений функции больше на данной части интервал. Изучим этот процесс на следующем примере.

    Пример \(\PageIndex{3}\): нахождение площади области, ограниченной пересекающимися функциями

    Если \(\textbf{R}\) — это область между графиками функций \(\displaystyle f (x)=\sin x \) и \(\displaystyle g(x)=\cos x\) на интервале \(\displaystyle [0,π]\), найти площадь области \(\textbf{R }\).

    Решение

    Область изображена на следующем рисунке.

    Рисунок \(\PageIndex{5}\): Область между двумя кривыми может быть разбита на две подобласти.

    Графики функций пересекаются в точке \(\displaystyle x=π/4\). Для \(\displaystyle x∈[0,π/4], \cos x≥\sin x,\) поэтому

    \(\displaystyle |f(x)−g(x)|=|\sin x −\cos x|=\cos x−\sin x .\)

    С другой стороны, для \(\displaystyle x∈[π/4,π], \sin x ≥\cos x,\), поэтому

    9π_{π/4} \\[4pt] =(\sqrt{2}−1)+(1+\sqrt{2})=2\sqrt{2}. \end{align*}\]

    Площадь области составляет \(\displaystyle 2\sqrt{2}\) единиц 2 .

    Упражнение \(\PageIndex{3}\)

    Если \(\textbf{R}\) область между графиками функций \(\displaystyle f(x)=\sin x \) и \( \displaystyle g(x)=\cos x\) на интервале \(\displaystyle [π/2,2π]\), найти площадь области \(\textbf{R}\).

    Подсказка

    Две кривые пересекаются в точке \(\displaystyle x=(5π)/4. \)

    Ответить

    \(\displaystyle 2+2\sqrt{2}\) единиц 2

    Пример \(\PageIndex{4}\): определение площади сложной области

    Рассмотрим область, изображенную на рисунке \(\PageIndex{6}\). Найдите площадь \(\textbf{R}\).

    Рисунок \(\PageIndex{6}\): Для вычисления площади этой области требуются два интеграла. 92_1=\dfrac{1}{2}.\)

    Складывая эти области вместе, мы получаем

    \(\displaystyle A=A_1+A_2=\dfrac{1}{3}+\dfrac{1}{2 }=\dfrac{5}{6}.\)

    Площадь области составляет \(\displaystyle 5/6\) единиц 2 .

    Упражнение \(\PageIndex{4}\)

    Рассмотрим область, изображенную на следующем рисунке. Найдите площадь \(\textbf{R}\).

    Подсказка

    Две кривые пересекаются в точке х=1

    Ответить

    \(\displaystyle \dfrac{5}{3}\) единиц 2

    Области, определенные относительно y

    В примере \(\PageIndex{4}\) нам пришлось вычислить два отдельных интеграла для вычисления площади области. 2\) как функции \(\displaystyle y \).Однако, судя по графику, нас интересует положительный квадратный корень.) Точно так же правый график представлен функцией \(\displaystyle y=g(x)=2−x\), но также легко может быть представлен функцией \(\displaystyle x=u(y)=2−y\). Когда графики представлены как функции \(\displaystyle y\), мы видим, что область ограничена слева графиком одной функции и справа графиком другой функции. Следовательно, если мы интегрируем по \(\displaystyle y\), нам нужно вычислить только один интеграл. Разработаем формулу для этого типа интеграции.

    Пусть \(\displaystyle u(y)\) и \(\displaystyle v(y)\) — непрерывные функции на интервале \(\displaystyle [c,d]\) такие, что \(\displaystyle u(y) )≥v(y)\) для всех \(\displaystyle y∈[c,d]\). Мы хотим найти площадь между графиками функций, как показано на рисунке \(\PageIndex{7}\).

    Рисунок \(\PageIndex{7}\): Мы можем найти площадь между графиками двух функций, \(\displaystyle u(y)\) и \(\displaystyle v(y)\).

    На этот раз мы собираемся разбить интервал на 9d_c[u(y)−v(y)]dy. \end{align*}\]

    Эти результаты резюмируются в следующей теореме.

    Нахождение площади между двумя кривыми, интегрирование по оси Y

    Пусть \(\displaystyle u(y)\) и \(\displaystyle v(y)\) — непрерывные функции, такие что \(\displaystyle u( y)≥v(y) \) для всех \(\displaystyle y∈[c,d]\). Пусть \(\textbf{R}\) обозначает область, ограниченную справа графиком \(\displaystyle u(y)\), слева графиком \(\displaystyle v(y)\ ), а сверху и снизу строками \(\displaystyle y=d\) и \(\displaystyle y=c\) соответственно. Тогда площадь \(\textbf{R}\) равна 9d_c[u(y)−v(y)]dy. \nonumber \]

    Пример \(\PageIndex{5}\): интегрирование по y

    Вернемся к примеру \(\PageIndex{4}\), только на этот раз интегрируем по \(\displaystyle y \). Пусть \(\textbf{R}\) будет регионом, изображенным на рисунке \(\PageIndex{9}\). Найдите площадь \(\textbf{R}\) путем интегрирования по \(\displaystyle y\).

    Рисунок \(\PageIndex{9}\): Площадь области \(\textbf{R}\) можно вычислить с помощью одного интеграла, только если кривые рассматриваются как функции \(\displaystyle y\).

    Решение

    Сначала мы должны представить графики как функции \(\displaystyle y\). Как мы видели в начале этого раздела, кривая слева может быть представлена ​​функцией \(\displaystyle x=v(y)=\sqrt{y}\), а кривая справа может быть представлена ​​функцией функция \(\displaystyle x=u(y)=2−y\).

    Теперь нам нужно определить пределы интегрирования. Область ограничена снизу осью x, поэтому нижний предел интегрирования равен \(\displaystyle y=0\). Верхний предел интегрирования определяется точкой пересечения двух графиков, которая является точкой \(\displaystyle (1,1)\), поэтому верхний предел интегрирования равен \(\displaystyle y=1\). Таким образом, мы имеем \(\displaystyle [c,d]=[0,1]\). 91_0\\[4pt] =\dfrac{5}{6}. \end{align*}\]

    Площадь области составляет \(\displaystyle 5/6\) единиц 2 .

    Упражнение \(\PageIndex{5}\)

    Давайте вернемся к контрольной точке, связанной с примером \(\PageIndex{4}\), только на этот раз давайте проинтегрируем относительно \(\displaystyle y\). Пусть \(\textbf{R}\) будет областью, изображенной на следующем рисунке. Найдите площадь \(\textbf{R}\) путем интегрирования по \(\displaystyle y\).

    Подсказка

    Повторите процесс из предыдущего примера.

    Ответить

    \(\displaystyle \dfrac{5}{3}\) единиц 2

    Ключевые понятия

    • Определенные интегралы можно использовать не только для нахождения площади под кривой, но и для нахождения площади между двумя кривыми.
    • Чтобы найти площадь между двумя кривыми, заданными функциями, проинтегрируйте разность функций.
    • Если графики функций пересекаются или область является сложной, используйте абсолютное значение разности функций. В этом случае может потребоваться вычислить два или более интеграла и сложить результаты, чтобы найти площадь области.
    • Иногда проще интегрировать по y, чтобы найти площадь. Принципы одни и те же независимо от того, какая переменная используется в качестве переменной интегрирования.

    Ключевые уравнения

      9d_c[u(y)−v(y)]dy\)


      Эта страница под названием 6.1: Области между кривыми распространяется в соответствии с лицензией CC BY-NC-SA 4.0 и была создана, изменена и/или курирована Гилбертом Стрэнгом и Эдвином «Джедом» Херманом (OpenStax) через исходный контент, который был отредактирован. к стилю и стандартам платформы LibreTexts; подробная история редактирования доступна по запросу.

      1. Наверх
        • Была ли эта статья полезной?
        1. Тип изделия
          Раздел или страница
          Автор
          ОпенСтакс
          Лицензия
          CC BY-NC-SA
          Версия лицензии
          4,0
          Программа OER или Publisher
          ОпенСтакс
          Показать страницу TOC
          нет
        2. Метки
          1. ОБЛАСТЬ МЕЖДУ ДВУМЯ КРИВЫМИ
          2. Площадь между двумя кривыми, интегрированная по оси x
          3. Площадь между двумя кривыми, интегрированная по оси Y
          4. Площади составных регионов
          5. автор @ Эдвин «Джед» Герман
          6. автор@Гилберт Странг
          7. источник@https://openstax. org/details/books/calculus-volume-1

        Площадь области, ограниченной кривыми

        Площадь в прямоугольных координатах

        Напомним, что площадь под графиком непрерывной функции f ( х ) между вертикальными линиями х = 9b {f\left( x \right)dx} = F\left( b \right) — F\left( a \right),\]

        , где F ( x ) — любая производная от f ( x ).

        Рисунок 1.

        Мы можем расширить понятие площади под кривой и рассмотреть площадь области между двумя кривыми.

        Если \(f\left( x \right)\) и \(g\left( x \right)\) две непрерывные функции и \(f\left( x \right) \ge g\left( x \ справа)\) на отрезке \(\left[ {a,b} \right],\) то площадь между кривыми \(y = f\left( x \right)\) и \(y = g \left( x \right)\) в этом интервале равно 9б {\ влево [ {е \ влево ( х \ вправо) — г \ влево ( х \ вправо)} \ вправо] dx} = F \ влево ( б \ вправо) — G \ влево ( б \ вправо) — F \ влево( а \вправо) + G\влево( а \вправо),\]

        где \(F\left( x \right)\) и \(G\left( x \right)\) — первообразные функций \(f\left( x \right)\) и \(g\left ( x \right),\) соответственно.

        Обратите внимание, что эта площадь всегда будет неотрицательной, как \(f\left( x \right) — g\left( x \right) \ge 0\) для всех \(x \in \left[ {a,b } \справа].\)

        При наличии точек пересечения следует разбить интервал на несколько подинтервалов и определить, какая кривая больше на каждом подинтервале. Затем мы можем определить площадь каждой области, интегрируя разность большей и меньшей функций. 9\prime\left( t \right),\) \(y\left( t \right)\) здесь предполагаются непрерывными на отрезке \(\left[ {a,b} \right].\) Кроме того что функция \(x\left( t \right),\) должна быть монотонной на этом интервале.

        Рис. 5.

        Если \(x = x\left( t \right),\) \(y = y\left( t \right),\) \(0 \le t \le T\) параметрические уравнения гладкая кусочно-замкнутая кривая \(С\), проходимая против часовой стрелки и ограничивающая область слева (рис. \(5\)), то площадь области определяется следующими интегралами: 92}\) на отрезке \(\left[{1,b}\right]\) равно \(1?\)

        Пример 3

        Найдите координату точки \(a\), которая разбивает площадь под корневой функцией \(y = \sqrt{x}\) на отрезке \(\left[{0,4}\right]\) на равные части.

        Пример 4

        Область ограничена вертикальными линиями \(x = t\), \(x = t + \frac{\pi }{2}\), осью \(x-\) и кривая \(y = a + \cos x,\), где \(a \ge 1.\) Определите значение \(t\), при котором область имеет наибольшую площадь. 9{t + \frac{\pi} {2}} = a\left( {t + \frac{\pi} {2}} \right) + \sin \left( {t + \frac{\pi} }{ 2}} \right) — at — \sin t = \cancel{at} + \frac{{a\pi}}{2} + \sin \left( {t + \frac{\pi }{2}} \right) — \cancel{at} — \sin t = \frac{{a\pi}}{2} + \sin \left( {t + \frac{\pi }{2}} \right) — \ sin т.\]

        Использование разности синусов тождества

        \[\sin\alpha — \sin\beta = 2\cos\frac{{\alpha + \beta}}{2}\sin\frac{{\alpha — \beta}}{2},\]

        получаем

        \[A = \frac{{a\pi}}{2} + 2\cos \frac{{t + \frac{\pi} {2} + t}}{2}\sin \frac{{\ cancel{t} + \frac{\pi }{2} — \cancel{t}}}{2} = \frac{{a\pi}}{2} + 2\cos \left( {t + \frac {\pi} {4}} \right)\sin \frac{\pi}}{4} = \frac{{a\pi}}{2} + 2\cos \left( {t + \frac{\pi }{4}} \right) \cdot \frac{{\sqrt 2 }}{2} = \frac{{a\pi}}{2} + \sqrt 2 \cos \left( {t + \frac{ \pi }{4}} \справа). \]

        Область имеет наибольшую площадь, когда \(\cos \left( {t + \frac{\pi }{4}} \right) = -1.\)

        Решая это уравнение, находим

        \[\cos \left( {t + \frac{\pi }{4}} \right) = — 1,\;\; \Rightarrow t + \frac{\pi }{4} = \pi + 2\pi n,\;\; \Rightarrow t = \frac{{3\pi }}{4} + 2\pi n,\,n \in \mathbb{Z}.\]

        Дополнительные проблемы см. на стр. 2.

        9.1 Площадь между кривыми

        Мы видели, как с помощью интегрирования можно найти область между кривая и ось $x$. С очень небольшим изменением мы можем найти некоторые области между кривыми; действительно, площадь между кривой и осью $x$ может интерпретируется как площадь между кривой и второй «кривой». с уравнением $y=0$. В самых простых случаях идея довольно проста чтобы понять. 92\кр &={16\более4}-{64\более3}+28-4-({1\более4}-{8\более3}+7-2)\кр &=23-{56\over3}-{1\over4}={49\over12}.\cr }$$ $\квадрат$

        Стоит рассмотреть эту проблему немного подробнее. Мы видели один из способов посмотрите на него, рассматривая желаемую область как большую область минус маленькую площади, что естественным образом приводит к различию между двумя интегралы. Но поучительно рассмотреть, как мы могли бы найти желаемая область напрямую. Мы можем аппроксимировать площадь, разделив площадь на тонкие срезы и аппроксимируя площадь каждого среза на прямоугольник, как показано на цифра 92 = 1$. Обратите внимание: $t$ фиксировано, плоскость $x$-$y$.

        Пример 9.1.15 Докажите, что площадь $R$ равна $t$.

        Область между двумя функциями | Superprof

        В этой статье мы обсудим, как вычислить площадь между двумя функциями. Мы специально сосредоточимся на том, как вычислить площадь между кривой и прямой линией, а также площадь между двумя кривыми.

        Площадь между двумя функциями

        Площадь между двумя функциями равна площади функции, расположенной выше, за вычетом площади функции, расположенной ниже. Математически мы можем обозначить эту область так:

         

        Лучшие репетиторы по математике

        Поехали

        Площадь между кривой и прямой

        Теперь давайте разберемся, как вычислить площадь между кривой и прямой на следующих примерах

        Пример 1

        Найдите площадь пространства, ограниченного параболой и прямой, проходящей через точки A(−1, 0) и B(1, 4).

        Решение

        Шаг 1 — Найдите уравнение прямой

        На этом шаге мы вычислим уравнение прямой, проходящей через две точки A и B. Для этого сначала мы должны вычислить наклон прямой, проходящей через точки A(-1, 0) и В(1, 4). Для расчета наклона мы будем использовать следующую формулу:

        Подставим значения точек A и B в приведенную выше формулу:

        Теперь подставим этот наклон в уравнение точки пересечения ниже:

        Следовательно, уравнение прямой линии имеет вид y = 2x + 2.

        Шаг 2. Нарисуйте график 

        На этом этапе мы нарисуем график функции и линии следующим образом:

        Шаг 3. Расчет границ

        Точки, в которых линия пересекает параболы, будут границами или пределами функции. Как видно из приведенного выше графика, линия пересекает параболу в точках и . Следовательно, это пределы функции.

        Шаг 4. Вычисление определенного интеграла

        Чтобы вычислить определенный интеграл, сначала используйте информацию из предыдущих шагов, чтобы записать функции в следующей форме:

        правило суммы/разности определенных интегралов, подобное этому:

        Чтобы вычислить определенный интеграл, мы сначала найдем первообразную функции. Первообразная функции равна

        Теперь воспользуемся фундаментальной теоремой исчисления:

        Подставим 2 и 0 в первообразную функции, например: линии y = x, при x = 0 и x = 2.

        Решение

        Шаг 1. Нарисуйте график

        В этом примере нам уже дано уравнение линии y = x. Следовательно, нам не нужно его вычислять. Мы просто начнем с построения графика функций и .

        На приведенном выше графике видно, что от x = 0 до x = 1 прямая линия находится выше параболы, а от x = 1 до x = 2 прямая линия находится ниже параболы. Следовательно, мы будем вычислять площади, используя эти пределы выше и ниже параболы отдельно.

         

        Шаг 2. Вычисление границ

        В этом примере уже заданы границы или пределы графика, которые равны 0 и 1.

        Шаг 3. Вычисление определенного интеграла

        Чтобы вычислить определенный интеграл, сначала используйте информацию из предыдущих шагов, чтобы записать функции в следующем виде:

        Площадь, где прямая проходит над параболой:

        Найдите первообразную функции . Первопроизводная функции равна

        Используйте основную теорему исчисления:

        Подставьте 1 и 0 в первообразную функции следующим образом:

        93}{3} —

         

        В следующем разделе мы увидим, как вычислить площадь между двумя кривыми, зная их уравнения.

         

        Площадь между двумя кривыми

        Следующие примеры помогут вам понять, как вычислить площадь между двумя кривыми.

        Пример 1

        Найдите площадь, ограниченную графиками функций и

        Решение

        Шаг 1 — Нарисуйте график

        Шаг 2 — Найдите границы

        Чтобы определить, где графики двух кривых пересекаются друг друга, мы приравниваем уравнения двух кривых:

        или

        Следовательно, границы — и 0,

        Шаг 3. — Вычислить определенный интеграл

        Чтобы вычислить определенный интеграл, сначала используйте информацию из предыдущих шагов, чтобы записать функции в следующем виде:

        Найдите первообразную функции. Первопроизводная функции равна

        . Используйте основную теорему исчисления:

        . Подставив 0 в первообразную функции, получим следующее значение площади:

        Пример 2

        Найдите площадь между двумя кривые и.

        Решение

        Выполните следующие действия, чтобы рассчитать площадь.

        Шаг 1. Нарисуйте график

        График двух кривых приведен ниже:

        Шаг 2. Найдите границы

        Вычислите границы функции по уравнению:

        или

        Следовательно, границы функции равны 5 и 902.

        Шаг 3. Вычисление определенного интеграла

        Чтобы вычислить определенный интеграл, сначала используйте информацию из предыдущих шагов, чтобы записать функции в следующем виде:

        Найдите первообразную функции. Первопроизводная функции равна

        Используйте основную теорему исчисления:

        Подставьте 2 и 0 в первообразную функции:

         

        9002 Исчисление I -0 Онлайн-заметки Пола
        Главная / Исчисление I / Приложения интегралов / Площадь между кривыми

        Показать мобильное уведомление Показать все примечания Скрыть все примечания

        Уведомление для мобильных устройств

        Похоже, вы используете устройство с «узкой» шириной экрана ( т. е. вы наверное на мобильном телефоне). Из-за характера математики на этом сайте лучше всего просматривать в ландшафтном режиме. Если ваше устройство не находится в ландшафтном режиме, многие уравнения будут отображаться сбоку вашего устройства (должна быть возможность прокрутки, чтобы увидеть их), а некоторые пункты меню будут обрезаны из-за узкой ширины экрана.

        Раздел 6-2: Площадь между кривыми

        В этом разделе мы рассмотрим нахождение площади между двумя кривыми. На самом деле есть два случая, которые мы собираемся рассмотреть.

        В первом случае мы хотим определить площадь между \(y = f\left( x \right)\) и \(y = g\left( x \right)\) на интервале \(\left[ {яркий]\). Мы также собираемся предположить, что \(f\left( x \right) \ge g\left( x \right)\). Взгляните на следующий эскиз, чтобы получить представление о том, на что мы изначально собираемся смотреть.

        В разделе «Формулы площади и объема» главы «Дополнительно» мы вывели следующую формулу для площади в этом случае. {{\,b}}{{f\left(x\right) — g\left(x\right)\,dx}} \метка{уравнение:уравнение1}\конец{уравнение}\] 9{{\,d}}{{f\left(y\right) — g\left(y\right)\,dy}}\label{eq:eq2}\end{equation}\]

        Теперь \(\eqref{eq:eq1}\) и \(\eqref{eq:eq2}\) вполне пригодные формулы, однако иногда легко забыть, что они всегда требуют, чтобы первая функция была больше из двух функций. Таким образом, вместо этих формул мы будем использовать следующие «словесные» формулы, чтобы убедиться, что мы помним, что площадь всегда представляет собой «большую» функцию минус «меньшую» функцию. 9{{\,d}}{{\left(\begin{array}{c}{\mbox{right}}\\ {\mbox{function}}\end{array} \right) — \left(\begin {массив} {c} {\ mbox {left}} \\ {\ mbox {function}} \ end {array} \ right) \, dy}}, \ hspace {0,5 дюйма} c \ le y \ le d \ метка{уравнение:уравнение4}\конец{уравнение}\]

        Использование этих формул всегда заставит нас думать о том, что происходит с каждой задачей, и убедиться, что мы получили правильный порядок функций, когда переходим к использованию формулы. 2}\) и \(y = \sqrt x \).

        Показать решение

        Прежде всего, что мы подразумеваем под «окруженной областью». Это означает, что интересующая нас область должна иметь одну из двух кривых на каждой границе области. Итак, вот график двух функций с заштрихованной областью.

        Обратите внимание, что мы не берем какую-либо часть области справа от крайней правой точки пересечения этих двух графиков. В этой области нет границы с правой стороны, поэтому эта область не является частью замкнутой области. Помните, что одна из заданных функций должна находиться на границе замкнутой области. 92}\) является верхней функцией, и они подходят для подавляющего большинства \(x\). Однако в данном случае это младшая из двух функций.

        Пределы интегрирования для этого будут точками пересечения двух кривых. В этом случае довольно легко увидеть, что они пересекаются в точках \(x = 0\) и \(x = 1\), так что это пределы интегрирования.

        Итак, интеграл, который нам нужно вычислить, чтобы найти площадь, равен

        . 1\\ & = \frac{1}{3}\end{align*}\]

        Прежде чем перейти к следующему примеру, следует отметить пару важных моментов.

        Во-первых, почти во всех этих задачах требуется граф. Часто граничную область, которая дает пределы интегрирования, трудно определить без графика.

        Кроме того, без графика часто бывает трудно определить, какая из функций является верхней, а какая — нижней. Это особенно верно в случаях, подобных последнему примеру, где ответ на этот вопрос на самом деле зависел от диапазона \(x\), который мы использовали.

        Наконец, в отличие от площади под кривой, которую мы рассматривали в предыдущей главе, площадь между двумя кривыми всегда будет положительной. Если мы получим отрицательное число или ноль, мы можем быть уверены, что где-то допустили ошибку, и нам нужно будет вернуться и найти ее.

        Также обратите внимание, что иногда вместо слова «область, заключенная в», мы будем говорить «область, ограниченная». Они означают одно и то же.

        Давайте рассмотрим еще несколько примеров. 2}}}\), \(y = x + 1\), \(x = 2\) и ось \(y\).

        Показать решение

        В этом случае две последние части информации, \(x = 2\) и ось \(y\), говорят нам о правой и левой границах области. Также напомним, что ось \(y\) задается линией \(x = 0\). Вот график с заштрихованной областью.

        Здесь, в отличие от первого примера, две кривые не пересекаются. Вместо этого мы полагаемся на две вертикальные линии, чтобы ограничить левую и правую стороны области, как мы отметили выше

        92} + 10\) и \(у = 4х + 16\).

        Показать решение

        В этом случае точки пересечения (которые нам в конечном итоге потребуются) будет нелегко определить на графике, поэтому давайте продолжим и получим их сейчас. Обратите внимание, что для большинства этих задач вы не сможете точно идентифицировать точки пересечения на графике, поэтому вам нужно уметь определять их вручную. В этом случае мы можем получить точки пересечения, приравняв два уравнения. 92} — 4x — 6 & = 0\\ 2\left( {x + 1} \right)\left( {x — 3} \right) & = 0\end{align*}\]

        Получается, что две кривые пересекаются в точках \(x = — 1\) и \(x = 3\). Если они нам нужны, мы можем получить значения \(y\), соответствующие каждому из них, подставив значения обратно в любое из уравнений. Мы предоставим вам проверить, что координаты двух точек пересечения на графике равны \(\left( { — 1,12} \right)\) и \(\left( {3,28} \right )\). 92} + 10\), \(у = 4х + 16\), \(х = — 2\) и \(х = 5\).

        Показать решение

        Итак, функции, используемые в этой задаче, идентичны функциям из первой задачи. Разница в том, что мы расширили ограниченную область от точек пересечения. Поскольку это те же самые функции, которые мы использовали в предыдущем примере, мы не будем снова искать точки пересечения.

        Вот график этого региона.

        Итак, у нас есть небольшая проблема. Наша формула требует, чтобы одна функция всегда была верхней функцией, а другая функция всегда была нижней функцией, чего здесь явно нет. Однако на самом деле это не проблема, как может показаться на первый взгляд. Есть три области, в которых одна функция всегда является верхней функцией, а другая всегда нижней функцией. Итак, все, что нам нужно сделать, это найти площадь каждой из трех областей, что мы можем сделать, а затем сложить их все. 95\\ & = \frac{{14}}{3} + \frac{{64}}{3} + \frac{{64}}{3}\\ & = \frac{{142}}{3 }\конец{выравнивание*}\]

        Пример 5. Определите площадь области, заключенной в \(y = \sin x\), \(y = \cos x\), \(x = \frac{\pi }{2}\), и \( у\)-ось.

        Показать решение

        Сначала создадим график региона.

        Итак, у нас есть еще одна ситуация, когда нам нужно будет сделать два интеграла, чтобы получить площадь. Точка пересечения будет там, где 92} — 3\) и \(у = х — 1\).

        Показать решение

        Не позволяйте первому уравнению вас расстроить. 2} — 2y — 8\\ 0 & = \left( {y — 4} \right)\left( {y + 2} \right)\end{align*}\]

        Итак, похоже, что две кривые будут пересекаться в точках \(y = — 2\) и \(y = 4\) или, если нам нужны полные координаты, они будут: \(\left( { — 1, — 2} \справа)\) и \(\слева({5,4} \справа)\).

        Вот эскиз двух кривых.

        Теперь у нас будут серьезные проблемы, если мы не будем осторожны. До сих пор мы использовали верхнюю функцию и нижнюю функцию. Для этого обратите внимание на то, что на самом деле есть две части области, которые будут иметь разные нижние функции. В диапазоне \(\left[ { — 3, — 1} \right]\) парабола фактически является как верхней, так и нижней функцией.

        Чтобы использовать формулу, которую мы использовали до сих пор, нам нужно решить параболу для \(y\). Это дает,

        \[y = \pm \sqrt {2x + 6} \]

        , где «+» обозначает верхнюю часть параболы, а «-» — нижнюю часть.

        Вот набросок полной области с заштрихованными областями, которые нам понадобятся, если мы собираемся использовать первую формулу.

        Тогда интегралы площади будут равны 9{{\,d}}{{\left(\begin{array}{c}{\mbox{right}}\\ {\mbox{function}}\end{array} \right) — \left(\begin {массив} {c} {\ mbox {left}} \\ {\ mbox {function}} \ end {array} \ right) \, dy}}, \ hspace {0,5 дюйма} c \ le y \ le d \ ]

        и в нашем случае у нас есть одна функция, которая всегда слева, а другая всегда справа. Так что в данном случае это определенно выход. Обратите внимание, что нам нужно будет переписать уравнение прямой, поскольку оно должно быть в форме \(x = f\left( y \right)\), но это достаточно легко сделать. Вот график для использования этой формулы. 94\\ & = 18\конец{выравнивание*}\]

        Это то же самое, что мы получили, используя первую формулу, и это было определенно проще, чем первый метод.

        Итак, в этом последнем примере мы видели случай, когда мы могли использовать любую формулу для нахождения площади. Однако второе было определенно легче.

        Студенты часто приходят на занятия по математическому анализу с идеей, что единственный простой способ работать с функциями — использовать их в виде \(y = f\left( x \right)\).

      Площадь треугольника средняя линия: Средняя линия треугольника и площадь

      Как найти площадь через среднюю линию треугольника

      Средняя линия фигур в планиметрии — отрезок, соединяющий середины двух сторон данной фигуры. Понятие употребляется для следующих фигур: треугольник, четырёхугольник, трапеция.

      Содержание

      Средняя линия треугольника [ править | править код ]

      Средняя линия треугольника — отрезок, соединяющий середины двух сторон этого треугольника [1] .

      Свойства [ править | править код ]

      • средняя линия треугольника параллельна основанию и равна его половине.
      • средняя линия отсекает треугольник, подобный и гомотетичный исходному с коэффициентом 1/2; его площадь равна одной четвёртой площади исходного треугольника.
      • три средние линии делят исходный треугольник на четыре равных треугольника. Центральный из этих треугольников называется дополнительным или серединным треугольником.

      Признаки [ править | править код ]

      • Если отрезок в треугольнике проходит через середину одной из его сторон, пересекает вторую и параллелен третьей, то этот отрезок – средняя линия.

      Средняя линия четырёхугольника [ править | править код ]

      Средняя линия четырёхугольника — отрезок, соединяющий середины противолежащих сторон четырёхугольника.

      Свойства [ править | править код ]

      Первая линия соединяет 2 противоположные стороны. Вторая соединяет 2 другие противоположные стороны. Третья соединяет центры двух диагоналей (не во всех четырёхугольниках диагонали пунктом пересечения делятся пополам).

      • Если в выпуклом четырёхугольнике средняя линия образует равные углы с диагоналями четырёхугольника, то диагонали равны.
      • Длина средней линии четырёхугольника меньше полусуммы двух других сторон или равна ей, если эти стороны параллельны, и только в этом случае.
      • Середины сторон произвольного четырёхугольника — вершины параллелограмма. Его площадь равна половине площади четырёхугольника, а его центр лежит на точке пересечения средних линий. Этот параллелограмм называется параллелограммом Вариньона;
      • Последний пункт означает следующее: В выпуклом четырёхугольнике можно провести четыре средние линии второго рода. Средние линии второго рода – четыре отрезка внутри четырёхугольника, проходящие через середины его смежных сторон параллельно диагоналям. Четыре средние линии второго рода выпуклого четырёхугольника разрезают его на четыре треугольника и один центральный четырёхугольник. Этот центральный четырёхугольник является параллелограммом Вариньона.
      • Точка пересечения средних линий четырёхугольника является их общей серединой и делит пополам отрезок, соединяющий середины диагоналей. Кроме того, она является центроидом вершин четырёхугольника.
      • В произвольном четырёхугольнике вектор средней линии равен полусумме векторов оснований.

      Средняя линия трапеции [ править | править код ]

      Средняя линия трапеции — отрезок, соединяющий середины боковых сторон этой трапеции. Отрезок, соединяющий середины оснований трапеции, называют второй средней линией трапеции.

      Она рассчитывается по формуле: E F = A D + B C 2 <displaystyle EF=<frac <2>>> , где AD и BC — основания трапеции.

      07.06.2019

      5 июня Что порешать по физике

      30 мая Решения вчерашних ЕГЭ по математике

      Площадь треугольника ABC равна 176, DE — средняя линия. Найдите площадь треугольника CDE.

      Средняя линия отсекает от треугольника подобный ему с коэффициентом подобия Площади подобных фигур относятся как квадрат коэффициента подобия. Тогда

      Выясним, как связаны средняя линия треугольника и его площадь.

      I. Площадь треугольника равна половине произведения основания на высоту, проведённую к этой стороне:

      Поскольку средняя линия треугольника, соединяющая середины двух сторон, равна половине третьей стороны:

      то можно найти площадь треугольника через его среднюю линию:

      Площадь треугольника равна произведению средней линии и высоты, перпендикулярной этой средней линии.

      II.Прямая, параллельная стороне треугольника и пересекающая две другие его стороны, отсекает от него подобный треугольник.

      Если MN- средняя линия треугольника ABC и MN параллельна AC, то треугольники ABC и MBN подобны.

      Так как площади подобных треугольников относятся как квадраты их соответствующих сторон, то

      Средняя линия треугольника отсекает от него треугольник, площадь которого равна четверти площади исходного треугольника.

      Например, если площадь треугольника ABC равна 40 см², то средняя линия MN, параллельная стороне AC, делит его площадь на части:

      Площадь трапеции AMNC составляет три четверти площади треугольника ABC

      или может быть найденакак разность площадей треугольников ABC и MBC.

      Формулы площадей всех основных фигур


      1. Формула площади равнобедренной трапеции через стороны и угол

      а — нижнее основание

      b — верхнее основание

      с — равные боковые стороны

      α — угол при нижнем основании

       

      Формула площади равнобедренной трапеции через стороны, (S ):

       

      Формула площади равнобедренной трапеции через стороны и угол, (S ):

       

       

      2. Формулы площади равнобедренной трапеции если в нее вписана  окружность

      R — радиус вписанной окружности

      D — диаметр вписанной окружности

      O — центр вписанной окружности

      H — высота трапеции

      α, β — углы трапеции

      а — нижнее основание

      b — верхнее основание

       

      Формула площади равнобедренной трапеции через радиус вписанной окружности, (S ):

       

       

      СПРАВЕДЛИВО, для вписанной окружности в равнобедренную трапецию:


       

       

      R — радиус вписанной окружности

      m — средняя линия

      O — центр вписанной окружности

      c — боковые стороны

      а — нижнее основание

      b — верхнее основание

       

      Формула площади равнобедренной трапеции через радиус вписанной окружности, стороны и среднюю линию (S ):

       

      СПРАВЕДЛИВО, для вписанной окружности в равнобедренную трапецию:



      3. Формула площади равнобедренной трапеции через диагонали и угол между ними

       

       

      d — диагональ трапеции

      α, β — углы между диагоналями

       

      Формула площади равнобедренной трапеции через диагонали и угол между ними, (S ):



       

      4. Формула площади равнобедренной трапеции через среднюю линию, боковую сторону и угол при основании

       

      c — боковая сторона

      m — средняя линия трапеции

      α, β — углы при основании

       

      Формула площади равнобедренной трапеции через среднюю линию, боковую сторону и угол при основании, (S ):



       

      5. Формула площади равнобедренной трапеции через основания и высоту

       

      a — нижнее основание

      b — верхнее основание

      h — высота трапеции

       

      Формула площади равнобедренной трапеции через основания и высоту, (S ):

      Как найти Среднюю Линию Треугольника? Свойства, Теорема

      Понятие треугольника

      Треугольник — это геометрическая фигура, которая получилась из трех отрезков. Их соединили тремя точками, которые не лежат на одной прямой. Отрезки принято называть сторонами, а точки — вершинами.

      Виды треугольника:

      • Прямой. Один угол прямой, два других меньше 90 градусов.
      • Острый. Градус угла больше 0, но меньше 90 градусов.
      • Тупой. Один угол тупой, два других — острые.

      Треугольник считают равнобедренным, если две его стороны равны. Эти стороны называют боковыми сторонами, а третью — основанием.

      Треугольник, у которого все стороны равны, называется равносторонним или правильным.

      Треугольник называется прямоугольным, если у него есть прямой угол, то есть угол в 90°. Сторона прямоугольного треугольника, которая лежит напротив прямого угла — гипотенуза, а две другие стороны — катеты.

      Правильный (равносторонний или равноугольный) треугольник — это правильный многоугольник, в котором все стороны равны между собой, все углы также равны и составляют 60°. В равностороннем треугольнике высота является и биссектрисой, и медианой.

      Свойства треугольников:

      • В треугольнике против большего угла лежит большая сторона — и наоборот.
      • Сумма углов треугольника равна 180 градусов.
      • Все углы равностороннего треугольника равны 60 градусам.
      • В прямоугольном треугольнике квадрат гипотенузы равен сумме квадратов катетов.

      Понятие средней линии треугольника

      Определение средней линии треугольника подходит для любого вида этой фигуры.

      ​Средняя линия треугольника — отрезок, который соединяет середины двух сторон. В любом треугольнике можно провести три средних линии.

      ​Основанием считается сторона, с которой средняя линия не пересекается.

      Как найти среднюю линию треугольника расскажем дальше, а для начала еще немного разберемся со всеми определениями.

      Запоминаем

      Средняя линия параллельна третьей стороне, а ее длина равна половине длины этой стороны.

      Понятие средней линии прямоугольного треугольника

      Математики говорят: в любом треугольнике можно провести три средних линии. В прямоугольном треугольнике этот отрезок будет равен половине основания — это и есть формула средней линии прямоугольного треугольника. Основанием считается сторона, с которой средняя линия не пересекается.


      Прямой угол помогает нам применить другие признаки равенства и подобия. Для углов в прямоугольном треугольнике можно использовать геометрические тождества без дополнительных построений, а любую из сторон можно найти по теореме Пифагора.

       

      В прямоугольном треугольнике две средние линии перпендикулярны катетам, а третья равна медиане, проведённой к гипотенузе. Средние линии острого и разностороннего треугольника не обладают подобными свойствами.

      Важное свойство

      Средняя линия прямоугольного треугольника делит его на четыре прямоугольные фигуры.

      Свойства средней линии треугольника

      Признак средней линии треугольника: если отрезок в треугольнике проходит через середину одной из его сторон, пересекает вторую и параллелен третьей — этот отрезок можно назвать средней линией этого треугольника.

      Свойства:

       
      1. Средняя линия равна половине длины основания и параллельна ему.

      2. Средняя линия отсекает треугольник, подобный данному с коэффициентом 1/2; его площадь равна четверти площади данного.

      3. Три средние линии разделяют исходную фигуру на четыре равных треугольника. Центральный из них называют дополнительным.

      4. Три средние линии разделяют исходный прямоугольный треугольник на четыре равных прямоугольных треугольника.

      Теорема о средней линии треугольника

      Теорема о средней линии треугольника звучит так:

      Средняя линия треугольника параллельна основанию и равна его половине. А так выглядит формула нахождения средней линии треугольника:


      Докажем теорему:

       
      1. По условию нам дано, что MA = MB, NA = NC


      2. Рассмотрим два образовавшихся треугольника ΔAMN и ΔABC.

        По второму признаку подобия треугольников:


      3. Поэтому ∠1 = ∠2 , как соответственные, а по признаку параллельности прямых: MN || BC.

        Параллельность средней линии и соответствующего ей основания доказана.


      4. Еще из подобия треугольников △AMN~△ABC можно выписать и отношение их третьих сторон

        То, что средняя линия равна половине соответствующего основания, доказано.

       

      Теорема доказана.

      Пример 1. В треугольнике ΔABC AB = 8, BC = 7, CA = 5, точки M, N, K — середины сторон AB, BC, CA. Найти периметр ΔMNK.


      Как найти периметр треугольника:

       
      1. Сначала проверим существует ли указанный в условии треугольник ΔABC. Проверим это при помощи неравенства для его наибольшей стороны:

        7 + 5 > 8.

        Неравенство выполнено, значит, такой треугольник действительно есть.


      2. Соединим середины сторон треугольника ΔABC и получим его средние линии. Найдем их длины по теореме о средней линии:

       

      Ответ: периметр треугольника равен 10.

      Пример 2. В прямоугольном треугольнике АВС есть три средние линии: MN, NP, MP. В получившемся прямоугольнике MNPA известно, что синус угла между диагоналями равен 0,5. А средние линии MN и NP равны 3 и 4 соответственно. Найти площадь большого прямоугольного треугольника.


      Как решаем:

       
      1. В прямоугольнике две диагонали между собой равны. Одна из диагоналей MP — это гипотенуза прямоугольного треугольника MNP. Катеты треугольника известны, значит можно найти гипотенузу через теорему Пифагора:


      2. Найдем площадь прямоугольника, как произведение диагоналей на синус угла между ними:

        S = 5 * 5 * 0,5 = 12,5


      3. В большом треугольнике четыре малых, а в прямоугольнике два малых треугольника. Все малые треугольники между собой равны, значит, чтобы найти площадь прямоугольного треугольнику, нужно умножить площадь прямоугольника на 2.

        S = 12,5 * 2 = 25

       

      Ответ: площадь большого прямоугольного треугольника равна 25.

      Средняя ⚠️ линия треугольника: определение, признаки, теорема, формулы

      ​Средняя линия треугольника — отрезок, соединяющий середины двух его сторон.

      Свойства и признаки

      Признак средней линии: если отрезок в треугольнике проходит через середину одной из его сторон, пересекает вторую и параллелен третьей, то этот отрезок называется средней линией данного треугольника.

      Свойства:

      Осторожно! Если преподаватель обнаружит плагиат в работе, не избежать крупных проблем (вплоть до отчисления). Если нет возможности написать самому, закажите тут.

      1. Равна половине длины основания и параллельна ему.
      2. Отсекает треугольник, подобный данному с коэффициентом 1/2; его площадь равна четверти площади данного.
      3. Три средние линии разделяют исходную фигуру на четыре равных треугольника. Центральный из них называют дополнительным треугольником.
      4. Три средние линии разделяют исходный прямоугольный треугольник на четыре равных прямоугольных треугольника.

      Формула для расчета

      Теорема

      Средняя линия треугольника параллельна основанию и равна её половине.

      \(A_1C_1=\frac12AC\)

      Доказательство

      Дано: 

      \(\triangle ABC\)

      \(A_1C_1\)- средняя линия

      Доказать:

      \(A_1C_1\parallel AC\)

      \(A_1C_1=\frac12AC\)

      Рассмотрим \(\triangle BA_1C_1\) и \(\triangle BAC\):

      \(\left\{\begin{array}{l}\angle B\;-\;общий\\\frac{BA_1}{BA}=\frac{BC_1}{BC}=\frac12\end{array}\right.\)

      Из этого следует, что треугольники подобны по двум пропорциональным сторонам и углу между ними.

      Следовательно, \(\angle BA_1C_1=\angle BAC\) , как соответственные элементы подобных треугольников. Следовательно \(A_1C_1\parallel AC\) по признаку параллельности.

      Кроме того, из подобия следует, что \(\frac{A_1C_1}{AC}=\frac12\)

      Следовательно, \(A_1C_1=\frac12AC\)

      Утверждение доказано.

      Примечание

      Данная формула одинаково работает для любого треугольника: равнобедренного, равностороннего (правильного).

      Задачи на использование теоремы

      Задача 1

      В прямоугольном треугольнике ABC проведены средние линии: MN; NP; MP. При этом MN=NP=2. Найти площадь треугольника ABC.

       

      Рассмотрим прямоугольный треугольник NMP: 

      \(S_{\triangle NMP}=\frac12\times MN\times NP=\frac12\times2\times2=2\)

      Все маленькие треугольники равны, следовательно \(S_{\triangle ABC}=2\times4=8\)

      Ответ: 8

      Задача 2

      Площадь треугольника ABC равна 8. MN — средняя линия. Необходимо вычислить площадь треугольника BMN.

       

      \(S_{\triangle BMN}=\frac14S_{\triangle ABC}=\frac14\times8=2\)

      Ответ: 2

      Задача 3

      В треугольнике ABC точки M, N, K – середины сторон AB, BC, AC соответственно, MN=12, MK=10, KN=8. Необходимо узнать периметр треугольника ABC.

       

      Средняя линия равна половине основания, следовательно находим:

      MN = 12 ⇒ AC = 24

      MK = 10 ⇒ BC = 20

      KN = 8 ⇒ BA = 16

      Значит, \(P_{\triangle ABC}=24+20+16=60\)

      Ответ: 60

      Средняя линия треугольника, теория в ЕГЭ по математике

      \[{\Large{\text{Подобие треугольников}}}\]

      Определения

      Два треугольника называются подобными, если их углы соответственно равны и стороны одного треугольника пропорциональны сходственным сторонам другого
      (стороны называются сходственными, если они лежат напротив равных углов).

       

      Коэффициент подобия (подобных) треугольников – это число, равное отношению сходственных сторон этих треугольников.


       

      Определение

      Периметр треугольника – это сумма длин всех его сторон.

       

      Теорема

      Отношение периметров двух подобных треугольников равно коэффициенту подобия.

       

      Доказательство

      Рассмотрим треугольники \(ABC\) и \(A_1B_1C_1\) со сторонами \(a,b,c\) и \(a_1, b_1, c_1\) соответственно (см. рисунок выше).

       

      Тогда \(P_{ABC}=a+b+c=ka_1+kb_1+kc_1=k(a_1+b_1+c_1)=k\cdot P_{A_1B_1C_1}\)

       

      Теорема

      Отношение площадей двух подобных треугольников равно квадрату коэффициента подобия.

       

      Доказательство

      Пусть треугольники \(ABC\) и \(A_1B_1C_1\) подобны, причём \(\dfrac{AB}{A_1B_1} = \dfrac{AC}{A_1C_1} = \dfrac{BC}{B_1C_1} = k\). Обозначим буквами \(S\) и \(S_1\) площади этих треугольников соответственно.\circ — \angle A_1 — \angle B_1 = \angle C_1\), то есть углы треугольника \(ABC\) соответственно равны углам треугольника \(A_1B_1C_1\).


       

      Так как \(\angle A = \angle A_1\) и \(\angle B = \angle B_1\), то \(\dfrac{S_{ABC}}{S_{A_1B_1C_1}} = \dfrac{AB\cdot AC}{A_1B_1\cdot A_1C_1}\) и \(\dfrac{S_{ABC}}{S_{A_1B_1C_1}} = \dfrac{AB\cdot BC}{A_1B_1\cdot B_1C_1}\).

       

      Из этих равенств следует, что \(\dfrac{AC}{A_1C_1} = \dfrac{BC}{B_1C_1}\).

       

      Аналогично доказывается, что \(\dfrac{AC}{A_1C_1} = \dfrac{AB}{A_1B_1}\) (используя равенства \(\angle B = \angle B_1\), \(\angle C = \angle C_1\)).

       

      В итоге, стороны треугольника \(ABC\) пропорциональны сходственным сторонам треугольника \(A_1B_1C_1\), что и требовалось доказать.

       

      Теорема (второй признак подобия треугольников)

      Если две стороны одного треугольника пропорциональны двум сторонам другого треугольника и углы, заключенные между этими сторонами, равны, то такие треугольники подобны.

       

      Доказательство

      Рассмотрим два треугольника \(ABC\) и \(A’B’C’\), таких что \(\dfrac{AB}{A’B’}=\dfrac{AC}{A’C’}\), \(\angle BAC = \angle A’\). Докажем, что треугольники \(ABC\) и \(A’B’C’\) – подобны. Учитывая первый признак подобия треугольников, достаточно показать, что \(\angle B = \angle B’\).


       

      Рассмотрим треугольник \(ABC»\), у которого \(\angle 1 = \angle A’\), \(\angle 2 = \angle B’\). Треугольники \(ABC»\) и \(A’B’C’\) подобны по первому признаку подобия треугольников, тогда \(\dfrac{AB}{A’B’} = \dfrac{AC»}{A’C’}\).

       

      С другой стороны, по условию \(\dfrac{AB}{A’B’} = \dfrac{AC}{A’C’}\). Из последних двух равенств следует, что \(AC = AC»\).

       

      Треугольники \(ABC\) и \(ABC»\) равны по двум сторонам и углу между ними, следовательно, \(\angle B = \angle 2 = \angle B’\).

       

      Теорема (третий признак подобия треугольников)

      Если три стороны одного треугольника пропорциональны трем сторонам другого треугольника, то такие треугольники подобны.

       

      Доказательство

      Пусть стороны треугольников \(ABC\) и \(A’B’C’\) пропорциональны: \(\dfrac{AB}{A’B’} = \dfrac{AC}{A’C’} = \dfrac{BC}{B’C’}\). Докажем, что треугольники \(ABC\) и \(A’B’C’\) подобны.


       

      Для этого, учитывая второй признак подобия треугольников, достаточно доказать, что \(\angle BAC = \angle A’\).

       

      Рассмотрим треугольник \(ABC»\), у которого \(\angle 1 = \angle A’\), \(\angle 2 = \angle B’\).

       

      Треугольники \(ABC»\) и \(A’B’C’\) подобны по первому признаку подобия треугольников, следовательно, \(\dfrac{AB}{A’B’} = \dfrac{BC»}{B’C’} = \dfrac{C»A}{C’A’}\).

       

      Из последней цепочки равенств и условия \(\dfrac{AB}{A’B’} = \dfrac{AC}{A’C’} = \dfrac{BC}{B’C’}\) вытекает, что \(BC = BC»\), \(CA = C»A\).

       

      Треугольники \(ABC\) и \(ABC»\) равны по трем сторонам, следовательно, \(\angle BAC = \angle 1 = \angle A’\).


       

      \[{\Large{\text{Теорема Фалеса}}}\]

      Теорема

      Если на одной из сторон угла отметить равные между собой отрезки и через их концы провести параллельные прямые, то эти прямые отсекут на второй стороне также равные между собой отрезки.

       

      Доказательство

      Докажем сначала лемму: Если в \(\triangle OBB_1\) через середину \(A\) стороны \(OB\) проведена прямая \(a\parallel BB_1\), то она пересечет сторону \(OB_1\) также в середине.

       

      Через точку \(B_1\) проведем \(l\parallel OB\). Пусть \(l\cap a=K\). Тогда \(ABB_1K\) — параллелограмм, следовательно, \(B_1K=AB=OA\) и \(\angle A_1KB_1=\angle ABB_1=\angle OAA_1\); \(\angle AA_1O=\angle KA_1B_1\) как вертикальные. Значит, по второму признаку \(\triangle OAA_1=\triangle B_1KA_1 \Rightarrow OA_1=A_1B_1\). Лемма доказана.

       

      Перейдем к доказательству теоремы. Пусть \(OA=AB=BC\), \(a\parallel b\parallel c\) и нужно доказать, что \(OA_1=A_1B_1=B_1C_1\).

       

      Таким образом, по данной лемме \(OA_1=A_1B_1\). Докажем, что \(A_1B_1=B_1C_1\). Проведем через точку \(B_1\) прямую \(d\parallel OC\), причем пусть \(d\cap a=D_1, d\cap c=D_2\). Тогда \(ABB_1D_1, BCD_2B_1\) — параллелограммы, следовательно, \(D_1B_1=AB=BC=B_1D_2\). Таким образом, \(\angle A_1B_1D_1=\angle C_1B_1D_2\) как вертикальные, \(\angle A_1D_1B_1=\angle C_1D_2B_1\) как накрест лежащие, и, значит, по второму признаку \(\triangle A_1B_1D_1=\triangle C_1B_1D_2 \Rightarrow A_1B_1=B_1C_1\).

       

      Теорема Фалеса

      Параллельные прямые отсекают на сторонах угла пропорциональные отрезки.

       

      Доказательство

      Пусть параллельные прямые \(p\parallel q\parallel r\parallel s\) разбили одну из прямых на отрезки \(a, b, c, d\). Тогда вторую прямую эти прямые должны разбить на отрезки \(ka, kb, kc, kd\) соответственно, где \(k\) – некоторое число, тот самый коэффициент пропорциональности отрезков.

       

      Проведем через точку \(A_1\) прямую \(p\parallel OD\) (\(ABB_2A_1\) — параллелограмм, следовательно, \(AB=A_1B_2\)). Тогда \(\triangle OAA_1 \sim \triangle A_1B_1B_2\) по двум углам. Следовательно, \(\dfrac{OA}{A_1B_2}=\dfrac{OA_1}{A_1B_1} \Rightarrow A_1B_1=kb\).

      Аналогично проведем через \(B_1\) прямую \(q\parallel OD \Rightarrow \triangle OBB_1\sim \triangle B_1C_1C_2 \Rightarrow B_1C_1=kc\) и т.д.


       

      \[{\Large{\text{Средняя линия треугольника}}}\]

      Определение

      Средняя линия треугольника – это отрезок, соединяющий середины любых двух сторон треугольника.

       

      Теорема

      Средняя линия треугольника параллельна третьей стороне и равна ее половине.

       

      Доказательство

      1) Параллельность средней линию основанию следует из доказанной выше леммы.

       

      2) Докажем, что \(MN=\dfrac12 AC\).

       

      Через точку \(N\) проведем прямую параллельно \(AB\). Пусть эта прямая пересекла сторону \(AC\) в точке \(K\). Тогда \(AMNK\) — параллелограмм (\(AM\parallel NK, MN\parallel AK\) по предыдущему пункту). Значит, \(MN=AK\).

       

      Т.к. \(NK\parallel AB\) и \(N\) – середина \(BC\), то по теореме Фалеса \(K\) – середина \(AC\). Следовательно, \(MN=AK=KC=\dfrac12 AC\).

       

      Следствие

      Средняя линия треугольника отсекает от него треугольник, подобный данному с коэффициентом \(\frac12\).

      ЕГЭ по математике (2019 год). Задания №1 с ответами, профильные

       

       

       

       

       

       

      

       

      содержание   ..  47  48  49  50   ..

       

       

       

       

      Задание №4875

       

      Дан треугольник АВС. Его площадь равна 125. DE – средняя линия, параллельная стороне AB. Рассчитайте площадь трапеции ABED

       

      Решение

       

       

      Площадь трапеции ABED можно найти как разность площадей двух треугольников:

       

      Площадь треугольника CED будет в 4 раза меньше площади треугольника ABC, так как линейные размеры треугольника CED в 2 раза меньше соответствующих размеров треугольника ABC

       

      По найденной формуле вычисляем, что площадь трапеции ABED=93,75 Ответ: 93,75

       

       

       

      Задание №1118

       

       

      Боковые стороны в равнобедренном треугольнике равны 20, основание равно 24 . Вычислите радиус окружности, вписанной в этот треугольник

       

      Решение

       

      Радиус вписанной окружности равен отношению площади к полупериметру. Для нахождения площади, воспользуемся формулой Герона:

       

       

      Подставим значения сторон треугольника и найдём площадь. Она равна S=192 Подствавим значения и найдём полупериметр P=32 Тогда: Подствавим значения и найдём радиус r=192/32=6 Ответ: 6

       

       

       

      Задание №3393

       

      Основания равнобедренной трапеции равны 72 и 36. Синус острого угла трапеции равен 0,8. Найдите боковую сторону

       

       

      Решение

       

      Треугольники ADH и BKC равны (так как AD=CD и DH=CK), значит, AH=KB Треугольник ADH прямой, поэтому гипотенуза AD = AH / cos(a)

       

      По найденной формуле вычисляем, что AD=30 Ответ: 30

       

       

       

      Задание №1126

       

       

      Два известных угла вписанного в окружность четырехугольника равны 42° и 88°. Вычислите больший из оставшихся углов. Ответ дайте в градусах.

       

      Решение

       

      Cумма противоположных углов в четырехугольнике, вписанном в окружность равна 180 градусов (теорема Птолемея) угол противоположный углу 42 градусов равен 180-42=138 градусов угол противоположный углу 88 градусов равен 180-88=92 градусов Больший из неизвестных углов 138 градусов Ответ: 138

       

       

      Задание №3121

       

      Площадь параллелограмма ABCD равна 136. Точка E – середина стороны CD. Найдите площадь треугольника ADE

       

      Решение

       

       

      Разобьём параллеаграмм ABCD на равные треугольники (как на рисунке) — всего их 4, треугольник ADE, состоит из одного такого треугольника, значит его площадь равна 1/4 от площади параллеаграмма ABCD По найденной формуле вычисляем, что площадь треугольника ADE=34 Ответ: 34

       

       

       

      Задание №4296

       

       

      У прямоугольной трапеции, описанной около окружности, периметр равен 83, большая боковая сторона равна 36 . Рассчитайте радиус окружности

       

      Решение

       

      Сторона AD равна диаметру окружности, значит R=AD/2 В выпуклый четырехугольник можно вписать окружность тогда и только тогда, когда AB+CD=BC+AD

       

      R = 2,75 Ответ: 2,75

       

       

      Задание №2744

       

       

      Дана трапеция, описанная около окружности. Боковые стороны трапеции, равны 22 и 37 . Рассчитайте среднюю линию трапеции

       

      Решение

       

      В выпуклый четырехугольник можно вписать окружность тогда и только тогда, когда AB+CD=BC+AD

       

      Средняя линия MK = (DC+AB) / 2 = (AD+BC) / 2 = 59 / 2 = 29,5 Ответ: 29,5

       

       

      Задание №1099

       

       

      Дан четырёхугольник ABCD. В него вписана окружность, периметр = 260, стророна AB= 61 . Найдите длину стороны CD

       

      Решение

       

      В выпуклый четырехугольник можно вписать окружность тогда и только тогда, когда AB+CD=BC+AD Значит P / 2 = AB + CD CD = P/2-AB=69 Ответ: 69

       

       

      Задание №2994

       

       

      Дан правильный шестиугольник. Его периметр равен 330. Вычислите диаметр описанной окружности

       

      Решение

       

       

      Периметр (P) — сумма длин всех сторон, поэтому: AB / 6 = P / 6 =330 / 6 = 55 Рассмотрим угол AOB. Он равен 60°, т.к. вся окружность 360°, а треугольников 6 (360°/6=60°) Рассмотрим треугольник AOB. Он равносторонний, т.к. AO=OB=R и угол AOB=60° и тогда Диаметр D=2R=2AB=2*55=110 Ответ: 110

       

       

      Задание №4379

       

      Дан правильный многоугольник, вписанный в окружность. Угол между двумя соседними сторонами многоугольника равен 120°. Найдите число вершин многоугольника

       

      Решение

       

      Каждый угол правильного многоугольника равен 180° * (n – 2) / n , где n – число его углов (вершин) Составляем уравнение: 180 * ( n – 2 ) / n=120 180*n – 360 = 120 * n n=6 Ответ: 6

       

       

      Задание №4893

       

       

      Катеты прямоугольного равнобедренного треугольника равны 14+7√2 . Рассчитайте радиус окружности, вписанной в этот треугольник

       

      Решение

       

      Пусть катеты прямоугольного треугольника равны a, тогда гипотенуза AB, равна:

       

      Радиус вписанной в прямоугольный треугольник окружности равен половине разности суммы катетов и гипотенузы:

       

      Подставим в формулу вместо а значение катетов и решим уравнение Радиус r=7 Ответ: 7

       

       

       

      Задание №1208

       

       

      В равнобедренный треугольник вписана окружность, которая делит в точке касания одну из боковых сторон на два отрезка, длины которых равны 31 и 11, считая от вершины, противолежащей основанию. Вычислите периметр треугольника

       

      Решение

       

       

      Пусть точки H и K являются точками касания окружности и сторон AB и СВ соответственно. Треугольники KOH и KOB равны, т.к. являются прямоугольными с общей гипотенузой и равными катетами, значит, HB=KB=11 Периметр треугольника равен P=AC+CB+AH+HB=2CB+2HB=84+22=106 Ответ: 106

       

       

       

      Задание №4188

       

       

      Окружность вписана в четырёхугольник ABCD, сторона CD= 94, AB= 110 . Найдите периметр четырёхугольника ABCD

       

      Решение

       

      В выпуклый четырехугольник можно вписать окружность тогда и только тогда, когда AB+CD=BC+AD Периметр (P) четырехугольника – это сумма длин всех его сторон, то есть P=AB+BC+AD+CD= 2*(AB+CD) P = 408 Ответ: 408

       

       

      Задание №4129

       

       

      Окружность вписана в треугольник ABC, к ней проведены три касательные. Периметры отсеченных треугольников равны 20, 47, 73. Вычислите периметр данного треугольника

       

      Решение

       

       

      EF и ED — отрезки касательных, проведенных из одной точки Е. Они по свойству касательных равны. Аналогично, GF = GH. То есть, GE = GH + ED, а периметр треугольника AGE запишется как

       

      =20+47+73=140 Ответ: 140

       

       

       

      Задание №4615

       

       

      Дан треугольник ABC. Стороны AC=24, BC=45, угол C равен 90° . Вычислите радиус окружности, вписанной в этот треугольник

       

      Решение

       

      Радиус вписанной в прямоугольный треугольник окружности равен половине разности суммы катетов и гипотенузы:

       

      Подставим в формулу вместо значение AC и BC и решим уравнение Радиус r=9 Ответ: 9

       

       

       

      Задание №2252

       

       

      Дана равнобедренная трапеция. Основания трапеции равны 60 и 32. Радиус описанной окружности равен 34. Центр окружности лежит внутри трапеции. Необходимо найти высоту трапеции

       

      Решение

       

      Сделаем построение, проведем высоту KH через центр окружности O

       

      Из рисунка видно, что треугольники DOC и AOB – равнобедренные и их высоты KO и HO делят стороны DC и AB пополам. Найдем эти высоты из прямоугольных треугольников DKO и AOH по теореме Пифагора, имеем:

       

      Подставим известные значения в формулы и вычислим KO и HO KO=30 HO=16 Следовательно, высота трапеции равна KH=KO+HO=30+16=46 Примечание: Если бы большее основание трапеции лежало выше центра окружности (то есть оба основания располагались по одну сторону от центра окружности) длина высоты равнялась бы не сумме, а разности найденных отрезков. Решая данную задачу необходимо принимать во внимание рисунок, данный в условии Ответ: 46

       

       

      Задание №5512

       

      Площадь параллелограмма ABCD равна 146. Середина стороны BC — точка E. Найдите площадь трапеции ADEB

       

      Решение

       

       

      Разобьём параллеаграмм ABCD на равные треугольники (как на рисунке) — всего их 4, трапеция ADEB, состоит из трёх таких треугольников, значит площадь трапеции ADEB равна 3/4 от площади параллеаграмма ABCD По найденной формуле вычисляем, что площадь трапеции ADEB=109,5 Ответ: 109,5

       

       

       

      Задание №5021

       

       

      В четырехугольник ABCD вписана окружность, AB= 12, BC=2, CD=15. Найдите четвертую сторону четырехугольника

       

      Решение

       

      В выпуклый четырехугольник можно вписать окружность тогда и только тогда, когда AB+CD=BC+AD Сторона AD=AB+CD-BC=12+15-2=25 Ответ: 25

       

       

       

      Задание №1346

       

      Площадь параллелограмма ABCD равна 141. Середины его сторон являются вершинами параллелаграмма A′B′C′D′. Вычислите площадь параллелограмма A′B′C′D′

       

      Решение

       

       

      Разобьём параллеаграмм ABCD на равные треугольники (как на рисунке) — всего их 8, параллелограмм A′B′C′D′ состоит из четырёх таких треугольников, значит, его площадь равна 1/2 от площади параллеаграмма ABCD По найденной формуле вычисляем, что площадь параллелограмма A′B′C′D′=70,5 Ответ: 70,5

       

       

       

      Задание №1716

       

       

      Дана трапеция, описанная около окружности. Периметр трапеции равен 66. Вычислите длину средней линии трапеции

       

      Решение

       

      Периметр (Р) — сумма всех сторон трапеции В выпуклый четырехугольник можно вписать окружность тогда и только тогда, когда AB+CD=BC+AD

       

       

      Средняя линия MK = 66 / 4 = 16,5 Ответ: 16,5

       

       

       

       

       

       

       

       

       

       

       

      содержание   ..  47  48  49  50   ..

       

       

       

       

      Средняя линия треугольника ABC: определение, свойства, признак, длина

      В данной публикации мы рассмотрим определение, свойства и признак средней линии треугольника, а также разберем пример решения задачи для лучшего понимания теоретического материала.

      Определение средней линии треугольника

      Отрезок, который соединяет середины двух сторон треугольника, называется его средней линией.

      • KL – средняя линия треугольника ABC
      • K – середина стороны AB: AK = KB
      • L – середина стороны BC: BL = LC

      Свойства средней линии треугольника

      Свойство 1

      Средняя линия треугольника параллельна одной из его сторон (которую не пересекает) и в два раза меньше этой стороны.

      На рисунке выше:

      • KL параллельна AC
      • KL = 1/2 ⋅ AC

      Свойство 2

      Средняя линия треугольника отсекает от него подобный треугольник (в соотношении 1:2), площадь которого в 4 раза меньше исходного.

      На рисунке выше:

      • △KBL ∼ △ABC (подобие по пропорциональности всех сторон)
      • Стороны △KBL в два раза меньше соответствующих сторон △ABC:
        AB = 2KB, BC = 2BL, AC = 2KL
        .
      • S△ABC = 4 ⋅ S△KBL

      Свойство 3

      В любом треугольнике можно провести три средние линии.

      KL, KM и ML – средние линии треугольника ABC.

      • KL || AC, KL = 1/2 ⋅ AC
      • KM || BC, KM = 1/2 ⋅ BC
      • ML || AB, ML = 1/2 ⋅ AB

      Свойство 4

      Три средние линии треугольника делят его на 4 равных по площади треугольника.

      S1 = S2 = S3 = S4

      Признак средней линии треугольника

      Отрезок, проходящий через середину одной из сторон треугольника, пресекающий вторую и параллельный третьей стороне, является средней линией этого треугольника.

      Пример задачи

      Дан треугольник, две стороны которого равны 6 и 8 см. Найдите длину средней линии, соединяющей эти стороны.

      Решение

      Треугольник с заданными сторонами является прямоугольным, причем известные значения – это длины катетов. Средняя линия, которая соединяет катеты, параллельна гипотенузе и равна половине ее длины.

      Мы можем найти гипотенузу, воспользовавшись теоремой Пифагора.

      BC2 = AB2 + AC2 = 62 + 82 = 100.
      BC = 10.

      Таким образом, средняя линия LM = 1/2 ⋅ BC = 1/2 ⋅ 10 = 5.

      Середина треугольника (теорема, формула и видео) // Tutors.com

      Geometry, несмотря на то, что она требует длинных доказательств, невероятно эффективна. Где еще можно сделать на бумаге одну-единственную пометку и получить не один, не два, а пять результатов? Так обстоит дело с серединами треугольника.

      Содержание

      1. Что такое середина треугольника?
      2. Теорема о срединном сегменте треугольника
      3. Как найти середину треугольника
      4. Теорема о срединном сегменте треугольника
      5. Треугольник Серпинского

      Что такое средний сегмент треугольника?

      Средний сегмент треугольника — это линия, построенная путем соединения середин любых двух сторон треугольника.Не имеет значения, есть ли у вас прямоугольный треугольник, равнобедренный треугольник или равносторонний треугольник, все три стороны треугольника можно разделить пополам (разрезать пополам), причем точка, равноудаленная от любой вершины, является средней точкой этой стороны.

      In △ ASH, внизу стороны AS и AH равны 24 см и 36 см соответственно. Поскольку нам известны длины сторон, мы знаем, что точка C, середина стороны AS, находится ровно в 12 см от обоих концов. Точка R на AH находится ровно в 18 см от обоих концов.

      Соединение середин сторон, точек C и R на ASH делает кое-что помимо того, что заставляет всю нашу фигуру РАЗБИРАТЬСЯ.Он создает промежуточный сегмент CR, обладающий пятью удивительными особенностями.

      Пять свойств среднего сегмента

      Поскольку у треугольников три стороны, они могут иметь три средних сегмента. Вы можете присоединиться к любым двум сторонам в их середине. Один средний сегмент составляет половину длины основания (третья сторона не участвует в создании среднего сегмента). Это только одна интересная особенность. Это также:

      • Всегда параллельна третьей стороне треугольника; база
      • Образует меньший треугольник, похожий на исходный треугольник
      • .
      • Аналогичный треугольник меньшего размера составляет одну четвертую площади исходного треугольника
      • Аналогичный треугольник меньшего размера имеет половину периметра исходного треугольника

      Поскольку меньший треугольник, созданный средним сегментом, подобен исходному треугольнику, соответствующие углы двух треугольников идентичны; соответствующие внутренние углы каждого треугольника имеют одинаковые размеры.

      Из пяти атрибутов промежуточного сегмента два наиболее важных заключены в теореме о промежуточном сегменте, утверждении, которое было математически доказано (так что вам не нужно доказывать его снова; вы можете извлечь из этого пользу, чтобы сэкономить время и силы. ).

      Теорема о срединном сегменте треугольника

      Теорема о срединном сегменте треугольника говорит нам, что средний сегмент составляет половину длины третьей стороны (основания), и он также параллелен основанию.

      Вам не нужно доказывать теорему о среднем сегменте, но вы можете доказать ее, используя вспомогательную линию, совпадающие треугольники и свойства параллелограмма.

      Формула среднего сегмента

      Средний сегмент = 12 оснований треугольников

      Средний сегмент ∥ Основание треугольников

      Это мощная штука; за счет простых затрат на рисование одного линейного сегмента вы можете создать аналогичный треугольник с площадью в четыре раза меньше, чем оригинал, с периметром в два раза меньше, чем оригинал, и с основанием, гарантированно параллельным оригиналу и только вдвое короче.

      Как найти середину треугольника

      Нарисуйте любой треугольник, назовите его треугольником ABC.Используя циркуль, карандаш и линейку, найдите середины любых двух сторон вашего треугольника. Вы делаете это в четыре этапа:

      1. Отрегулируйте циркуль для рисования так, чтобы он повернул дугу, превышающую половину длины любой одной стороны треугольника
      2. Поместив стрелку циркуля в каждую вершину, проведите дугу через сторону треугольника с обоих концов, образуя две противоположные, пересекающиеся дуги
      3. Соедините точки пересечения обеих дуг с помощью линейки
      4. Точка, в которой линейка пересекает сторону треугольника, является средней точкой этой стороны)

      Соедините любые две середины сторон, и вы получите середину треугольника.Независимо от того, какой средний сегмент вы создали, он будет составлять половину длины основания треугольника (стороны, которую вы не использовали), а средний сегмент и основание будут параллельными линиями!

      Примеры теорем о срединном сегменте треугольника

      Вот правая △ СОБАКА, с боковым DO 46 дюймов и боковым DG 38,6 дюйма. Боковой ОГ (который будет базовым) — 25 дюймов. Площадь треугольника 482,5 кв. Дюйма.

      Какие точки вы соедините, чтобы создать средний сегмент?

      Только соединив точки V и Y, вы можете создать средний сегмент треугольника.Это сделает боковую OG базой.

      Вы сможете ответить на все эти вопросы:

      1. Каков периметр оригинальной △ СОБАКИ?
      2. Какова длина среднего сегмента VY?
      3. Какова длина бокового ДВ?
      4. Какова длина стороны DY?
      5. Каков периметр вновь созданного, подобного △ DVY?
      6. На какой территории находится вновь созданный △ DVY?

      Вот наши ответы:

      Добавьте длины: 46 «+ 38.6 «+ 25» = 109,6 «

      Средний сегмент VY = 12,5 дюйма

      Сторона DV = 23 «

      Сторона DY = 19,3 дюйма

      Периметр △ DVY = 54,8 «

      Площадь △ DVY = 120,625 дюйма2

      Треугольник Серпинского

      Используя теорему о срединном сегменте , вы можете построить фигуру, используемую во фрактальной геометрии, — треугольник Серпинского. Шаги просты, а результаты визуально приятны:

      1. Нарисуйте три средних сегмента любого треугольника, хотя равносторонние треугольники работают очень хорошо
      2. Либо игнорируйте, либо закрасьте большой центральный треугольник и сосредоточьтесь на трех оставшихся треугольниках одинакового размера.
      3. Для каждого углового треугольника соедините три новых средних сегмента.
      4. Снова игнорируйте (или закрасьте) каждый из их центральных треугольников и сосредоточьтесь на угловых треугольниках
      5. Для каждого из этих угловых треугольников соедините три новых средних сегмента.

      Эта непрерывная регрессия даст визуально мощную фрактальную фигуру:

      Следующий урок:

      Биссектриса сегмента

      Свойства среднего сегмента треугольника — Концепция

      Средний отрезок — это отрезок прямой, соединяющий середины двух сторон треугольника.Поскольку у треугольника три стороны, каждый треугольник имеет три средних сегмента. Средний сегмент треугольника параллелен третьей стороне треугольника и составляет половину длины третьей стороны. Еще один важный набор свойств среднего сегмента многоугольника, с которым необходимо ознакомиться, — это свойства среднего сегмента трапеции.

      Когда мы говорим о среднем сегменте треугольника, следует помнить о двух ключевых моментах. Первый — что такое средний сегмент? Хорошо, если бы я нашел середину одной стороны треугольника, если бы я нашел середину другой стороны треугольника, и если бы я соединил их с линейным сегментом, который является определением среднего сегмента, что-то, что соединяет 2 средние точки треугольника.Итак, первое, что важно в этом сегменте, это то, что он будет параллелен третьей стороне.
      Ну как мне узнать, какая третья сторона? Это будет сторона, которая не задействована в конечных точках нашего среднего сегмента, поэтому на этой стороне прямо здесь нет конечной точки этого среднего сегмента. Итак, я собираюсь сказать, что эти две линии должны быть параллельны, второй ключевой момент в этом среднем сегменте — это то, что он будет составлять половину длины третьей стороны. Итак, если бы я сказал, что этот средний сегмент был 10 дюймов, тогда длина этой третьей стороны была бы вдвое больше 20 дюймов.Но здесь не только один средний сегмент треугольника, их будет три, и мы найдем два других, найдя середину этой третьей стороны. Итак, если бы я соединил эти две средние точки, я бы получил средний сегмент, параллельный этой третьей стороне. Итак, я собираюсь использовать две стрелки, чтобы показать, что эти две параллельны. Этот средний сегмент будет составлять половину длины этой третьей стороны. И третий средний сегмент, который я мог бы нарисовать, — это средний сегмент прямо здесь, который будет параллелен этой третьей стороне, поэтому я собираюсь использовать другое количество стрелок, чтобы показать, что они параллельны.Итак, я собираюсь использовать 3 стрелки, которые я собираюсь подойти сюда 1, 2, 3. Итак, у нас есть 3 пары параллельных сторон со средними сегментами, и другой ключевой момент заключается в том, что средний сегмент составляет половину длины третьей стороны. .

      Промежуточный сегмент: теорема и формула — видео и стенограмма урока

      Свойства теоремы о срединном и срединном сегментах

      Срединный сегмент треугольника имеет несколько полезных свойств:

      • Срединный сегмент составляет половину длины основания
      Средний сегмент составляет половину длины основания.
      • Промежуточный сегмент параллелен основанию
      • Треугольник, образованный средним сегментом и двумя половинными сторонами, имеет те же углы, что и исходный треугольник
      • Периметр треугольника, образованного средним сегментом и двумя половинными сторонами, равен половине периметра исходного треугольника
      • Площадь треугольника, образованного серединным сегментом и двумя половинными сторонами, равна одной четвертой площади исходного треугольника

      Тот факт, что средний сегмент составляет половину длины основания, часто называют теоремой о среднем сегменте .Вы можете вспомнить, что теорема — это просто математическое утверждение, которое формально доказано. Давайте посмотрим на быстрый пример, который использует теорему о среднем сегменте.

      На рисунке ниже сегмент DE является средним сегментом треугольника ABC. Если DE параллелен AC и AC имеет длину 10 футов, какова длина DE ?

      DE — средний сегмент, параллельный AC.

      Мы знаем из теоремы о промежуточном сегменте, что DE составляет половину длины AC. Следовательно, DE должен составлять половину 10 футов или 5 футов.

      Давайте рассмотрим несколько примеров.

      Примеры использования среднего сегмента

      На рисунке ниже мы видим, что Б-стрит, Смит-роуд и Пауэлл-стрит образуют треугольник. Джонс Уэй образует середину этого треугольника. Если бы вы знали, что Пауэлл-стрит имеет длину 1000 метров, вы могли бы найти длину Джонс-Уэй. Поскольку Jones Way представляет собой средний сегмент, он должен составлять половину длины основания, Powell St.

      Джонс-Уэй — это середина треугольника, образованного Б., Смит-роуд и Пауэлл-стрит.

      Половина 1000 метров составляет 500 метров, следовательно, длина Jones Way 500 метров.

      Попробуем еще один пример.

      В треугольнике ABC ниже DE — это средний сегмент треугольника. Если величина угла BAC составляет 55 градусов, какова величина угла BDE ?

      На этом рисунке сравните размеры углов левой стороны, BAC и BDE.

      Третье свойство среднего сегмента говорит нам, что средний сегмент образует второй, меньший треугольник, который имеет те же размеры углов, что и исходный треугольник. Это означает, что если угол BAC имеет размер 55 градусов, угол BDE также должен иметь размер 55 градусов.

      Соотношение площадей часто наблюдается в треугольнике Серпинского . Этот особый дизайн создается путем рисования треугольника с последующим рисованием трех средних сегментов.Созданный новый центральный треугольник (показан красным в левом треугольнике ниже) имеет площадь, равную одной четвертой площади исходного треугольника. Процесс рисования средних сегментов незатененных треугольников и закрашивание нового центрального треугольника можно повторить для создания дизайна, как показано в правом треугольнике ниже.

      Треугольник Серпинского формируется путем рисования средних сегментов исходного треугольника, затемнения центрального треугольника и последующего повторения процесса.

      Резюме урока

      Средний сегмент треугольника определяется как сегмент, образованный соединением средних точек любых двух сторон треугольника.Он имеет следующие свойства:

      1) Это половина длины основания треугольника.

      2) Параллельно базе.

      3) Он образует меньший треугольник с теми же углами, что и исходный треугольник.

      4) Он образует меньший треугольник с периметром, равным половине периметра исходного треугольника.

      5) Он образует меньший треугольник с площадью, равной одной четвертой площади исходного треугольника.

      Свойства середины треугольника

      Линия DE — это середина треугольника ABC.
      • Он соединяет две середины двух сторон треугольника.
      • Равен половине длины основания.
      • Параллельно базе.
      • Он образует меньший треугольник со всеми теми же углами, половину периметра и одну четвертую площади исходного треугольника.

      Результаты обучения

      Изучение этой информации о среднем сегменте может позволить вам сделать следующее:

      • Обратите внимание на определение и назначение среднего сегмента треугольника
      • Укажите свойства среднего сегмента треугольника
      • Используйте теорему о срединном сегменте

      Трапеция, средняя линия и средний сегмент трапеции и треугольника

      Четырехугольник с двумя противоположными параллельными сторонами называется трапецией (трапеция) .

      Параллельные стороны трапеции называются основаниями (AB и CD), а те, которые не параллельны, называются ногами (AD и BC).
      Если ноги равны по длине, трапеция называется , равнобедренная, .
      DE и CF — высота .

      Средняя линия трапеции

      Линия, соединяющая середины сторон, которые не параллельны, называется средней линией (или средним сегментом) трапеции.

      Линия MN является средней линией ABCD. А сегмент MN — это средний сегмент ABCD.

      AM = MD
      BN = NC

      Средняя линия трапеции параллельна ее сторонам.
      В нашем случае — MN || AB || ОКРУГ КОЛУМБИЯ.

      Теорема 1:

      Если линия, проходящая через середину отрезка трапеции, параллельна ее основаниям, затем линия проходит через середину другой ноги.

      Теорема 2:

      Средний отрезок трапеции составляет половину длины двух параллельных сторон.

      Другими словами:
      $ \ overline {MN} = \ frac {\ overline {AB} + \ overline {DC}} {2} $

      Середина треугольника

      Отрезок, соединяющий середины двух сторон треугольника, называется средним сегментом треугольника.

      Он параллелен третьей стороне, а его длина вдвое меньше длины третьей стороны.

      Теорема : Если отрезок прямой пересекает середину одной стороны треугольника и параллелен другой стороне того же треугольника, то этот отрезок делит третью сторону пополам.

      $ \ overline {AM} = \ overline {MC} $ и $ \ overline {BN} = \ overline {NC} $ =>

      $ MN || AB $
      $ \ overline {MN} = \ frac {\ overline {AB}} {2} $

      Применение свойств средних сегментов

      Разделите отрезок на равные отрезки без измерения.

      Задание: Разделите данный сегмент $ \ overline {AB} $ на 5 равных сегментов без измерения.

      Решение:

      Пусть p — произвольный луч с началом A, не лежащий на AB.На п. Рисуем последовательно пять равных отрезков.
      $ \ overline {AA_1} = \ overline {A_1A_2} = \ overline {A_2A_3} = \ overline {A_3A_4} = \ overline {A_4A_5} $
      Соединяем A 5 с B и проводим линии через A 4 , A 3 , A 2 и A 1 , которые параллельны A 5 B.

      Они пересекают AB в точках B 4 , B 3 , B 2 и B 1 соответственно. Эти точки делят отрезок $ \ overline {AB} $ на пять равных отрезков.

      Действительно, из трапеции BB 3 A 3 A 5 мы видим, что $ \ overline {BB_4} = \ overline {B_4B_3} $. Таким же образом из трапеции B 4 B 2 A 2 A 4 , получаем $ \ overline {B_4B_3} = \ overline {B_3B_2} $

      При этом от трапеции B 3 B 1 A 1 A 3 ,
      $ \ overline {B_3B_2} = \ overline {B_2B_1} $.
      Тогда из B 2 AA 2 следует, что $ \ overline {B_2B_1} = \ overline {B_1A} $.В итоге получаем:
      $ \ overline {AB_1} = \ overline {B_1B_2} = \ overline {B_2B_3} = \ overline {B_3B_4} = \ overline {B_4B} $

      Понятно, что если AB нужно разделить на другое количество равных отрезков, мы должны спроецировать такое же количество равных отрезков на p. Далее поступаем так же.

      Середина треугольника — Cuemath

      Замкнутая фигура, состоящая из трех отрезков, формирует форму треугольника.

      Давайте исследуем мир треугольника

      Треугольник состоит из множества частей.Например, углы, стороны, медиана, середина, середина сегмента и т. Д. Вот упражнение для вас. Теперь вы можете визуализировать различные типы треугольников в математике на основе их сторон и углов. Попробуйте изменить положение вершин, чтобы понять взаимосвязь между сторонами и углами треугольника.

      В более поздней части этой главы мы обсудим середину и середину треугольника.

      Для любых двух точек, скажем, \ (A \) и \ (C \), середина — это точка \ (B \), которая расположена на полпути между точками \ (A \) и \ (B \).

      Обратите внимание, что точка \ (B \) равноудалена от \ (A \) и \ (C \).

      Средняя точка существует только для линейного сегмента.

      Линия, соединяющая среднюю точку, называется мидсегментом.

      В этом мини-уроке мы исследуем мир среднего сегмента треугольника, найдя ответы на такие вопросы, как средний сегмент треугольника, теорема о среднем сегменте треугольника и доказательство с помощью интерактивных вопросов.

      Итак, приступим!

      План урока

      Что такое средний сегмент треугольника ?

      Середина треугольника Определение

      Середина треугольника — это отрезок прямой, соединяющий средние точки или центр двух противоположных или смежных сторон треугольника

      На приведенном выше рисунке D — это середина AB, а E — середина AC.

      Здесь DE — середина треугольника ABC.


      Теорема о промежуточном сегменте треугольника

      Теорема о среднем сегменте гласит, что отрезок прямой, соединяющий середины любых двух сторон треугольника, параллелен третьей стороне треугольника и составляет его половину.

      В треугольнике ABC имеем

      \ (AD = DB \) и \ (AE = EC \)

      Тогда согласно теореме среднего сегмента

      \ (DE∥BC \) и \ (DE = \ dfrac {1} {2} \ BC \)

      Аналогично

      \ (AD = DB \) и \ (BF = FC \)

      Тогда согласно теореме среднего сегмента

      \ (DF∥AC \) и \ (DF = \ dfrac {1} {2} \ AC \)

      Аналогично

      \ (AE = EC \) и \ (BF = FC \)

      Тогда согласно теореме среднего сегмента

      \ (EF∥AB \) и \ (EF = \ dfrac {1} {2} \ AB \)

      Проба для середины треугольника

      В предыдущем разделе мы видели треугольник \ (ABC \) с тремя серединами в виде \ (D, \) \ (E, \) и \ (F \).

      Нам нужно доказать две вещи, чтобы оправдать доказательство теоремы о середине треугольника:

      • \ (DE∥BC \)
      • \ (DE = \ dfrac {1} {2} \ BC \)

      Дано: D и E — средние точки AB и AC

      .

      Чтобы доказать, что \ (DE∥BC \) и \ (DE = \ dfrac {1} {2} \ BC \) нам нужно провести линию, параллельную AB, пересекающую E, полученную в F.

      В \ (\ bigtriangleup {ADE} \) и \ (\ bigtriangleup {CFE} \)

      \ (\ begin {align} AE & = EC \ text {(E — средняя точка AC)} \\\ \ angle {1} & = \ angle {2} \ text {(Вертикально противоположные углы)} \\ \ \ angle {3} & = \ angle {4} \ text {(Альтернативные углы)} \ end {align} \)

      По AAS конгруэнтности треугольника имеем,

      \ (\ bigtriangleup {ADE} \ cong \ bigtriangleup {CFE} \)

      По CPCT у нас,

      \ (DE = FE \)

      \ (AD = CF \)

      D — это середина AB

      .

      \ (AD = BD \)

      \ (BD = CF \)

      DBCF — параллелограмм,

      \ (DF || BC \) и \ (DF = BC \)

      \ (DE || BC \) и \ (DF = BC \)

      \ (DE = \ dfrac {1} {2} DF \)

      с, DF = BC

      \ (DE = \ dfrac {1} {2} BC \)

      Следовательно доказано

      Середина треугольника формулы
      Средний сегмент \ (= \) \ (\ dfrac {1} {2} \ times \) Основание треугольника

      Что такое обратная теорема о срединном сегменте треугольника?

      Теорема, обратная теореме о среднем сегменте, определяется как: Когда сегмент линии соединяет две средние точки двух противоположных сторон треугольника и параллелен третьей стороне треугольника и составляет половину его, тогда это средний сегмент треугольника.

      В треугольнике ABC имеем

      \ (DE∥BC \) и \ (DE = \ dfrac {1} {2} \ BC \)

      Тогда согласно обратной теореме треугольника о среднем сегменте

      \ (AD = DB \) и \ (AE = EC \)
      \ (DE \) — середина треугольника \ (ABC \)

      Доказательство обратного теоремы о срединном сегменте треугольника

      В предыдущем разделе мы видели \ (\ bigtriangleup {ABC} \) с \ (D, \) \ (E, \) и \ (F \) в качестве трех средних точек.

      Нам нужно доказать любое из перечисленных ниже вещей, чтобы оправдать доказательство обратной теоремы о середине треугольника:

      • \ (DE \) — средний сегмент \ (\ bigtriangleup {ABC} \)
      • \ (AD = DB \) и \ (AE = EC \)

      У нас есть D как середина AB, тогда \ (AD = DB \) и \ (DE || BC \)

      \ (AB \) \ (= \) \ (AD + DB \) \ (= \) \ (DB + DB \) \ (= \) \ (2DB \)

      DBCF — параллелограмм.

      \ (DE || BC \) и \ (BD || CF \)

      Противоположные стороны параллелограмма равны.

      \ (BD = CF \)

      \ (DA = CF \)

      В \ (\ bigtriangleup {ADE} \) и \ (\ bigtriangleup {CFE} \)

      \ (\ begin {align} \ angle {1} & = \ angle {2} \ text {(Вертикально противоположные углы)} \\\ \ angle {3} & = \ angle {4} \ text {(Альтернативные углы )} \\\ DA & = CF \ end {align} \)

      По AAS конгруэнтности треугольника имеем,

      \ (\ bigtriangleup {ADE} \ cong \ bigtriangleup {CFE} \)

      По CPCT у нас

      \ (AE = EC \)

      E — средняя точка AC и DF.

      Следовательно, DE является средней частью \ (\ bigtriangleup {ABC} \).

      Важные примечания

      a) Отрезок, проходящий через среднюю точку, всегда параллелен одной стороне треугольника.
      б) Средний отрезок \ (= \) \ (\ dfrac {1} {2} \) длина третьей стороны треугольника.
      в) Треугольник может иметь не более трех средних сегментов.
      г) Средняя часть теоремы о треугольнике также известна как теорема о средней точке.

      Решенные примеры

      Чтобы лучше понять средний сегмент треугольника, давайте рассмотрим несколько решенных примеров.

      На данном рисунке H и M — середины треугольника EFG. Помогите Джейми доказать \ (HM || FG \) для следующих двух случаев.

      а) EH = 6, FH = 9, EM = 2 и GM = 3
      б) EH = 16, FH = 12, EM = 4 и GM = 3

      Решение

      а) У нас EH = 6, FH = 9, EM = 2 и GM = 3

      \ (\ dfrac {EH} {FH} = \ dfrac {6} {9} = \ dfrac {2} {3} \)

      \ (\ dfrac {EM} {GM} = \ dfrac {EH} {FH} = \ dfrac {2} {3} \)

      б) У нас EH = 16, FH = 12, EM = 4 и GM = 3

      \ (\ dfrac {EH} {FH} = \ dfrac {16} {12} = \ dfrac {4} {3} \)

      \ (\ dfrac {EM} {GM} = \ dfrac {EH} {FH} = \ dfrac {4} {3} \)

      HM делит EF и EG треугольника EFG в равных пропорциях.

      Следовательно, HM — это середина треугольника EFG.

      \ (\ следовательно \) \ (HM || FG \)

      Помогите Рону найти значение x и значение отрезка AB, учитывая, что A и B являются серединами треугольника PQR.

      Решение

      У нас есть две средние точки A и B.

      Согласно теореме о треугольнике среднего сегмента

      \ (\ begin {align} QR & = 2AB \\\
      36 & = 2 (9х) \
      х & = 2 \\\
      AB & = 18 \ end {align} \)

      \ (\ следовательно \) Значение x равно 2
      Стоимость AB составляет 18

      Интерактивные вопросы

      Вот несколько занятий для вас.Выберите / введите свой ответ и нажмите кнопку «Проверить ответ», чтобы увидеть результат.

      Сложные вопросы

      a) Рассмотрим треугольник ABC, и пусть D — любая точка на BC. Пусть X и Y — середины отрезков AB и AC. Покажите, что XY делит пополам AD.

      б) Рассмотрим параллелограмм ABCD. E и F — середины AB и CD соответственно. Покажите, что отрезки AF и EC пересекают диагональ BD.

      Подведем итоги

      Мини-урок был посвящен увлекательной концепции средней части треугольника. Математическое путешествие по средней части треугольника начинается с того, что студент уже знает, и переходит к творческому созданию новой концепции в молодых умах. Сделано таким образом, чтобы оно не только было понятным и понятным, но и навсегда осталось с ними. В этом заключается магия Куэмат.

      О компании Cuemath

      В Cuemath наша команда экспертов по математике стремится сделать обучение интересным для наших любимых читателей, студентов!

      Благодаря интерактивному и увлекательному подходу «обучение-обучение-обучение» учителя исследуют тему со всех сторон.

      Будь то рабочие листы, онлайн-классы, сеансы сомнений или любые другие формы отношений, мы в Cuemath верим в логическое мышление и интеллектуальный подход к обучению.


      Часто задаваемые вопросы (FAQ)

      1. Что такое средний сегмент треугольника?

      Середина треугольника — это отрезок прямой, соединяющий средние точки или центр двух противоположных или смежных сторон треугольника

      В треугольнике может быть 3 средних сегмента.

      На приведенном выше рисунке D — это середина AB, E — середина AC, а F — середина BC.

      Здесь DE, DF и EF — 3 средних сегмента треугольника ABC.

      2. Как найти середину треугольника?

      Мы можем найти средний сегмент треугольника, используя средний сегмент формулы треугольника,

      Средний сегмент \ (= \) \ (\ dfrac {1} {2} \ times \) Основание треугольника.

      3. Что такое теорема о средней точке?

      Теорема о средней точке утверждает, что отрезок прямой, соединяющий середины любых двух сторон треугольника, параллелен третьей стороне и равен половине третьей стороны.Рассмотрим произвольный треугольник \ (\ bigtriangleup {ABC} \). Пусть D и E — середины отрезков AB и AC. Предположим, вы присоединяетесь к D и E:

      Теорема о средней точке гласит, что DE будет параллельно BC и равно ровно половине BC.

      Площадь трапеции с медианой

      В дополнение к стандартной формуле для площади трапеции с использованием ее оснований, мы также можем вычислить площадь трапеции с ее серединой и ее высотой. Медиана — это линия, соединяющая две средние точки ног трапеции — непараллельные стороны трапеции.Медиана также называется срединным сегментом или средней линией.

      Задача

      BCD — трапеция, AB || CD . EF — это линия, соединяющая середины ног AD, и BC , AE = ED и BF = FC . h — высота трапеции. Найдите формулу для его площади, используя h и | EF |

      Стратегия

      Давайте посмотрим, как мы можем связать то, что мы знаем о медиане трапеции, с формулой, которая у нас уже есть для площади трапеции.Площадь трапеции составляет (короткое основание + длинное основание) · высота / 2, или A = ½ ( AB + DC ) · час.

      В этой задаче у нас есть высота и средний или средний сегмент. Из теоремы о срединном сегменте трапеции мы получаем связь между средним сегментом и основаниями: | EF | = ½ ( AB + DC ). Глядя на две формулы, мы видим, что можем просто заменить EF на ½ ( AB + DC ) в формуле для площади и получить A = | EF | · h

      Решение

      (1) A = ½ ( AB + DC ) · h // Площадь трапеции
      (2) AE = ED , BF = FC // задано
      (3) EF — средний сегмент // (2), Определение среднего сегмента
      (4) | EF | = ½ ( AB + DC ) // (3), теорема о среднем сегменте трапеции
      (5) A = | EF | · H // (1), (4), подстановка

      Другой способ решения этой проблемы

      В предыдущем разделе мы полагались на признание того, что формула для площади трапеции — A = ½ ( AB + DC ) · h очень похожа на формулу длины мидсегмента — | EF | = ½ ( AB + DC ), и произведена замена, которая привела к очень компактному и элегантному решению.

      Но что, если мы не сразу узнаем, что формулы похожи, или не вспомним, что средний сегмент равен половине суммы оснований? Давайте посмотрим на другой способ решить эту проблему, не полагаясь на это.

      Поскольку EF — это линия, соединяющая середины сторон, мы могли бы использовать теорему о треугольнике середины отрезка, но для этого нам понадобится треугольник. Итак, давайте нарисуем один, используя диагональ AC:

      Решение, используя теорему о среднем сегменте треугольника

      В треугольнике ΔACD, | EG | — это линия, параллельная основанию CD, которая начинается от середины стороны AD, поэтому согласно обратной теореме о среднем сегменте треугольника, это средний сегмент, равный половине основания.Положим | EG | = x. Если x равен половине базы, то CD базы должен быть равен 2x.

      Теперь посмотрим на другой треугольник ΔACB. Используя те же рассуждения, что и выше, | GF | начинается от середины стороны BC и параллельна AB — так что, согласно обратной теореме о среднем сегменте треугольника, это средний сегмент, равный половине основания. Длина | GF | равно | EF | -x, поэтому основание AB равно 2 · (| EF | -x) или 2 · | EF | -2x.

      Теперь давайте подставим эти значения в формулу для площади трапеции:

      (1) A = ½ ( AB + CD ) · h
      (2) AB = 2 · | EF | — 2x
      (3) CD = 2x
      (4) A = ½ (2 · | EF | -2x + 2x) · h = ½ (2 · | EF |) · h = | EF | · h

      % PDF-1.4 % 550 0 объект > эндобдж xref 550 97 0000000016 00000 н. 0000003148 00000 п. 0000003233 00000 н. 0000003471 00000 н. 0000004015 00000 н. 0000004091 00000 н. 0000004168 00000 п. 0000004243 00000 н. 0000004321 00000 п. 0000004610 00000 н. 0000008084 00000 н. 0000008496 00000 н. 0000008907 00000 н. 0000009240 00000 п. 0000009495 00000 н. 0000009831 00000 н. 0000010215 00000 п. 0000016821 00000 п. 0000017332 00000 п. 0000017755 00000 п. 0000018113 00000 п. 0000018288 00000 п. 0000019099 00000 н. 0000019411 00000 п. 0000019760 00000 п. 0000020118 00000 п. 0000020391 00000 п. 0000020833 00000 п. 0000021556 00000 п. 0000021976 00000 п. 0000022471 00000 п. 0000023205 00000 п. 0000023633 00000 п. 0000023866 00000 п. 0000024285 00000 п. 0000024332 00000 п. 0000024380 00000 п. 0000024417 00000 п. 0000024470 00000 п. 0000024518 00000 п. 0000025256 00000 п. 0000025334 00000 п. 0000026741 00000 п. 0000027069 00000 п. 0000027255 00000 п. 0000027770 00000 п. 0000029076 00000 п. 0000029989 00000 н. 0000031455 00000 п. 0000031755 00000 п. 0000032180 00000 п. 0000032873 00000 п. 0000032924 00000 п. 0000034255 00000 п. 0000034628 00000 п. 0000034890 00000 н. 0000035620 00000 п. 0000036148 00000 п. 0000039117 00000 п. 0000040222 00000 п. 0000044895 00000 п. 0000047882 00000 п. 0000051027 00000 п. 0000056557 00000 п. 0000059249 00000 п. 0000069750 00000 п. 0000070705 00000 п. 0000082112 00000 п. 0000082267 00000 п. 0000082861 00000 п. 0000083440 00000 п. 0000083500 00000 п. 0000084126 00000 п. 0000084345 00000 п. 0000084634 00000 п.

      Бухгалтерское с ответами дело тесты: Ответы на тесты по дисциплине «Бухгалтерское дело. Тест 10»

      Ответы к тесту по Бухгалтерскому делу

      Общее методологическое руководство бухгалтерским учетом в России осуществляет:

      а) Президент РФ

      б) Правительство РФ

      в) Институт профессиональных бухгалтеров

      Активы организации по времени использования подразделяются на:

      а) текущие и долгосрочные;

      б) внеоборотные и оборотные;

      в) предмета труда и средства труда.

      Цель бухгалтерского учета определяется — интересами:

      а) государства;

      б) пользователей информации;

      в) работников бухгалтерской службы.

      К юридически контролируемому имуществу относится имущество:

      а) принадлежащее организации на праве собственности;

      б) не принадлежащее организации на праве собственности;

      в) принадлежащее и не принадлежащее организации на праве собственности.

      Фактическая стоимость- это:

      а) сумма денежных средств или их эквивалентов уплачиваемая или начисленная при приобретении или производстве объекта;

      б) сумма денежных средств или их эквивалентов, которая должна быть уплачена в настоящее время в случае необходимости замены объекта;

      в) сумма денежных средств или их эквивалентов, которая может быть получена в результате продажи объекта или при наступлении срока его ликвидации.

      В соответствии с осмотрительностью в бухгалтерском учете создаются резервы:

      а) резерв по сомнительным долгам, под обесценивание финансовых вложений, под снижение стоимости товарно-материальных ценностей;

      б) резерв под снижение стоимости товарно-материальных ценностей, основных средств и нематериальных активов;

      в) резерв по сомнительным долгам, под снижение стоимости основных средств, под обесценивание финансовых вложений.

      Упрощенная система бухгалтерского учета предназначена для:

      а) всех организаций;

      б) субъектов малого предпринимательства;

      в) иностранных организаций.

      Ответственность за организацию бухгалтерского дела на предприятии возлагается на:

      а) главного бухгалтера

      б) руководителя предприятия

      в) руководителя и главного бухгалтера

      Налоговые органы являются внешними пользователями информации:

      а) с прямым финансовым интересом;

      б) с косвенным финансовым интересом;

      в) без финансового интереса.

      При ликвидации предприятия без правопреемника документы личного состава работников:

      а) выдаются им на руки;

      б) уничтожаются по акту;

      в) сдаются в территориальное отделение Росархива.

      Имущество, полученное в капитализированный финансовый лизинг:

      а) является активом организации;

      б) не является активом организации;

      в) зависит от условий договора.

      Основные направления реформирования бухгалтерского учета в России – это:

      а) совершенствование нормативно правового регулирования, формирование стандартов, методическое и кадровое обеспечение, международное сотрудничество;

      б) пересмотр плана счетов и первичных документов, взвешенное использование международных стандартов в национальном регулировании;

      в) переориентация национального учета с нормативного регулирования учетного процесса на МСФО, система профессиональной подготовки бухгалтерских кадров.

      Целью реформирования бухгалтерского учета в России является:

      а) переход российского учета на МСФО;

      б) переориентация национального учета с нормативного регулирования учетного процесса на учетную политику организаций;

      в) приведение национальной системы бухгалтерского учета в соответствие с требованиями рыночной экономики и МСФО.

      Задачи реформирования бухгалтерского учета в России:

      а) сформировать систему национальных стандартов, увязать реформу с МСФО, оказать методическую помощь заинтересованным пользователям;

      б) совершенствовать нормативную базу, повысить квалификацию бухгалтеров;

      в) организовать ведения бухгалтерского учета в соответствии с МСФО, организовать международное сотрудничество, разработать новый план счетов бухгалтерского учета.

      Программой реформирования бухгалтерского учета полагалось разработать и утвердить:

      а) 22 ПБУ;

      б) 23 ПБУ;

      в) 24 ПБУ.

      В рамках Программы реформирования бухгалтерского учета в России в 1998г. были приняты:

      а) Положение по ведению бухгалтерского учета и бухгалтерской отчетности;

      б) Федеральный закон «О бухгалтерском учете»;

      в) План счетов.

      Ежегодное повышение квалификации профессионального бухгалтера:

      а) дело сугубо добровольное;

      б) обязательно;

      в) желательно, но не обязательно.

      Членами ИПБ России могут быть:

      а) только физические лица;

      б) только юридические лица;

      в) физические и юридические лица.

      Институт профессиональных бухгалтеров России:

      а) государственная организация;

      б) профессиональный союз бухгалтеров;

      в) некоммерческое партнерство.

      Территориальные институты профессиональных бухгалтеров имеют статус:

      а) корпоративного члена ИПБ России;

      б) ассоциативного члена ИПБ России;

      в) действующего члена ИПБ России.

      Профессиональными организациями аудиторов России являются:

      а) хозяйственное общество;

      б) государственное предприятие;

      в) некоммерческая организация.

      Система регулирования бухгалтерского учета в России имеет:

      а) два уровня;

      б) три уровня;

      в) четыре уровня.

      Организация бухгалтерского дела на предприятии определяется:

      а) Федеральным законом «О бухгалтерском учете»;

      б) Положением по ведению бухгалтерского учета и бухгалтерской отчетности в РФ;

      в) Приказом руководителя.

      Положение по ведению бухгалтерского учета и бухгалтерской отчетности в РФ относится к:

      а) первому уровню системы регулирования бухгалтерского учета;

      б) второму уровню системы регулирования бухгалтерского учета;

      в) третьему уровню системы регулирования бухгалтерского учета;

      г) четвертому уровню системы регулирования бухгалтерского учета.

      План счетов бухгалтерского учета относится к:

      а) первому уровню системы регулирования бухгалтерского учета;

      б) второму уровню системы регулирования бухгалтерского учета;

      в) третьему уровню системы регулирования бухгалтерского учета;

      г) четвертому уровню системы регулирования бухгалтерского учета.

      Методические указания по бухгалтерскому учету материально-производственных запасов,утвержденные Приказом Минфина РФ от 28.12.01 № 119н:

      а) являются обязательными;

      б) не являются обязательными;

      в) не обязательны, но желательны.

      Собственники имущества, находящегося в хозяйственном ведении и оперативном управлении,являются:

      а) внешними пользователями информации;

      б) внутренними пользователями информации;

      в) внешними и внутренними пользователями информации.

      Для составления бухгалтерской отчетности за отчетный год признается период:

      а) с 1 января по 31 декабря включительно;

      б) с 1 января отчетного года по 1 января следующего за отчетным годом;

      в) по усмотрению руководства организации.

      В машинно-ориентированных системах бухгалтерского учета использование двойной записи насчетах:

      а) обязательно;

      б) необязательно;

      в) необязательно, но желательно.

      Более современным считается использование при автоматизации бухгалтерского дела:

      а) системы учетных алгоритмов формирования экономической информации;

      б) модельный принцип систематизации данных бухгалтерского учета;

      в) использование западных систем.

      Основой системы нормативного регулирования бухгалтерского учета в РФ является:

      а) Кодекс профессиональной этики бухгалтера;

      б) Федеральный закон «О бухгалтерском учете;

      в) Закон «О государственном регулировании бухгалтерского учета в РФ.

      Положение о бухгалтерии предприятия утверждается

      а) обязательно;

      б) необязательно;

      в) желательно.

      Контроль над соблюдением исполнителями графика документооборота по предприятиюосуществляет:

      а) главный бухгалтер организации;

      б) руководитель организации;

      в) руководитель и главный бухгалтер организации.

      Положение о бухгалтерии предприятия:

      а) разработано на государственном уровне как нормативный документ;

      б) разрабатывается самой организацией;

      в) разрабатывается Росстатом.

      Положение о бухгалтерии предприятия разрабатывается:

      а) руководителем;

      б) главным бухгалтером;

      в) руководителем и главным бухгалтером.

      К организационно-распорядительным документам относятся:

      а) график документооборота;

      б) положение о бухгалтерии;

      в) номенклатура дел.

      Распоряжения главного бухгалтера передаются старшим бухгалтерам соответствующих отделов,которые определяют конкретного исполнителя при:

      а) иерархичной структуре организации бухгалтерии;

      б) линейно-штабной структуре организации бухгалтерии;

      в) функциональной структуре организации бухгалтерии.

      Все работники бухгалтерии подчиняются непосредственно главному бухгалтеру при:

      а) линейной структуре организации бухгалтерии;

      б) вертикальной структуре организации бухгалтерии;

      в) комбинированной структуре организации бухгалтерии.

      Административная ответственность главного бухгалтера наступает:

      а) за неуплату налогов;

      б) за неуплату налогов в крупных размерах;

      в) за неуплату налогов в особо крупных размерах.

      Подписывать приходные кассовые ордера при приеме наличных денег главный бухгалтер:

      а) обязан;

      б) не обязан;

      в) может доверить подписывать кассиру.

      Разработка документооборота в организации:

      а) обязательно;

      б) необязательно;

      в) желательно.

      Формировать учетную политику организации обязан:

      а) главный бухгалтер;

      б) руководитель;

      в) руководитель и главный бухгалтер.

      Периодичность учета материальных ценностей включается в:

      а) организационный аспект учетной политики;

      б) технический аспект учетной политики;

      в) методический аспект учетной политики.

      Рабочий план счетов включается в:

      а) технический аспект учетной политики;

      б) организационный аспект учетной политики;

      в) методический аспект учетной политики.

      Уровень централизации учета включается в:

      а) технический аспект учетной политики;

      б) организационный аспект учетной политики;

      в) методический аспект учетной политики.

      Учетный процесс рассредоточен по отдельным производственным подразделениям при:

      а) централизованном учете;

      б) децентрализованном учете;

      в) смешанном учете.

      Порядок проведения инвентаризации включается в:

      а) организационный аспект учетной политики;

      б) технический аспект учетной политики;

      в) методический аспект учетной политики.

      Первичный учетный документ должен содержать следующие обязательные реквизиты:

      а) наименование и дата составления документа, наименование организации, содержание и измерители хозяйственной операции, наименование должностного лица и его подпись;

      б) наименование и дата составления документа, наименование организации, содержание и измерители хозяйственной операции, подпись руководителя и печать.

      в) наименование и период составления документа, содержание и измерители хозяйственной операции, подпись.

      Подписывать расходные кассовые ордера при выдаче наличных денег главный бухгалтер:

      а) обязан;

      б) не обязан;

      в) может доверить подписывать кассиру.

      Отсроченными обязательствами считается задолженность:

      а) срок погашения которой не наступил;

      б) срок погашения истек;

      в) срок погашения продлен.

      Выдача первичных учетных документов, регистров и отчетности из архива производиться с разрешения:

      а) руководителя организации;

      б) главного бухгалтера;

      в) руководителя и главного бухгалтера предприятия.

      Казенное предприятие на выделенное ему имущество имеет:

      а) право собственности;

      б) право хозяйственного ведения;

      в) право оперативного управления.

      Некоммерческие организации:

      а) потребительские кооперативы;

      б) общество с ограниченной ответственностью;

      в) государственные и муниципальные унитарные предприятия.

      Государственное унитарное предприятие на выделенное ему имущество имеет:

      а) право собственности;

      б) право хозяйственного ведения;

      в) право оперативного управления.

      Некоммерческое партнерство прибыль, полученную по результатам предпринимательскойдеятельности:

      а) могут распределять между своими членами в полном объеме;

      б) могут распределять 50% прибыли между своими членами;

      в) не могут распределять между членами партнерства.

      Источник финансирования потребительских кооперативов:

      а) имущественные паевые взносы;

      б) вступительные и членские взносы;

      в) добровольные имущественные взносы.

      Источник финансирования общественных организаций:

      а) имущественные паевые взносы;

      б) вступительные и членские взносы;

      в) добровольные имущественные взносы.

      Источник финансирования автономных некоммерческих организаций:

      а) имущественные паевые взносы;

      б) вступительные и членские взносы;

      в) добровольные имущественные взносы.

      Некоммерческое партнерство может реорганизоваться в:

      а) автономная некоммерческая организация;

      б) учреждение;

      в) союз.

      Под грантом понимаются:

      а) любое целевое поступление денежных средств или иного имущества;

      б) поступление денежных средств или иного имущества от зарубежных правительственных организаций;

      в) целевое поступление денежных средств или иного имущества на осуществление конкретныхпрограмм.

      Уставный фонд формируют:

      а) хозяйственные общества;

      б) хозяйственные товарищества;

      в) унитарные предприятия.

      Погашение обязательств приводит:

      а) к оттоку капитала организации;

      б) к оттоку активов организации;

      в) притоку активов организации.

      Под обязательством в бухгалтерском учете понимается:

      а) дебиторская задолженность организации;

      б) кредиторская задолженность организации;

      в) дебиторская и кредиторская задолженность организации.

      Показатели платежеспособности характеризуют:

      а) краткосрочную платежеспособность организации;

      б) долгосрочную платежеспособность организации;

      в) краткосрочную и долгосрочную платежеспособность организации.

      Коэффициент финансовой устойчивости характеризует:

      а) структуру доходов организации;

      б) рентабельность расходов организации;

      в) долгосрочную платежеспособность организации.

      Быстрореализуемые активы:

      а) денежные средства;

      б) дебиторская задолженность до 12 месяцев;

      в) дебиторская задолженность более 12 месяцев.

      Наиболее ликвидные активы:

      а) денежные средства;

      б) дебиторская задолженность до 12 месяцев;

      в) дебиторская задолженность более 12 месяцев.

      Медленнореализуемые активы:

      а) нематериальные активы;

      б) дебиторская задолженность до 12 месяцев;

      в) дебиторская задолженность более 12 месяцев.

      Труднореализуемые активы:

      а) нематериальные активы;

      б) дебиторская задолженность до 12 месяцев;

      в) дебиторская задолженность более 12 месяцев.

      Наиболее срочные обязательства:

      а) краткосрочные кредиты;

      б) доходы будущих периодов;

      в) кредиторская задолженность.

      Показатели состояния основных и оборотных средств – это:

      а) коэффициент реальной стоимости имущества;

      б) коэффициент оборачиваемости запасов;

      в) коэффициент финансовой устойчивости.

      Коэффициент постоянных пассивов характеризует:

      а) финансовую независимость организации;

      б) финансовую устойчивость организации;


      в) финансовую маневренность собственного капитала.

      Тест: Тесты по бухгалтерскому делу и ответы к компьютерному тестированию

      Тема: Тесты по бухгалтерскому делу и ответы к компьютерному тестированию

      Раздел: Бесплатные рефераты по бухгалтерскому делу

      Тип: Тест | Размер: 10. 80K | Скачано: 267 | Добавлен 28.09.13 в 21:25 | Рейтинг: +1 | Еще Тесты

      Вопрос: Актив признается в бухгалтерском балансе, когда: 
      Ответ: вероятно получение организацией экономических выгод в будущем от этого актива 
      Ответ: его стоимость может быть измерена с достаточной степенью надежности 

      Вопрос: Бухгалтерский счет предназначен для: 
      Ответ: текущей регистрации и группировки изменений в активе организации в процессе движения 

      Вопрос: В соответствии с Концепцией бухгалтерского учета в рыночной экономике России активами считаются: 
      Ответ: хозяйственные средства, контроль над которыми организация получила в результате свершившихся фактов ее хозяйственной деятельности и которые должны принести ей экономические выгоды в будущем 

      Вопрос: В состав прочих доходов включаются: 
      Ответ: чрезвычайные доходы 

      Вопрос: В условиях инфляционной экономики в ходе анализа необходимо исключить при проведении расчетов влияние инфляционных тенденций, для сравнения данных отчетности с аналогичными показателями предыдущих лет применяют: 
      Ответ: индекс-дефлятор 

      Вопрос: Взаимодействие бухгалтерии с другими структурными подразделениями организации определяется: 
      Ответ: движением документов в соответствии с утвержденным графиком документооборота 

      Вопрос: Вступительная бухгалтерская отчетность возникшей в результате реорганизации в форме слияния организации на дату ее государственной регистрации формируется: 
      Ответ: на основе данных передаточного акта и построчного объединения числовых показателей заключительной бухгалтерской отчетности реорганизованных организаций 

      Вопрос: Выберите понятие капитала: 
      Ответ: вложения собственника и прибыль, накопленная за все время деятельности организации 

      Вопрос: Главный бухгалтер приступает к руководству бухгалтерией: 
      Ответ: с даты, указанной в приказе руководителя организации 

      Вопрос: Главный бухгалтер, сдающий дела, освобождается от руководства бухгалтерией: 
      Ответ: с момента подписания акта проверки кассы 

      Вопрос: Государственная регистрация осуществляется в срок: 
      Ответ: не более 5 рабочих дней со дня представления документов в регистрирующий орган 

      Вопрос: Данные передаточного акта или разделительного баланса, изменения величины уставного и добавочного капиталов, других числовых показателей бухгалтерского баланса раскрываются: 
      Ответ: в пояснительной записке к заключительной бухгалтерской отчетности 

      Вопрос: Деятельность организации начинается: 
      Ответ: с даты государственной регистрации 

      Вопрос: Для оценки активов, обязательств, доходов и расходов согласно п. 9 Концепции бухгалтерского учета в рыночной экономике России могут быть использованы следующие методы: 
      Ответ: фактическая (первоначальная) стоимость (себестоимость) 
      Ответ: текущая (восстановительная) стоимость (себестоимость) 

      Вопрос: Должностные инструкции на работников бухгалтерии составляются: 
      Ответ: главным бухгалтером и утверждаются руководителем организации 

      Вопрос: Доля собственного капитала в оборотных средствах (коэффициент обеспеченности собственными средствами) рассчитывается как: 
      Ответ: отношение собственных средств в обороте ко всей величине оборотных средств 

      Вопрос: Доходами организации признаются поступления: 
      Ответ: от штрафов, пеней, неустоек за нарушение условий договоров купли-продажи 
      Ответ: авансы в счет оплаты продукции, товаров, работ, услуг 

      Вопрос: Доходом признается: 
      Ответ: увеличение экономических выгод в течение отчетного периода 

      Вопрос: Изменение учетной политики должно: 
      Ответ: вводиться с 1 января года, следующего за годом его утверждения и раскрываться в пояснительной записке к бухгалтерской отчетности организации 

      Вопрос: Инвентаризации подлежат: 
      Ответ: все имущество организации независимо от его местонахождения 

      Вопрос: Инвентаризация осуществляется на основании: 
      Ответ: приказа руководителя 

      Вопрос: Инициаторами реорганизации может выступать: 
      Ответ: учредители или по решению суда 

      Вопрос: Информационное обеспечение аудита с применением компьютеров включает: 
      Ответ: данные бухгалтерского учета экономического субъекта на бумажных носителях или в виде базы данных бухгалтерии и нормативно-справочную базу и систему форм рабочей документации аудитора 

      Вопрос: Информация, раскрываемая в бухгалтерской отчетности, должна: 
      Ответ: полно и достоверно отражать имущественное и финансовое положение организации 

      Вопрос: Информация, формируемая в подсистеме бухгалтерского учета, представляется внешним пользователям в виде отчетности: 
      Ответ: статистической 
      Ответ: налоговой 
      Ответ: бухгалтерской 

      Вопрос: Искажение бухгалтерской отчетности — это: 
      Ответ: неверное отражение и представление данных из-за нарушения установленных правил его организации и ведения 

      Вопрос: К бухгалтерской информации предъявляются такие требования, как: 
      Ответ: объективность, достоверность, своевременность, оперативность 

      Вопрос: К допущениям бухгалтерского учета относятся: 
      Ответ: непрерывность деятельности, имущественная обособленность, метод начисления, последовательность применения учетной политики 

      Вопрос: Какие правовые базы используют бухгалтер и аудитор в своей работе? 
      Ответ: «Гарант», «Консультант Плюс», «Кодекс» 

      Вопрос: Концепции бухгалтерского учета в рыночной экономике России от 29. 12.1997 г. одобрена: 
      Ответ: Методическим советом по бухгалтерскому учету при Минфине России и Президентским советом Института профессиональных бухгалтеров РФ 

      Вопрос: Коэффициент оборотных средств в производстве вычисляется как: 
      Ответ: отношение стоимости оборотных средств в производстве к среднемесячной выручке 

      Вопрос: Коэффициент автономии (финансовой независимости) определяется: 
      Ответ: отношением стоимости капитала и резервов организации, очищенных от убытков, к сумме средств организации в виде оборотных и внеоборотных активов 

      Вопрос: Коэффициент внутреннего долга хозяйствующего субъекта определяется как частное от деления суммы: 
      Ответ: обязательств по строкам «Задолженность перед персоналом организации», «Задолженность участникам (учредителям) по выплате доходов», «Доходы будущих периодов», «Резервы предстоящих расходов», «Прочие краткосрочные обязательства» на среднемесячную выручку 

      Вопрос: Коэффициент задолженности фискальной системе вычисляется по формуле: 
      Ответ: (сумма обязательств по строкам «Задолженность перед государственными внебюджетными фондами» и «Задолженность перед бюджетом») / (среднемесячна выручка) 

      Вопрос: Коэффициент обеспеченности оборотными средствами характеризует объем оборотных активов, выраженный в среднемесячных доходах организации, а также их оборачиваемость, и вычисляется путем деления: 
      Ответ: оборотных активов организации на среднемесячную выручку 

      Вопрос: Коэффициент оборотных средств в расчетах вычисляется как отношение стоимости оборотных средств: 
      Ответ: за вычетом оборотных средств в производстве к среднемесячной выручке 

      Вопрос: Коэффициент текущей ликвидности характеризует: 
      Ответ: платежеспособность организации с учетом предстоящих расчетов от дебиторов 

      Вопрос: Коэффициент, вычисляемый как отношение стоимости всех оборотных средств в виде запасов, дебиторской задолженности, краткосрочных финансовых вложений, денежных средств и прочих оборотных активов к текущим обязательствам организации, называют: 
      Ответ: коэффициент покрытия текущих обязательств оборотными активами 

      Вопрос: Коэффициенты исполнения текущих обязательств перед Федеральным бюджетом, бюджетом субъекта РФ, местным бюджетом, а так же перед государственными внебюджетными фондами Российской Федерации вычисляются как отношение: 
      Ответ: величины уплаченных налогов (взносов) к величине начисленных налогов (взносов) за тот же период 

      Вопрос: На начальной стадии жизненного цикла организации налогоплательщик встает на учет: 
      Ответ: во внебюджетные фонды, налоговый орган 

      Вопрос: Наличие объектов бухгалтерского учета на определенную дату и в определенном месте представляет собой: 
      Ответ: состояние 

      Вопрос: Налогоплательщик обязан сообщить в налоговый орган по месту учета об открытии или закрытии любого банковского счета: 
      Ответ: в 5-дневный срок 

      Вопрос: Налогоплательщики-организации могут выбрать упрощенную систему налогообложения: 
      Ответ: с численностью работников до 100 человек 

      Вопрос: Нормативной документ, в котором установлены понятия «допущения» и «требования»: 
      Ответ: Положении по бухгалтерскому учету «Учетная политика организации» (ПБУ 1/98), утвержденным приказом Минфина России от 09. 12.98 г. № 60н 

      Вопрос: Общее методологическое руководство организацией бухгалтерского учета в Российской Федерации осуществляется: 
      Ответ: Правительством РФ 

      Вопрос: Объем доходов организации за рассматриваемый период и определение основного финансового ресурса организации, который используется для осуществления текущей, финансовой и инвестиционной деятельности, определяет: 
      Ответ: среднемесячная выручка 

      Вопрос: Обязательством считается: 
      Ответ: задолженность организации, являющаяся следствием свершившихся проектов ее хозяйственной деятельности и расчеты по которой должны привести к оттоку активов 

      Вопрос: Организационная структура бухгалтерии формируется в соответствии: 
      Ответ: с расчетным количеством работников бухгалтерии 

      Вопрос: Основанием внесения в государственный Реестр соответствующей записи является: 
      Ответ: решение о регистрации организации, принятое регистрирующим органом 

      Вопрос: Основные правила (принципы) ведения бухгалтерского учета определены нормативными документами: 
      Ответ: Федеральным законом от 21. 11.96 № 129-ФЗ «О бухгалтерском учете» 
      Ответ: Положением по ведению бухгалтерского учета и бухгалтерской отчетности в Российской Федерации, утвержденным Приказом Минфина РФ от 29.07.98 № 34н 

      Вопрос: Основополагающие признаки, характеризующие статус главного бухгалтера организации, определены: 
      Ответ: ФЗ «О бухгалтерском учете» 

      Вопрос: Ответственность за организацию бухгалтерского учета, соблюдение законодательства при совершении фактов хозяйственной деятельности несет: 
      Ответ: руководитель организации 

      Вопрос: Оценка имущества, произведенного в самой организации осуществляется: 
      Ответ: по стоимости его изготовления 

      Вопрос: Оценка передаваемого (принимаемого) при реорганизации организации имущества производится: 
      Ответ: по остаточной стоимости, либо по текущей рыночной стоимости 

      Вопрос: Передача дел бухгалтерии производится на основе: 
      Ответ: последнего представленного бухгалтерского баланса 

      Вопрос: Положение о бухгалтерии представляет собой документ: 
      Ответ: распорядительный внутренний 

      Вопрос: Положение о бухгалтерии утверждается: 
      Ответ: руководителем организации 

      Вопрос: Последовательность разделов Положения о бухгалтерии: 
      Ответ: Общие положения 
      Ответ: Цели и задачи 
      Ответ: Функции бухгалтерии 
      Ответ: Права и обязанности руководства 
      Ответ: Взаимоотношения с другими структурными подразделениями 
      Ответ: Организация работы 

      Вопрос: Постановка на учет и снятие с учета осуществляются: 
      Ответ: бесплатно 

      Вопрос: Потенциальную возможность имущества прямо или косвенно способствовать притоку денежных средств или их эквивалентов в организацию называют: 
      Ответ: будущими экономическими выгодами 

      Вопрос: Правовой статус бухгалтерской службы определяется: 
      Ответ: положением о бухгалтерии 

      Вопрос: При реорганизации хозяйствующего субъекта в форме разделения составляется: 
      Ответ: разделительный баланс, содержащий положения о правопреемстве имущества и обязательств реорганизуемой организации на основании решения учредителей 

      Вопрос: Прием и сдача дел главными бухгалтерами производятся на основании: 
      Ответ: приказа руководителя организации 

      Вопрос: Признак классификации фактов хозяйственной деятельности (ФХД) по отношению к контрагенту позволяет выделить: 
      Ответ: односторонние и двусторонние ФХД 

      Вопрос: Программа реформирования бухгалтерского учета в соответствии с международными стандартами финансовой отчетности, утвержденная Постановлением Правительства РФ имеет разделов: 
      Ответ: два 

      Вопрос: Программные продукты, автоматизирующие учет в организации, можно разделить по степени охвата на: 
      Ответ: «коробочный» продукт, программный комплекс и интегрированный программный комплекс 

      Вопрос: Процесс реорганизации организации может проходить в формах: 
      Ответ: преобразования, слияния, присоединения, разделения, выделения 

      Вопрос: Рабочее место работника считается стационарным, если оно: 
      Ответ: создается на срок не менее одного месяца 

      Вопрос: Расходами организации признаются выбывшие активы: 
      Ответ: проценты, начисленные организацией за предоставление ей в пользование денежных средств 

      Вопрос: Расходами признаются: 
      Ответ: уменьшение экономических выгод в течение отчетного периода 

      Вопрос: Рентабельность оборотного капитала вычисляется как частное от деления: 
      Ответ: прибыли, остающейся в распоряжении акционеров после уплаты налогов и всех отчислений, на сумму оборотных средств 

      Вопрос: Рентабельность продаж вычисляется как частное отделения: 
      Ответ: прибыли, полученной в результате продажи продукции, на выручку организации за тот же период 

      Вопрос: Соблюдение общих методологических принципов бухгалтерского учета (допущений и требований) и постановку бухгалтерского дела осуществляет: 
      Ответ: главный бухгалтер 

      Вопрос: Собственный капитал в обороте вычисляется как: 
      Ответ: разность между собственными капиталом организации и ее внеоборотными активами 

      Вопрос: Соответствие между признаком классификации и группой фактов хозяйственной деятельности: 
      Ответ номер столбца: 2-13-4 

      Вопрос: Среднемесячная выработка на одного работника вычисляется как частное от деления: 
      Ответ: среднемесячной выручки на среднесписочную численность работников 

      Вопрос: Среднемесячная выручка рассчитывается как отношение: 
      Ответ: валовой выручки организации, включающей выручку от реализации за отчетный период(по оплате), НДС, акцизы и другие обязательные платежи, к количеству месяцев в отчетном периоде 

      Вопрос: Степень платежеспособности определяется как отношение: 
      Ответ: текущих заемных средств (краткосрочных обязательств) организации к среднемесячной выручке 

      Вопрос: Требование последовательности применения учетной политики содержится в: 
      Ответ: Положении по бухгалтерскому учету «Учетная политика организации» (ПБУ 1/98) 

      Вопрос: Унифицированные формы первичной учетной документации являются: 
      Ответ: обязательными к применению 

      Вопрос: Установление информационной связи между синтетическими счетами называют: 
      Ответ: корреспонденцией счетов 

      Вопрос: Учетная политика для целей бухгалтерского учета разрабатывается на основании: 
      Ответ: ПБУ 1/98 

      Вопрос: Учетная политика для целей налогового учета разрабатывается на основании: 
      Ответ: гл. 25 НК РФ и гл. 21 НК РФ 

      Вопрос: Факт хозяйственной деятельности, возникающий в результате каких-либо случайных явлений, влияющих на результаты деятельности организации, представляет собой: 
      Ответ: событие 

      Вопрос: Факт хозяйственной деятельности, возникающий в результате осуществления текущей, финансовой и инвестиционной деятельности, представляет собой: 
      Ответ: действие 

      Вопрос: Факты хозяйственной деятельности, у которых сальдо и оборот всегда равны, отражаются на счетах: 
      Ответ: забалансовых 

      Вопрос: Федеральный закон «О бухгалтерском учете» устанавливает: 
      Ответ: единые правовые и методологические основы организации и ведения бухгалтерского учета в РФ 

      Вопрос: Филиалы и представительства юридического лица действуют на основании: 
      Ответ: положения 

      Вопрос: Формирование учетной политики и контроль за ее исполнением возложен на: 
      Ответ: главного бухгалтера 

      Вопрос: Эффективность внеоборотного капитала (фондоотдача) определяется как отношение: 
      Ответ: среднемесячной выручки к стоимости внеоборотного капитала

      Чтобы скачать бесплатно Тесты на максимальной скорости, зарегистрируйтесь или авторизуйтесь на сайте.

      Важно! Все представленные Тесты для бесплатного скачивания предназначены для составления плана или основы собственных научных трудов.

      Друзья! У вас есть уникальная возможность помочь таким же студентам как и вы! Если наш сайт помог вам найти нужную работу, то вы, безусловно, понимаете как добавленная вами работа может облегчить труд другим.

      Если Тест, по Вашему мнению, плохого качества, или эту работу Вы уже встречали, сообщите об этом нам.

      Тест с ответами по бухучету — упражнение — Теория бухгалтерского учета (2) | Упражнения и задачи Бухгалтерское дело

      Скачай Тест с ответами по бухучету — упражнение — Теория бухгалтерского учета (2) и больше Упражнения и задачи в PDF из Бухгалтерское дело только в Docsity! А 1. Активы в бухгалтерском балансе, составленном в соответствии с требованиями МСФО, располагаются в порядке: — по убыванию ликвидности 2. Арифметическая проверка документов – это проверка: — правильности подсчета стоимостных показателей Б 3. Бухгалтерский баланс – это: — способ экономической группировки и обобщения активов по составу и размещению источников его формирования, выраженной в денежной оценке и составленный на определенную дату 4. Базовыми принципами бухгалтерского учета являются: — автономность, двойная запись, денежное измерение, преемственность начислений, осмотрительность В 5. В каком разделе баланса показывают сумму оборотных активов: — раздел II актива баланса 6. В каких случаях допускается изменение учетной политики: — законодательно-нормативные акты, разработка новых способов введения бухгалтерского учета, а также… 7. Выберите выражение, отражающее более правильное определение бухгалтерских документов: — письменное свидетельство, которое подтверждает факт совершения хозяйственных операций 8. Выпущенная из производства готовая продукция оценивается по: — Фактической производственной себестоимости 9. В какой группе отражены регулирующие счета: — 02, 05, 26, 44 10. В каком году в России появилось понятие «двойная запись» и «бухгалтерский учет»? — 1722 11. В чем выражается сущность двойной записи: — факт хозяйственной деятельности отражается на счетах дважды: в дебет одного счета и одновременно в кредит другого взаимосвязанного с ним счета на одинаковую сумму 12. Взаимосвязь между бухгалтерскими счетами и балансом устанавливается: — по остаткам статей баланса открываются бухгалтерские счета, а на основании остатков бухгалтерских счетов составляется баланс 13. В соответствии с группировкой активов хозяйствующего субъекта по видам, дебиторская задолженность является: — средствами в расчетах 14. В какой группе отражены распределительные счета? — 96, 25, 26, 98 15. В какой группе счетов при классификации по экономическому содержанию относятся счета: 66 «Расчеты по краткосрочным кредитам и займам» и 67 «Расчеты по долгосрочным кредитам и займам»? — К группе счетов заемных источников формирования активов 16. В основу активов организации положен принцип: — единообразия и реальности 17. В состав годовой бухгалтерской отчетности включают: — бухгалтерский баланс, отчеты о прибылях и убытках, пояснения к бухгалтерскому балансу и отчету о прибылях и убытках, аудиторское заключение 18. В каком разделе учетной политики организации отражается изменение учетной политики? — во втором 19. В какой части бухгалтерского баланса отражается кредиторская задолженность? — в V разделе пассива 20. В каком году в России появилось слово «бухгалтер»? — в 1710 21. В чем отличие между счетами бухгалтерского учета и балансом? — Бухгалтерские счета отражают текущие факты хозяйственной деятельности и итоговые данные за отчетные периоды в денежных, натуральных и трудовых показателях. В балансе отражаются только итоговые данные, служащие основанием для анализа деятельности организации 22. В чем заключается разница между накопительными и сводными документами? — Сводный документ, составляется на основании первичных документов, накопительный представляет собой первичный документ 23. В зависимости от сроков погашения обязательств различают: — долгосрочный и краткосрочный заемный капитал 24. В основе текущих активов выделяют: — денежные средства 25. В каких случаях проведение инвентаризации обязательно? — перед составлением годовой отчетности, при передаче имущества в аренду, продаже, выкупе, при смене материально-ответственных лиц, выявлении фактов хищения и стихийных бедствиях 26. В каком разделе отражается дебиторская задолженность: — во II разделе актива бухгалтерского баланса 27. В чем выражается взаимосвязь между синтетическими и аналитическими счетами: — сальдо один, обороты и сальдо два по всем аналитическим счетам равно сальдо один, оборотам и сальдо два синтетического счета 28. В каком разделе учетной политики организации отражается изменение учетной политики: — во втором Г 29. Где впервые появился бухгалтерский термин – кредит? — в Риме 30. Готовая продукция на счетах отражается по: — фактической производственной себестоимости 31. Годовая бухгалтерская отчетность представляется: — в течении 90 дней по окончании года 32. Готовая продукция – это продукция: — выпущенная из производства и сданная на склад 33. Главный бухгалтер подчиняется: — акционерам Д 34. Документы при поступлении в бухгалтерию подвергаются проверке: — формальной — по существу отражаемых операций — арифметической 35. Документы по способу составления подразделяются на: — машинные и ручные 36. Документы по объему подразделяются на: — разовые и накопительные 37. Допущениями бухгалтерского учета являются: — непрерывность деятельности, имущественная обособленность, приоритет содержания над формой, непротиворечивость 38. Документы по назначению классифицируются на: — распорядительные и оправдательные 39. Документооборот представляет собой: — путь, который совершает документ от момента его составления до сдачи в архив 40. Для осуществления бухгалтерских записей основанием являются документы: — оправдательные 41. Документы можно классифицировать по таким признакам, как: — назначение, способ охвата операций, строение, место составления, способ составления, структура 42. Документы бухгалтерского оформления применяются для: — подготовки информации к отражению в учетных регистрах 81. К какому типу относится факт хозяйственной деятельности «Оприходованы поступившие материалы от поставщиков»? — к третьему типу 82. Какие измерители применяются в хозяйственном учете для отражения имущества организаций? — Натуральные, трудовые и денежные 83. Кому представляется отчетность в обязательном порядке? — учредителям, органам государственной статистики, органам исполнительной власти, банкам, налоговой инспекции и другим пользователям в соответствии с действующим законодательством России 84. Какое из определений характеризует содержание термина «Международные стандарты бухгалтерской отчетности»? — это свод правил бухгалтерского учета, носящих рекомендательный характер 85. Какие принципы лежат в основе Кодекса профессиональной этики бухгалтера? — объективность, независимость, компетентность и честность 86. Какие способы применяются в бухгалтерском учете для исправления ошибочных записей? — корректирующий способ, дополнительная проводка и способ «красного сторно» 87. Кем проводится инвентаризация в организации? — работниками бухгалтерии 88. Какой регистр является основным при журнально-ордерной форме бухгалтерского учета? — Главная книга 89. Какими признаками характеризуется полезность бухгалтерской информации? — ценность, надежность 90. Как определить пассивность счета? — По главной книге, оборотной ведомости, активной части баланса 91. Какой регистр является основным при простой форме бухгалтерского учета? — Книга учета хозяйственных операций 92. Куда относится недостача материальных запасов в пределах норм естественной убыли? — Д20 – К10 93. К какой группе счетов при классификации по структуре относятся счета 90 «Продажи» и 91 «Прочие доходы и расходы»? — К группе сопоставляющих счетов 94. Каким способом исправляются ошибки в расходных и приходных ордерах? — Исправления не допускаются 95. К какой классификационной группе относятся счета 90 и 91? — к сопоставляющей 96. Когда бухгалтерский учет зародился в России как наука? — 1845-1850 97. К какому уровню системы нормативно-правовых актов относятся издаваемые ПБУ? — к первому 98. Какие из приведенных являются оправдательными документами? — квитанция о внесении денег в кассу другой организации 99. Какое из равенств обязательно в бухгалтерском балансе? — равенство итогов актива и пассива баланса 100. К объектам, составляющим текущую, финансовую и инвестиционную деятельность, относят: — хозяйственные и финансовые процессы, финансовые результаты 101. К какому уровню относятся рабочие документы организации, формирующие ее учетную политику? — ко второму 102. Какие из приведенных разделов и групп относят к пассиву баланса? — долгосрочные и краткосрочные обязательства 103. К требованиям бухгалтерского учета относят: — полнота, осмотрительность, рациональность, временная определенность фактов хозяйственной деятельности, последовательность применения учетной политики 104. Каким основным документом организации регламентируется порядок проведения инвентаризации и методы оценки видов имущества и обязательств? — инвентаризационной ведомостью 105. Каково основное счетное назначение оборотной ведомости по синтетическим счетам? — Сделать периодическое обобщение сумм оборотов и остатков по всем синтетическим счетам для проверки учетных записей. Составления баланса и общего ознакомления с состоянием и изменениями активов организации 106. К какому типу регистров относится Главная книга? — Синтетическому 107. К какой категории бухгалтерских документов по назначению относится ведомость распределения косвенных расходов: — бухгалтерского оформления 108. Какова роль бухгалтерских счетов? — Предназначены для группировки активов текущего отражения, обобщения и контроля за данными фактов хозяйственной деятельности качественно однородным признакам 109. К какому уровню системы относятся законы о бухгалтерском учете, издаваемые в России? — к третьему 110. Какой записью отражается недостача топлива на общезаводском складе, если о взыскании ущерба с виновных лиц отказано судом: — Д91 – К73 111. Как определить конечный остаток на пассивных счетах? — к начальному кредитовому сальдо прибавляют оборот по кредиту и вычитают оборот по дебету 112. Какое из приведенных выражений соответствует переводу «Международные стандарты финансовой отчетности»? — IFRS 113. Каковы основные требования к ведению бухгалтерского учета? — обязательное соблюдение в течении года принятой учетной политики и ведение активов и обязательств, а также хозяйственных операций в рублях Л 114. Лицевые счета и личные дела рабочих и служащих хранятся: — 75 лет 115. Ликвидационный баланс составляется: — с начала ликвидационного периода организации М 116. Методом бухгалтерского учета является: — совокупность способов и приемов с помощью которых признается предмет (объекты) бухгалтерского учета 117. Может ли главный бухгалтер принимать к исполнению документы, по операциям, противоречащим законодательству и нарушающие договорную и финансовую дисциплину: — Может по согласованию с Министерством по налогам и сборам 118. МСФО определяют, что промежуточная отчетность считается менее надежной, чем годовая, т.к.: — для промежуточной отчетности не требуется аудит 119. Может ли главный бухгалтер исполнять обязанности связанные с материальной ответственностью? — Может по согласованию с банком и инспекцией Министерства по налогам и сборам 120. Может ли руководитель организации поручить ведение бухгалтерского учета и отчетности другой специализированной организации на договорных началах? — Может, если в организации отсутствует бухгалтерская служба 121. «Международные стандарты бухгалтерского учета» — это: — свод правил, методов и процедур бухгалтерского учета, разработанных высокопрофессиональными международными организациями, которые носят рекомендательный характер Н 122. На какие разделы делятся счета при классификации по экономическому содержанию? — счета активов предприятия, его обязательства и факты хозяйственной деятельности 123. На какие основные группы подразделяются счета по структуре — счета основные, регулирующие, распределительные, калькуляционные, сопоставляющие, финансово-результативные, забалансовые 124. На выявленную недостачу незавершенного производства по цеху основного производства производится запись: — Д94 «Недостачи и потери от порчи ценностей» — К20 «Основное производство» 125. На списание общепроизводственных расходов сборочного цеха производится запись: — Д20 «Основное производство» — К25 «Общепроизводственные расходы» 126. Надежность показателей финансовой отчетности означает отсутствие: — существенных ошибок и искажений, который могут неправильно информировать пользователя 127. На счете 90 «Продажи» отражается: — полная фактическая себестоимость проданной продукции 128. На какие разделы делятся счета по экономическому содержанию? — счета активов предприятия, его обязательства и факты хозяйственной деятельности О 129. Основные задачи бухгалтерского учета включают: — формирование, обеспечение информацией, своевременное предотвращение негативных явлений, выявление внутрихозяйственных резервов и прогнозирования результатов деятельности организации на текущей период и на перспективу 130. Отметьте основные виды бухгалтерских балансов: — промежуточные, годовые, вступительные, разделительные, санируемые, ликвидационные, сводные 131. Отметьте в приведенных группах калькуляционные счета: — 20, 29, 08, 44 132. Отметьте в приведенных группах основные пассивные счета: — 90, 98, 84, 80 133. Отметьте, в какой группе распределены бюджетно-распределительные счета: — 97, 96, 98 134. Ответственность за организацию бухгалтерского учета несет: — руководитель организации 135. Основными недостатками мемориально-ордерной формы учета являются: — неприспособленность к автоматизации — большой объем учетной работы приходится на конец периода 136. Отметьте в приведенных ответах основные активные счета: — 01, 08, 04, 10 137. Определите правильный порядок записи на активном счете: — На активном счете начальный остаток записывается в дебет, увеличение — в дебет, уменьшение – в кредит 138. Обязательными элементами финансовой отчетности в системе МСФО являются: — активы, обязательства, доходы, расходы, капитал 139. Отпуск материалов в цехи вспомогательных производств отражается записью: — Д23 «Вспомогательные производства» — К10 «Материалы» П 140. По состоянию на 1.01.2004 утверждено и принято к исполнению международных стандартов финансовой отчетности: — 41 141. По МСФО результатами деятельности организации признаются:

      Тест по бухгалтерскому делу с ответами тренажер проводок онлайн

      Тест с ответами по теме «РФЭЕ» Бухгалтерское дело. Тест 1″

      1. Выберите верное определение термина «бухгалтерское дело»?

      а)ведение бухгалтерского учета в организации;
      б)практическая деятельность бухгалтерии во всех ее проявлениях; (верно)
      в)ведение управленческого и налогового учета в организации.

      2. Кто из сотрудников организации несет ответственность за работу всех бухгалтеров?

      а)бухгалтер по материалам;
      б)бухгалтер по денежным средствам;
      в)главный бухгалтер. (Верно)

      3. К какому уровню системы государственного регулирования бухгалтерского дела в России относятся методические рекомендации по инвентаризации имущества основных средств?

      а)первый уровень;
      б)второй уровень; (Верно)
      в)третий уровень.

      4. Что является основной частью бухгалтерского дела?

      а)бухгалтерский учет; (Верно)
      б)управленческий учет;
      в)налоговый учет.

      5. На какие две группы можно разделить всех пользователей бухгалтерской информации?

      а)внутренние пользователи и внешние пользователи; (верно)
      б)пользователи с прямым финансовым интересом и пользователи с косвенным финансовым интересом;
      в)пользователи с прямым финансовым интересом и пользователи без финансового интереса

      Нет времени или сил пройти тест онлайн? Поможем сдать тест дистанционно для любого учебного заведения: подробности.

      Вопрос 1. Активные счета — это счета для учёта

      • имущества
      • источников образования имущества
      • результатов хозяйственной деятельности

      Вопрос 2. Активы организации — это

      • действия, связанные с движением имущества в процессе хозяйственной деятельности
      • имущество, принадлежащее организации на правах собственности
      • источники приобретения имущества организации
      • хозяйственные средства, контроль над которыми организация получила в результате свершившихся фактов её хозяйственной деятельности и которые должны принести ей экономические выгоды в будущем

      Вопрос 3. Активы организации подразделяются на

      • внеоборотные и оборотные активы
      • основные активы и не основные активы
      • предметы труда и методы труда

      Вопрос 4. Амортизация в бухгалтерском учете — это

      • постепенный перенос стоимости имущества в течение всего срока его полезного использования на себестоимость продукции (работ, услуг)
      • способность имущества к использованию в хозяйственной деятельности организации
      • физический износ имущества

      Вопрос 5. Амортизация основных средств начисляется в течение

      • 12 лет
      • 20 лет
      • всего срока нахождения их в организации
      • срока их полезного использования

      Вопрос 6. Аналитические счета — это счета для

      • подробной характеристики объектов учета
      • текущего контроля за хозяйственными операциями
      • укрупненной группировки и учета однородных объектов

      Вопрос 7. Баланс считается абсолютно ликвидным, если имеют место следующие соотношения:

      • А1 П2; A3 > ПЗ; А4
      • А1 >П1; А2 ПЗ; А4
      • А1 >П1; А2>П2; A3 > ПЗ; А4
      • А1 >П1; А2>П2; A3 > ПЗ; А4>П4

      Вопрос 8. Без подписи главного бухгалтера считаются недействительными

      • акты выполненных работ с поставщиками и подрядчиками
      • денежные и расчетные документы
      • договора с покупателями и заказчиками
      • финансовые и кредитные обязательства

      Вопрос 9. Бухгалтерская запись (проводка) – это запись

      • дебетуемого (кредитуемого) счета
      • корреспондирующих счетов
      • суммы по хозяйственной операции
      • хозяйственной операции в денежном выражении путем отражения на корреспондирующих счетах

      Вопрос 10. Бухгалтерская отчетность составляется на основе данных учета

      • бухгалтерского
      • налогового
      • оперативного
      • статистического

      Вопрос 11. Бухгалтерские проводки активно-активного типа меняют

      • валюту баланса
      • структуру актива баланса
      • структуру пассива баланса

      Вопрос 12. Бухгалтерские проводки активно-пассивного типа валюту баланса

      • не изменяют
      • увеличивают
      • уменьшают

      Вопрос 13. Бухгалтерские проводки пассивно-активного типа валюту баланса

      • не изменяют
      • увеличивают
      • уменьшают

      Вопрос 14. Бухгалтерские проводки пассивно-пассивного типа меняют

      • валюту баланса
      • структуру актива баланса
      • структуру пассива баланса

      Вопрос 15. Бухгалтерский баланс — это обобщенное отражение информации об имуществе организации

      • в денежной оценке по его видам и источникам образования на определенную дату
      • в натурально-вещественной форме
      • на определенную дату в натурально-стоимостных показателях
      • по его видам и источникам образования за определенный период времени

      Вопрос 16. Бухгалтерский баланс представляет собой таблицу, состоящую из

      • актива и пассива
      • дебета и кредита
      • доходов и расходов
      • прибылей и убытков

      Вопрос 17. Бухгалтерский учет дает информацию об объектах бухгалтерского учета в … выражении

      • денежном
      • натуральном
      • натурально-стоимостном
      • трудовом

      Вопрос 18. Бухгалтерский учет по валютным счетам ведется

      • в иностранной валюте
      • в рублях на основании пересчета по курсу банка, в котором открыт валютный счет
      • в рублях на основании пересчета по курсу Центрального Банка РФ

      Вопрос 19. В активе баланса имущество группируется по

      • видам и источникам образования
      • источникам образования и назначению
      • степени ликвидности

      Вопрос 20. В активе баланса отражаются

      • долги поставщикам за товары и услуги
      • задолженность покупателей за продукцию
      • уставный капитал

      Вопрос 21. В активе баланса сгруппированы

      • имущество и дебиторская задолженность
      • источники формирования имущества
      • хозяйственные процессы за отчетный период

      Вопрос 22. В зависимости от объема учетной работы руководитель организации может

      • не организовывать ведение бухгалтерского учета
      • поручить ведение учета работникам сферы производства
      • учредить бухгалтерскую службу или вести учет лично

      Вопрос 23. В пассиве баланса отражаются

      • дебиторская задолженность
      • капитал
      • основные средства

      Вопрос 24. В пассиве баланса сгруппированы

      • дебиторская задолженность
      • имущество и денежные средства
      • источники формирования имущества и кредиторская задолженность

      Вопрос 25. В первичные документы (кроме кассовых и банковских) исправления

      • могут вноситься по согласованию с вышестоящей организацией
      • могут вноситься по согласованию с налоговыми органами
      • могут вноситься по согласованию с участниками хозяйственной операции
      • не могут вноситься

      Вопрос 26. В состав годовой бухгалтерской отчетности обязательно включаются

      • бухгалтерский баланс
      • декларация по налогу на прибыль
      • отчет о затратах на производство
      • отчет о продукции
      • отчет о финансовых результатах

      Вопрос 27. Величина, обратная времени, необходимому для превращения активов в денежные средства называется

      • кредитоспособность
      • ликвидность активов
      • ликвидность баланса
      • финансовая устойчивость

      Вопрос 28. Внесение изменений в кассовые и банковские документы

      • допускается
      • допускается в отдельных случаях
      • не допускается

      Вопрос 29. Вновь созданная организация оформляет учетную политику не позднее

      • 30 дней с начала календарного года
      • 60 дней со дня государственной регистрации
      • 90 дней с начала календарного года
      • 90 дней со дня государственной регистрации

      Вопрос 30. Впервые необходимость двойной записи на счетах была обоснована

      • А. П.Рудановским
      • Бенедиктом Котрульи
      • Гюгли и Шерром
      • Лукой Пачоли

      Вопрос 31. Выдача денежных средств подотчетным лицам отражается записью

      • Д-т сч.50 «Касса» Кт сч.71 «Расчеты с подотчетными лицами»
      • Д-т сч.71 «Расчеты с подотчетными лицами» Кт сч. 26 «Общехозяйственные расходы»
      • Д-т сч.71 «Расчеты с подотчетными лицами» Кт сч. 51 «Расчетные счета»
      • Д-т сч.71 «Расчеты с подотчетными лицами» Кт сч.50 «Касса»

      Вопрос 32. Выплачена из кассы зарплата работникам

      • Д 70 » Расчеты с персоналом по оплате труда» К 50 «Касса»
      • Д 70 » Расчеты с персоналом по оплате труда» К сч.51 «Расчетные счета»
      • Д-т сч. 25 «Общехозяйственные расходы» Кт сч.70 «Расчеты с персоналом по оплате труда»

      Вопрос 33. Выявленные при инвентаризации расхождения между фактическим наличием имущества и данными бухгалтерского учета могут быть в виде

      • избытка
      • излишка
      • недобора
      • недостачи
      • неполноты

      Вопрос 34. Главный бухгалтер назначается (освобождается) на должность

      • Министерством финансов РФ
      • налоговыми органами
      • руководителем организации
      • финансовым директором

      Вопрос 35. Главный бухгалтер несет ответственность за

      • ведение бухгалтерского учета и представление отчетности
      • действия материально ответственных лиц
      • обеспечение организации средствами оргтехники

      Вопрос 36. Двойная запись — это способ

      • группировки объектов учета
      • обобщения данных бухгалтерского учета
      • отражения хозяйственных операций

      Вопрос 37. Действия, связанные с движением активов и пассивов называются

      • активизация
      • хозяйственная деятельность
      • хозяйственные операции

      Вопрос 38. Для осуществления бухгалтерских записей основанием являются документы

      • бухгалтерского оформления
      • первичные
      • распорядительные

      Вопрос 39. Для отражения в учете денежных средств используются счета

      • с 01 по 09
      • с 20 по 29
      • с 50 по 59
      • с 60 по 69

      Вопрос 40. Для следующих случаев проведение инвентаризации обязательно

      • в случаях стихийных бедствий и чрезвычайных ситуаций
      • перед составлением квартальной отчетности
      • при изменении норм расхода материалов

      Сдадим ваш тест на хорошо или отлично

      Пройти тест синергии Бухгалтерское дело

    • … положил начало рассмотрению бухгалтерского учета как орудия управления отдельным предприятием с одной стороны и как универсальной методологической науки с другой

      Зенон Элейский

      Христоф Штехер

      Бенедетто Котрульи

      Лука Пачоли

    • Бухгалтерское дело включает в себя …

      только ведение бухгалтерского учета и составление отчетности, а также контроль и анализ отчетных данных

      только ведение бухгалтерского учета

      ведение учета, составление отчетности, контроль, аудит, анализ отчетных данных, участие в профессиональных организациях и т. д.

    • Сумма превышения доходов над расходами предприятия – это …

    • Слово «бухгалтер» возникло в …

      самом конце Средневековья

      конце эпохи Возрождения

      XVI

      XIX в.

    • Способ экономической группировки имущества по его составу и по источникам его формирования в денежном выражении на определенную дату – это …

      документация

      инвентаризация

      калькуляция

      бухгалтерский баланс

    • Бухгалтерский учет возник в …

      в то время, когда наши далекие предки стали осознавать себя в обществе

      Средние века

      эпоху Возрождения

      середине XIX в.

    • Чистая стоимость имущества, определяемая как разница между стоимостью имущества организации и ее обязательствами, – это … капитал

      уставный

      добавочный

      собственный

      резервный

      заемный

    • Бухгалтерский учет в рф регулируется на …

      двух уровнях – законодательном и методическом

      двух уровнях –законодательном и на уровне самой организации

      трех уровнях – законодательном, нормативном и на уровне самой организации

      четырех уровнях – законодательном, нормативном, методическом и на уровне самой организации

    • В современных условиях сложилось …

      два основных типа организации структуры бухгалтерии: линейно-штабная и функциональная

      два основных типа организации структуры бухгалтерии: иерархическая и линейно-штабная

      два основных типа организации структуры бухгалтерии: линейная и комбинированная

      три основных типа организации структуры бухгалтерии: линейная (иерархическая), по вертикали (линейно-штабная) и комбинированная (функциональная)

    • Указание дебета и кредита счетов, затрагиваемых данной хозяйственной операцией, на которые следует на основании первичных документов отнести стоимостную оценку показателя, характеризующего конкретный хозяйственный объект, – это …

      бухгалтерский баланс

      счет бухгалтерского учета

      дебетовый оборот

      кредитовый оборот

      бухгалтерская проводка

    • Неверно, что … относят к основным элементам и средствам системы организации бухгалтерского учета

      рабочий план счетов бухгалтерского учета

      регистры бухгалтерского учета

      первичные учетные документы

      итоговое обобщение информации

      внутреннюю бухгалтерскую отчетность

      документооборот

    • По степени влияния на финансовое положение хозяйственные операции можно классифицировать на …

      односторонние и двусторонние

      внутренние и внешние

      моментальные, длительные и абсолютные

      фактически проведенные и признанные в отчетном периоде и условные

      правомерные и неправомерные

    • Документы, необходимые для организации учета хозяйственных операций, перечисляют в плане …

      документации

      отчетности

      технического оформления учета

      организации труда и повышения квалификации работников бухгалтерии

      (проекте) корреспонденции счетов рабочего плана счетов

    • Совокупность вкладов учредителей в имущество при создании предприятия в размерах, определенных учредительными документами, – это …

      уставный капитал

      добавочный капитал

      резервный капитал

      прибыль

    • Все хозяйственные операции, происходящие в организации, могут быть отнесены к одному из … типов операций

      двух

      трех

      четырех

      пяти

      шести

    • Определенное направление деятельности, которое имеет к системе бухгалтерского учета прямое отношение, но не всегда влечет за собой хозяйственную операцию, – это …

      хозяйственная ситуация

      факт хозяйственной жизни

      условный факт хозяйственной деятельности

    • Имущество, которое можно использовать многократно в течение длительного (более 12 месяцев) времени без потери им своей физической формы, признается …

      оборотными активами

      внеоборотными активами

      материально-производственными запасами

      средствами в расчетах

    • Первые упоминается о бухгалтерском учете как о науке датируются …

      XII в.

      XV в.

      XIX в.

      началом XX в.

    • Система элементов и средств наиболее оптимального построения учетного процесса с целью получения достоверной, своевременной и полезной для управления информации о деятельности организации, а также для осуществления контроля за эффективностью использования производственных ресурсов – это …

      рациональная организация бухгалтерского учета

      бухгалтерское дело

      бухгалтерский учет

      анализ отчетных данных

    • Международная федерация бухгалтеров (мфб) в … приняла кодекс этики профессиональных бухгалтеров – свод этических требований, правил, предъявляемых к бухгалтерам во всем мире (впоследствии он был пересмотрен)

      1933 г.

      1946 г.

      1974 г.

      1996 г.

    • Проверка документа при его поступлении в бухгалтерию, в ходе которой проверяется законность операции, – это …

      формальная проверка

      проверка по существу

      фактическая проверка

    • Способ группировки затрат и определения себестоимости – это …

      оценка

      инвентаризация

      калькуляция

      бухгалтерский баланс

    • Первая стадия кругооборота капитала – это процесс …

      производства

      реализации

      снабжения

    • Роль бухгалтерского дела в современном обществе …

      переоценить невозможно, т. к. правильное ведение бухгалтерского учета и составление отчетности влияет не только на благополучие предприятия, но и стабильность экономики региона и государства

      высока, но работа бухгалтера важна только для собственников предприятия

      падает, так как современные ИТ-технологии заменяют работу бухгалтеров

    • Отдельное хозяйственное действие (факт), вызывающее изменения в объеме, составе, размещении и использовании средств, а также в составе и назначении источников этих средств – это …

    • Неверно, что в ряду основных требований, предъявляемых к бухгалтерской профессии, – …

      достоверность

      профессионализм

      качество услуг

      объем услуг

      доверие

    • По отношению к контрагенту хозяйственные операции можно классифицировать на …

      односторонние и двусторонние

      внутренние и внешние

      моментальные, длительные и абсолютные

      фактически проведенные и признанные в отчетном периоде и условные

      правомерные и неправомерные

    • Положения по бухгалтерскому учету (пбу) относятся к такому уровню регулирования бухгалтерского учета, как

      законодательный уровень

      нормативный уровень

      методический уровень

      уровень документов, утвержденных в самой организации

    • Тест с ответами на тему: “Бухгалтерская отчетность”

      1. Какие формы бухгалтерской отчетности не включаются в состав годовой бухгалтерской отчетности?
      а) бухгалтерский баланс, отчет о прибылях и убытках и пояснения к ним+
      б) Непредвиденные расходы
      в) Внеплановые расходы

      2. В составе каких расходов при заполнении отчета о прибылях и убытках отражаются расходы, связанные с предоставлением за плату во временное пользование своих активов, не являющихся предметом деятельности организации?
      а) составе прочих расходов+
      б) Расходы первой важности
      в) Покупки

      3. Пользователем бухгалтерской отчетности организации является:
      а) юридическое и физическое лицо, заинтересованное в конкретной информации+
      б) Физическое лицо
      в) Юридическое лицо

      4. По какой статье формы 2 отражается сумма превышения фактической производственной себестоимости выпущенной продукции, если организация использует в текущем учете счет 40 «Выпуск продукции (работ, услуг)»:
      а) Прочие расходы
      б) по статье «Себестоимость проданных товаров, продукции, работ, услуг». +
      в) Продажа услуг иных лиц

      5. Что характеризует Отчет о движении денежных средств?
      а) Изменения в финансовом положении организации в разрезе текущей, инвестиционной и финансовой деятельности.+
      б) Перевод средств
      в) Осуществление платежей

      6. Учетная политика – это:
      а) совокупность способов ведения экономическим субъектом бухгалтерского учета.+
      б) Отчёт за проведённый период финансовой компании
      в) Задолженность

      7. Вправе ли организация представлять в промежуточной отчетности (кроме предусмотренных нормативными документами форм отчетности) иные отчетные показатели, в том числе пояснительную записку:
      а) Да+
      б) Нет
      в) Есть исключения

      8. Объектом статистического наблюдения являются….
      а) некоторое конечное множество единиц наблюдения – элементарных (неделимых) носителей признаков, подлежащих регистрации+
      б) Поток денежных средств
      в) Переход от одной валюты к другой

      9. В пассиве баланса сгруппированы:
      а) источники образования активов+
      б) Расходы за прошлый месяц
      в) Доходы за прошлый месяц

      10. Включается ли в сводную отчетность финансовые вложения головной организации в уставные капиталы дочерних обществ:
      а) Нет+
      б) Да
      в) Зависит от ситуации

      11. Может ли организация после итоговых данных об ее активах, капитале, резервах и обязательствах в бухгалтерском балансе приводить информацию о наличии ценностей, учитываемых на забалансовых счетах:
      а) Да+
      б) Нет
      в) В зависимости от ситуации

      12. Подлежат ли разграничению в отчете о прибылях и убытках доходы и расходы организации с разделением на обычные и прочие:
      а) Да+
      б) Нет
      в) Есть исключения

      13. Отчетной датой для составления бухгалтерской отчетности считается:
      а) последний календарный день отчетного периода+
      б) Первый календарный день
      в) За 10 дней до окончания месяца

      14. Может ли экономический субъект самостоятельно формировать свою учетную политику
      а) Да, руководствуясь законодательством Российской Федерации о бухгалтерском учете+
      б) Нет

      15. Собственным капиталом организации являются:
      а) уставный, добавочный, резервный капиталы, нераспределенная прибыль+
      б) Уставный и добавочный
      в) Резервный капитал

      16. Сумма уценки НМА в бухгалтерском учете относится:
      а) на финансовый результат в составе прочих расходов+
      б) На финансовом результате в составе прочих доходов+
      в) На всех доходах и расходах

      17. Имущество, приобретенное за плату, в бухгалтерском балансе отражается:
      а) в сумме фактических затрат на приобретение+
      б) в прибыли
      в) в разнице между доходами и расходами

      18. Возможно ли в бухгалтерском балансе или отчете о прибылях и убытках организации отражение показателей об отдельных видах ее активов, обязательств, доходов, расходов и хозяйственных операций общей суммой, если каждый из них несущественен для заинтересованных пользователей при оценке финансового положения и результатов деятельности организации:
      а) да, при условии раскрытия информации об отдельных видах активов, обязательствах, доходах и расходах и хозяйственных операциях организации в приложениях к бухгалтерскому балансу и отчету о прибылях и убытках.+
      б) нет

      19. Бухгалтерская отчетность представляет:
      а) единую систему данных об имущественном и финансовом положении организации и результатах ее хозяйственной деятельности, составляемую на основе данных бухгалтерского учета по установленным формам+
      б) все непредвиденные расходы и дополнительные доходы
      в) Доходную часть предприятия

      20. Не является объектом бухгалтерского учета:
      а) бизнес-план+
      б) Медиаплан+
      в) Контент-план+

      21. Имеют ли право некоммерческие организации не представлять в составе годовой бухгалтерской отчетности отчет о движении денежных средств:
      а) имеют при отсутствии соответствующих данных+
      б) не имеют
      в) всегда да

      22. Минфин РФ применяет ПБУ 4/99 при установлении:
      а) типовых форм бухгалтерской отчетности и инструкции о порядке составления отчетности+
      б) при условии, что доходы превышают расходы
      в) при условии, что расходы превышают доходы

      Опрос и тест по принципам бухгалтерского учета

    • 1.

      Личные активы владельца компании , а не появятся на балансе компании, в силу какого принципа/руководства?

      Стоимость

      Неправильно.

      Принцип затрат связан с балансовой стоимостью бизнес-активов.

      Хозяйственный субъект

      Правильно!

      Активы владельца , а не показаны в балансе предприятия. Это верно даже в том случае, если предприятие является индивидуальным предпринимателем.

      Денежная единица

      Неправильно.

      Денежная единица предполагает выражение сумм в долларах и предположение, что покупательная способность доллара не меняется (нет инфляции).

    • 2.

      Какой принцип/рекомендация требует, чтобы в балансовом отчете компании земля отражалась в сумме, которую компания заплатила за приобретение земли, даже если сегодня земля может быть продана по значительно более высокой цене?

      Стоимость

      Верно!

      Принцип затрат требует, чтобы бухгалтер отражал активы по себестоимости и расходы по себестоимости, а не в более высоких суммах. Бухгалтерам не разрешается признавать прибыль от простого владения землей. Чтобы иметь возможность признать прибыль от земли, компания должна продать землю.

      Хозяйственный субъект

      Неправильно.

      Допущение об экономической сущности предполагает отделение личных операций владельца от деловых операций.

      Денежная единица

      Неправильно.

      Лучшим ответом является принцип затрат. Удержание суммы актива по себестоимости обусловлено принципом затрат. Предположение о денежной единице состоит в том, что доллар стабилен с течением времени — инфляция отсутствует.

    • 3.

      Какой принцип/рекомендация позволяет компании игнорировать изменение покупательной способности доллара с течением времени?

      Стоимость

      Неправильно.

      Лучшим ответом будет предположение о денежной единице. Принцип затрат требует, чтобы бухгалтеры регистрировали операции по себестоимости (объективная оценка) и сохраняли актив по себестоимости.

      Хозяйственный субъект

      Неправильно.

      Допущение об экономической сущности предполагает отделение личных операций владельца от деловых операций.

      Денежная единица

      Верно!

      Предположение о денежной единице состоит в том, что доллар стабилен во времени — инфляция отсутствует.

    • 4.

      Какой принцип/рекомендация требует наличия в финансовой отчетности компании сносок, содержащих информацию, важную для пользователей финансовой отчетности?

      Консерватизм

      Неправильно.

      Консерватизм предполагает выбор между приемлемыми альтернативами. Другими словами, консерватизм используется, чтобы разорвать связь между двумя приемлемыми вариантами того, как объяснить что-то. Это также связано с признанием потерь, но не доходов в определенных ситуациях.

      Хозяйственный субъект

      Неправильно.

      Допущение об экономической сущности предполагает отделение личных операций владельца от деловых операций.

      Полное раскрытие информации

      Верно!

      Принцип полного раскрытия информации требует, чтобы предприятия раскрывали информацию, имеющую отношение к решениям инвесторов и кредиторов.

    • 5.

      Какой принцип/рекомендация оправдывает нарушение компанией принципа бухгалтерского учета, поскольку суммы несущественны?

      Консерватизм

      Неправильно.

      Консерватизм предполагает выбор между приемлемыми альтернативами. Другими словами, консерватизм используется, чтобы разорвать связь между двумя приемлемыми вариантами того, как объяснить что-то. Это также связано с признанием потерь, но не доходов в определенных ситуациях.

      Полное раскрытие информации

      Неправильно.

      Принцип полного раскрытия информации требует, чтобы предприятия раскрывали информацию, имеющую отношение к решениям инвесторов и кредиторов.

      Материальность

      Верно!

      Когда сумма настолько мала/несущественна, бухгалтер может принять решение игнорировать принцип бухгалтерского учета. Например, крупная компания может приобрести цифровую камеру за 300 долларов, которая будет использоваться в течение следующих пяти лет. Принцип соответствия потребует расходов (амортизации) в размере 60 долларов в год в течение пяти лет. Большинство бухгалтеров нарушили бы принцип соответствия и потратили бы все 300 долларов в год приобретения. Обоснование состоит в том, что лица, принимающие решения, не будут введены в заблуждение небольшой разницей в 240 долларов в год покупки и 60 долларов в год в каждый из следующих четырех лет.

    • 6.

      Какой принцип/рекомендация связана с предположением о том, что компания будет существовать достаточно долго, чтобы выполнять свои цели и обязательства?

      Хозяйственный субъект

      Неправильно.

      Допущение об экономической сущности предполагает отделение личных операций владельца от деловых операций.

      Непрерывная деятельность

      Верно!

      Это правильный ответ.

      Период времени

      Неправильно.

      Допущение о периоде времени (также известное как периодичность) — это предположение о том, что текущую деловую активность можно разделить на периоды времени в год, месяц, неделю и т. д.

    • 7.

      В финансовых отчетах очень крупной корпорации суммы в долларах округлены до ближайшей 1000 долларов. Какой принцип/рекомендация бухгалтерского учета оправдывает непредставление сумм с точностью до копейки?

      Полное раскрытие информации

      Неправильно.

      Полное раскрытие информации связано с надлежащим представлением информации, которая имеет отношение к читателям финансовой отчетности. Часто это осуществляется в примечаниях (сносках) к финансовой отчетности.

      Материальность

      Верно!

      Пока пропущенные цифры малы по сравнению с реальными суммами, компании будут округлять числа, чтобы подчеркнуть соответствующие цифры. Смысл в том, что пропуск незначащих цифр никого не введет в заблуждение.

      Денежная единица

      Неправильно.

      В США денежная единица связана с отчетностью обо всех товарах в долларах США и с тем, что покупательная способность доллара США не меняется с течением времени.

    • 8.

      Бухгалтеры могут признавать убытки, но не доходы в определенных ситуациях. Например, компания может списать стоимость запасов, но не будет увеличивать стоимость запасов. Какой принцип/рекомендация связана с этим действием?

      Консерватизм

      Верно!

      Консерватизм предполагает выбор между приемлемыми альтернативами. Другими словами, консерватизм используется, чтобы разорвать связь между двумя приемлемыми вариантами того, как объяснить что-то. Это также связано с признанием потерь, но не доходов в определенных ситуациях.

      Существенность

      Неправильно.

      Существенность включает незначительные суммы. Например, многие компании представляют свои финансовые отчеты в тысячах долларов. Причина в том, что копейки, доллары и сотни долларов не имеют значения для лиц, принимающих решения, использующих финансовые отчеты. Другим примером является немедленное списание в расход степлера стоимостью 15 долларов вместо амортизации степлера в течение срока его полезного использования, равного 5 годам.

      Денежная единица

      Неправильно.

      Денежная единица предполагает выражение сумм в долларах и предположение, что покупательная способность доллара не меняется (нет инфляции).

    • 9.

      Какой принцип/рекомендация предписывает компании показывать все расходы, связанные с ее доходами за определенный период, даже если расходы не были оплачены в этот период?

      Стоимость

      Неправильно.

      Принцип затрат требует, чтобы бухгалтер отражал активы и расходы по себестоимости, а не по более высоким суммам.

      Соответствие

      Верно!

      Это правильный ответ.

      Денежная единица

      Неправильно.

      Денежная единица предполагает выражение сумм в долларах и предположение, что покупательная способность доллара не меняется (нет инфляции).

    • 10.

      Когда бухгалтеру приходится выбирать между двумя приемлемыми альтернативами, бухгалтер должен выбрать альтернативу, которая будет отражать меньшую прибыль, меньшую сумму активов или большую сумму обязательств. На каком принципе/рекомендации это основано?

      Консерватизм

      Верно!

      Консерватизм используется для того, чтобы «разорвать ничью». Бухгалтеры должны стремиться быть объективными и проявлять консерватизм, когда существуют сомнения между двумя вариантами.

      Стоимость

      Неверно.

      Себестоимость включает учет операций по их денежной стоимости на момент совершения операции.

      Существенность

      Неправильно.

      Существенность включает незначительные суммы и учет этих сумм.

    • 11.

      В балансах предприятий коммунального хозяйства активы заводов указаны перед текущими активами. В соответствии с каким принципом/рекомендацией бухгалтерского учета это приемлемо?

      Консерватизм

      Неправильно.

      Стоимость

      Неверно.

      Промышленная практика

      Верно!

      Некоторые отрасли (обычно те, которые регулируются государством) имеют уникальные требования к отчетности, которые соблюдаются в финансовой отчетности, а также в отчетах правительству.

    • 12.

      Крупная компания покупает цифровую камеру за 250 долларов и немедленно тратит ее на расходы вместо того, чтобы учитывать ее как актив и амортизировать в течение срока ее полезного использования. Эта практика может быть приемлемой, из-за какого принципа/рекомендации?

      Стоимость

      Неправильно.

      Себестоимость включает учет операций по их денежной стоимости на момент совершения операции.

      Соответствует

      Неправильно.

      Принцип сопоставления будет подчеркивать амортизацию, например, «сопоставление» 50 долларов Амортизационных расходов каждый год в течение пяти лет с доходами за эти пять лет.

      Материальность

      Верно!

      Поскольку это крупная компания, 250 долларов считаются незначительной суммой. Следовательно, если компания амортизирует камеру по 50 долларов в год в течение пяти лет или тратит 250 долларов в год ее покупки, инвестора или кредитора не введут в заблуждение дополнительные расходы в размере 200 долларов в год в первый год и разница в 50 долларов в год. следующие четыре года.

    • 13.

      Корпорация оплачивает свой ежегодный счет по налогу на имущество в размере приблизительно 12 000 долларов США одним платежом 28 декабря. В течение года ежемесячные отчеты о доходах корпорации показывают расходы по налогу на имущество в размере 1 000 долларов США. Это пример какого принципа/рекомендации бухгалтерского учета?

      Консерватизм

      Неправильно.

      Консерватизм требует, чтобы при наличии двух приемлемых вариантов бухгалтерского учета бухгалтер выбрал вариант, который приводит к меньшим активам, меньшей прибыли или большей сумме обязательств, чтобы «разорвать связь» между вариантами.

      Соответствие

      Верно!

      Принцип согласования требует, чтобы компания уплачивала 1/12 годового налога на недвижимость за каждый месяц, когда доходы получены от собственности.

      Денежная единица

      Неправильно.

      В США денежная единица связана с отчетностью обо всех товарах в долларах США и с тем, что покупательная способность доллара США не меняется с течением времени.

    • 14.

      В декабре компания продала покупателю товаров на 8000 долларов. Условия продажи компании требуют, чтобы клиент заплатил компании в течение 30 дней. В отчете о прибылях и убытках компании сообщается о продаже в декабре. В соответствии с каким принципом/рекомендацией бухгалтерского учета это является правильным?

      Полное раскрытие информации

      Неверно.

      Полное раскрытие включает передачу информации, чтобы читатели финансовой отчетности могли принимать обоснованные решения.

      Денежная единица

      Неправильно.

      В США денежная единица связана с отчетностью обо всех товарах в долларах США и с тем, что покупательная способность доллара США не меняется с течением времени.

      Признание доходов

      Верно!

      Принцип признания выручки требует, чтобы выручка отражалась в момент получения выручки (когда продаются товары или предоставляются услуги), а не в момент получения платежа.

    • 15.

      Учет по методу начисления основан на этом принципе/рекомендации.

      Стоимость

      Неправильно.

      Себестоимость включает учет операций по их денежной стоимости на момент совершения операции.

      Полное раскрытие информации

      Неправильно.

      Полное раскрытие включает передачу информации, чтобы читатели финансовой отчетности могли принимать обоснованные решения.

      Соответствие

      Верно!

      Принцип сопоставления требует, чтобы расходы сопоставлялись с соответствующими доходами или с отчетным периодом, когда были понесены расходы. Когда расходы оплачены, значения не имеет.

    • 16.

      Творческий исполнительный директор корпорации, лично ответственный за многочисленные изобретения и инновации, не указывается в качестве актива на балансе корпорации. Принцип/рекомендация бухгалтерского учета, запрещающая корпорации указывать это лицо в качестве актива,

      Консерватизм

      Неправильно.

      Консерватизм требует, чтобы при наличии двух приемлемых вариантов бухгалтерского учета бухгалтер выбрал вариант, который приводит к меньшим активам, меньшей прибыли или большей сумме обязательств, чтобы «разорвать связь» между вариантами.

      Стоимость

      Верно!

      Принцип затрат требует, чтобы активы и другие операции учитывались по себестоимости. Исполнительный директор не был приобретен по себестоимости и поэтому не отражается в качестве актива на балансе корпорации. Предположение о денежной единице также является еще одной причиной, по которой исполнительный директор не регистрируется — мы не знаем, как измерить исполнительного директора в долларах США.

      Непрерывность деятельности

      Неправильно.

      Непрерывность деятельности — это предположение о том, что компания будет продолжать свою деятельность достаточно долго, чтобы выполнять свои цели и обязательства.

    • 17.

      Актив стоимостью 120 000 долл. США амортизируется в течение срока его полезного использования, равного 10 годам, вместо того, чтобы списывать на расходы всю сумму при его покупке. Это соответствует какому принципу/рекомендации?

      Стоимость

      Неправильно.

      Себестоимость включает учет операций по себестоимости или денежной стоимости на момент совершения операции.

      Полное раскрытие информации

      Неправильно.

      Полное раскрытие включает передачу информации, чтобы читатели финансовой отчетности могли принимать обоснованные решения.

      Соответствие

      Верно!

      Принцип сопоставления требует, чтобы расходы сопоставлялись с соответствующими доходами или с отчетным периодом, когда были понесены расходы. Когда расходы оплачены, значения не имеет.

    • 18.

      Ближе к концу текущего года компания потребовала от клиента внести 200 000 долларов США в качестве залога за работу, которая должна начаться в следующем году. В конце текущего года компания отразила сумму в размере 200 000 долларов США в качестве обязательства в своем балансе. Какой принцип/руководство бухгалтерского учета помешал компании указать 200 000 долларов США в своем отчете о прибылях и убытках за текущий год?

      Непрерывность деятельности

      Неправильно.

      Непрерывность деятельности — это предположение о том, что компания будет продолжать свою деятельность достаточно долго, чтобы выполнять свои цели и обязательства.

      Существенность

      Неправильно.

      Существенность включает незначительные суммы и учет этих сумм.

      Признание доходов

      Верно!

      Принцип признания выручки требует, чтобы выручка признавалась в момент ее получения, а не в момент получения денежных средств.

    • 19.

      Предприятие розничной торговли желает отразить запасы товаров в своем балансе по их розничной стоимости. Какой принцип/рекомендация бухгалтерского учета будет нарушена?

      Стоимость

      Верно!

      Принцип затрат требует, чтобы активы учитывались по себестоимости на момент их приобретения. Принцип затрат запрещает увеличивать стоимость товаров в запасах до того, как они будут проданы.

      Полное раскрытие информации

      Неправильно.

      Полное раскрытие включает передачу информации, чтобы читатели финансовой отчетности могли принимать обоснованные решения.

      Денежная единица

      Неправильно.

      В США денежная единица связана с отчетностью обо всех товарах в долларах США и с тем, что покупательная способность доллара США не меняется с течением времени.

    • 20.

      Компания заняла 100 000 долларов в декабре и произведет единственную выплату процентов, когда срок погашения векселя наступит через шесть месяцев. Общая сумма процентов за шесть месяцев составит 3600 долларов. В отчете о прибылях и убытках за декабрь бухгалтер сообщил о расходах по процентам в размере 600 долларов. Это действие было результатом применения какого принципа/рекомендации бухгалтерского учета?

      Стоимость

      Неправильно.

      Себестоимость включает учет операций по себестоимости или денежной стоимости на момент совершения операции.

      Соответствие

      Верно!

      Принцип сопоставления требует, чтобы расходы сопоставлялись с соответствующими доходами или с соответствующим периодом времени. В этом случае компания несет процентные расходы каждую минуту, что у нее есть кредит. За один месяц использования денег у компании есть процентные расходы в размере 600 долларов, и об этом необходимо сообщить в отчете о прибылях и убытках за декабрь, чтобы соответствовать принципу соответствия и методу начисления в бухгалтерском учете.

      Признание доходов

      Неправильно.

      Принцип признания выручки требует, чтобы выручка отражалась в момент получения выручки (когда продаются товары или предоставляются услуги), а не в момент получения платежа.

    • Дебет и кредит Викторина и тест


      1. Используйте следующую информацию для ответов на вопросы 1 и 2:
        Компания получает 500 долларов наличными в качестве дополнительных инвестиций в компанию от ее владельца, Мэри Смит. Компания Увеличен счет Cash и Mary Smith, Capital увеличен.

      2. 1.

        Должна ли запись в размере 500 долларов США на кассовый счет быть дебетовой ?

        Да

        Верно!

        Наличные всегда списывается при получении наличных.

        Нет

        Неправильно.

        Помните, что при получении наличных со счета Наличные ДЕБЕТОВАЕТСЯ. Также помните, что мы дебетуем активные счета (кроме счетов контрасных активов), чтобы увеличить их нормальный дебетовый баланс.

      3. 2.

        Должна ли запись на сумму 500 долларов в Mary Smith, Capital быть дебетовой ?

        Да

        Неправильно.

        Акционерный счет владельца, Mary Smith, Capital , должен быть ЗАЧЕТ. Этому есть две причины. Одна из причин заключается в том, что счет Cash был дебетован (поскольку компания получила наличные). Поэтому другой частью сделки должен быть кредит.

        Вторая причина — нормальный баланс на Мэри Смит, Капитал — это кредитовый баланс, и чтобы увеличить его баланс, нам нужно ЗАПИСАТЬСЯ на счет. Напомним, что счет собственного капитала, Mary Smith, Capital , находится на правой или кредитной стороне уравнения бухгалтерского учета, и поэтому его баланс обычно является кредитовым балансом.

        Нет

        Верно!

        Акционерный счет этого владельца должен быть кредитован, а не дебетован.

      4. Используйте следующую информацию для ответов на вопросы с 3 по 6:
        Предприятие, использующее метод учета по методу начисления, в августе оказало услуги на расчетный счет. Услуги стоили 2000 долларов, и компания предоставила клиенту условия кредита, в которых указано, что сумма должна быть выплачена компании в сентябре.

      5. 3.

        Если предположить, что компания составляет ежемесячные отчеты о прибылях и убытках, какой будет сумма счета , дебетованная на сумму 2000 долларов в августе ?

        Наличные

        Неправильно.

        Правильный ответ: Дебиторская задолженность . Слова «на счету» и «условия кредита» указывают на то, что наличные деньги были , а не получены.

        Дебиторская задолженность

        Верно!

        Счет Дебиторская задолженность должен быть дебетован в августе. Дебиторская задолженность , актив, был увеличен в августе.

        Доход от услуг

        Неправильно.

        Дебетуемый счет: Дебиторская задолженность . Счет Доходы от услуг должны быть ЗАЧЕТНЫ в августе. Очень редко какой-либо счет доходов будет дебетован.

      6. 4.

        На какой счет компания должна зачислить 2000 долларов в августе ?

        Наличные

        Неправильно.

        Денежные средства в августе не поступали. Кроме того, наличные зачисляются, когда компания ПЛАТИТ наличными.

        Дебиторская задолженность

        Неправильно.

        Вы должны иметь ДЕБЕТИРОВАНИЕ Дебиторская задолженность в августе. Кредит Дебиторская задолженность уменьшил бы остаток на счете, чего не было в августе.

        Доход от услуг

        Верно!

        Вы должны кредитовать доходы, когда они получены.

      7. 5.

        В сентябре , когда компания получает 2000 долларов от клиента, с какого счета компания должна дебетовать ?

        Наличные

        Верно!

        Мы всегда дебетуем Наличные при получении наличных.

        Дебиторская задолженность

        Неправильно.

        Правильный ответ: дебетовать Денежные средства , так как деньги были получены. Дебиторская задолженность должна быть КРЕДИТОВАНА, так как этот актив уменьшается, когда компания взыскивает свою дебиторскую задолженность. (Дебет Дебиторская задолженность или любой другой актив увеличит остаток на счете.)

        Доход от услуг

        Неправильно.

        В сентябре выручки нет. Доход был получен и признан в августе. Кроме того, счет «Доход от услуг» почти всегда КРЕДИТИРОВАН — редко дебетуется.

      8. 6.

        В сентябре , когда компания получает 2000 долларов от клиента, на какой счет компания должна кредитовать ?

        Наличные

        Неправильно.

        Когда наличные деньги получены, счет Cash будет ДЕБЕТИРОВАН, а не кредитован.

        Дебиторская задолженность

        Верно!

        Когда актив, такой как Дебиторская задолженность , уменьшается, вы кредитуете счет.

        Доход от услуг

        Неправильно.

        Доход от услуги был зачислен в августе, когда услуга была оказана. Вы не можете признавать выручку дважды за одну и ту же услугу. В сентябре компания просто собирает дебиторскую задолженность. Это означает, что кредит должен быть Дебиторская задолженность .

      9. 7. Чтобы увеличить остаток на следующих счетах, вы дебетуете или кредитуете счет?

        Кредиторская задолженность

        Дебет

        Неправильно.

        Кредиторская задолженность — счет пассивов. Счета пассивов имеют кредитовые остатки, и для увеличения баланса вам необходимо КРЕДИТировать счет.

        Кредит

        Верно!

        Чтобы увеличить пассив, вы кредитуете счет пассива.

        Наличные

        Дебет

        Верно!

        Денежные средства и прочие активы списываются в счет увеличения их остатков.

        Кредит

        Неправильно.

        Кредит уменьшит Денежный баланс счета (или любого счета активов).

        Земля

        Дебет

        Верно!

        Поскольку земля является активом, вы дебетуете счет, чтобы увеличить его баланс.

        Кредит

        Неправильно.

        Правильный ответ — дебет. Поскольку земля является активом, вам необходимо ДЕБЕТОВАТЬ счет Land , чтобы увеличить его баланс.

        Векселя к оплате

        Дебет

        Неправильно.

        Так как Векселя к оплате является пассивным счетом, вам необходимо ПОПОЛНИТЬ счет, чтобы увеличить его.

        Кредит

        Верно!

        Как и в случае любого пассивного счета, вы кредитуете счет, чтобы увеличить его баланс.

        Дебиторская задолженность

        Дебет

        Верно!

        Дебиторская задолженность является активом, и дебет увеличит счет актива.

        Кредит

        Неправильно.

        Кредит УМЕНЬШИТ такой актив, как Дебиторская задолженность . Для увеличения Дебиторская задолженность необходимо дебетовать счет.

        Мэри Смит, Столица

        Дебет

        Неправильно.

        Мэри Смит, Капитал является счетом собственного капитала, и его обычное сальдо является кредитовым сальдо. Поэтому, чтобы увеличить счет, вам нужно ЗАПИСАТЬСЯ на него.

        Кредит

        Верно!

        Мэри Смит, Капитал является счетом собственного капитала с нормальным кредитовым балансом. Следовательно, вы кредитуете счет, чтобы увеличить его баланс.

        Расходные материалы

        Дебет

        Верно!

        Расходные материалы — счет активов. Счета активов обычно имеют дебетовые остатки и дебетуются для увеличения их остатков.

        Кредит

        Неправильно.

        Расходные материалы — счет активов. Счета активов обычно имеют дебетовые остатки и дебетуются для увеличения их остатков.

        Расходы на расходные материалы

        Дебет

        Верно!

        Расходы на поставки должны быть дебетованы. За исключением особых ситуаций (корректирующие проводки, закрывающие проводки и некоторые корректирующие проводки) расходы всегда дебетуются.

        Кредит

        Неправильно.

        Расходы на поставки должны быть дебетованы. За исключением особых ситуаций (корректирующие проводки, закрывающие проводки и некоторые корректирующие проводки) расходы всегда дебетуются.

        Предоплаченная страховка

        Дебет

        Верно!

        Предоплаченное страхование является активом, и активы увеличиваются по дебету.

        Кредит

        Неправильно.

        Предоплаченное страхование является активом, и активы увеличиваются по ДЕБЕТУ. При предоплате страховки вы платите наличными. Это означает, что в записи должен быть CREDIT Денежные средства . Это, в свою очередь, означает, что вам нужно будет ДЕБЕТОВАТЬ счет — и счет будет Prepaid Insurance .

        Доход от услуг

        Дебет

        Неправильно.

        Счета доходов почти всегда ЗАЧЕТНЫ. Исключениями являются: корректирующие, закрывающие и некоторые корректирующие записи. Подумайте об оказании услуги за наличные. Вы бы дебетовали Наличные , потому что вы получили наличные, и вам нужно было бы кредитовать счет из-за двойной записи. Поскольку вы зарабатываете деньги, оказывая услугу, вы должны кредитовать счет доходов. Доходы также имеют эффект увеличения собственного капитала, который обычно имеет кредитовое сальдо.

        Кредит

        Верно!

        Счета доходов почти всегда ЗАЧЕТНЫ. (Исключениями являются: исправление, закрытие и некоторые корректирующие проводки.) Доходы имеют эффект увеличения собственного капитала, поэтому для доходов разумно иметь кредитовый баланс.

        Мэри Смит, Рисование

        Дебет

        Верно!

        Счет заимствования обычно имеет дебетовое сальдо и должно дебетоваться, когда владелец выводит активы из бизнеса для личного использования. Вы также можете визуализировать запись журнала. Когда владелец выводит деньги из бизнеса, бизнес получает КРЕДИТ 9.0004 Денежные средства . Это означает, что другой задействованный счет должен быть дебетован. Мэри Смит, рисунок — контрсчет собственного капитала владельца.

        Кредит

        Неправильно.

        Счет заимствования обычно имеет дебетовое сальдо и должно дебетоваться, когда владелец выводит активы из бизнеса для личного использования. Вы также можете визуализировать запись журнала. Когда владелец вытягивает деньги из бизнеса, бизнес будет КРЕДИТ наличными. Это означает, что другой задействованный счет должен быть дебетован. Мэри Смит, Рисование — это счет капитала контра-владельца.

        Оборудование

        Дебет

        Верно!

        Оборудование является активом и должен быть дебетован для увеличения баланса счета.

        Кредит

        Неправильно.

        Кредит уменьшит баланс счета активов. Чтобы увеличить баланс счета актива, вам необходимо дебетовать счет.

        Незаработанный доход

        Дебет

        Неправильно.

        Незаработанный доход — счет обязательств. Дебет уменьшит пассив счета. Вы хотите кредитовать пассивный счет, чтобы увеличить его.

        Кредит

        Верно!

        Поскольку Незаработанный доход является пассивным счетом, кредит увеличит его баланс.

      10. 8. Чтобы уменьшить остаток на следующих счетах, вы дебетуете или кредитуете счет?

        Кредиторская задолженность

        Дебет

        Верно!

        Кредиторская задолженность — счет пассивов. Чтобы уменьшить счет пассива, вы дебетуете счет.

        Кредит

        Неправильно.

        Кредиторская задолженность — счет пассивов. Счета обязательств имеют кредитовое сальдо, и для уменьшения сальдо необходимо ДЕБЕТОВАТЬ счет. (Если вы должны были погасить обязательство, вам пришлось бы кредитовать Денежные средства , поэтому запись на счет пассива должна быть дебетовой.)

        Наличные

        Дебет

        Неправильно.

        Дебет УВЕЛИЧИТ баланс счета Cash (или любого счета активов). КРЕДИТ уменьшит Денежный счет.

        Кредит

        Верно!

        Кредит уменьшит баланс денежного счета (или любого счета активов).

        Земля

        Дебет

        Неправильно.

        Правильный ответ — кредит. Поскольку земля является активом, вам необходимо ЗАПОЛНИТЬ счет Land , чтобы уменьшить его баланс.

        Кредит

        Верно!

        Поскольку земля является активом, вы кредитуете счет Земля , чтобы уменьшить его баланс.

        Векселя к оплате

        Дебет

        Верно!

        Как и в случае любого пассивного счета, вы дебетуете счет Векселя к оплате , чтобы уменьшить его остаток.

        Кредит

        Неправильно.

        Поскольку счет Векселя к оплате является пассивным счетом с нормальным кредитовым сальдо, для уменьшения сальдо счета требуется ДЕБЕТ.

        Дебиторская задолженность

        Дебет

        Неправильно.

        Дебиторская задолженность является активом, и КРЕДИТ необходим для уменьшения нормального дебетового сальдо.

        Кредит

        Верно!

        Кредит уменьшит такой актив, как Дебиторская задолженность .

        Мэри Смит, Столица

        Дебет

        Верно!

        Мэри Смит, Капитал является счетом собственного капитала с нормальным кредитовым балансом. Следовательно, вы дебетуете счет, чтобы уменьшить его баланс.

        Кредит

        Неправильно.

        Мэри Смит, Капитал является счетом собственного капитала с нормальным кредитовым балансом. Если вы пополняете счет, вы увеличиваете его баланс. ДЕБЕТ уменьшит баланс.

        Расходные материалы

        Дебет

        Неправильно.

        Расходные материалы — счет активов. Счета активов обычно имеют дебетовые остатки, а дебет увеличивает остатки активов. Вы должны НАЧАТЬ актив, чтобы уменьшить баланс актива.

        Кредит

        Верно!

        Поскольку Расходные материалы является счетом актива, он будет уменьшен за счет кредита.

        Расходы на расходные материалы

        Дебет

        Неправильно.

        Дебет УВЕЛИЧИТСЯ Расходы на поставки . Очень необычно, что предыдущие расходы, уже зарегистрированные в счете расходов, будут уменьшены. Тем не менее, КРЕДИТ уменьшит нормальное дебетовое сальдо расходов.

        Кредит

        Верно!

        Очень необычно, что предыдущие расходы, уже зарегистрированные в счете расходов, будут уменьшены. Однако кредит уменьшит нормальное дебетовое сальдо расходов.

        Предоплаченная страховка

        Дебет

        Неправильно.

        Предоплаченная страховка является активом, и для уменьшения актива вам необходимо ЗАЧЕТНУТЬ счет.

        Кредит

        Верно!

        Предоплаченная страховка является активом и будет уменьшена с кредитом.

        Доход от услуг

        Дебет

        Верно!

        Поскольку счета доходов имеют кредитовые сальдо, вы правильно указываете, что дебетование уменьшит сальдо.

        Кредит

        Неправильно.

        Счета доходов имеют кредитовое сальдо, поэтому кредитование счета доходов УВЕЛИЧИТ баланс.

        Мэри Смит, Рисование

        Дебет

        Неправильно.

        Дебет на расчетный счет увеличит (а не уменьшит) остаток в Mary Smith, Drawing . (Поскольку этот счет для получения средств является противоположным счету собственного капитала владельца, дебетование приведет к уменьшению собственного капитала владельца. Однако дебетование приведет к увеличению баланса счета для получения.) учетная запись.

        Кредит

        Верно!

        Поскольку Мэри Смит, рисунок является счетом акционерного капитала против собственника с дебетовым сальдо, вы правильно указываете, что кредит необходим для уменьшения сальдо.

        Оборудование

        Дебет

        Неправильно.

        Оборудование является активом, и дебетование увеличит баланс счета. Вам придется КРЕДИТНОЕ оборудование, чтобы уменьшить его баланс.

        Кредит

        Верно!

        Кредит уменьшит баланс счета этого актива.

        Незаработанный доход

        Дебет

        Верно!

        Незаработанный доход является пассивным счетом, и его остаток будет уменьшен по дебету.

        Кредит

        Неправильно.

        Поскольку Незаработанный доход является пассивным счетом, вам необходимо ДЕБЕТОВАТЬ этот счет, чтобы уменьшить его остаток.

      11. 9. Каков нормальный баланс для следующих счетов?

        Кредиторская задолженность

        Дебет

        Неправильно.

        Кредиторская задолженность — счет пассивов. Счета пассивов имеют КРЕДИТНЫЕ остатки. Обязательства находятся в правой или кредитной части уравнения бухгалтерского учета.

        Кредит

        Верно!

        Пассивные счета обычно имеют кредитовое сальдо.

        Наличные

        Дебет

        Верно!

        Денежные средства и прочие активы имеют дебетовые остатки.

        Кредит

        Неправильно.

        Денежные средства и прочие активы имеют ДЕБЕТОВЫЕ остатки. Активы находятся в левой или дебетовой части уравнения бухгалтерского учета.

        Земля

        Дебет

        Верно!

        Поскольку Земля является активом, то ее нормальный баланс является дебетовым.

        Кредит

        Неправильно.

        Правильный ответ — дебет. Поскольку Земля является активом, ее нормальный баланс является ДЕБЕТНЫМ.

        Векселя к оплате

        Дебет

        Неправильно.

        С Векселя к оплате — это пассивный счет, его остаток обычно является КРЕДИТНЫМ.

        Кредит

        Верно!

        Пассивные счета обычно имеют кредитовое сальдо.

        Дебиторская задолженность

        Дебет

        Верно!

        Дебиторская задолженность является активом. Поэтому его нормальное сальдо является дебетовым сальдо.

        Кредит

        Неправильно.

        Дебиторская задолженность является активом. Следовательно, его нормальный баланс является ДЕБЕТНЫМ балансом.

        Мэри Смит, Столица

        Дебет

        Неправильно.

        Мэри Смит, Капитал является счетом собственного капитала, и его обычный баланс является КРЕДИТНЫМ балансом.

        Кредит

        Верно!

        Мэри Смит, Капитал является счетом собственного капитала с нормальным кредитовым балансом.

        Расходные материалы

        Дебет

        Верно!

        Расходные материалы — счет активов. Активные счета обычно имеют дебетовые остатки.

        Кредит

        Неправильно.

        Расходные материалы — счет активов. Активные счета обычно имеют дебетовых остатков .

        Расходы на расходные материалы

        Дебет

        Верно!

        Расходы на поставки (и все расходы) обычно должны иметь дебетовое сальдо.

        Кредит

        Неправильно.

        Расходы на поставки (и все расходы) обычно должны иметь ДЕБЕТОВОЕ сальдо.»

        Предоплаченная страховка

        Дебет

        Верно!

        Предоплаченное страхование является активом, и активы обычно имеют дебетовые остатки.

        Кредит

        Неправильно.

        Предоплаченное страхование является активом, и активы обычно имеют дебетовые остатки.

        Доход от услуг

        Дебет

        Неправильно.

        Счета доходов обычно имеют КРЕДИТНЫЕ остатки. (Выручка приведет к увеличению собственного капитала, а собственный капитал обычно имеет кредитовое сальдо.)

        Кредит

        Верно!

        Счета доходов обычно имеют КРЕДИТНЫЕ остатки. (Доходы приводят к увеличению собственного капитала, а собственный капитал обычно имеет кредитовое сальдо.)

        Мэри Смит, Рисование

        Дебет

        Верно!

        Счет заимствования обычно имеет дебетовое сальдо и должно дебетоваться, когда владелец выводит активы из бизнеса для личного использования.

        Кредит

        Неправильно.

        Счет заимствования обычно имеет дебетовое сальдо и должно дебетоваться, когда владелец выводит активы из бизнеса для личного использования.

        Оборудование

        Дебет

        Верно!

        Оборудование является активом и поэтому обычно имеет дебетовое сальдо.

        Кредит

        Неправильно.

        Оборудование является активом и поэтому обычно имеет ДЕБЕТОВОЕ сальдо.

        Незаработанный доход

        Дебет

        Неправильно.

        Незаработанный доход — счет обязательств. В результате нормальный баланс этого счета является КРЕДИТОМ.

        Кредит

        Верно!

        Поскольку Незаработанный доход является пассивным счетом, его обычное сальдо является кредитовым сальдо.

      12. 10.

        Как правило, когда в транзакции участвуют расходы, счет расходов будет __________.

        Списано

        Верно!

        Расходы почти всегда дебетуются. Исключениями могут быть закрытие записей и, возможно, исправление и корректировка записей.

        Зачислено

        Неправильно.

      13. 11.

        Как правило, когда в транзакции участвуют доходы, счет доходов будет __________.

        Списано

        Неправильно.

        Зачислено

        Верно!

        Доходы почти всегда кредитуются. Исключениями могут быть закрытие записей и, возможно, исправление и корректировка записей.

      14. 12.

        Слово бухгалтера, указывающее, что запись будет сделана в левой части счета, __________.

        Дебет

        Верно!

        Кредит

        Неверно.

      15. 13.

        Какое сальдо, скорее всего, будет иметь счет контрактива, такой как накопленная амортизация?

        Дебет

        Неправильно.

        Кредит

        Верно!

        Поскольку активные счета, скорее всего, будут иметь дебетовые остатки, противоположный активный счет будет иметь противоположный баланс.

      16. 14.

        Какое сальдо, скорее всего, будет иметь счет контрответственности, такой как Скидка на векселя к оплате?

        Дебет

        Верно!

        Поскольку счета обязательств, скорее всего, будут иметь кредитовое сальдо, на счете встречных обязательств будет противоположное сальдо.

        Кредит

        Неверно.

      Бухгалтерский учет и финансы Онлайн-тест

      Ознакомьтесь с общедоступными вопросами ниже

      Решите вопросы, чтобы получить обратную связь, или
      пройдите практический тест и получите бесплатный сертификат.

      Экран с рабочими примерами вопросов

      Рабочие образцы являются лучшим показателем эффективности работы. Более 8000 компаний используют премиум-вопросы TestDome.

      О тесте

      Тест по бухгалтерскому учету и финансам оценивает способность кандидата измерять, обрабатывать и передавать финансовую информацию о бизнесе или корпорации.

      Эта оценка может использоваться в качестве бухгалтерского теста для отбора перед приемом на работу кандидатов, претендующих на различные должности, включая штатного бухгалтера, финансового бухгалтера и управленческого бухгалтера, или в качестве финансового теста для кандидатов, претендующих на должность финансового аналитика или финансового менеджера. позиции.

      Этот тест требует, чтобы кандидаты ответили на вопросы с несколькими вариантами ответов и на вопросы расчета о принципах и основных предметах бухгалтерского учета и финансов.

      Примеры общих вопросов

      Общая сумма процентов

      Публичная

      Основная финансовая отчетность

      Бухгалтерский учет и финансы

      Процентные расходы

      Где можно найти информацию об общей сумме процентов, начисленных в течение отчетного периода в один год?

      Решить вопрос

      Снятие наличных

      Государственный

      Двойная бухгалтерия

      Бухгалтерский учет и финансы

      Бухгалтерский баланс

      План счетов

      план счетов банка будет затронут?

      Решить вопрос

      Краткосрочный долг

      Новый

      Государственный

      Финансовый анализ

      Коэффициент текущей ликвидности

      Коэффициент ликвидности

      Коэффициент быстрой ликвидности

      Прочитайте приведенное ниже утверждение и заполните пропуски правильными ответами.

      Коэффициенты ликвидности указывают на платежеспособность предприятия _________________________, тогда как коэффициенты платежеспособности указывают на платежеспособность предприятия __________________. Если мы видим, что Коэффициент быстрой ликвидности с течением времени составляет ____________________, а Коэффициент текущей ликвидности — ____________________ с течением времени, мы можем сделать вывод, что способность компании погасить краткосрочный долг, вероятно, улучшается.

      Решить вопрос

      Корпоративный автомобиль

      Публичный

      Двойная бухгалтерия

      Бухгалтерский учет и финансы

      План счетов

      Дебеты и кредиты

      Дебеты и кредиты

      Основные средства 90 Ваша компания с наличными 90 a30 Компания только что купила a30 Основные средства 90.

      Решить вопрос

      Просмотреть все общедоступные вопросы

      Для соискателей: получить сертификат

      Получите бесплатный сертификат, набрав 25% лучших результатов в тесте «Бухгалтерский учет и финансы» с общедоступными вопросами.

      Пройти сертификационный тест

      Образец серебряного сертификата

      Sunshine Caprio

      Java и SQL

      TestDome
      Сертификат

      Посмотреть сертификат

      Пройти сертификационный тест практиковаться.
      Верните деньги, если найдете ответ на какой-либо премиальный вопрос в Интернете.

      Зарегистрируйтесь, чтобы предложить этот тест

      Еще 27 дополнительных вопросов по бухгалтерскому учету и финансам

      Бухгалтерский баланс, Офисные стулья, Стоимость производственной линии, Деревообрабатывающий станок, Дебет или кредит, Новое корпоративное здание, Типы счетов, Служебные автомобили, Продажа товаров, Перевод, Счет-фактура, Собственный капитал и активы, Итого активы, Прибыль и доход, Чистый доход , Капитал, Сбор дебиторской задолженности, Снижение чистой прибыли, Criterion Inc., Zero Debts, Brightsense Inc., Webster and Sons, Plurality Corp. , TDEC Inc., Балансовые отчеты, AP Williams, Strawberry Inc..

      Протестированные навыки и темы

      • Основные финансовые отчеты
      • Бухгалтерский учет и финансы
      • БАЛЕКТЫ
      • Двойной бухгалтерский бухгалтер
      • Делуя
      • Диаграмма
      • DEBRECIATION
      • Диаграмма
      • DEBRECIATION
      • .
      • Счета-фактуры
      • Обязательства
      • Заработная плата
      • Процентные расходы
      • Финансовый анализ
      • Activity Ratios
      • Receivables Turnover Ratio
      • Profit Margins
      • Profitability Ratios
      • Investment Ratios
      • Payout Ratio
      • Retained Earnings
      • Debt Ratio
      • Leverage Ratios
      • Cash Ratio
      • Liquidity Ratios
      • Debt-to -Коэффициент собственного капитала

      Для должностей

      • Бухгалтер
      • Финансовый бухгалтер
      • Финансовый аналитик
      • Финансовый менеджер
      • Управленческий бухгалтер
      • Штатный бухгалтер

      Образец отчета кандидата

      Что говорят другие

      Простое и понятное техническое тестирование выбрать из, и не отнимает у кандидата чрезмерное количество времени. Он также имитирует рабочее давление с ограничениями по времени.

      Ян Опперман, Grindrod Bank

      Обзоры продуктов

      Используется

      Решите все ваши задачи по тестированию навыков

      Более 150 готовых тестов

      От веб-разработки и администрирования баз данных до управления проектами и поддержки клиентов. Посмотреть все готовые тесты.

      90+ навыков

      От JavaScript и SQL до английского и поддержки клиентов. Просмотреть все вопросы для фильтрации по навыкам.

      Тест на несколько навыков

      Смешайте вопросы для разных навыков или даже специальные вопросы в одном тесте. См. пример.

      Как работает TestDome

      1

      Выберите предварительно сделанный тест
      или создайте пользовательский тест

      2

      Приглашения кандидатов по
      Электронная почта, URL или ваш ATS

      3

      Кандидаты.

      4

      Сортировка кандидатов и
      получение индивидуальных отчетов

      Хотите узнать больше?

      Характеристики

      Цены

      Не совсем то, что вы ищете?

      Related Accounting & Finance Tests:

      • Financial Analyst
      Follow us

      Sitemap

      Product

      Features

      Use Cases

      Customers

      ROI Calculator

      Integrations

      Pricing

      Assessments

      Tests

      Вопросы

      Для соискателей

      Ресурсы

      Книга: наем на основе фактических данных

      Блог

      Служба поддержки

      Свяжитесь с нами

      Юридический отдел

      Следуйте за нами

      Карта сайта

      Вернуться к началуВ начало норма для тестов и викторин. Но некоторые факультеты бухгалтерского учета меняют эту модель, разрабатывая новые творческие подходы к оценке.

      Придуманные ими необычные тесты и викторины в некотором смысле являются ответом на вызовы времени. Теперь, когда учащиеся имеют широкий и быстрый доступ к информации, запоминание фактов стало менее значимым. «Интернет сам по себе является поводом для переоценки того, что вы тестируете и что вы цените», — сказал Тимоти Фогарти, дипломированный бухгалтер, доктор юридических наук, профессор бухгалтерского учета в Университете Кейс Вестерн Резерв в Кливленде, который позволяет студентам принимать открытые решения. -книжные экзамены. По его словам, преподаватели должны задавать вопросы, требующие от учащихся обдумывания своих ответов, а не просто воспроизводить информацию, которую они нашли в другом месте.

      Студенты могут легко получить доступ ко многим основанным на фактах тестовым вопросам и ответам через банков тестов , отметил Фогарти. Более того, вопросы, проверяющие запоминание, менее важны при подготовке учащихся к экзамену CPA, который был скорректирован таким образом, чтобы требовать критического мышления. «Вы должны задавать вдумчивые вопросы, на которые [учебник] не отвечает», — сказал он.

      Вот несколько способов, которыми преподаватели бухгалтерского учета переосмыслили тесты и викторины для развития критического мышления:

      Тесты с открытой книгой (и с открытым ноутбуком). Фогарти, преподающий бухгалтерский учет 35 лет, в 2018 году переработал свои тесты, чтобы позволить учащимся использовать внешние источники, включая учебники и поисковые запросы Google. Идея, которая на самом деле пришла от студентов, состоит в том, чтобы позволить им продемонстрировать более глубокое обучение. В «среде открытых книг», сказал он, «все ресурсы мира не помогут вам, потому что вы должны продумать, каким будет ответ. Главное, за что я хочу вознаграждать, — это то, насколько хорошо [студенты] собирают информацию воедино».

      Вопросы, созданные студентами. Джоэл Ланц, CPA/CITP/CFF, CGMA, приглашенный доцент в SUNY Old Westbury, позволяет своим студентам помогать в подготовке экзамена. В течение последних двух лет он предлагал каждому учащемуся представить потенциальные тестовые вопросы с несколькими вариантами ответов через онлайн-форум. Каждая заявка должна включать объяснение того, почему ответ правильный, включая документацию из учебника, видеолекции, дополнительные материалы и/или занятия в классе, а также обсуждение того, почему другие ответы неверны и откуда пришла идея — учебник или лекцию, например.

      Ланц делится всеми присланными вопросами в электронном виде с другими учащимися, чтобы помочь им подготовиться к экзаменам. По его словам, просматривая онлайн-форум, «учащиеся могут увидеть, как думают другие ученики».

      Перед каждым из двух семестровых онлайн-тестов из 50 вопросов он просматривает отправленные материалы, чтобы выбрать до 25 или около того для использования. Отобранные получают учащимся «гонорар» в размере половины балла, добавляемого к их общей оценке в классе.

      Однако, чтобы сделать разрез, представления не могут быть легкими. В программу занятий Ланц включил это примечание для студентов, в котором говорится, что он с большей вероятностью выберет их вопросы, если они проверят способность сверстников применять знания, а не просто запоминать их. «Я не собираюсь выбирать вопрос, который вы можете найти в Google», — сказал он. «Это противоречит всей цели. Я должен защитить целостность теста».

      Предоставление учащимся возможности участвовать в викторинах в группах. ДеАнна Мартин, дипломированный бухгалтер, профессор бухгалтерского учета в колледже Сантьяго-Каньон в Оранже, штат Калифорния, позволяет своим ученикам проходить часовые викторины в командах из трех-пяти человек лично или в переговорных комнатах в Zoom. (Те, кто не может встретиться, могут выполнять работу индивидуально.) Она позволяет ученикам помогать друг другу, хотя задачи каждого ученика имеют разные числа для расчета. (Она использует программное обеспечение, которое автоматически генерирует различные числа для вопросов и помогает выставлять оценки. ) Идея групповых викторин, по ее словам, состоит в том, чтобы помочь учащимся учиться, объясняя понятия друг другу. «Желание состоит в том, чтобы заставить студентов общаться друг с другом и взаимодействовать», — сказала она.

      У Мартина улучшились результаты после внесения изменений несколько лет назад. Студенты рассказали ей, что во время викторин у них были «моменты озарения», когда они впервые поняли материал, когда обсуждали его с членами своей группы или объясняли кому-то другому. «Я вижу гораздо большие успехи в викторинах», — сказала она.

      Мартин дает несколько тестов по материалу или проблеме, которую она рассмотрела за последние 24 часа. Во многих случаях информация или проблемы почти идентичны тому, что было только что представлено. Цель состоит в том, чтобы заставить учащихся быть внимательными во время урока и делать тщательные записи. «Содержание викторины — это именно то, что мы обсуждали в классе», — сказал Мартин, который преподает несколько курсов по бухгалтерскому учету. «Я хочу настроить их на успех».

      Дон Вотапка — писатель-фрилансер из Атланты. Чтобы прокомментировать эту статью или предложить идею для другой статьи, свяжитесь с Кортни Виен, старшим редактором JofA , по адресу [email protected].

      Найдите ответы на часто задаваемые вопросы об экзамене CPA | Ресурсы

      Ресурсы

      10 месяцев назад · 1 мин чтения

      В разделе часто задаваемых вопросов вы найдете ответы на наиболее распространенные вопросы об экзамене CPA и связанных с ним темах. Чтобы найти информацию о приложениях, приспособлениях, правах на участие, экзаменационных кредитах, сборах, юрисдикциях, лицензировании и других общих административных темах, мы рекомендуем вам посетить CPA Central NASBA. По вопросам, связанным с расписанием экзаменов и центрами тестирования, вы можете обратиться на веб-сайт Prometric. Если вы не можете найти ответ на свой вопрос, обратитесь напрямую в AICPA или NASBA.

      Что такое Единый экзамен CPA® (экзамен CPA)?

      Экзамен — это 16-часовой тест, состоящий из четырех частей, который требуется пройти во всех юрисдикциях для получения лицензии CPA. Он предназначен для проверки минимальных знаний и навыков, необходимых CPA. Экзамен CPA принадлежит AICPA и проводится в центрах тестирования Prometric NASBA и AICPA от имени государственных советов по бухгалтерскому учету.

      Каковы требования для сдачи экзамена CPA?

      В 55 юрисдикциях (50 штатов и 5 территорий США) есть бухгалтерские советы. Эти советы определяют квалификационные требования и требования к лицензированию кандидатов в своей юрисдикции.

      Доступен ли экзамен CPA на другом языке, кроме английского?

      Нет. Экзамен CPA предлагается только на английском языке.

      Есть ли ограничение по времени для сдачи экзамена CPA?

      Вы должны сдать все четыре раздела экзамена CPA в течение 18 месяцев. Расчет начала 18-месячного периода зависит от юрисдикции. Для получения подробной информации обратитесь в свою конкретную бухгалтерию.

      Сколько стоит сдать экзамен CPA?

      Сборы зависят от юрисдикции, в которой вы хотите получить лицензию. Вы можете узнать подробности, обратившись к вашему конкретному совету бухгалтеров.

      Как мне подать заявку на экзамен CPA?

      Сначала вы должны определиться с юрисдикцией, в которую вы будете подавать заявление. Выбрав свою юрисдикцию, вы можете получить материалы заявки и подать заполненные заявки в соответствии с указаниями.

      Есть ли одна центральная организация, куда я могу подать заявку на сдачу экзамена CPA?

      Нет. Вы можете сдать экзамен (и получить квалификацию CPA) только в том случае, если вы соответствуете требованиям бухгалтерского совета в одной из 55 юрисдикций США.

      Где я могу сдать экзамен CPA?

      Вы можете сдать экзамен в авторизованных центрах тестирования Prometric в 55 юрисдикциях США, а также в некоторых странах мира. См. «Международные часто задаваемые вопросы» ниже для тестирования стран.

      Какова структура и формат экзамена CPA?

      Экзамен CPA состоит из четырех четырехчасовых секций. Вы будете брать по одному разделу за раз. Каждый из четырех разделов состоит из пяти небольших разделов, известных как «тестлеты», в которых представлены вопросы с несколькими вариантами ответов, симуляции на основе задач и, в случае раздела BEC, письменные коммуникативные задания.

      Сдам ли я тот же экзамен CPA, что и другие кандидаты?

      Вы будете сдавать другой, но эквивалентный экзамен. Представленные вам вопросы взяты из набора тестовых вопросов в соответствии с определенными спецификациями. Несмотря на то, что вы сдаете разные экзамены, спецификации обеспечивают сопоставимость результатов.

      Некоторые администрации экзамена CPA сложнее, чем другие?

      Между разными администрациями могут быть незначительные различия, но эти различия учитываются при подсчете баллов. AICPA повышает безопасность тестирования, создавая несколько форм экзамена CPA с разными вопросами для разных администраций. Каждая форма сопоставима, но не идентична.

      Большое внимание уделяется соответствию форм с точки зрения содержания и сложности элементов. Помните, что вам могут задавать вопросы различной сложности в зависимости от вашей успеваемости. Сложность вопроса учитывается при подсчете очков. Следовательно, это не означает, что легче получить более высокий балл просто потому, что вы получаете более простые вопросы.

      Какие компьютерные навыки мне необходимы для сдачи экзамена CPA?

      Для сдачи экзамена CPA необходимы только базовые навыки работы с компьютером. Предполагается, что вы знакомы с использованием мыши и клавиатуры, а также с основными функциями работы с электронными таблицами и текстовыми редакторами.

      Чтобы ответить на экзаменационные вопросы, вам может потребоваться:

      • Выберите ответ из доступных вариантов, щелкнув переключатель

      • Выполните стандартные финансовые расчеты, используя электронную таблицу Microsoft® Excel или четырехфункциональную онлайн-систему. калькулятор

      • Меморандум или буква типа А в письменных задачах связи

      • Выполните авторитетный поиск в литературе в исследовательской части моделирования

      • Копировать и вставьте текст с использованием стандартного соротика

      • Использование Scrollbars

      • 7777
      • .

      • Изменение размера или перемещение окон

      • Выделение отрывков из экспонатов, которые вы хотели бы запомнить

      Чтобы узнать больше о функциональности экзамена CPA, ознакомьтесь с примерами тестов.

      Используются ли на экзамене CPA Microsoft® Word и Microsoft® Excel?

      У вас будет доступ к версии Excel, в которой некоторые функции недоступны. Вас не будут проверять на способность использовать Excel. Программа доступна только как инструмент для использования при тестировании. Экзамен CPA использует приложение для обработки текстов, которое похоже, но не идентично Word.

      Чтобы узнать больше о функциональности экзамена CPA, ознакомьтесь с образцами тестов.

      Могу ли я сдать экзамен CPA на бумаге?

      Нет. Это компьютерный тест.

      Международное тестирование

      Какие темы тестируются на экзамене CPA?

      Темы каждого раздела экзамена CPA можно найти в планах экзамена CPA.

      Каковы правила для новых бухгалтерских и аудиторских заключений?

      Вы можете узнать, когда новый контент подходит для тестирования, ознакомившись с Политикой экзаменационной комиссии в отношении новых постановлений.

      Какие информационные материалы/базы данных доступны мне во время экзамена CPA?

      Для всех симуляций у вас есть доступ к официальной литературе, которая включает некоторые разделы Профессиональных стандартов AICPA (в разделе «Аудит и аттестация»), Кодификации FASB (в разделе «Финансовый учет и отчетность») и Налогового кодекса (в Положении раздел).

      Образцы экзаменационных тестов

      Что такое образцы тестов?

      Образцы тестов позволяют вам попрактиковаться в формате и функциях экзамена CPA с помощью программного обеспечения, которое вы будете использовать в центре тестирования (вам нужно будет использовать свой собственный Microsoft® Excel и авторитетную литературу, представленную в образце). тест представляет собой сжатую версию).

      Несмотря на то, что для каждого раздела существует один образец теста, они не являются полными разделами экзамена CPA и не определяют вашу готовность сдать настоящий экзамен CPA.

      Что входит в образец теста?

      Примеры тестов включают в себя два теста с множественным выбором и три теста на основе задач для каждого из трех разделов экзамена CPA: аудит и аттестация (AUD), финансовый учет и отчетность (FAR) и регулирование (REG). Образец теста Business Environment and Concepts (BEC) включает в себя два тестлета с несколькими вариантами ответов, два тестлета моделирования на основе задач и один письменный коммуникационный тестлет. У вас также будет доступ к учебным темам, которые можно найти, нажав кнопку «Справка» в каждом образце теста.

      Какой компьютер мне нужен, чтобы использовать образцы тестов?

      Примеры тестов доступны через Интернет и требуют подключения к Интернету. Они будут работать на большинстве устройств и операционных систем. Вам следует использовать 23-дюймовый HD-монитор (1920×1080), чтобы получить опыт, аналогичный реальному испытательному центру.

      Примеры тестов оптимизированы для среды тестирования в центрах Prometric. Если у вас возникнут какие-либо проблемы, мы рекомендуем использовать самую последнюю версию браузера Chrome на настольном HD-мониторе.

      Поскольку в программе CPA Exam используются мониторы высокой четкости (HD), нужен ли мне монитор HD для выполнения пробных тестов?

      Нет. Однако, если вам нужна реалистичная картина того, как текст будет выглядеть в центре тестирования Prometric, мы рекомендуем использовать монитор высокой четкости.

      Почему я вижу полосы прокрутки справа или внизу при просмотре образцов тестов?

      Примеры тестов можно запускать на мониторах с разрешением ниже минимального рекомендуемого. Если вы это сделаете, вы увидите полосы прокрутки. Вы можете убрать полосы прокрутки на мониторах с меньшим разрешением, чем рекомендуемое, с помощью сочетаний клавиш масштабирования в браузере:

      Уменьшение масштаба изменит размер образца теста, чтобы он поместился на экране без полос прокрутки, но вы можете обнаружить, что меньший текст труднее читать.

      Будет ли работать правая кнопка мыши при тестировании в центрах тестирования Prometric?

      При выполнении пробного теста при нажатии правой кнопки мыши появляется контекстное меню. Это позволит вам копировать, вставлять и выполнять другие операции. Примечание. Эта функция недоступна в центрах тестирования Prometric.

      Что делать, если у меня возникли проблемы с запуском пробного теста?

      Убедитесь, что ваше устройство соответствует минимальным техническим требованиям. Если у вас по-прежнему возникают проблемы с запуском пробных тестов, обратитесь за помощью в AICPA.

      Какой проходной балл?

      Проходной балл — 75 по шкале от 0 до 99. Оценки не кривые. Узнайте, как определяется ваш балл, прочитав о подсчете баллов на экзамене CPA.

      Кто устанавливает проходной балл для экзамена CPA?

      Проходной балл определяется Экзаменационной комиссией AICPA (BOE), которая учитывает множество факторов, включая результаты исследований, устанавливающих стандарты, исторические тенденции и изменения содержания экзамена. BOE также получает информацию от NASBA, психометрических консультантов, академического сообщества и лицензированных CPA. Проходной балл является основанием для принятия решения о прохождении или провале, рекомендованного бухгалтерским советам в консультативном отчете об оценке.

      Является ли оценка автоматизированным процессом?

      Оценка полностью автоматизирована для всех компонентов экзамена CPA, за исключением письменных коммуникативных задач. Большинство письменных коммуникативных ответов оцениваются с помощью компьютерной программы оценки, которая откалибрована с использованием людей. В некоторых случаях ответы оцениваются сетью оценщиков (все CPA). Если ваш балл близок к проходному баллу, ваши письменные коммуникативные задания будут автоматически переоценены оценщиками. Если ответ оценивается более чем одним оценщиком, в качестве окончательной оценки используется среднее значение баллов.

      AICPA использует Теорию ответных заданий (IRT) для объективной части экзамена. IRT — это хорошо зарекомендовавший себя психометрический подход к подсчету баллов, используемый на лицензионных и сертификационных экзаменах, которые проводят множество различных форм тестов.

      Все процедуры подсчета очков, независимо от того, автоматизированы они или нет, проверяются на различных этапах процесса подсчета очков.

      Когда публикуются результаты?

      Результаты публикуются на непрерывной основе, даты публикуются два раза в год.

      Как экзамен CPA проверяет мои знания и навыки?

      Мы используем многоступенчатую адаптивную модель доставки тестов для всех тестов с несколькими вариантами ответов. Ваш первый тестлет будет дан на уровне средней сложности. Следующий тест будет того же уровня или немного сложнее, в зависимости от ваших результатов. Адаптивная модель не используется для моделирования задач или задач письменного общения.

      Является ли многоэтапное тестирование справедливым? Почему вы его используете?

      Да, честно. Поскольку при подсчете баллов учитываются характеристики тестовых вопросов, нет никаких преимуществ или недостатков в назначении тестов разной сложности. Мы используем многоэтапное тестирование, поскольку вопросы теста соответствуют уровням владения языком, и поэтому для получения точных оценок уровня владения языком требуется меньшее количество вопросов.

      Как вы решаете, какие вопросы сложные, а какие средние?

      Уровни сложности тестовых вопросов (и другие статистические данные, используемые для описания каждого тестового вопроса) определяются путем статистического анализа ответов кандидатов. На уровне вопроса сложность определяется не как категория (например, умеренная или сложная), а как числовое значение по шкале. Тестлеты классифицируются как средние или сложные в зависимости от средней сложности вопросов в этом тестлете. Все тестлеты содержат вопросы разной степени сложности. Просто вопросы в сложных тестлетах имеют более высокий средний уровень сложности, чем в средних тестлетах.

      Что такое вопросы предварительного тестирования и оцениваются ли они?

      Вопросы для предварительного тестирования включены в каждый экзамен CPA (они могут быть вопросами с несколькими вариантами ответов, имитационными задачами или письменными коммуникативными заданиями) только с целью сбора данных. Данные необходимы для оценки качества вопросов и для сбора информации о подсчете баллов для последующего использования, когда вопросы станут рабочими элементами. Они не являются частью вашего расчета очков.

      Вносятся ли коррективы в баллы для кандидатов, которые испытывают трудности во время тестирования?

      Нет. Ваши результаты оцениваются с использованием одного и того же процесса и метода, чтобы обеспечить единообразие и обоснованность решения о прохождении или не прохождении теста. В редких случаях, когда во время тестирования возникают серьезные технические проблемы, NASBA может предложить вам бесплатное повторное тестирование.

      Могу ли я получить проходной балл, только хорошо ответив на вопросы с несколькими вариантами ответов?

      Нет. Ваши результаты оцениваются с использованием одного и того же процесса и метода, чтобы обеспечить единообразие и обоснованность решения о прохождении или не прохождении теста.

      Что такое проверка результатов?

      Процесс проверки оценок включает в себя проверку того, что утвержденные ключи ответов использовались и применялись правильно при определении оценки кандидата, и не дает возможности рассмотреть альтернативные ответы. Это просто дополнительная независимая проверка вашего балла на экзамене CPA. Пожалуйста, имейте в виду, что все партитуры перед выпуском проходят тщательную проверку качества.

      Учитывая, что все проверки качества уже завершены, маловероятно, что ваша оценка изменится в результате проверки.

      Как и когда я могу запросить пересмотр результатов?

      Вы можете связаться со своим бухгалтерским советом или назначенным им агентом для получения инструкций по запросу о пересмотре оценки, необходимых сборах и соблюдении крайнего срока запроса на пересмотр оценки. Если вы подадите заявку после крайнего срока, ваш запрос не будет обработан. Возможность подать заявку на пересмотр оценки доступна только в течение короткого периода времени после того, как вам сообщили о вашей оценке. Обработка и публикация оценки может занять до восьми недель.

      Как будут сообщены результаты просмотра оценок?

      После того, как NASBA отправит ваш запрос в AICPA, AICPA проверит вашу оценку и сообщит о результате через NASBA вашему бухгалтерскому совету или назначенному им лицу. Затем НАСБА, правление или уполномоченное им лицо передаст вам результат.

      Что такое апелляция?

      В юрисдикциях, допускающих апелляции, этот процесс дает вам возможность обжаловать неудовлетворительную оценку. Опция подачи апелляции, если она доступна, позволяет вам просмотреть тестовые вопросы с несколькими вариантами ответов или задачи объективного моделирования, на которые вы ответили неправильно, вместе с их ответами, а также отправить комментарии в Интернете. В обращение не включены письменные коммуникативные задания.

      Конфиденциальность экзамена CPA требует, чтобы такие сеансы просмотра происходили только в разрешенных местах, в условиях строгой безопасности и в присутствии представителя вашего бухгалтерского совета или его представителя.

      Почему мне следует подумать о подаче апелляции?

      Подавать апелляцию следует только в том случае, если вы хотите пересмотреть свои неверные ответы, потому что считаете, что существует вопрос или проблема моделирования, которую вы хотели бы оспорить.

      При просмотре вопросов или задач моделирования, на которые вы ответили неправильно, вы можете решить оспорить достоверность одного или нескольких пунктов. Если вы решите это сделать, вы должны быть готовы представить убедительную, энергичную и убедительную защиту своих неправильных ответов.

      Обратите внимание, что вы не сможете подавать новые ответы во время апелляции. Тем не менее, у вас будет возможность оспорить вопрос (вопросы) теста с множественным выбором или моделирование (я) и защитить ответы, которые вы дали на экзамене.

      Как подать апелляцию?


      Свяжитесь со своим Советом по бухгалтерскому учету или его представителем, чтобы определить, доступна ли возможность подачи апелляции в вашей юрисдикции.

      Какова плата за апелляцию?

      Вы должны связаться с вашим Советом по бухгалтерскому учету или его представителем, чтобы узнать точные суммы этих сборов. С вас будет взиматься отдельная плата за каждый пункт (тестовый вопрос с несколькими вариантами ответов или симуляционная задача), который вы решите оспорить.

      Как моя апелляция будет рассмотрена и мне будет сообщен результат?

      Если ваш бухгалтерский совет решит, что вы имеете право на апелляцию, совет направит ваш запрос в AICPA через NASBA. Правление или его уполномоченный составят для вас расписание сеанса просмотра. Ваши онлайн-комментарии во время этой сессии будут переданы в AICPA через NASBA.

      После сессии AICPA рассмотрит ваши ответы по разделу, на который вы подаете апелляцию, рассмотрит отправленные вами онлайн-комментарии, проверит ваш балл и направит результат в NASBA. (Примечание: вы не получите подробной информации о вопросе(ах), с которыми вы оспариваете, из-за необходимости сохранения конфиденциальности содержания экзамена.) NASBA сообщит о результатах вашему совету или назначенному им лицу, и результат будет передан ты.

      Что такое международное тестирование?

      Международное тестирование позволяет гражданам США, а также имеющим право кандидатам из других стран сдавать Единый экзамен CPA® (экзамен) в некоторых странах. Если вы не являетесь гражданином США, этот процесс позволяет вам получить лицензию CPA США. AICPA, NASBA и Prometric предоставляют те же услуги для международного тестирования, что и для тестирования в США. Процесс экзамена и лицензирования для иностранных кандидатов такой же, как и для кандидатов в пределах юрисдикции США.

      Если я не гражданин США, как мне узнать, имею ли я право сдавать экзамен CPA?

      Веб-сайт NASBA предоставляет информацию для иностранных кандидатов. Вы также можете рассмотреть возможность использования международных оценочных услуг NASBA. Свяжитесь с NASBA, чтобы узнать о международных требованиях к кандидатам.

      Когда я могу пройти тестирование за границей?

      Тестирование для иностранных кандидатов проводится постоянно, как и для кандидатов из США

      Как записаться на экзамен CPA в международном центре тестирования?

      После завершения процесса международной регистрации посетите веб-сайт Prometric, чтобы запланировать экзамен CPA.

      Где я могу пройти тестирование на международном уровне?

      Международные кандидаты, соответствующие требованиям, могут сдать экзамен CPA в центрах тестирования Prometric в Бахрейне, Бразилии, Египте, Англии, Германии, Индии, Ирландии, Японии, Иордании, Кувейте, Ливане, Непале, Шотландии и Объединенных Арабских Эмиратах (ОАЭ). Все кандидаты могут пройти тестирование в США. Чтобы определить, можете ли вы сдать экзамен CPA в одном из международных регионов, ознакомьтесь с конкретными часто задаваемыми вопросами ниже.

      Кто может пройти тестирование в Японии?

      Если вы являетесь гражданином или резидентом Японии или США, вы можете пройти тестирование в Японии.

      Кто может пройти тестирование в Индии?

      Если вы являетесь гражданином или резидентом Индии, Бутана, Бангладеш, Мьянмы, Мальдивских островов, Непала, Шри-Ланки или США, вы можете пройти тестирование в Индии.

      Кто может пройти тестирование в Бахрейне, Египте, Иордании, Кувейте, Ливане, Саудовской Аравии и Объединенных Арабских Эмиратах (ОАЭ)?

      Граждане или резиденты Бахрейна, Египта, Индии, Иордании, Кувейта, Ливана, Омана, Катара, Саудовской Аравии, Йемена, Объединенных Арабских Эмиратов (ОАЭ), США или Йемена.

      Кто может пройти тестирование в Бразилии?

      Граждане или резиденты любой из следующих стран могут проходить тестирование в Бразилии: Аргентина, Антигуа/Барбуда, Багамы, Барбадос, Белиз, Боливия, Бразилия, Каймановы острова, Чили, Колумбия, Коста-Рика, Доминика, Доминиканская Республика, Эквадор, Сальвадор, Французская Гвиана, Гватемала, Гренада, Гайана, Гаити, Гондурас, Ямайка, Мексика, Никарагуа, Панама, Парагвай, Перу, Сент-Китс/Невис, Сент-Люсия, Сент-Винсент/Гренадины, Суринам, Тринидад и Тобаго, Уругвай, США и Венесуэла

      Кто может пройти тестирование в Англии, Германии, Ирландии и Шотландии?

      Любой отвечающий критериям кандидат CPA может пройти тестирование в этих странах.

      Кто может пройти тестирование в Израиле?

      Граждане или жители Израиля или США могут пройти тестирование в Израиле.

      Кто может пройти тестирование в Непале?

      Если вы являетесь гражданином или резидентом Бутана, Бангладеш, Индии, Мьянмы, Мальдивских островов, Непала, Шри-Ланки или США, вы можете пройти тестирование в Непале.

      Могу ли я сдать экзамен, если я являюсь резидентом страны международного тестирования, но у меня нет паспорта?

      Нет. Вы должны предъявить свой паспорт в качестве основного документа, удостоверяющего личность. Сюда входят граждане США, проживающие за границей.

      Я гражданин страны, в которой проводится международное тестирование. Бухгалтерский совет предоставил мне право сдавать экзамен CPA, но, согласно вашей политике, я не могу сдавать международные тесты и должен поехать в США. Если я имею право, почему я не могу сдавать тесты в любом месте, где хочу к?

      AICPA, NASBA и Prometric управляют центрами тестирования с соблюдением самых строгих мер безопасности, гарантируют целостность и безопасность данных и защищают конфиденциальность кандидатов. Три партнера решили, что требования к гражданству и месту жительства, а также достоверность некоторых видов удостоверений личности обеспечивают необходимый уровень безопасности и позволяют нам лучше обслуживать кандидатов на сдачу экзаменов.

      Как вы определяете международные места тестирования?

      Международные места тестирования оцениваются на основе набора критериев, включая:

      • Объем спроса на экзамен от кандидатов в этих странах

      • Способность сдать экзамен без юридических препятствий

      • Угроза безопасности экзамена (как физическая безопасность, так и безопасность интеллектуальной собственности), оцененные на уровне, равном уровню, представленному внутри страны

      • Наличие установленных центров тестирования Prometric

      Страны, которые не соответствуют этим критериям в совокупности NASBA, AICPA и Prometric, не считаются местами тестирования. Мы ценим стремление к дополнительным международным тестовым площадкам и регулярно проверяем новые места на предмет возможного расширения.

      Где я могу пройти тестирование, если моя юрисдикция не участвует в международном тестировании?

      Если вы регистрируетесь через юрисдикцию, не участвующую в программе, вы можете пройти тестирование только в утвержденных центрах тестирования в США, на Гуаме, в Пуэрто-Рико или на Виргинских островах. Пожалуйста, свяжитесь с NASBA, чтобы узнать, участвует ли ваша юрисдикция в международной программе.

      Где я могу подать заявку на международное тестирование?

      Пожалуйста, посетите веб-сайт NASBA для получения информации о применении.

      Могу ли я использовать свое международное уведомление о расписании (NTS) для записи на испытательный полигон в США?

      Нет. После получения NTS для одного из международных центров тестирования его нельзя обменять на NTS для любого из центров тестирования в США. Вы должны уведомить NASBA о переходе обратно на местную NTS.

      Могу ли я подать заявку на возмещение или продление моего нынешнего NTS, чтобы записаться на один из международных полигонов?

      Отказ от сдачи экзамена CPA и/или запрос на продление текущего NTS не предусмотрены. Заявка и/или сборы не возвращаются. Если срок действия вашего NTS истекает до начала тестирования или вы пропустите запланированное назначение тестирования, вы не сможете перенести или получить возмещение любой из уплаченных комиссий. Вам нужно будет повторно подать заявку на экзамен и оплатить соответствующую заявку/регистрацию и сборы.

      Если я пройду международный тест, когда я получу свои баллы?

      Пожалуйста, ознакомьтесь с датами выпуска партитуры для получения подробной информации.

      Где я могу найти дополнительную информацию о международном тестировании, когда она станет доступной?

      AICPA предоставляет информацию через объявления об экзаменах, а также обновляет эти международные часто задаваемые вопросы по мере поступления новой информации. Вы также можете найти информацию на веб-сайте NASBA.

      Что вы думаете об этом?

      Каждый ваш отзыв поможет нам улучшить ваш опыт

      Что вы думаете об этом?

      Каждый бит обратной связи, который вы предоставляете, поможет нам улучшить ваш опыт

      , упомянутые в этой статье

      Темы

      Люди, лидерство и продвижение

      Подтопики

      Development

      CPA Exam

      1Найти точное значениеsin(30)
      2Найти точное значениеsin(45)
      3Найти точное значениеsin(30 град. )
      4Найти точное значениеsin(60 град. )
      5Найти точное значениеtan(30 град. )
      6Найти точное значениеarcsin(-1)
      7Найти точное значениеsin(pi/6)
      8Найти точное значениеcos(pi/4)
      9Найти точное значениеsin(45 град. )
      10Найти точное значениеsin(pi/3)
      11Найти точное значениеarctan(-1)
      12Найти точное значениеcos(45 град. )
      13Найти точное значениеcos(30 град. )
      14Найти точное значениеtan(60)
      15Найти точное значениеcsc(45 град. )
      16Найти точное значениеtan(60 град. )
      17Найти точное значениеsec(30 град. )
      18Найти точное значениеcos(60 град. )
      19Найти точное значениеcos(150)
      20Найти точное значениеsin(60)
      21Найти точное значениеcos(pi/2)
      22Найти точное значениеtan(45 град. )
      23Найти точное значениеarctan(- квадратный корень из 3)
      24Найти точное значениеcsc(60 град. )
      25Найти точное значениеsec(45 град. )
      26Найти точное значениеcsc(30 град. )
      27Найти точное значениеsin(0)
      28Найти точное значениеsin(120)
      29Найти точное значениеcos(90)
      30Преобразовать из радианов в градусыpi/3
      31Найти точное значениеtan(30)
      32Преобразовать из градусов в радианы45
      33Найти точное значениеcos(45)
      34Упроститьsin(theta)^2+cos(theta)^2
      35Преобразовать из радианов в градусыpi/6
      36Найти точное значениеcot(30 град. )
      37Найти точное значениеarccos(-1)
      38Найти точное значениеarctan(0)
      39Найти точное значениеcot(60 град. )
      40Преобразовать из градусов в радианы30
      41Преобразовать из радианов в градусы(2pi)/3
      42Найти точное значениеsin((5pi)/3)
      43Найти точное значениеsin((3pi)/4)
      44Найти точное значениеtan(pi/2)
      45Найти точное значениеsin(300)
      46Найти точное значениеcos(30)
      47Найти точное значениеcos(60)
      48Найти точное значениеcos(0)
      49Найти точное значениеcos(135)
      50Найти точное значениеcos((5pi)/3)
      51Найти точное значениеcos(210)
      52Найти точное значениеsec(60 град. )
      53Найти точное значениеsin(300 град. )
      54Преобразовать из градусов в радианы135
      55Преобразовать из градусов в радианы150
      56Преобразовать из радианов в градусы(5pi)/6
      57Преобразовать из радианов в градусы(5pi)/3
      58Преобразовать из градусов в радианы89 град.
      59Преобразовать из градусов в радианы60
      60Найти точное значениеsin(135 град. )
      61Найти точное значениеsin(150)
      62Найти точное значениеsin(240 град. )
      63Найти точное значениеcot(45 град. )
      64Преобразовать из радианов в градусы(5pi)/4
      65Найти точное значениеsin(225)
      66Найти точное значениеsin(240)
      67Найти точное значениеcos(150 град. )
      68Найти точное значениеtan(45)
      69Вычислитьsin(30 град. )
      70Найти точное значениеsec(0)
      71Найти точное значениеcos((5pi)/6)
      72Найти точное значениеcsc(30)
      73Найти точное значениеarcsin(( квадратный корень из 2)/2)
      74Найти точное значениеtan((5pi)/3)
      75Найти точное значениеtan(0)
      76Вычислитьsin(60 град. )
      77Найти точное значениеarctan(-( квадратный корень из 3)/3)
      78Преобразовать из радианов в градусы(3pi)/4
      79Найти точное значениеsin((7pi)/4)
      80Найти точное значениеarcsin(-1/2)
      81Найти точное значениеsin((4pi)/3)
      82Найти точное значениеcsc(45)
      83Упроститьarctan( квадратный корень из 3)
      84Найти точное значениеsin(135)
      85Найти точное значениеsin(105)
      86Найти точное значениеsin(150 град. )
      87Найти точное значениеsin((2pi)/3)
      88Найти точное значениеtan((2pi)/3)
      89Преобразовать из радианов в градусыpi/4
      90Найти точное значениеsin(pi/2)
      91Найти точное значениеsec(45)
      92Найти точное значениеcos((5pi)/4)
      93Найти точное значениеcos((7pi)/6)
      94Найти точное значениеarcsin(0)
      95Найти точное значениеsin(120 град. )
      96Найти точное значениеtan((7pi)/6)
      97Найти точное значениеcos(270)
      98Найти точное значениеsin((7pi)/6)
      99Найти точное значениеarcsin(-( квадратный корень из 2)/2)
      100Преобразовать из градусов в радианы88 град.

      Sin(1°)0. 0175
      Sin(2°)0.0349
      Sin(3°)0.0523
      Sin(4°)0.0698
      Sin(5°)0.0872
      Sin(6°)0.1045
      Sin(7°)0.1219
      Sin(8°)0.1392
      Sin(9°)0.1564
      Sin(10°)0.1736
      Sin(11°)0.1908
      Sin(12°)0.2079
      Sin(13°)0.225
      Sin(14°)0.2419
      Sin(15°)0.2588
      Sin(16°)0.2756
      Sin(17°)0.2924
      Sin(18°)0.309
      Sin(19°)0.3256
      Sin(20°)0.342
      Sin(21°)0.3584
      Sin(22°)0.3746
      Sin(23°)0.3907
      Sin(24°)0.4067
      Sin(25°)0.4226
      Sin(26°)0. 4384
      Sin(27°)0.454
      Sin(28°)0.4695
      Sin(29°)0.4848
      Sin(30°)0.5
      Sin(31°)0.515
      Sin(32°)0.5299
      Sin(33°)0.5446
      Sin(34°)0.5592
      Sin(35°)0.5736
      Sin(36°)0.5878
      Sin(37°)0.6018
      Sin(38°)0.6157
      Sin(39°)0.6293
      Sin(40°)0.6428
      Sin(41°)0.6561
      Sin(42°)0.6691
      Sin(43°)0.682
      Sin(44°)0.6947
      Sin(45°)0.7071
      Sin(46°)0.7193
      Sin(47°)0.7314
      Sin(48°)0.7431
      Sin(49°)0. 7547
      Sin(50°)0.766
      Sin(51°)0.7771
      Sin(52°)0.788
      Sin(53°)0.7986
      Sin(54°)0.809
      Sin(55°)0.8192
      Sin(56°)0.829
      Sin(57°)0.8387
      Sin(58°)0.848
      Sin(59°)0.8572
      Sin(60°)0.866
      Sin(61°)0.8746
      Sin(62°)0.8829
      Sin(63°)0.891
      Sin(64°)0.8988
      Sin(65°)0.9063
      Sin(66°)0.9135
      Sin(67°)0.9205
      Sin(68°)0.9272
      Sin(69°)0.9336
      Sin(70°)0.9397
      Sin(71°)0.9455
      Sin(72°)0.9511
      Sin(73°)0.9563
      Sin(74°)0. 9613
      Sin(75°)0.9659
      Sin(76°)0.9703
      Sin(77°)0.9744
      Sin(78°)0.9781
      Sin(79°)0.9816
      Sin(80°)0.9848
      Sin(81°)0.9877
      Sin(82°)0.9903
      Sin(83°)0.9925
      Sin(84°)0.9945
      Sin(85°)0.9962
      Sin(86°)0.9976
      Sin(87°)0.9986
      Sin(88°)0.9994
      Sin(89°)0.9998
      Sin(90°)1
      Sin(91°)0.9998
      Sin(92°)0.9994
      Sin(93°)0.9986
      Sin(94°)0.9976
      Sin(95°)0.9962
      Sin(96°)0. 9945
      Sin(97°)0.9925
      Sin(98°)0.9903
      Sin(99°)0.9877
      Sin(100°)0.9848
      Sin(101°)0.9816
      Sin(102°)0.9781
      Sin(103°)0.9744
      Sin(104°)0.9703
      Sin(105°)0.9659
      Sin(106°)0.9613
      Sin(107°)0.9563
      Sin(108°)0.9511
      Sin(109°)0.9455
      Sin(110°)0.9397
      Sin(111°)0.9336
      Sin(112°)0.9272
      Sin(113°)0.9205
      Sin(114°)0.9135
      Sin(115°)0.9063
      Sin(116°)0.8988
      Sin(117°)0.891
      Sin(118°)0.8829
      Sin(119°)0.8746
      Sin(120°)0. 866
      Sin(121°)0.8572
      Sin(122°)0.848
      Sin(123°)0.8387
      Sin(124°)0.829
      Sin(125°)0.8192
      Sin(126°)0.809
      Sin(127°)0.7986
      Sin(128°)0.788
      Sin(129°)0.7771
      Sin(130°)0.766
      Sin(131°)0.7547
      Sin(132°)0.7431
      Sin(133°)0.7314
      Sin(134°)0.7193
      Sin(135°)0.7071
      Sin(136°)0.6947
      Sin(137°)0.682
      Sin(138°)0.6691
      Sin(139°)0.6561
      Sin(140°)0.6428
      Sin(141°)0.6293
      Sin(142°)0. 6157
      Sin(143°)0.6018
      Sin(144°)0.5878
      Sin(145°)0.5736
      Sin(146°)0.5592
      Sin(147°)0.5446
      Sin(148°)0.5299
      Sin(149°)0.515
      Sin(150°)0.5
      Sin(151°)0.4848
      Sin(152°)0.4695
      Sin(153°)0.454
      Sin(154°)0.4384
      Sin(155°)0.4226
      Sin(156°)0.4067
      Sin(157°)0.3907
      Sin(158°)0.3746
      Sin(159°)0.3584
      Sin(160°)0.342
      Sin(161°)0.3256
      Sin(162°)0.309
      Sin(163°)0.2924
      Sin(164°)0.2756
      Sin(165°)0.2588
      Sin(166°)0. 2419
      Sin(167°)0.225
      Sin(168°)0.2079
      Sin(169°)0.1908
      Sin(170°)0.1736
      Sin(171°)0.1564
      Sin(172°)0.1392
      Sin(173°)0.1219
      Sin(174°)0.1045
      Sin(175°)0.0872
      Sin(176°)0.0698
      Sin(177°)0.0523
      Sin(178°)0.0349
      Sin(179°)0.0175
      Sin(180°)0

      Sin(181°)-0.0175
      Sin(182°)-0.0349
      Sin(183°)-0.0523
      Sin(184°)-0.0698
      Sin(185°)-0. 0872
      Sin(186°)-0.1045
      Sin(187°)-0.1219
      Sin(188°)-0.1392
      Sin(189°)-0.1564
      Sin(190°)-0.1736
      Sin(191°)-0.1908
      Sin(192°)-0.2079
      Sin(193°)-0.225
      Sin(194°)-0.2419
      Sin(195°)-0.2588
      Sin(196°)-0.2756
      Sin(197°)-0.2924
      Sin(198°)-0.309
      Sin(199°)-0.3256
      Sin(200°)-0.342
      Sin(201°)-0.3584
      Sin(202°)-0.3746
      Sin(203°)-0.3907
      Sin(204°)-0.4067
      Sin(205°)-0.4226
      Sin(206°)-0.4384
      Sin(207°)-0.454
      Sin(208°)-0.4695
      Sin(209°)-0. 4848
      Sin(210°)-0.5
      Sin(211°)-0.515
      Sin(212°)-0.5299
      Sin(213°)-0.5446
      Sin(214°)-0.5592
      Sin(215°)-0.5736
      Sin(216°)-0.5878
      Sin(217°)-0.6018
      Sin(218°)-0.6157
      Sin(219°)-0.6293
      Sin(220°)-0.6428
      Sin(221°)-0.6561
      Sin(222°)-0.6691
      Sin(223°)-0.682
      Sin(224°)-0.6947
      Sin(225°)-0.7071
      Sin(226°)-0.7193
      Sin(227°)-0.7314
      Sin(228°)-0.7431
      Sin(229°)-0.7547
      Sin(230°)-0. 766
      Sin(231°)-0.7771
      Sin(232°)-0.788
      Sin(233°)-0.7986
      Sin(234°)-0.809
      Sin(235°)-0.8192
      Sin(236°)-0.829
      Sin(237°)-0.8387
      Sin(238°)-0.848
      Sin(239°)-0.8572
      Sin(240°)-0.866
      Sin(241°)-0.8746
      Sin(242°)-0.8829
      Sin(243°)-0.891
      Sin(244°)-0.8988
      Sin(245°)-0.9063
      Sin(246°)-0.9135
      Sin(247°)-0.9205
      Sin(248°)-0.9272
      Sin(249°)-0.9336
      Sin(250°)-0.9397
      Sin(251°)-0.9455
      Sin(252°)-0.9511
      Sin(253°)-0.9563
      Sin(254°)-0. 9613
      Sin(255°)-0.9659
      Sin(256°)-0.9703
      Sin(257°)-0.9744
      Sin(258°)-0.9781
      Sin(259°)-0.9816
      Sin(260°)-0.9848
      Sin(261°)-0.9877
      Sin(262°)-0.9903
      Sin(263°)-0.9925
      Sin(264°)-0.9945
      Sin(265°)-0.9962
      Sin(266°)-0.9976
      Sin(267°)-0.9986
      Sin(268°)-0.9994
      Sin(269°)-0.9998
      Sin(270°)-1
      Sin(271°)-0.9998
      Sin(272°)-0.9994
      Sin(273°)-0.9986
      Sin(274°)-0.9976
      Sin(275°)-0. 9962
      Sin(276°)-0.9945
      Sin(277°)-0.9925
      Sin(278°)-0.9903
      Sin(279°)-0.9877
      Sin(280°)-0.9848
      Sin(281°)-0.9816
      Sin(282°)-0.9781
      Sin(283°)-0.9744
      Sin(284°)-0.9703
      Sin(285°)-0.9659
      Sin(286°)-0.9613
      Sin(287°)-0.9563
      Sin(288°)-0.9511
      Sin(289°)-0.9455
      Sin(290°)-0.9397
      Sin(291°)-0.9336
      Sin(292°)-0.9272
      Sin(293°)-0.9205
      Sin(294°)-0.9135
      Sin(295°)-0.9063
      Sin(296°)-0.8988
      Sin(297°)-0.891
      Sin(298°)-0.8829
      Sin(299°)-0. 8746
      Sin(300°)-0.866
      Sin(301°)-0.8572
      Sin(302°)-0.848
      Sin(303°)-0.8387
      Sin(304°)-0.829
      Sin(305°)-0.8192
      Sin(306°)-0.809
      Sin(307°)-0.7986
      Sin(308°)-0.788
      Sin(309°)-0.7771
      Sin(310°)-0.766
      Sin(311°)-0.7547
      Sin(312°)-0.7431
      Sin(313°)-0.7314
      Sin(314°)-0.7193
      Sin(315°)-0.7071
      Sin(316°)-0.6947
      Sin(317°)-0.682
      Sin(318°)-0.6691
      Sin(319°)-0.6561
      Sin(320°)-0.6428
      Sin(321°)-0. 6293
      Sin(322°)-0.6157
      Sin(323°)-0.6018
      Sin(324°)-0.5878
      Sin(325°)-0.5736
      Sin(326°)-0.5592
      Sin(327°)-0.5446
      Sin(328°)-0.5299
      Sin(329°)-0.515
      Sin(330°)-0.5
      Sin(331°)-0.4848
      Sin(332°)-0.4695
      Sin(333°)-0.454
      Sin(334°)-0.4384
      Sin(335°)-0.4226
      Sin(336°)-0.4067
      Sin(337°)-0.3907
      Sin(338°)-0.3746
      Sin(339°)-0.3584
      Sin(340°)-0.342
      Sin(341°)-0.3256
      Sin(342°)-0.309
      Sin(343°)-0.2924
      Sin(344°)-0.2756
      Sin(345°)-0. 2588
      Sin(346°)-0.2419
      Sin(347°)-0.225
      Sin(348°)-0.2079
      Sin(349°)-0.1908
      Sin(350°)-0.1736
      Sin(351°)-0.1564
      Sin(352°)-0.1392
      Sin(353°)-0.1219
      Sin(354°)-0.1045
      Sin(355°)-0.0872
      Sin(356°)-0.0698
      Sin(357°)-0.0523
      Sin(358°)-0.0349
      Sin(359°)-0.0175
      Sin(360°)-0

    • 1414141418 и 14141418 и4141418 и 141414 и 14666 и , 141418 , 2 , . 2

      Потому что они подходят для обоих cos и sin :

           

      Ваша рука может помочь вам вспомнить:

            

      Например, есть 3 пальца выше 30°, поэтому cos(30°) = √3 2

      Что насчет загара?

      Итак, tan = sin/cos , поэтому мы можем вычислить это так:

      tan(30°) = sin(30°) cos(30°)  =  1/2 √3/2 = 1 √3 = √3 3 *

      tan(45°) = sin(45°) cos(45°)  =  √2/2 √2/2 =

      tan(60°) = sin(60°) cos(60°)  =  √3/2 1/2 = √3 

      * Примечание: написание 1 √3 может стоить вам баллов, поэтому вместо этого используйте √3 3 (подробнее см. Рациональные знаменатели).

      Быстрый набросок

      Еще один способ запомнить значения 30° и 60° — сделать быстрый набросок:

      Начертите треугольник со стороной 2  

      Разрезать пополам. Пифагор говорит, что новая сторона равна √3

      .

      1 2 + (√3) 2 = 2 2

      1 + 3 = 4

       
      Затем используйте sohcahtoa для sin, cos или tan  

      Пример: sin(30°)

      Синус: soh cahtoa

      синус противоположно деленному на гипотенузу

      sin(30°) = напротив гипотенуза знак равно 1 2

       

      Весь круг

      Для всего круга нам нужны значения в каждом квадранте с правильным знаком плюс или минус в декартовых координатах:

       

      Обратите внимание, что потому что первое, а sin второе, поэтому получается (cos, sin) :

      Сохранить как PDF

      Пример: Что такое cos(330°) ?

       

      Сделайте такой набросок, и мы увидим, что это «длинное» значение:   √3 2

      А это тот же единичный круг в радианах .

      Пример: Что такое sin(7π/6) ?

       

      Подумайте «7π/6 = π + π/6», затем сделайте набросок.

      Затем мы видим, что это отрицательное и является «коротким» значением: −½

       

      7708, 7709, 7710, 7711, 8903, 8904, 8906, 8907, 8905, 8908

       

      Сноска: откуда берутся значения?

      Мы можем использовать уравнение x 2 + y 2 = 1, чтобы найти длины x и y (которые равны cos и sin , когда радиус равен 18 1912) :

      45 Degrees

      For 45 degrees, x and y are equal, so y=x :

      x 2 + x 2 = 1

      2x 2 = 1

      x 2 = ½

      х = у = √(½)

      60 градусов

      Возьмите равносторонний треугольник (все стороны равны и все углы равны 60°) и разделите его посередине.

      Сторона «x» теперь составляет ½ ,

      , а сторона «y» составляет:

      (½) 2 + y 2 = 1

      ¼ + y 2 = 1

      ¼ + y 2 = 1

      ¼ + y 2 = 1

      ¼ + y 2 = 1

      y 2 = 1-¼ = ¾

      y = √(¾)

      30 градусов

      30 ° равно 60 ° с перестановкой x и y, поэтому х = √(¾) и у = ½

      И:

      √1/2 = √2/4 = √2 √4 = √2 2

      Также:

      √3/4 = √3 √4 = √3 2

      И вот результат (как и раньше):

      Угол Кос Грех Tan=Sin/Cos
      30 ° √3 2 1 2 1 √3 знак равно √3 3
      45 ° √2 2 √2 2 1
      60 ° 1 2 √3 2 √3

       

      Значение sin left 40circ 35 rightcos left 19circ class 11 maths CBSE

      Ответ

      Verified

      189. \circ + 35’\], получаем 9\circ 25′} \right)\] равно \[\dfrac{{\sqrt 3}}{2}\].
      Правильный вариант — вариант (б).

      Примечание: Мы использовали унитарный метод для преобразования 35 минут и 25 минут в градусы. Унитарный метод — это метод, при котором сначала рассчитывается количество на единицу, а затем количество единиц умножается. Здесь мы сначала вычислили значение 1 минуты в градусах, а затем умножили его на 35 и 25, чтобы получить значение 35 минут и 25 минут в градусах соответственно.

      Недавно обновленные страницы

      Рассчитать изменение энтропии, связанное с конверсией класса 11 химии JEE_Main

      Закон, сформулированный доктором Нернстом, является первым законом термодинамики класса 11 химии JEE_Main

      Для реакции при rm0rm0rmC и нормальном давлении класса A 11 химия JEE_Main

      Двигатель, работающий между rm15rm0rm0rmC и rm2rm5rm0rmC класс 11 химия JEE_Main

      Для реакции rm2Clg в rmCrmlrm2rmg признаки 11 класса химии JEE_Main

      Изменение энтальпии перехода жидкой воды в химический класс 11 JEE_Main

      Рассчитайте изменение энтропии при переходе в химический класс 11 JEE_Main

      Закон, сформулированный доктором Нернстом, представляет собой Первый закон термодинамики 11-го класса химии JEE_Main

      Для реакция при rm0rm0rmC и нормальном давлении А химический класс 11 JEE_Main

      Двигатель, работающий между rm15rm0rm0rmC и rm2rm5rm0rmC химический класс 11 JEE_Main

      0003

      Изменение энтальпии перехода жидкой воды класса 11 химии JEE_Main

      Тенденции сомнения

      Sin Calculator — Sine

      ClickCalculators. com

      91
        90
      • Диаграмма
      • Примеры

      Калькулятор функции синуса

      Введите значение, например: 60, -30, пи/3, 3пи/2 и т. д.

      Угол:

      Использование калькулятора

      Чтобы использовать этот калькулятор, просто введите значение угла и нажмите «Рассчитать». Вы можете выбрать радианы (rad) или градусы (°) в качестве единицы измерения угла. Единицей по умолчанию является градус (°)

      Примеры допустимых входных значений

      • 30 → sin(30°) = 0,5
      • пи → грех(пи рад ) = 0
      • 3pi/4 → sin(3pi/4) = 0,707 …
      • 1/(2pi) → sin(1/(2pi) рад ) = 0,158 … (обратите внимание на скобки в знаменателе)
      • 1/2pi → sin(1/2pi рад ) = sin((1/2) x pi рад ) = 1 ровно

      Примечание: этот калькулятор принимает числа, дроби, ‘пи’, ‘π’, ‘+’, ‘-‘, ‘*’, ‘/’, ‘(‘, ‘)’ и некоторые (не все) комбинации их в качестве входных данных. Используйте его с осторожностью!

      Единица измерения:

      градусы (°)

      радианы (рад)

      Результат здесь!

      Тригонометрические функции. Таблица специальных углов

      х° x рад грех(х) кос(х) рыжевато-коричневый (х) КСК(х) сек(х) детская кроватка(х)
      0 0 1 0 0 1 0
      30° №/6 1/2 √3/2 √3/3 2 2√3/3 √3
      45° №/4 √2/2 √2/2 1 √2 √2 1
      60° №/3 √3/2 1/2 √3 2√3/3 2 √3/3
      90° №/2 1 0 (*) 1 0 0
      120° 2π/3 √3/2 -1/2 -√3 2√3/3 -2 -√3/3
      135° 3π/4 √2/2 -√2/2 -1 √2 -√2 -1
      150° 5π/6 1/2 -√3/2 -√3/3 2 -2√3/3 -√3
      180° 0 -1 0 0 -1 (*)
      210° 7π/6 -1/2 -√3/2 √3/3 -2 -2√3/3 -√3
      225° 5π/4 -√2/2 -√2/2 1 -√2 -√2 1
      240° 4π/3 -√3/2 -1/2 -√3 -2√3/3 -2 -√3/3
      270° 3π/2 -1 0 (*) -1 0 (*)
      300° 5π/3 -√3/2 1/2 -√3 -2√3/3 2 -√3/3
      315° 7π/4 -√2/2 √2/2 -1 -√2 √2 -1
      330° 11π/6 -1/2 √3/2 -√3/3 -2 2√3/3 -√3

      (*) означает неопределенное значение.

      Большая таблица синусов в градусах и радианах от 0 до 360 градусов с шагом 0,5 градуса

      град = рад1401 = sin
      0 = 0 = 0
      0.5 = π/360 = 0.0087
      1 = π/180 = 0.0175
      1.5 = π/120 = 0.0262
      2 = π/90 = 0.0349
      2.5 = π/72 = 0.0436
      3 = π/60 = 0.0523
      3.5 = 7π/360 = 0.061
      4 = π/45 = 0. 0698
      4.5 = π/40 = 0.0785
      5 = π/36 = 0.0872
      5.5 = 11π/360 = 0.0958
      6 = π/ 30 = 0.1045
      6.5 = 13π/360 = 0.1132
      7 = 7π/180 = 0.1219
      7.5 = π/24 = 0.1305
      8 = 2π/45 = 0.1392
      8.5 = 17π/360 = 0.1478
      9 = π/20 = 0.1564
      9.5 = 19π/360 = 0. 165
      10 = π/18 = 0.1736
      10.5 = 7π/120 = 0.1822
      11 = 11π/180 = 0.1908
      11.5 = 23π/360 = 0.1994
      12 = π/15 = 0.2079
      12.5 = 5π/72 = 0.2164
      13 = 13π/180 = 0.225
      13.5 = 3π/40 = 0.2334
      14 = 7π/90 = 0.2419
      14.5 = 29π/360 = 0.2504
      15 = π/12 = 0. 2588
      15.5 = 31π/360 = 0.2672
      16 = 4π/45 = 0.2756
      16.5 = 11π/120 = 0.284
      17 = 17π/180 = 0.2924
      17.5 = 7π/72 = 0.3007
      18 = π/10 = 0.309
      18.5 = 37π/360 = 0.3173
      19 = 19π/180 = 0.3256
      19.5 = 13π/120 = 0.3338
      20 = π/9 = 0.342
      20.5 = 41π/360 = 0. 3502
      21 = 7π/60 = 0.3584
      21.5 = 43π/360 = 0.3665
      22 = 11π/90 = 0.3746
      22.5 = π/8 = 0.3827
      23 = 23π/180 = 0.3907
      23.5 = 47π/360 = 0.3987
      24 = 2π/15 = 0.4067
      24.5 = 49π/360 = 0.4147
      25 = 5π/36 = 0.4226
      25.5 = 17π/120 = 0.4305
      26 = 13π/90 = 0. 4384
      26.5 = 53π/360 = 0.4462
      27 = 3π/20 = 0.454
      27.5 = 11π/72 = 0.4617
      28 = 7π/45 = 0.4695
      28.5 = 19π/120 = 0.4772
      29 = 29π/180 = 0.4848
      29.5 = 59π/360 = 0.4924
      30 = π/6 = 1/2
      30.5 = 61π/360 = 0.5075
      31 = 31π /180 = 0.515
      31.5 = 7π/40 = 0. 5225
      32 = 8π/45 = 0.5299
      32.5 = 13π/72 = 0.5373
      33 = 11π/60 = 0.5446
      33.5 = 67π/360 = 0.5519
      34 = 17π/90 = 0.5592
      34.5 = 23π/120 = 0.5664
      35 = 7π/36 = 0.5736
      35.5 = 71π/360 = 0.5807
      36 = π/5 = 0.5878
      36.5 = 73π/360 = 0.5948
      37 = 37π/180 = 0. 6018
      37.5 = 5π/24 = 0.6088
      38 = 19π/90 = 0.6157
      38.5 = 77π/360 = 0.6225
      39 = 13π/60 = 0.6293
      39.5 = 79π/360 = 0.6361
      40 = 2π/9 = 0.6428
      40.5 = 9π/40 = 0.6494
      41 = 41π/180 = 0.6561
      41.5 = 83π/360 = 0.6626
      42 = 7π/30 = 0.6691
      42.5 = 17π/72 = 0. 6756
      43 = 43π/180 = 0.682
      43.5 = 29π/120 = 0.6884
      44 = 11π/45 = 0.6947
      44.5 = 89π/360 = 0.7009
      45 = π/4 = √2/2
      45.5 = 91π/360 = 0.7133
      46 = 23π/90 = 0.7193
      46.5 = 31π/120 = 0.7254
      47 = 47π/180 = 0.7314
      47.5 = 19π/72 = 0.7373
      48 = 4π/15 = 0. 7431
      48.5 = 97π/ 360 = 0.749
      49 = 49π/180 = 0.7547
      49.5 = 11π/40 = 0.7604
      50 = 5π/18 = 0.766
      50.5 = 101π/360 = 0.7716
      51 = 17π/ 60 = 0.7771
      51.5 = 103π/360 = 0.7826
      52 = 13π/45 = 0.788
      52.5 = 7π/24 = 0.7934
      53 = 53π/180 = 0.7986
      53.5 = 107π/ 360 = 0. 8039
      54 = 3π/10 = 0.809
      54.5 = 109π/360 = 0.8141
      55 = 11π/36 = 0.8192
      55.5 = 37π/120 = 0.8241
      56 = 14π/ 45 = 0.829
      56.5 = 113π/360 = 0.8339
      57 = 19π/60 = 0.8387
      57.5 = 23π/72 = 0.8434
      58 = 29π/90 = 0.848
      58.5 = 13π/ 40 = 0.8526
      59 = 59π/180 = 0. 8572
      59.5 = 119π/360 = 0.8616
      60 = π/3 = √3/2
      60.5 = 121π/360 = 0.8704
      61 = 61π/180 = 0.8746
      61.5 = 41π/120 = 0.8788
      62 = 31π/90 = 0.8829
      62.5 = 25π/72 = 0.887
      63 = 7π/20 = 0.891
      63.5 = 127π/ 360 = 0.8949
      64 = 16π/45 = 0.8988
      64.5 = 43π/120 = 0. 9026
      65 = 13π/36 = 0.9063
      65.5 = 131π/360 = 0.91
      66 = 11π /30 = 0.9135
      66.5 = 133π/360 = 0.9171
      67 = 67π/180 = 0.9205
      67.5 = 3π/8 = 0.9239
      68 = 17π/45 = 0.9272
      68.5 = 137π /360 = 0.9304
      69 = 23π/60 = 0.9336
      69.5 = 139π/360 = 0.9367
      70 = 7π/18 = 0. 9397
      70.5 = 47π/120 = 0.9426
      71 = 71π /180 = 0.9455
      71.5 = 143π/360 = 0.9483
      72 = 2π/5 = 0.9511
      72.5 = 29π/72 = 0.9537
      73 = 73π/180 = 0.9563
      73.5 = 49π /120 = 0.9588
      74 = 37π/90 = 0.9613
      74.5 = 149π/360 = 0.9636
      75 = 5π/12 = 0.9659
      75.5 = 151π/360 = 0. 9681
      76 = 19π/45 = 0.9703
      76.5 = 17π/40 = 0.9724
      77 = 77π/180 = 0.9744
      77.5 = 31π/72 = 0.9763
      78 = 13π/30 = 0.9781
      78.5 = 157π/360 = 0.9799
      79 = 79π/180 = 0.9816
      79.5 = 53π/120 = 0.9833
      80 = 4π/9 = 0.9848
      80.5 = 161π/360 = 0.9863
      81 = 9π/20 = 0. 9877
      81.5 = 163π/360 = 0.989
      82 = 41π/90 = 0.9903
      82.5 = 11π/24 = 0.9914
      83 = 83π/180 = 0.9925
      83.5 = 167π/360 = 0.9936
      84 = 7π/15 = 0.9945
      84.5 = 169π/360 = 0.9954
      85 = 17π/36 = 0.9962
      85.5 = 19π/40 = 0.9969
      86 = 43π/90 = 0.9976
      86.5 = 173π/360 = 0. 9981
      87 = 29π/60 = 0.9986
      87.5 = 35π/72 = 0.999
      88 = 22π/45 = 0.9994
      88.5 = 59π/120 = 0.9997
      89 = 89π/180 = 0.9998
      89.5 = 179π/360 = 1
      90 = π/2 = 1
      90.5 = 181π/360 = 1
      91 = 91π/180 = 0.9998
      91.5 = 61π/120 = 0.9997
      92 = 23π/45 = 0. 9994
      92.5 = 37π/72 = 0.999
      93 = 31π/60 = 0.9986
      93.5 = 187π/360 = 0.9981
      94 = 47π/90 = 0.9976
      94.5 = 21π/40 = 0.9969
      95 = 19π/36 = 0.9962
      95.5 = 191π/360 = 0.9954
      96 = 8π/15 = 0.9945
      96.5 = 193π/360 = 0.9936
      97 = 97π/180 = 0.9925
      97.5 = 13π/24 = 0. 9914
      98 = 49π/90 = 0.9903
      98.5 = 197π/360 = 0.989
      99 = 11π/20 = 0.9877
      99.5 = 199π/360 = 0.9863
      100 = 5π/9 = 0.9848
      100.5 = 67π/120 = 0.9833
      101 = 101π/180 = 0.9816
      101.5 = 203π/360 = 0.9799
      102 = 17π/30 = 0.9781
      102.5 = 41π/72 = 0.9763
      103 = 103π/180 = 0. 9744
      103.5 = 23π/40 = 0.9724
      104 = 26π/45 = 0.9703
      104.5 = 209π/360 = 0.9681
      105 = 7π/12 = 0.9659
      105.5 = 211π/360 = 0.9636
      106 = 53π/90 = 0.9613
      106.5 = 71π/120 = 0.9588
      107 = 107π/180 = 0.9563
      107.5 = 43π/72 = 0.9537
      108 = 3π/5 = 0.9511
      108.5 = 217π/360 = 0. 9483
      109 = 109π/180 = 0.9455
      109.5 = 73π/120 = 0.9426
      110 = 11π/18 = 0.9397
      110.5 = 221π/360 = 0.9367
      111 = 37π/60 = 0.9336
      111.5 = 223π/360 = 0.9304
      112 = 28π/45 = 0.9272
      112.5 = 5π/8 = 0.9239
      113 = 113π/180 = 0.9205
      113.5 = 227π/360 = 0.9171
      114 = 19π/30 = 0. 9135
      114.5 = 229π/360 = 0.91
      115 = 23π/36 = 0.9063
      115.5 = 77π/120 = 0.9026
      116 = 29π/45 = 0.8988
      116.5 = 233π/360 = 0.8949
      117 = 13π/20 = 0.891
      117.5 = 47π/72 = 0.887
      118 = 59π/90 = 0.8829
      118.5 = 79π/120 = 0.8788
      119 = 119π/180 = 0.8746
      119.5 = 239π/360 = 0. 8704

      deg = rad = sin
      120 = 2π/3 = √3/2
      120.5 = 241π/360 = 0.8616
      121 = 121π/180 = 0.8572
      121.5 = 27π/40 = 0.8526
      122 = 61π/90 = 0.848
      122.5 = 49π/72 = 0.8434
      123 = 41π/60 = 0.8387
      123.5 = 247π/360 = 0.8339
      124 = 31π/45 = 0. 829
      124.5 = 83π/120 = 0.8241
      125 = 25π/36 = 0.8192
      125.5 = 251π/360 = 0.8141
      126 = 7π/10 = 0.809
      126.5 = 253π/360 = 0.8039
      127 = 127π/180 = 0.7986
      127.5 = 17π/24 = 0.7934
      128 = 32π/45 = 0.788
      128.5 = 257π/360 = 0.7826
      129 = 43π/60 = 0.7771
      129.5 = 259π/360 = 0. 7716
      130 = 13π/18 = 0.766
      130.5 = 29π/40 = 0.7604
      131 = 131π/180 = 0.7547
      131.5 = 263π/360 = 0.749
      132 = 11π/15 = 0.7431
      132.5 = 53π/72 = 0.7373
      133 = 133π/180 = 0.7314
      133.5 = 89π/120 = 0.7254
      134 = 67π/90 = 0.7193
      134.5 = 269π/360 = 0.7133
      135 = 3π/4 = √2/2
      135. 5 = 271π/360 = 0.7009
      136 = 34π/45 = 0.6947
      136.5 = 91π/120 = 0.6884
      137 = 137π/180 = 0.682
      137.5 = 55π/72 = 0.6756
      138 = 23π/30 = 0.6691
      138.5 = 277π/360 = 0.6626
      139 = 139π/180 = 0.6561
      139.5 = 31π/40 = 0.6494
      140 = 7π/9 = 0.6428
      140.5 = 281π/360 = 0. 6361
      141 = 47π/60 = 0.6293
      141.5 = 283π/360 = 0.6225
      142 = 71π/90 = 0.6157
      142.5 = 19π/24 = 0.6088
      143 = 143π/180 = 0.6018
      143.5 = 287π/360 = 0.5948
      144 = 4π/5 = 0.5878
      144.5 = 289π/360 = 0.5807
      145 = 29π/36 = 0.5736
      145.5 = 97π/120 = 0.5664
      146 = 73π/90 = 0. 5592
      146.5 = 293π/360 = 0.5519
      147 = 49π/60 = 0.5446
      147.5 = 59π/72 = 0.5373
      148 = 37π/45 = 0.5299
      148.5 = 33π/40 = 0.5225
      149 = 149π/180 = 0.515
      149.5 = 299π/360 = 0.5075
      150 = 5π/6 = 1/2
      150.5 = 301π/360 = 0.4924
      151 = 151π/180 = 0.4848
      151.5 = 101π/120 = 0. 4772
      152 = 38π/ 45 = 0.4695
      152.5 = 61π/72 = 0.4617
      153 = 17π/20 = 0.454
      153.5 = 307π/360 = 0.4462
      154 = 77π/90 = 0.4384
      154.5 = 103π/ 120 = 0.4305
      155 = 31π/36 = 0.4226
      155.5 = 311π/360 = 0.4147
      156 = 13π/15 = 0.4067
      156.5 = 313π/360 = 0.3987
      157 = 157π/180 = 0. 3907
      157.5 = 7π/8 = 0.3827
      158 = 79π/90 = 0.3746
      158.5 = 317π/360 = 0.3665
      159 = 53π/60 = 0.3584
      159.5 = 319π/360 = 0.3502
      160 = 8π/9 = 0.342
      160.5 = 107π/120 = 0.3338
      161 = 161π/180 = 0.3256
      161.5 = 323π/360 = 0.3173
      162 = 9π/10 = 0.309
      162.5 = 65π/72 = 0. 3007
      163 = 163π/180 = 0.2924
      163.5 = 109π/120 = 0.284
      164 = 41π/45 = 0.2756
      164.5 = 329π/360 = 0.2672
      165 = 11π/12 = 0.2588
      165.5 = 331π/360 = 0.2504
      166 = 83π/90 = 0.2419
      166.5 = 37π/40 = 0.2334
      167 = 167π/180 = 0.225
      167.5 = 67π/72 = 0.2164
      168 = 14π/15 = 0. 2079
      168.5 = 337π/360 = 0.1994
      169 = 169π/180 = 0.1908
      169.5 = 113π/120 = 0.1822
      170 = 17π/18 = 0.1736
      170.5 = 341π/360 = 0.165
      171 = 19π/20 = 0.1564
      171.5 = 343π/360 = 0.1478
      172 = 43π/45 = 0.1392
      172.5 = 23π/24 = 0.1305
      173 = 173π/180 = 0.1219
      173. 5 = 347π/360 = 0.1132
      174 = 29π/30 = 0.1045
      174.5 = 349π/360 = 0.0958
      175 = 35π/36 = 0.0872
      175.5 = 39π/40 = 0.0785
      176 = 44π/45 = 0.0698
      176.5 = 353π/360 = 0.061
      177 = 59π/60 = 0.0523
      177.5 = 71π/72 = 0.0436
      178 = 89π/90 = 0.0349
      178.5 = 119π/120 = 0. 0262
      179 = 179π/ 180 = 0.0175
      179.5 = 359π/360 = 0.0087
      180 = π = 0
      180.5 = 361π/360 = -0.0087
      181 = 181π/180 = -0.0175
      181.5 = 121π/120 = -0.0262
      182 = 91π/90 = -0.0349
      182.5 = 73π/72 = -0.0436
      183 = 61π/60 = -0.0523
      183.5 = 367π/360 = -0.061
      184 = 46π/45 = -0. 0698
      184.5 = 41π/40 = -0.0785
      185 = 37π/36 = -0.0872
      185.5 = 371π/360 = -0.0958
      186 = 31π/30 = -0.1045
      186.5 = 373π/360 = -0.1132
      187 = 187π/180 = -0.1219
      187.5 = 25π/24 = -0.1305
      188 = 47π/45 = -0.1392
      188.5 = 377π/360 = — 0.1478
      189 = 21π/20 = -0.1564
      189. 5 = 379π/360 = -0.165
      190 = 19π/18 = -0.1736
      190.5 = 127π/120 = -0.1822
      191 = 191π/180 = -0.1908
      191.5 = 383π/360 = -0.1994
      192 = 16π/15 = -0.2079
      192.5 = 77π/72 = -0.2164
      193 = 193π/180 = -0.225
      193.5 = 43π/40 = -0.2334
      194 = 97π/90 = -0.2419
      194.5 = 389π/360 = -0. 2504
      195 = 13π/12 = -0.2588
      195.5 = 391π/360 = -0.2672
      196 = 49π /45 = -0.2756
      196.5 = 131π/120 = -0.284
      197 = 197π/180 = -0.2924
      197.5 = 79π/72 = -0.3007
      198 = 11π/10 = -0.309
      198.5 = 397π/360 = -0.3173
      199 = 199π/180 = -0.3256
      199.5 = 133π/120 = -0.3338
      200 = 10π/9 = -0. 342
      200.5 = 401π/360 = -0.3502
      201 = 67π/60 = -0.3584
      201.5 = 403π/360 = -0.3665
      202 = 101π/90 = -0.3746
      202.5 = 9π/8 = -0.3827
      203 = 203π/180 = -0.3907
      203.5 = 407π/360 = -0.3987
      204 = 17π/15 = -0.4067
      204.5 = 409π/360 = -0.4147
      205 = 41π/36 = -0.4226
      205. 5 = 137π/120 = — 0.4305
      206 = 103π/90 = -0.4384
      206.5 = 413π/360 = -0.4462
      207 = 23π/20 = -0.454
      207.5 = 83π/72 = -0.4617
      208 = 52π/45 = -0.4695
      208.5 = 139π/120 = -0.4772
      209 = 209π/180 = -0.4848
      209.5 = 419π/360 = -0.4924
      210 = 7π/6 = -1/2
      210.5 = 421π /360 = -0. 5075
      211 = 211π/180 = -0.515
      211.5 = 47π/40 = -0.5225
      212 = 53π/45 = -0.5299
      212.5 = 85π/72 = -0.5373
      213 = 71π/ 60 = -0.5446
      213.5 = 427π/360 = -0.5519
      214 = 107π/90 = -0.5592
      214.5 = 143π/120 = -0.5664
      215 = 43π/36 = -0.5736
      215.5 = 431π/360 = -0.5807
      216 = 6π/5 = -0. 5878
      216.5 = 433π/360 = -0.5948
      217 = 217π/180 = -0.6018
      217.5 = 29π/24 = -0.6088
      218 = 109π/90 = -0.6157
      218.5 = 437π/360 = -0.6225
      219 = 73π/60 = -0.6293
      219.5 = 439π/360 = -0.6361
      220 = 11π/9 = -0.6428
      220.5 = 49π/40 = -0.6494
      221 = 221π/180 = -0.6561
      221. 5 = 443π/360 = -0.6626
      222 = 37π/30 = -0.6691
      222.5 = 89π/72 = -0.6756
      223 = 223π/180 = -0.682
      223.5 = 149π/120 = -0.6884
      224 = 56π/45 = -0.6947
      224.5 = 449π/360 = -0.7009
      225 = 5π/4 = — √2/2
      225.5 = 451π/360 = -0.7133
      226 = 113π/90 = -0.7193
      226.5 = 151π/120 = -0. 7254
      227 = 227π/180 = -0.7314
      227.5 = 91π/72 = -0.7373
      228 = 19π/15 = -0.7431
      228.5 = 457π/360 = -0.749
      229 = 229π/180 = -0.7547
      229.5 = 51π/40 = -0.7604
      230 = 23π/18 = -0.766
      230.5 = 461π/360 = -0.7716
      231 = 77π/60 = -0.7771
      231.5 = 463π/360 = -0.7826
      232 = 58π/45 = -0. 788
      232.5 = 31π /24 = -0.7934
      233 = 233π/180 = -0.7986
      233.5 = 467π/360 = -0.8039
      234 = 13π/10 = -0.809
      234.5 = 469π/360 = -0.8141
      235 = 47π/36 = -0.8192
      235.5 = 157π/120 = -0.8241
      236 = 59π/45 = -0.829
      236.5 = 473π/360 = -0.8339
      237 = 79π/60 = -0.8387
      237. 5 = 95π/72 = -0.8434
      238 = 119π/90 = -0.848
      238.5 = 53π/40 = -0.8526
      239 = 239π/180 = -0.8572
      239.5 = 479π/360 = -0.8616

      deg = rad = sin
      240 = 4π/3 = -√3/2
      240.5 = 481π/360 = -0.8704
      241 = 241π/180 = -0.8746
      241.5 = 161π/120 = -0.8788
      242 = 121π/90 = -0. 8829
      242.5 = 97π/72 = -0.887
      243 = 27π/20 = -0.891
      243.5 = 487π/360 = -0.8949
      244 = 61π/ 45 = -0.8988
      244.5 = 163π/120 = -0.9026
      245 = 49π/36 = -0.9063
      245.5 = 491π/360 = -0.91
      246 = 41π/30 = -0.9135
      246.5 = 493π/360 = -0.9171
      247 = 247π/180 = -0.9205
      247. 5 = 11π/8 = -0.9239
      248 = 62π/45 = -0.9272
      248.5 = 497π/360 = -0.9304
      249 = 83π/60 = -0.9336
      249.5 = 499π/360 = -0.9367
      250 = 25π/18 = -0.9397
      250.5 = 167π/120 = -0.9426
      251 = 251π/180 = -0.9455
      251.5 = 503π/360 = -0.9483
      252 = 7π/5 = -0.9511
      252.5 = 101π/72 = -0. 9537
      253 = 253π/180 = -0.9563
      253.5 = 169π/120 = -0.9588
      254 = 127π/90 = -0.9613
      254.5 = 509π/360 = -0.9636
      255 = 17π/12 = -0.9659
      255.5 = 511π/360 = -0.9681
      256 = 64π/45 = -0.9703
      256.5 = 57π/40 = -0.9724
      257 = 257π/180 = -0.9744
      257.5 = 103π/72 = -0.9763
      258 = 43π/30 = -0. 9781
      258.5 = 517π/360 = -0.9799
      259 = 259π/180 = -0.9816
      259.5 = 173π/120 = -0.9833
      260 = 13π/9 = -0.9848
      260.5 = 521π/360 = -0.9863
      261 = 29π/ 20 = -0.9877
      261.5 = 523π/360 = -0.989
      262 = 131π/90 = -0.9903
      262.5 = 35π/24 = -0.9914
      263 = 263π/180 = -0.9925
      263. 5 = 527π/360 = -0.9936
      264 = 22π/15 = -0.9945
      264.5 = 529π/360 = -0.9954
      265 = 53π/36 = -0.9962
      265.5 = 59π/40 = -0.9969
      266 = 133π/90 = -0.9976
      266.5 = 533π/360 = -0.9981
      267 = 89π/60 = -0.9986
      267.5 = 107π/72 = -0.999
      268 = 67π/45 = -0.9994
      268.5 = 179π/120 = -0. 9997
      269 = 269π/180 = -0.9998
      269.5 = 539π/360 = -1
      270 = 3π/2 = -1
      270.5 = 541π/360 = — 1
      271 = 271π/180 = -0.9998
      271.5 = 181π/120 = -0.9997
      272 = 68π/45 = -0.9994
      272.5 = 109π/72 = -0.999
      273 = 91π/60 = — 0.9986
      273.5 = 547π/360 = -0.9981
      274 = 137π/90 = -0. 9976
      274.5 = 61π/40 = -0.9969
      275 = 55π/36 = -0.9962
      275.5 = 551π/360 = -0.9954
      276 = 23π/15 = -0.9945
      276.5 = 553π/360 = -0.9936
      277 = 277π/180 = -0.9925
      277.5 = 37π/24 = -0.9914
      278 = 139π/90 = -0.9903
      278.5 = 557π/360 = -0.989
      279 = 31π/20 = -0.9877
      279. 5 = 559π/360 = -0.9863
      280 = 14π/9 = -0.9848
      280.5 = 187π /120 = -0.9833
      281 = 281π/180 = -0.9816
      281.5 = 563π/360 = -0.9799
      282 = 47π/30 = -0.9781
      282.5 = 113π/72 = -0.9763
      283 = 283π/180 = -0.9744
      283.5 = 63π/40 = -0.9724
      284 = 71π/45 = -0.9703
      284.5 = 569π/360 = -0. 9681
      285 = 19π/12 = -0.9659
      285.5 = 571π/360 = -0.9636
      286 = 143π/90 = -0.9613
      286.5 = 191π/120 = -0.9588
      287 = 287π/180 = -0.9563
      287.5 = 115π/72 = — 0.9537
      288 = 8π/5 = -0.9511
      288.5 = 577π/360 = -0.9483
      289 = 289π/180 = -0.9455
      289.5 = 193π/120 = -0.9426
      290 = 29π/18 = -0. 9397
      290.5 = 581π/360 = -0.9367
      291 = 97π/60 = -0.9336
      291.5 = 583π/360 = -0.9304
      292 = 73π/45 = -0.9272
      292.5 = 13π/8 = -0.9239
      293 = 293π/180 = -0.9205
      293.5 = 587π/360 = -0.9171
      294 = 49π/30 = -0.9135
      294.5 = 589π/360 = -0.91
      295 = 59π /36 = -0.9063
      295. 5 = 197π/120 = -0.9026
      296 = 74π/45 = -0.8988
      296.5 = 593π/360 = -0.8949
      297 = 33π/20 = -0.891
      297.5 = 119π/72 = -0.887
      298 = 149π/90 = -0.8829
      298.5 = 199π/120 = -0.8788
      299 = 299π/180 = -0.8746
      299.5 = 599π/360 = -0.8704
      300 = 5π/3 = -√3/2
      300.5 = 601π/360 = -0. 8616
      301 = 301π/180 = -0.8572
      301.5 = 67π/40 = -0.8526
      302 = 151π/90 = -0.848
      302.5 = 121π/72 = -0.8434
      303 = 101π/60 = -0.8387
      303.5 = 607π/360 = -0.8339
      304 = 76π/45 = -0.829
      304.5 = 203π/120 = — 0.8241
      305 = 61π/36 = -0.8192
      305.5 = 611π/360 = -0.8141
      306 = 17π/10 = -0. 809
      306.5 = 613π/360 = -0.8039
      307 = 307π/180 = -0.7986
      307.5 = 41π/24 = -0.7934
      308 = 77π/45 = -0.788
      308.5 = 617π/360 = -0.7826
      309 = 103π/60 = -0.7771
      309.5 = 619π/360 = -0.7716
      310 = 31π/18 = -0.766
      310.5 = 69π/40 = -0.7604
      311 = 311π/180 = -0.7547
      311. 5 = 623π/360 = -0.749
      312 = 26π/ 15 = -0.7431
      312.5 = 125π/72 = -0.7373
      313 = 313π/180 = -0.7314
      313.5 = 209π/120 = -0.7254
      314 = 157π/90 = -0.7193
      314.5 = 629π/360 = -0.7133
      315 = 7π/4 = -√2/2
      315.5 = 631π/360 = -0.7009
      316 = 79π/45 = -0.6947
      316.5 = 211π/120 = -0. 6884
      317 = 317π/180 = -0.682
      317.5 = 127π/72 = -0.6756
      318 = 53π/30 = -0.6691
      318.5 = 637π/360 = -0.6626
      319 = 319π/180 = -0.6561
      319.5 = 71π/40 = -0.6494
      320 = 16π/9 = -0.6428
      320.5 = 641π/360 = -0.6361
      321 = 107π/60 = -0.6293
      321.5 = 643π/360 = -0.6225
      322 = 161π/90 = -0. 6157
      322.5 = 43π/24 = -0.6088
      323 = 323π/180 = -0.6018
      323.5 = 647π/360 = -0.5948
      324 = 9π/5 = — 0.5878
      324.5 = 649π/360 = -0.5807
      325 = 65π/36 = -0.5736
      325.5 = 217π/120 = -0.5664
      326 = 163π/90 = -0.5592
      326.5 = 653π/360 = -0.5519
      327 = 109π/60 = -0.5446
      327. 5 = 131π/72 = -0.5373
      328 = 82π/45 = -0.5299
      328.5 = 73π/40 = -0.5225
      329 = 329π/180 = -0.515
      329.5 = 659π/360 = -0.5075
      330 = 11π/6 = -1/2
      330.5 = 661π/360 = -0.4924
      331 = 331π/180 = -0.4848
      331.5 = 221π/ 120 = -0.4772
      332 = 83π/45 = -0.4695
      332.5 = 133π/72 = -0. 4617
      333 = 37π/20 = -0.454
      333.5 = 667π/360 = -0.4462
      334 = 167π/90 = -0.4384
      334.5 = 223π/120 = -0.4305
      335 = 67π/36 = -0.4226
      335.5 = 671π/360 = -0.4147
      336 = 28π/15 = -0.4067
      336.5 = 673π/360 = -0.3987
      337 = 337π/180 = -0.3907
      337.5 = 15π/8 = -0.3827
      338 = 169π/90 = -0. 3746
      338.5 = 677π/360 = -0.3665
      339 = 113π/60 = -0.3584
      339.5 = 679π/360 = -0.3502
      340 = 17π/9 = -0.342
      340.5 = 227π/120 = -0.3338
      341 = 341π/180 = -0.3256
      341.5 = 683π/360 = -0.3173
      342 = 19π/10 = -0.309
      342.5 = 137π/72 = -0.3007
      343 = 343π/180 = -0.2924
      343. 5 = 229π/120 = -0.284
      344 = 86π/45 = -0.2756
      344.5 = 689π/360 = -0.2672
      345 = 23π/12 = -0.2588
      345.5 = 691π/360 = -0.2504
      346 = 173π/90 = -0.2419
      346.5 = 77π/40 = -0.2334
      347 = 347π/180 = -0.225
      347.5 = 139π/72 = -0.2164
      348 = 29π/15 = -0.2079
      348.5 = 697π/ 360 = -0. 1994
      349 = 349π/180 = -0.1908
      349.5 = 233π/120 = -0.1822
      350 = 35π/18 = -0.1736
      350.5 = 701π/360 = -0.165
      351 = 39π/20 = -0.1564
      351.5 = 703π/360 = -0.1478
      352 = 88π/45 = -0.1392
      352.5 = 47π/24 = -0.1305
      353 = 353π/180 = -0.1219
      353.5 = 707π/360 = -0.1132
      354 = 59π/30 = -0. 1045
      354.5 = 709π/360 = -0.0958
      355 = 71π/36 = -0.0872
      355.5 = 79π/40 = -0.0785
      356 = 89π/45 = -0.0698
      356.5 = 713π/360 = -0.061
      357 = 119π/60 = -0.0523
      357.5 = 143π/72 = -0.0436
      358 = 179π/90 = -0.0349
      358.5 = 239π/120 = -0.0262
      359 = 359π/180 = -0.0175
      360 = = 0

      SINE Расчеты

      SIN (290 °) SIN (-437π/360 RAD 9142 ).

    • Теория сложения вероятностей: Математическое Бюро. Страница 404

      Теоремы сложения и умножения вероятностей. Примеры решения задач

      Теоремы сложения и умножения вероятностей

      Основные понятия
      События называются несовместными, если появление одного из них исключает появление других событий в одном и том же испытании. В противном случае они называются совместными.
      Полной группой называют совокупность событий, объединение которых есть событие достоверное.
      Противоположными называют два единственно возможных события, образующих полную группу.
      События называются зависимыми, если вероятность появления одного из них зависит от наступления или ненаступления других событий.
      События называются независимыми, если вероятность одного из них не зависит от наступления или ненаступления других.
      Теорема сложения вероятностей несовместных событий
      Р(A+B)=Р(A)+Р(B),
      где А, В — несовместные события.

      Теорема сложения вероятностей совместных событий
      Р(A+B)=Р(A)+Р(B)-P(AB), где А и В — совместные события.


      Теорема умножения вероятностей независимых событий
      ,
      где А и В независимые события.
      Теорема умножения вероятностей зависимых событий
      Р(АВ)=Р(А)РA(B),
      где РA(B) — вероятность наступления события В при условии, что произошло событие А; А и В- зависимые события.

      Задача 1.
      Стрелок производит два выстрела по мишени. Вероятность попадания при каждом выстреле 0,8. Составить полную группу событий и найти их вероятности.
      Решение.
      Испытание — Производится два выстрела по мишени.
      Событие А — оба раза промахнулся.
      Событие В — попал один раз.
      Событие С — оба раза попал.
      .

      Контроль: P(A) + P(B) + P(C) = 1.
      Задача 2.
      Согласно прогнозу метеорологов Р(дождь)=0,4; Р(ветер)=0,7; Р(дождь и ветер)=0,2. Какова вероятность того, что будет дождь или ветер?
      Решение. По теореме сложения вероятностей и в силу совместности предложенных событий имеем:
      Р(дождь или ветер или то и другое)=Р(дождь) +Р(ветер) –Р(дождь и ветер)=0,4+0,7-0,2=0,9.
      Задача 3.
      На станции отправления имеется 8 заказов на отправку товара: пять – внутри страны, а три – на экспорт. Какова вероятность того, что два выбранных наугад заказа окажутся предназначенными для потребления внутри страны?
      Решение. Событие А – первый взятый наугад заказ – внутри страны. Событие В – второй тоже предназначен для внутреннего потребления. Нам необходимо найти вероятность  Тогда по теореме об умножении вероятностей зависимых событий имеем

      Задача 4.
      Из партии изделий товаровед наудачу отбирает изделия высшего сорта. Вероятность того, что выбранная вещь окажется высшего сорта равна, 0,8; первого сорта – 0,7; второго сорта – 0,5. Найти вероятность того, что из трех наудачу отобранных изделий будут:
      а) только два высшего сорта;
      б) все разные.
      Решение. Пусть событие  — изделие высшего сорта; событие — изделие первого сорта; событие — изделие второго сорта.
      По условию задачи ; ;  События — независимы.
      а) Событие А – только два изделия высшего сорта будет выглядеть так тогда

      б) Событие В – все три изделия различны — выразим так:, тогда .
      Задача 5.
      Вероятности  попадания в цель при стрельбе из трех орудий таковы: p1=0,8;  p2=0,7;  p3=0,9. Найти вероятность хотя бы одного попадания (событие А) при одном залпе из всех орудий.
      Решение. Вероятность попадания в цель каждым из орудий не зависит от результатов стрельбы из других орудий, поэтому рассматриваемые события  (попадание первого орудия),  (попадание второго орудия) и  (попадание третьего орудия) независимы в совокупности.
      Вероятности событий, противоположных событиям  (т.е. вероятности промахов), соответственно равны:

      Искомая вероятность
      Задача 6.
      В типографии имеется 4 печатных машины. Для каждой машины вероятность того, что она работает в данный момент, равна 0,9. Найти вероятность того, что в данный момент работает хотя бы одна машина (событие А).
      Решение. События «машина работает» и «машина не работает» (в данный момент) – противоположные, поэтому сумма их вероятностей равна единице:
      Отсюда вероятность того, что машина в данный момент не работает, равна
      Искомая вероятность . Задача 7. В читальном зале имеется 6 учебников по теории вероятностей , из которых три в переплете . Библиотекарь наудачу взял два учебника. Найти вероятность того, что оба учебника окажутся в переплете.

      Решение. Рассмотрим следующие события:
      А1- первый взятый учебник в переплете;
      A2- второй взятый учебник в переплете.
      Событие, состоящее в том, что оба взятых учебника в переплете . События А1 и А2 являются зависимыми, так как вероятность наступления события А2 зависит от наступления события А1. Для решения указанной задачи воспользуемся теоремой умножения вероятностей зависимых событий: .
      Вероятность наступления события А1  p(A1) в соответствии с классическим определением вероятности:
      P(A1)=m/n=3/6=0,5.
      Вероятность наступления события А2 определяется условной вероятностью наступления события А2 при условии наступления события А1 , т.е. (A2)==0,4.
      Тогда искомая вероятность наступления события:
      P(A)=0,5*0,4=0,2.

      Основные теоремы. Теорема сложения вероятностей, Теорема умножения вероятностей, Формула полной вероятности, Формула Байеса

       |  Основные теоремы  |   |   |   |   |   |   | 

       

       

      2.       Основные теоремы

      2.1. Теоремы сложения и умножения вероятностей

      События А и В называются несовместными, если они не могут произойти одновременно

      События А и В называются совместными, если они могут произойти одновременно.

      Суммой двух события А и В называется событие с, состоящее в выполнении события А или события В, или обоих вместе.

      Сумой нескольких событий называется событие, состоящее в том, что появится хотя бы одно из этих событий.

      Теорема сложения вероятностей несовместных событий

       

      Теорема сложения вероятностей совместных событий

      В случае четырех и более события данная формула еще больше усложняется

      События А и В называются независимыми, если вероятность появления события А не зависит от появления события В и наоборот: вероятность события в не зависит от появления события А.

      События А и  В называются зависимыми, если  вероятность события В зависит от того появилось ли событие А или наоборот.

      Произведением двух события А и В называется событие С, состоящее в том, что события А и В появятся одновременно.

      Произведением нескольких событий называется событие, состоящее вы том, что данные события появятся одновременно.

      Теорема умножения вероятностей для независимых событий

       

      Теорема умножения вероятностей для зависимых событий

      Где — условная вероятность появления события В, при условии что появилось событие А.

       

      2.2. Вероятность появления хотя бы одного события

      Пусть события   независимые, причем

       

      Вероятность появления события А, состоящее в том, что появится хотя бы одно событие :

       

       

      2.3. Формула полной вероятности

      Вероятность события А, которое может наступить лишь  при появлении одного из несовместных событий (гипотез)  , образующих полную группу событий, равна сумму произведений вероятностей каждой из гипотез на соответствующую условную вероятность появления события А:

       

      Данная формула называется формулой полной вероятности

      2.4. Формула Байеса (Бейеса)

      Имеется полная группа несовместных гипотез . Вероятности этих гипотез до опыта известны и равны соответственно:  . Произведен опыт, в результате которого наблюдено появление события А. Вероятность того, что появилась i-ая гипотеза, при условии того, что произошло событие А

       , где вероятность события А находится с помощью формулы полной вероятности

      Данная формула и есть формула Байеса (Бейеса).

       

      Теория вероятности — Кафедра биоинформатики

      Мясникова Екатерина Марковна

       проф., к.ф.-м.н.

      Санкт-Петербургский государственный политехнический университет

      Кафедра прикладной математики

       

      Описание курса

      Цель изучения дисциплины — освоение основных понятий и методов теории вероятностей, развитие способностей к логическому мышлению, получение навыков построения вероятностных моделей и решения на их основе задач различного уровня сложности.

      Изучение курса ставит перед собой следующие задачи:

      • освоение основных понятий и методов теории вероятностей;

      • изучение основных методов решения вероятностных задач;

      • ознакомление с наиболее важными для приложений законами распределения вероятностей;

      • приобретение фундаментальных знаний по теории вероятностей для обеспечения освоения дисциплин, базирующихся на понятиях и методах теории вероятностей.

      Темы курса:

      1. Классическая модель вероятностного пространства

      Случайные события и соотношения между ними. Классическое определение вероятности. Геометрические вероятности. Простейшие комбинаторные теоремы. Условная вероятность. Теоремы умножения и сложения вероятностей. Независимость событий. Формулы полной вероятности и Байеса. Схема испытаний Бернулли. Биномиальный закон распределения вероятностей. Полиномиальная схема испытаний. Полиномиальный закон распределения вероятностей. Производящие функции распределений.

      2. Аксиоматическое построение теории вероятностей.

      Аксиома непрерывности и ее эквивалентность аксиоме счетной аддитивности. Свойства вероятности. Борелевская сигма-алгебра множеств вещественной прямой. Функция распределения на прямой. Борелевская сигма-алгебра множеств и функция распределения в пространстве. Способы задания вероятностных мер на построенных измеримых пространствах. Типы вероятностных мер.

      3. Случайные величины и случайные векторы

      Случайная величина, ее распределение вероятностей и функция распределения. Типы случайных величин. Борелевские функции. Случайный вектор, его распределение вероятностей и функция распределения. Независимость случайных величин. Законы рас-пределения функций случайных величин. Композиция (свертка) распределений.

      4. Числовые характеристики распределений случайных величин

      Математическое ожидание, его свойства и теорема о его вычислении. Дисперсия и ее свойства. Неравенство Чебышева. Математическое ожидание и дисперсия независимых случайных величин. Моменты высших порядков. Неравенства Гельдера, Йенсена и Ляпунова. Ковариационная матрица случайного вектора. Коэффициент корреляции. Условные распределения и условные математические ожидания.

      5. Производящие и характеристические функции случайных величин и векторов

      Производящие функции и факториальные моменты целочисленных неотрицательных случайных величин. Производящие функции случайных векторов. Характеристические функции случайных величин. Формула обращения. Теорема единственности. Теорема непрерывности. Семиинварианты случайных величин. Характеристические функции случайных векторов.

      6. Предельные теоремы

      Типы сходимости последовательности случайных величин. Закон больших чисел. Теорема Хинчина. Теорема Бернулли. Теорема Чебышева. Центральная предельная теорема. Теорема Леви. Локальная и интегральная теоремы Муавра-Лапласа. Теорема Ляпунова. Теорема Линдеберга. Следствия теоремы Линдеберга.

      Рекомендуемая литература

      1. А.А.Боровков. Теория вероятностей. М., Наука, 1986

      2. А.Ширяев Вероятность, М., МЦНМО , 2007

      3. Б.В.Гнеденко. Курс теории вероятностей. Москва Эдиториал. УРСС 2001.

      Список публикаций:
      1. V.Vitaly, M.Gursky, L.Panok, E.Myasnikova, A.Manu, G.Maria, Samsonova, J.Reinitz, and A.Samsonov (2011). Mechanisms of gap gene expression canalization in the Drosophila blastoderm. BMC Systems Biology, 5:118. doi:10.1186/1752-0509-5-118.
      2. E.Myasnikova, S.Surkova, G.Stein, A.Pisarev, M.Samsonova. (2011) A regression system for estimation of errors introduced by confocal imaging into gene expression data in situ. BMC Bioinformatics 12: 320, doi:10.1186/1471-2105-12-320.
      3. K.Kozlov, S.Surkova, E.Myasnikova, J.Reinitz, M.Samsonova . (2012) Modeling of Gap Gene Expression in Drosophila Kruppel Mutants. PLoS Comput Biol 8(8): e1002635. doi:10.1371/journal.pcbi.1002635

      Теорема сложения вероятностей совместных событий

      На прошлом уроке мы рассмотрели теорему сложения вероятностей только для несовместных событий. В случае, когда два события A и B – совместны, справедлива следующая теорема.

      Теорема

      Вероятность появления хотя бы одного из двух совместных событий равна сумме вероятностей этих событий без вероятности их совместного появления:

         (1)

      Доказательство

      Событие наступит, если наступит одно из трех несовместных событий: . По теореме сложения вероятностей несовместных событий имеем:

          (2)

      Событие A произойдет, если наступит одно из двух несовместных событий: . Вновь применяя теорему сложения вероятностей для несовместных событий, получаем , откуда

      .     (3)

      Аналогично для события B: , откуда

           (4)

      Теперь подставим (3) и (4) в формулу (2), отсюда получаем формулу сложения вероятностей совместных событий (1).

      Как вы уже поняли формула, которую я дал вам на прошлом уроке это лишь частный случай формулы (1). Действительно, если события несовместны, то их произведение – пустое множество, то есть невозможное событие. А вероятность невозможного события равна нулю.

      Вероятность суммы трех совместных событий

      Аналогично выражению (1) запишем вероятность суммы трех совместных событий:

         (5)

      Кстати, справедливость формул (1) и (5) можно наглядно проиллюстрировать:

      Также из выражения (1) можно получить формулу для вероятности произведения двух событий. Выходит:

           (6)

      ПРИМЕР 1. Бросаются две игральные кости. Какова вероятность появления хотя бы одной шестерки?

      Решение. Обозначим события: — появление шестерки на первой кости, — на второй кости. Понятно, что эти события совместные, т.е. шестерка может выпасть как на первой, так и на второй кости.

      а) Для вычислений воспользуемся формулой (1). Однако здесь возникла сложность, как вычислить вероятность произведения, т.е. вероятность того, что на каждой из двух костей выпали шестерки. По формуле классической вероятности, количество «удачных» комбинаций равно 1, а для вычисления числа всех равновозможных комбинаций используем правило произведения (комбинаторика):

      б) Рассмотрим другой способ решения, воспользовавшись следствием закона сложения вероятностей:

      Ответ: вероятность появления хоть одной шестерки равна 11/36 или 0,3056 или 30,56%

      На этом все! Всем Спасибо!

      Если кто-то не понял или не разобрался в теме или в примерах, задавайте вопросы в комментариях.

      Правила вероятности

      Условная вероятность

      Формула полной вероятности

      Формула Байеса

      Оценка вероятности в схеме испытаний Бернулли

      Мы можем применять правила вероятности для того, чтобы складывать и умножать вероятности.

      Например, у взрослого пациента все зубы сохранены, некоторые зубы отсутствуют или он беззубый; вероятности равны 0,67, 0,24 и 0,09 соответственно.

      • Правило сложения. Если два события, и , взаимоисключающие, несовместимые, то вероятность события или равна сумме их вероятностей:

        Вероятность того, что у пациента есть несколько зубов, равна 0,67 + 0,24 = 0,91.

      • Правило умножения. Если два события, и , независимы (т. е. возникновение одного события не влияет на возможность появления другого), то вероятность того, что оба события произойдут, равна произведению вероятности каждого:

        Например, если 2 не имеющих отношения друг к другу больных ожидают приема в кабинете хирургической стоматологии то вероятность того, что у обоих больных есть все зубы, равна 0,67 • 0.67 =  0,45.

      Условная вероятность

      Условная вероятность — вероятность одного события при условии, что другое событие уже произошло. 

      Пусть  — фиксированное вероятностное пространство. Пусть  — два случайных события, причём . Тогда условной вероятностью события при условии события называется

      Формула полной вероятности

      Пусть событие может наступать только при условии появления одного из событий , образующих полную систему событий. Тогда вероятность события равна сумме произведений вероятностей каждого из событий на соответствующую условную вероятность события :

      Эта формула носит название формулы полной вероятности.

      Формула Байеса

      Если вероятности событий до опыта были , то с учетом появления в результате опыта события условная вероятность вычисляется по формуле Байеса:

      Оценка вероятности в схеме испытаний Бернулли

      Мы приводим пример классического статистического рассуждения, которое полезно иметь в виду при анализе реальных данных. 

      Бытует мнение, что при рождении ребенка вероятность мальчика такая же, как и девочки. 

      Примем это за гипотезу. 

      Для её проверки имеется огромный статистический материал. 

      Воспользуемся данными по Швейцарии с 1871 по 1900 гг., когда там родилось человек и среди них мальчиков и девочек. 

      Согласуется ли гипотеза о равновероятности рождения мальчика и девочки с этими числами? 

      Условно назвав «успехом» рождение мальчика, поставим этот вопрос по-другому, обратившись к схеме Бернулли с вероятностью «успеха» . 

      Согласуется ли гипотеза с тем, что в серии из испытаний частота «успеха» оказалось равной 

      Очевидно, если вместо гипотезы выдвинуть, скажем, предположение о том, что , то это предположение будет сразу же отвергнуто как маловероятное (или даже невозможное). 

      Уместно спросить: почему? Ответ здесь можно дать, основываясь на том, что частота как случайная величина (обозначим её ) подчиняется известному закону распределения. 

      Эта величина имеет биномиальное распределение. При больших n имеет место нормальное приближение (в силу центральной предельной теоремы). 

      Воспользовавшись нормальным приближением и задавшись малым  (будем называть  уровнем значимости), можно утверждать, например, что

      с вероятностью, где   определяется из условия с помощью нормальной функции распределения

      ( называется квантилем уровня). Скажем,  отвечает , а  уже соответствует 

      Это легко проверить с помощью калькулятора вероятностных распределений STATISTICA. Вернемся к нашим числовым данным и гипотезе , согласно которым мы имеем значение

      Оно далеко выходит за границу 

      Какое же значение, основываясь на этих данных, следует приписать неизвестной вероятности ?

      Мы знаем, что по закону больших чисел есть предел частоты (при ), и при имеющемся у нас можно в качестве оценки взять уже приводившееся ранее значение . Эту оценку можно уточнить следующим образом. Поскольку всегда имеет место неравенство , получаем

      с вероятностью, не меньшей (точнее, допущение о том, что истинное значение лежит вне этих границ, означает наступление события, дополнительного к (2) и имеющего вероятность не больше ).

      В этом смысле можно утверждать, например, что  с вероятностью не меньшей 0.9973 (это получается при  с уровнем значимости ).

      Данное рассуждение приведено в книге Ю.А. Розанова «Теория вероятностей, случайные процессы и математическая статистика: Учебник для вузов», М.: Наука, Главная редакция физико-математической литературы.

      Связанные определения:
      Вероятность события
      Независимые повторные испытания Бернулли
      Независимые события

      В начало

      Содержание портала

      Теорема сложения вероятностей | matematicus.ru

      Теоремы сложения вероятностей

      Теорема 1
      Для любых двух событий А и В, вероятность равна выражению:

      Р(А + В) = Р(А) + Р(В)-Р(А·В)


      Теорема 2
      Вероятность появления одного из двух несовместных событий равна сумме вероятностей этих событий:

       Р(А + В) = Р(А) + Р(В)


      Следствие 1
      Если события А12,…,Аn образуют полную группу, то получаем

      P(A1+A2+…+An) = P(A1) + P(A2) + …+ P(An)

      Следствие 2
      Сумма вероятностей событий, образующих полную группу событий равна 1, т.е.

      P(A1) + P(A2) + …+ P(An) = 1

      Следствие 3
      Сумма противоположных событий равна 1, т. е.


      Пример 1
      Из колоды в 36 карт наудачу вынимается одна. Какова вероятность того, что будет вынута пика или туз?
      Решение
      Здесь события
      А«вытащили из колоды  карту масти пики»;
      В«вытащили из колоды туз»;
      А·В«вытащили из колоды пиковый туз».
      По теореме сложения вероятностей имеем:

      Р(А+В)=Р(А) + Р(В)-Р(А·В)

      Так как в колоде карт 4 туза и 9 карт, имеющие масть пики, то получаем вероятности

      P(A) = 4/36
      P(B) = 9/36

      Так как пиковый туз единственный в колоде карт, то вероятность Р(А·В) для события А·В«вытащили пиковый туз» равна 1/36

      Р(А·В)=1/36

      Искомая вероятность равна:

      Р(А + В)=Р(А)+Р(В)–Р(А·В)=

      =4/36+9/36+1/36=12/36=1/3


      Пример 2
      В ящике лежат 8 шаров, из которых 2 белых, 3 красных и 3 зеленых. Наугад берется один шар. Какова вероятность того, что этот шар цветной.
      Решение 
      А — «появление красного шара»

      P(A) = 3/8

      В — «появление зеленого шара»

      P(B) = 3/8

      А + В — «появление цветного шара»

      Р(А+В)=Р(А)+Р(В)=

      =3/8+3/8=6/8=3/4


      Пример 3
      Студент берет билет 1,2,3,4,5,6,7,8,9,10. Какова вероятность того,
      что он выберет билет с четным номером?
      Решение
      Номера чётных билетов: 2,4,6,8,10. Всего 5 билетов, следовательно, вероятность выбрать чётный билет равна:

      1/10+1/10+1/10+1/10+1/10=5/10=1/2

      Сложение и умножение вероятностей 9 класс онлайн-подготовка на Ростелеком Лицей

      Тема 22.

      Сложение и умножение вероятностей.

      Рассмотрим пример. Пусть в ящике находится двадцать кубиков: десять белых, четыре красных и шесть синих. Из ящика наугад вынимают один кубик. Рассмотрим такие события: Событие А – кубик оказался красным, Событие В – кубик оказался синим.

      События А и В не могут произойти одновременно. Говорят, что события А и В являются несовместными.

      Два события называют несовместными, если в одном и том же испытании они не могут произойти одновременно, то есть наступление одного из них исключает наступление другого.

      Пусть событие С означает, что извлечённый из ящика кубик оказался не белым (то есть красным или синим).

      Выясним, как вероятность события С связана с вероятностями каждого из событий А и В. Найдем вероятности событий А, В и С. Для каждого извлечения кубика из ящика равновозможными являются двадцать исходов. Из них для события А благоприятными являются четыре исхода, для события В – шесть исходов, для события С – десять исходов. Отсюда, вероятность события А равна 420(четырем двадцатым), вероятность события В – 620 (шести двадцатым), вероятность события С – 1020 (десяти двадцатым).

      Мы видим, что вероятность события С равна сумме вероятностей событий А и В.

      Итак, если событие С означает, что наступает одно из двух несовместных событий А или В, то вероятность события С равна сумме вероятностей событий А и В.

      Вообще

      Если событие С означает, что наступает одно из двух несовместных событий А или В, то вероятность события С равна сумме вероятностей события А и В.

      Пример первый. Есть десять экзаменационных билетов. Ученик вытянул один из них. Какова вероятность того, что номером билета является простое число, или число большее шести.

      Событие А — простое число: 4 благоприятных исхода из 10 возможных

      Это числа 2,3,5 и 7

      Событие B — число больше 8: 2 благоприятных исхода из 10 возможных

      Это 9 и 10

      Вероятность события А равна 0,4, а вероятность события В равна 0,2

      Событие С наступает тогда, когда наступает одно из событий A или В, которые являются несовместными. Значит, вероятность события С равна сумме вероятностей событий А и В, то есть

      Р(С)= Р(А)+ Р(В)=0,4+0,2=0,6

      При решении некоторых задач бывает удобно воспользоваться свойством вероятностей противоположных событий.

      Разъясним смысл этого понятия на примере бросания игрального кубика. Пусть событие А означает, что выпало шесть очков, Б – что выпало менее шести очков. Всякое наступление события А означает, что наступление Б не наступит. А наступление события Б означает, что событие А не наступит. В таких случаях говорят, что события А и Б – противоположные события.

      Найдем вероятности событий А и Б. Для события А благоприятным является один исход из шести равновозможных исходов. Для события Б – пять исходов из шести. Значит, Вероятность события А равна 16 (одной шестой), а вероятность события Б равна 56(пяти шестым). Нетрудно заметить, что их сумма равна единице.

      Итак, сумма вероятностей противоположных событий равна единице.

      Два события называются независимыми, если наступление одного из них не влияет на вероятность наступления другого события.

      Приведем пример. Пусть в одной из двух коробок находится восемнадцать шаров, три из которых красные, а в другой двадцать четыре шара, четыре из которых красные. Из каждой коробки наугад вынимают по одному шару. Какова вероятность того, что оба шара окажутся красными?

      Рассмотрим такие события: А – из первой коробки вынимают красный шар, Б- из второй коробки вынимают красный шар.

      Для события А благоприятными являются три исхода из восемнадцати, для события Б благоприятными являются четыре исхода из двадцати четырех. Значит, вероятность события А равна трем восемнадцатым, вероятность события Б равна четырем двадцати четвертым.

      Очевидно, что события А и Б являются независимыми. Рассмотрим событие, которое состоит в совместном появлении событий А и Б. Обозначим его буквой С.

      Благоприятными для события С являются те исходы, при которых оба вытянутых шара окажутся красными. Каждому из трех возможных извлечений красного шара из первой коробки соответствует четыре возможности извлечения красного шара из второй коробки, то есть число благоприятных исходов для события С, равно произведению три и четыре. Следовательно, вероятность извлечения двух шаров будет равна

      Итак,

      Если событие C означает совместное наступление событий A и B, то вероятность события C равна произведению вероятностей событий А и B.

      Правило сложения для определения вероятностей

      Что такое правило сложения вероятностей?

      Правило сложения для вероятностей описывает две формулы: одна для вероятности одного из двух взаимоисключающих событий, а другая — для вероятности двух не исключающих друг друга событий.

      Первая формула — это просто сумма вероятностей двух событий. Вторая формула — это сумма вероятностей двух событий за вычетом вероятности того, что оба они произойдут.

      Ключевые выводы

      • Правило сложения для вероятностей состоит из двух правил или формул, одна из которых учитывает два взаимоисключающих события, а другая — два не исключающих друг друга события.
      • Не исключающие друг друга означает, что между двумя рассматриваемыми событиями существует некоторое перекрытие, и формула компенсирует это путем вычитания вероятности перекрытия P (Y и Z) из суммы вероятностей Y и Z.
      • Теоретически первая форма правила является частным случаем второй формы.

      Формулы для правил сложения вероятностей —

      Математически вероятность двух взаимоисключающих событий обозначается:

      Взаимодействие с другими людьми п ( Y или Z ) знак равно п ( Y ) + п ( Z ) P (Y \ text {или} Z) = P (Y) + P (Z) P (Y или Z) = P (Y) + P (Z)

      Математически вероятность двух не исключающих друг друга событий обозначается следующим образом:

      Взаимодействие с другими людьми п ( Y или Z ) знак равно п ( Y ) + п ( Z ) — п ( Y и Z ) P (Y \ text {или} Z) = P (Y) + P (Z) — P (Y \ text {и} Z) P (Y или Z) = P (Y) + P (Z) −P (Y и Z)

      Что вам говорит правило сложения вероятностей?

      Чтобы проиллюстрировать первое правило правила сложения вероятностей, рассмотрим кубик с шестью гранями и шансами на выпадение 3 или 6.Поскольку шансы выпадения 3 равны 1 из 6, а шансы выпадения 6 также 1 из 6, вероятность выпадения 3 или 6 составляет:

      1/6 + 1/6 = 2/6 = 1/3

      Чтобы проиллюстрировать второе правило, рассмотрим класс, в котором 9 мальчиков и 11 девочек. В конце семестра 5 девочек и 4 мальчика получают оценку B. Если студент выбран случайно, каковы шансы, что он будет девочкой или четвертым? Поскольку шансы выбрать девушку составляют 11 из 20, шансы выбрать ученицу B равны 9 из 20, а шансы выбрать девушку, которая является ученицей B, равны 5/20, шансы выбрать девушку или ученицу B являются:

      11/20 + 9/20 — 5/20 = 15/20 = 3/4

      На самом деле два правила упрощаются до одного правила, второго.Это потому, что в первом случае вероятность двух взаимоисключающих событий равна 0. В примере с кубиком невозможно бросить одновременно 3 и 6 при одном броске одного кубика. Таким образом, эти два события исключают друг друга.

      Взаимная эксклюзивность

      Взаимоисключающие — это статистический термин, описывающий два или более событий, которые не могут совпадать. Обычно он используется для описания ситуации, когда возникновение одного результата заменяет другой. В качестве основного примера рассмотрим бросание игральных костей.Вы не можете бросить одновременно пятерку и тройку на одном кубике. Кроме того, получение тройки при начальном броске не влияет на то, дает ли последующий бросок пятерку. Все броски кубика — независимые события.

      Вероятностное правило сложения

      Если А и B являются двумя событиями в вероятностном эксперименте, то вероятность того, что произойдет одно из событий, равна:

      п ( А или B ) знак равно п ( А ) + п ( B ) — п ( А и B )

      Это можно представить в виде Диаграмма Венна в виде:

      п ( А ∪ B ) знак равно п ( А ) + п ( B ) — п ( А ∩ B )

      Если А и B два взаимоисключающие события , п ( А ∩ B ) знак равно 0 .Тогда вероятность того, что произойдет одно из событий, равна: п ( А или B ) знак равно п ( А ) + п ( B )

      Это может быть представлено на диаграмме Венна как:

      п ( А ∪ B ) знак равно п ( А ) + п ( B )

      Пример:

      Если вы вытащите одну карту из обычной колоды карт, какова вероятность того, что это будет туз или пика?

      Позволять Икс быть событием выбора туза и Y быть событием выбора лопаты.

      п ( Икс ) знак равно 4 52

      п ( Y ) знак равно 13 52

      Эти два события не исключают друг друга, так как есть один благоприятный исход, при котором карта может быть как тузом, так и пикой.

      п ( Икс и Y ) знак равно 1 52

      п ( Икс или Y ) знак равно 4 52 + 13 52 — 1 52 знак равно 16 52 знак равно 4 13

      правил сложения в вероятности и статистике

      Правила сложения важны с точки зрения вероятности.Эти правила предоставляют нам способ вычислить вероятность события « A или B, » при условии, что мы знаем вероятность A и вероятность B . Иногда «или» заменяется на U, символ из теории множеств, обозначающий объединение двух множеств. Точное правило сложения зависит от того, являются ли событие A и событие B взаимоисключающими или нет.

      Правило сложения для взаимоисключающих событий

      Если события A, и B, являются взаимоисключающими, то вероятность A или B является суммой вероятности A и вероятности B .Запишем это компактно следующим образом:

      P ( A или B ) = P ( A ) + P ( B )

      Обобщенное правило сложения для любых двух событий

      Вышеупомянутая формула может быть обобщена для ситуаций, когда события не обязательно могут быть взаимоисключающими. Для любых двух событий A и B вероятность A или B является суммой вероятности A и вероятности B за вычетом общей вероятности A, и . B :

      P ( A или B ) = P ( A ) + P ( B ) — P ( A и B )

      Иногда слово «и» заменяется на ∩, который является символом из теории множеств, обозначающим пересечение двух множеств.

      Правило сложения для взаимоисключающих событий на самом деле является частным случаем обобщенного правила. Это связано с тем, что если A, и B являются взаимоисключающими, то вероятность как A, и B равна нулю.

      Пример # 1

      Мы увидим примеры того, как использовать эти правила сложения. Предположим, мы берем карту из хорошо перемешанной стандартной колоды карт. Мы хотим определить вероятность того, что вытянутая карта — это двойка или лицевая карта.Событие «нарисована лицевая карта» является взаимоисключающим с событием «выпала двойка», поэтому нам просто нужно сложить вероятности этих двух событий вместе.

      Всего имеется 12 лицевых карт, поэтому вероятность вытягивания лицевой карты составляет 12/52. В колоде четыре двойки, поэтому вероятность вытащить двойку составляет 4/52. Это означает, что вероятность вытащить двойку или лицевую карту составляет 12/52 + 4/52 = 16/52.

      Пример # 2

      Теперь предположим, что мы берем карту из хорошо перемешанной стандартной колоды карт.Теперь мы хотим определить вероятность получения красной карты или туза. В этом случае два события не исключают друг друга. Червовый туз и бубновый туз являются элементами набора красных карт и набора тузов.

      Мы рассматриваем три вероятности, а затем объединяем их, используя обобщенное правило сложения:

      • Вероятность розыгрыша красной карточки 26/52
      • Вероятность выпадения туза 4/52
      • Вероятность получения красной карты и туза 2/52

      Это означает, что вероятность вытягивания красной карты или туза составляет 26/52 + 4/52 — 2/52 = 28/52.

      Что такое теоремы сложения и умножения о вероятности?

      Что такое теоремы сложения и умножения о вероятности?

      Теорема вероятности сложения и умножения

      Сформулируйте и докажите вероятностную теорему сложения и умножения на примерах

      Уравнение теоремы сложения и умножения

      Замечания:

      1. P (A + B) или P (A∪B) = Вероятность наступления событий A или B
        = Вероятность наступления событий A или B или обоих
        = Вероятность наступления хотя бы одного события A или B
      2. P (AB) или P (A∩B) = Вероятность совершения событий A и B вместе.

      (1) Когда события не являются взаимоисключающими:
      Если A и B — два не исключающих друг друга события, то
      P (A∪B) = P (A) + P (B) — P (A ∩B)
      или P (A + B) = P (A) + P (B) — P (AB)
      Для любых трех событий A, B, C
      P (A∪B∪C) = P (A) + P (B) + P (C) — P (A∩B) — P (B∩C) — P (C∩A) + P (A∩B∩C)
      или P (A + B + C) = P (A) + P (B) + P (C) — P (AB) — P (BC) — P (CA) + P (ABC)

      .

      (2) Когда события являются взаимоисключающими:
      Если A и B являются взаимоисключающими событиями, то
      n (A∩B) = 0 ⇒ P (A∩B) = 0
      ∴ P (A ∪B) = P (A) + P (B).
      Для любых трех событий A, B, C, которые являются взаимоисключающими,
      P (A∩B) = P (B∩C) = P (C∩A) = P (A∩B∩C) = 0
      ∴ P (A∪B∪C) = P (A) + P (B) + P (C).
      Вероятность наступления любого из нескольких взаимоисключающих событий равна сумме их вероятностей, , т.е. , если A 1 , A 2 ……… A n являются взаимоисключающими событиями, тогда
      P (A 1 + A 2 +… + A n ) = P (A 1 ) + P (A 2 ) + …… + P (A n )
      i.е. P (Σ A i ) = Σ P (A i ).

      (3) Когда события независимы:
      Если A и B являются независимыми событиями, то P (A∩B) = P (A) .P (B)
      ∴ P (A∪B) = П (А) + П (В) — П (А). П (В)

      (4) Некоторые другие теоремы

      1. Пусть A и B — два события, связанные со случайным экспериментом, тогда
      2. Обобщение теоремы сложения:
        Если A 1 , A 2 ……… A n — n событий, связанных со случайным экспериментом, то
      3. Неравенство Були: если A 1 , A 2 ……… A n — n событий, связанных со случайным экспериментом, то

        Эти результаты могут быть легко получены с помощью принципа математической индукции.

      Условная вероятность

      Пусть A и B — два события, связанные со случайным экспериментом. Тогда вероятность появления A при условии, что B уже произошло и P (B) ≠ 0, называется условной вероятностью и обозначается P (A / B).
      Таким образом, P (A / B) = вероятность появления A, учитывая, что B уже произошло.

      Аналогично, P (B / A) = вероятность появления B, при условии, что A уже произошло.

      Иногда P (A / B) также используется для обозначения вероятности появления A при возникновении B.Точно так же P (B / A) используется для обозначения вероятности появления B при возникновении A.

      Теорема вероятности умножения

      1. Если A, и B — два события, связанные со случайным экспериментом, то P (A∩B) = P (A) .P (B / A), если P ( A ) ≠ 0 или P (A∩B) = P (B) .P (A / B), если P (B) ≠ 0.
      2. Расширение теоремы умножения:
        Если A 1 , A 2 ……… A n — n событий, связанных со случайным экспериментом, то
        P (A 1 ∩A 2 ∩A 3 ∩… ∩A n ) = P (A 1 ) P (A 2 / A 1 ) P (A 3 / A 1 ∩A 2 ) …… P (A n / A 1 ∩A 2 ∩… ∩A n − 1 ),
        где P (A i / A 1 ∩A 2 ∩… ∩A i − 1 ) представляет собой условную вероятность события, учитывая, что события A 1 , A 2 ……… A i 1 уже произошли.
      3. Теоремы умножения для независимых событий:
        Если A, и B являются независимыми событиями, связанными со случайным экспериментом, то P (A∩B) = P (A) .P (B) т.е. вероятность одновременного наступления двух независимых событий равно произведению их вероятностей. По теореме умножения P (A∩B) = P (A) .P (B / A). Поскольку A, и B, являются независимыми событиями, поэтому P (B / A) = P (B). Следовательно, P (A∩B) = P (A).П (В).
      4. Расширение теоремы умножения для независимых событий:
        Если A 1 , A 2 ……… A n — независимые события, связанные со случайным экспериментом, то
        P (A 1 ∩A 2 ∩A 3 ∩… ∩A n ) = P (A 1 ) P (A 2 ) ..… P (A n ).
        По теореме умножения имеем
        P (A 1 ∩A 2 ∩A 3 ∩… ∩A n ) = P (A 1 ) P (A 2 / A 1 ) P (A 3 / A 1 ∩A 2 ) …… P (A n / A 1 ∩A 2 ∩… ∩A n − 1 )
        Так как A 1 , A 2 ……… A n-1 , A n являются независимыми событиями, поэтому
        P (A 2 / A 1 ) = P (A 2 ), P (A 3 / A 1 ∩A 2 ) = P (A 3 ), ……, P (A n / A 1 ∩A 2 ∩… ∩A n −1 ) = P (A n )
        Следовательно, P (A 1 ∩A 2 ∩A 3 ∩… ∩A n ) = P (A 1 ) P (A 2 )..… P (A n ).

      Вероятность хотя бы одного из n независимых событий:
      Если p 1 , p 2 ……… p n быть вероятностями наступления n независимых событий A 1 , A 2 ……… A n соответственно, затем

      Полная вероятность и правило Бая

      (1) Закон полной вероятности:
      Пусть S будет пространством выборки и пусть E 1 , E 2 ……… E n будет n взаимоисключающими и исчерпывающими событиями связанный со случайным экспериментом.Если A — любое событие, которое происходит с E 1 или E 2 или… или E n , то
      P (A) = P (E 1 ) P (A / E 1 ) + P (E 2 ) P (A / E 2 ) +… .. P (E n ) P (A / E n ).

      (2) Правило Бая:
      Пусть S будет пробным пространством, а E 1 , E 2 ……… E n n взаимоисключающими событиями, так что

      Мы можем думать о E i как причины, которые приводят к исходу эксперимента.Вероятности P (E i ), i = 1, 2,… .., n называются априорными вероятностями. Предположим, что эксперимент приводит к результату события A , где P (A)> 0. Нам нужно найти вероятность того, что наблюдаемое событие A было вызвано причиной E i , то есть мы ищем условное вероятность P (E i / A). Эти вероятности называются апостериорными вероятностями, которые, согласно правилу Байя, равны

      .

      4.3: Правила сложения и умножения вероятности

      При вычислении вероятности необходимо учитывать два правила при определении, являются ли два события независимыми или зависимыми и являются ли они взаимоисключающими.

      Правило умножения

      Если A и B — два события, определенные в пространстве выборки, то:

      \ [P (A \ text {AND} B) = P (B) P (A | B) \ label {eq1} \]

      Это правило можно также записать как:

      \ [P (A | B) = \ dfrac {P (A \ text {AND} B)} {P (B)} \ nonumber \]

      (Вероятность \ (A \) при заданном \ (B \) равна вероятности \ (A \) и \ (B \), деленной на вероятность \ (B \).)

      Если \ (A \) и \ (B \) независимы , то

      \ [P (A | B) = P (A).\ nonumber \]

      и уравнение \ ref {eq1} становится

      .

      \ [P (A \ text {AND} B) = P (A) P (B). \ nonumber \]

      Правило сложения

      Если A и B определены в пространстве выборки, то:

      Если A и B являются взаимоисключающими , то

      \ [P (A \ text {AND} B) = 0. \ nonumber \]

      и уравнение \ ref {eq5} становится

      .

      \ [P (A \ text {OR} B) = P (A) + P (B). \ nonumber \]

      Пример \ (\ PageIndex {1} \)

      Клаус пытается выбрать, куда поехать в отпуск.Его два варианта: \ (\ text {A} = \ text {Новая Зеландия} \) и \ (\ text {B} = \ text {Alaska} \).

      • Клаус может позволить себе только один отпуск. Вероятность того, что он выберет \ (\ text {A} \), равна \ (P (\ text {A}) = 0,6 \), а вероятность того, что он выберет \ (\ text {B} \), равна \ (P (\ текст {B}) = 0,35 \).
      • \ (P (\ text {A AND B}) = 0 \), потому что Клаус может позволить себе только один отпуск
      • Следовательно, вероятность того, что он выберет Новую Зеландию или Аляску, равна \ (P (\ text {A OR B}) = P (\ text {A}) + P (\ text {B}) = 0.6 + 0,35 = 0,95 \). Обратите внимание, что вероятность того, что он никуда не поедет в отпуск, должна составлять 0,05.

      Карлос играет в американский футбол. Он забивает в 65% случаев, когда бьет. В следующей игре Карлос забьет два мяча подряд. \ (\ text {A} = \) событие, которое Карлос успешно с первой попытки. \ (P (\ text {A}) = 0,65 \). \ (\ text {B} = \) событие, которое Карлос успешен со второй попытки. \ (P (\ text {B}) = 0,65 \). Карлос любит стрелять сериями. Вероятность того, что он забьет второй гол ДАЕТ , что он забил первый гол, равна 0.90.

      1. Какова вероятность, что он забьет оба гола?
      2. Какова вероятность того, что Карлос забьет первый или второй гол?
      3. Независимы ли \ (\ text {A} \) и \ (\ text {B} \)?
      4. Являются ли \ (\ text {A} \) и \ (\ text {B} \) взаимоисключающими?

      Решения

      а. Проблема состоит в том, чтобы найти \ (P (\ text {A AND B}) = P (\ text {B AND A}) \). Поскольку \ (P (\ text {B | A}) = 0,90: P (\ text {B AND A}) = P (\ text {B | A}) P (\ text {A}) = (0.90) (0,65) = 0,585 \)

      Карлос забивает первый и второй гол с вероятностью 0,585.

      г. Проблема заключается в том, чтобы найти \ (P (\ text {A OR B}) \).

      Карлос с вероятностью 0,715 забивает либо первый гол, либо второй гол.

      г. Нет, это не так, потому что \ (P (\ text {B AND A}) = 0,585 \).

      \ [P (\ text {B}) P (\ text {A}) = (0,65) (0,65) = 0,423 \]

      \ [0,423 \ neq 0,585 = P (\ text {B AND A}) \]

      Итак, \ (P (\ text {B AND A}) \) — это , а не , равное \ (P (\ text {B}) P (\ text {A}) \).

      г. Нет, это не так, потому что \ (P (\ text {A and B}) = 0,585 \).

      Чтобы быть взаимоисключающими, \ (P (\ text {A AND B}) \) должен быть равен нулю.

      Упражнение \ (\ PageIndex {1} \)

      Хелен играет в баскетбол. При штрафных бросках она выполняет бросок в 75% случаев. Теперь Хелен должна выполнить два штрафных броска. \ (\ text {C} = \) событие, когда Хелен делает первый выстрел. \ (P (\ text {C}) = 0,75 \). \ (\ text {D} = \) событие, которое Хелен делает второй выстрел. \ (P (\ text {D}) = 0,75 \). Вероятность того, что Хелен выполнит второй штрафной бросок, с учетом того, что она выполнила первый, равна 0.85. Какова вероятность того, что Хелен выполнит оба штрафных броска?

      Ответ

      \ [P (\ text {D | C}) = 0,85 \]

      \ [P (\ text {C AND D}) = P (\ text {D AND C}) \]

      \ [P (\ text {D AND C}) = P (\ text {D | C}) P (\ text {C}) = (0,85) (0,75) = 0,6375 \]

      Хелен выполняет первый и второй штрафные с вероятностью 0,6375.

      Пример \ (\ PageIndex {2} \)

      Общественная команда по плаванию насчитывает 150 членов. Семьдесят пять участников — опытные пловцы. Сорок семь участников — пловцы среднего уровня. Остальные — начинающие пловцы. Сорок пловцов продвинутого уровня занимаются четыре раза в неделю. Тридцать пловцов среднего уровня занимаются четыре раза в неделю. Десять начинающих пловцов тренируются четыре раза в неделю. Предположим, случайным образом выбран один член команды по плаванию.

      1. Какова вероятность того, что член — начинающий пловец?
      2. Какова вероятность того, что участник тренируется четыре раза в неделю?
      3. Какова вероятность того, что участник — опытный пловец и тренируется четыре раза в неделю?
      4. Какова вероятность того, что участник является пловцом продвинутого уровня и пловцом среднего уровня? Являются ли пловец продвинутого уровня и пловец среднего уровня взаимоисключающими? Почему или почему нет?
      5. Вы новичок в плавании и четыре раза в неделю тренируетесь в самостоятельных видах спорта? Почему или почему нет?

      Ответ

      1. \ (\ dfrac {28} {150} \)
      2. \ (\ dfrac {80} {150} \)
      3. \ (\ dfrac {40} {150} \)
      4. \ (P (\ text {расширенный И промежуточный}) = 0 \), поэтому это взаимоисключающие события.Пловец не может одновременно быть пловцом высокого уровня и пловцом среднего уровня.
      5. Нет, это не независимые события. \ [P (\ text {новичок И практикует четыре раза в неделю}) = 0,0667 \] \ [P (\ text {новичок}) P (\ text {практикует четыре раза в неделю}) = 0,0996 \] \ [0,0667 \ neq 0,0996 \]

      Упражнение \ (\ PageIndex {2} \)

      В школе 200 старшеклассников, 140 из которых будут поступать в колледж в следующем году. Сорок пойдут прямо на работу. Остальные берут перерыв в год.Пятьдесят старшеклассников, поступающих в колледж, занимаются спортом. Тридцать пенсионеров, идущих сразу на работу, занимаются спортом. Пятеро пожилых людей, взявших перерыв на год, занимаются спортом. Какова вероятность того, что пенсионер возьмет перерыв на год?

      Ответ

      \ [P = \ dfrac {200-140-40} {200} = \ dfrac {20} {200} = 0,1 \]

      Пример \ (\ PageIndex {3} \)

      Фелисити посещает Modesto JC в Модесто, Калифорния. Вероятность того, что Фелисити зачислится в класс математики, равна 0,2, а вероятность того, что она зачислится в класс речи, равна 0.65. Вероятность того, что она зачислится в математический класс, ПРИ ДАННОЙ, что она зачислена в речевой класс, равна 0,25.

      Пусть: \ (\ text {M} = \) урок математики, \ (\ text {S} = \) урок речи, \ (\ text {M | S} = \) математика заданная речь

      1. Какова вероятность того, что Фелисити будет изучать математику и речь?
        Найдите \ (P (\ text {M AND S}) = P (\ text {M | S}) P (\ text {S}) \).
      2. Какова вероятность того, что Фелисити поступит на уроки математики или речи?
        Найдите \ (P (\ text {M OR S}) = P (\ text {M}) + P (\ text {S}) — P (\ text {M AND S}) \).
      3. Независимы ли \ (\ text {M} \) и \ (\ text {S} \)? \ (P (\ text {M | S}) = P (\ text {M}) \)?
      4. Являются ли \ (\ text {M} \) и \ (\ text {S} \) взаимоисключающими? \ (P (\ text {M AND S}) = 0 \)?

      Ответ

      а. 0.1625, г. 0,6875, г. Кивок. №

      Упражнение \ (\ PageIndex {3} \)

      Студент идет в библиотеку. Пусть события \ (\ text {B} = \) студент проверяет книгу, а \ (\ text {D} = \) студент проверяет DVD. Предположим, что \ (P (\ text {B}) = 0.40, P (\ text {D}) = 0,30 \) и \ (P (\ text {D | B}) = 0,5 \).

      1. Найдите \ (P (\ text {B AND D}) \).
      2. Найдите \ (P (\ text {B OR D}) \).

      Ответ

      1. \ (P (\ text {B AND D}) = P (\ text {D | B}) P (\ text {B}) = (0,5) (0,4) = 0,20 \).
      2. \ (P (\ text {B OR D}) = P (\ text {B}) + P (\ text {D}) — P (\ text {B AND D}) = 0,40 + 0,30 — 0,20 = 0,50 \)

      Пример \ (\ PageIndex {4} \)

      Исследования показывают, что примерно каждая седьмая женщина (примерно 14.3%), дожившие до 90 лет, заболеют раком груди. Предположим, что у женщин, у которых развивается рак груди, тест дает отрицательный результат в 2% случаев. Также предположим, что в общей популяции женщин тест на рак груди дает отрицательный результат примерно в 85% случаев. Пусть \ (\ text {B} = \) женщина заболевает раком груди, а тест \ (\ text {N} = \) дает отрицательный результат. Предположим, одна женщина выбрана случайным образом.

      1. Какова вероятность того, что у женщины разовьется рак груди? Какова вероятность того, что у женщины отрицательный результат теста?
      2. Учитывая, что у женщины рак груди, какова вероятность того, что у нее будет отрицательный результат?
      3. Какова вероятность того, что у женщины рак груди И результаты анализов отрицательны?
      4. Какова вероятность того, что у женщины рак груди или результаты анализов отрицательны?
      5. Есть ли у вас рак груди и отрицательные результаты тестирования?
      6. Являются ли диагноз рака груди и отрицательный результат взаимоисключающими?

      Ответы

      1. \ (P (\ text {B}) = 0.143; P (\ text {N}) = 0,85 \)
      2. \ (P (\ text {N | B}) = 0,02 \)
      3. \ (P (\ text {B AND N}) = P (\ text {B}) P (\ text {N | B}) = (0,143) (0,02) = 0,0029 \)
      4. \ (P (\ text {B OR N}) = P (\ text {B}) + P (\ text {N}) — P (\ text {B AND N}) = 0,143 + 0,85 — 0,0029 = 0,9901 \)
      5. № \ (P (\ text {N}) = 0,85; P (\ text {N | B}) = 0,02 \). Итак, \ (P (\ text {N | B}) \) не равно \ (P (\ text {N}) \).
      6. № \ (P (\ text {B AND N}) = 0,0029 \). Чтобы \ (\ text {B} \) и \ (\ text {N} \) были взаимоисключающими, \ (P (\ text {B AND N}) \) должен быть равен нулю

      Упражнение \ (\ PageIndex {4} \)

      В школе 200 старшеклассников, 140 из которых будут поступать в колледж в следующем году.Сорок пойдут прямо на работу. Остальные берут перерыв в год. Пятьдесят старшеклассников, поступающих в колледж, занимаются спортом. Тридцать пенсионеров, идущих сразу на работу, занимаются спортом. Пятеро пожилых людей, взявших перерыв на год, занимаются спортом. Какова вероятность того, что выпускник пойдет в институт и займется спортом?

      Ответ

      Пусть \ (\ text {A} = \) студент — старшекурсник, идущий в колледж.

      Пусть \ (\ text {B} = \) студент занимается спортом.

      \ (P (\ text {B}) = \ dfrac {140} {200} \)

      \ (P (\ text {B | A}) = \ dfrac {50} {140} \)

      \ (P (\ text {A AND B}) = P (\ text {B | A}) P (\ text {A}) \)

      \ (P (\ text {A AND B}) = (\ dfrac {140} {200} \)) (\ (\ dfrac {50} {140}) = \ dfrac {1} {4} \)

      Пример \ (\ PageIndex {5} \)

      См. Информацию в примере \ (\ PageIndex {4} \).\ (\ text {P} = \) — положительный результат.

      1. Учитывая, что у женщины развивается рак груди, какова вероятность того, что у нее будет положительный результат теста. Найдите \ (P (\ text {P | B}) = 1 — P (\ text {N | B}) \).
      2. Какова вероятность того, что у женщины разовьется рак груди и положительный результат теста. Найдите \ (P (\ text {B AND P}) = P (\ text {P | B}) P (\ text {B}) \).
      3. Какова вероятность того, что у женщины не разовьется рак груди. Найдите \ (P (\ text {B ′}) = 1 — P (\ text {B}) \).
      4. Какова вероятность того, что у женщины положительный результат теста на рак груди.Найдите \ (P (\ text {P}) = 1 — P (\ text {N}) \).

      Ответ

      а. 0,98; б. 0,1401; c. 0,857; d. 0,15

      Упражнение \ (\ PageIndex {5} \)

      Студент идет в библиотеку. Пусть события \ (\ text {B} = \) студент проверяет книгу и \ (\ text {D} = \) студент проверяет DVD. Предположим, что \ (P (\ text {B}) = 0,40, P (\ text {D}) = 0,30 \) и \ (P (\ text {D | B}) = 0,5 \).

      1. Найдите \ (P (\ text {B ′}) \).
      2. Найдите \ (P (\ text {D AND B}) \).
      3. Найдите \ (P (\ text {B | D}) \).
      4. Найдите \ (P (\ text {D AND B ′}) \).
      5. Найдите \ (P (\ text {D | B ′}) \).

      Ответ

      1. \ (P (\ text {B ′}) = 0.60 \)
      2. \ (P (\ text {D AND B}) = P (\ text {D | B}) P (\ text {B}) = 0,20 \)
      3. \ (P (\ text {B | D}) = \ dfrac {P (\ text {B AND D})} {P (\ text {D})} = \ dfrac {(0.20)} {(0.30) } = 0,66 \)
      4. \ (P (\ text {D AND B ′}) = P (\ text {D}) — P (\ text {D AND B}) = 0,30 — 0,20 = 0,10 \)
      5. \ (P (\ text {D | B ′}) = P (\ text {D AND B ′}) P (\ text {B ′}) = (P (\ text {D}) — P (\ text {D И B})) (0.60) = (0,10) (0,60) = 0,06 \)

      Обзор формулы

      Правило умножения: \ (P (\ text {A AND B}) = P (\ text {A | B}) P (\ text {B}) \)

      Правило сложения: \ (P (\ text {A OR B}) = P (\ text {A}) + P (\ text {B}) — P (\ text {A AND B}) \)

      Используйте следующую информацию, чтобы ответить на следующие десять упражнений. Сорок восемь процентов всех зарегистрированных избирателей Калифорнии предпочитают пожизненное заключение без права досрочного освобождения смертной казни для человека, признанного виновным в убийстве первой степени.Среди зарегистрированных в Латинской Калифорнии избирателей 55% предпочитают пожизненное заключение без права досрочного освобождения смертной казни для человека, признанного виновным в убийстве первой степени. 37,6% всех калифорнийцев — латиноамериканцы.

      В этой задаче пусть:

      • \ (\ text {C} = \) Калифорнийцы (зарегистрированные избиратели) предпочитают жизнь в тюрьме без права досрочного освобождения смертной казни для человека, осужденного за убийство первой степени.
      • \ (\ text {L} = \) Латиноамериканцы из Калифорнии

      Предположим, случайным образом выбран один калифорнийец.

      Упражнение \ (\ PageIndex {5} \)

      Найдите \ (P (\ text {C}) \).

      Упражнение \ (\ PageIndex {6} \)

      Найдите \ (P (\ text {L}) \).

      Ответ

      0,376

      Упражнение \ (\ PageIndex {7} \)

      Найдите \ (P (\ text {C | L}) \).

      Упражнение \ (\ PageIndex {8} \)

      Проще говоря, что такое \ (\ text {C | L} \)?

      Ответ

      \ (\ text {C | L} \) означает, что, учитывая, что выбранный человек является латиноамериканцем из Калифорнии, этот человек является зарегистрированным избирателем, который предпочитает жизнь в тюрьме без права досрочного освобождения лицу, признанному виновным в убийстве первой степени.

      Упражнение \ (\ PageIndex {9} \)

      Найдите \ (P (\ text {L AND C}) \)

      Упражнение \ (\ PageIndex {10} \)

      На словах, что такое \ (\ text {L AND C} \)?

      Ответ

      \ (\ text {L AND C} \) — это событие, когда выбранным лицом является зарегистрированный избиратель из Латинской Америки, Калифорния, который предпочитает жизнь без права досрочного освобождения смертной казни для человека, осужденного за убийство первой степени.

      Упражнение \ (\ PageIndex {11} \)

      Являются ли \ (\ text {L} \) и \ (\ text {C} \) независимыми событиями? Покажите, почему или почему нет.

      Упражнение \ (\ PageIndex {12} \)

      Найдите \ (P (\ text {L OR C}) \).

      Ответ

      0,6492

      Упражнение \ (\ PageIndex {13} \)

      На словах, что такое \ (\ text {L OR C} \)?

      Упражнение \ (\ PageIndex {14} \)

      Являются ли события \ (\ text {L} \) и \ (\ text {C} \) взаимоисключающими? Покажите, почему или почему нет.

      Ответ

      Нет, потому что \ (P (\ text {L AND C}) \) не равно 0.

      Дополнительное правило вероятности: определение и примеры — видео и стенограмма урока

      Взаимоисключающие события

      Помните, бросание кубика было бы примером взаимоисключающего события.Кость не может приземлиться с двух сторон одновременно; поэтому вероятность того, что каждая сторона кубика исключает друг друга. Вы также можете слышать о взаимоисключающих событиях, называемых непересекающимися событиями. При работе с взаимоисключающими событиями по вероятности используйте следующую формулу:

      Формула расчета вероятности взаимоисключающего события

      Эта формула читается как:

      Вероятность события A или B равна вероятности события A плюс вероятность события B.

      Чтобы определить вероятность взаимоисключающих событий, выполните следующие действия:

      1. Найдите сумму возможных результатов
      2. Найдите желаемый результат
      3. Создайте соотношение для каждого события
      4. Сложите доли или доли каждого события

      Во-первых, всего шесть возможных исходов шестигранной кости. У вас есть шесть различных возможных результатов при броске кубика.

      Во-вторых, найдите желаемый результат.Шайенну нужно выбросить 3 или 6. Следовательно, желаемым результатом будет 3 или 6. На шестигранном кубике один раз появляется цифра 3 и 6. Запомните эту информацию для следующего шага.

      В-третьих, создайте коэффициент для каждого события. Первое событие, бросок 3, будет иметь коэффициент 1/6, потому что у кубика только одна сторона с тремя точками. Второе событие, выпадение 6, также будет иметь отношение 1/6, потому что у кубика только одна сторона с шестью точками.

      В-четвертых, сложите отношения или доли каждого события.Этот шаг даст вам вероятность бросить кубик и получить 3 или 6.

      1/6 + 1/6 = 2/6 или 1/3

      Таким образом, Шайенн имеет шанс 1 из 3 бросая 3 или 6. Как только она выбрасывает 3 или 6, Шайенн может приземлиться на поле, которое позволяет ей выбрать карту. Ей нужно выбрать черную карту или семерку.

      Неисключительные события

      Помните, что выбор черной карты или семи карт из колоды обычных игральных карт является примером несовместимых событий.Если вы ищете вероятность того, что два события произойдут одновременно, это называется пересечением двух событий. Узнайте больше о пересечении в нашем уроке «Правило вероятности умножения». Это формула для не исключающих друг друга событий:

      Формула для расчета вероятности не исключающих друг друга событий

      Эта формула читается как:

      Вероятность события A или B равна вероятности события A плюс вероятность события B минус вероятность событий A и B.

      Чтобы определить вероятность не исключающих друг друга событий, выполните следующие действия:

      1. Найдите сумму возможных исходов
      2. Найдите желаемый результат
      3. Создайте соотношение для каждого события
      4. Сложите доли или доли каждого события
      5. Вычесть перекрытие двух событий

      Во-первых, общее количество возможных исходов колоды обычных игральных карт равно 52, поскольку в обычной колоде 52 карты.

      Во-вторых, найдите желаемый результат. Шайенну нужно выбрать черную карту или семерку. Следовательно, желаемым исходом будет черная или семерка. Есть две масти, которые являются черными картами: пики и трефы. Для каждой масти по 13 карт. Следовательно, желаемый результат возможных вариантов для черной карты равен 26. В обычной колоде игральных карт четыре семерки, по одной семерки для каждой масти. Следовательно, желаемые возможности исхода для семи карт равны 4.

      В-третьих, создайте соотношение для каждого события.Первое событие, выбрав черную карту, будет иметь соотношение 26/52. Второе событие, выбрав семь карт, будет иметь соотношение 4/52. Я получил эти соотношения, используя желаемое число результатов в качестве числителя и общее количество возможных результатов в качестве знаменателя.

      В-четвертых, сложите отношения или доли каждого события следующим образом:

      26/52 + 4/52 = 30/52

      Вы можете остановиться здесь и сказать, что есть 30 из 52 шансов выбрать черную карту или семерку.Но обратите внимание, что в этом утверждении есть слово «или». Это означает, что вы ищете не карту, которая является черной семеркой, а просто все карты, которые являются черными и семеркой. Следовательно, вам нужно вычесть перекрытие двух событий из вероятности. Вероятность выпадения черной семерки составляет 2/52, потому что в колоде всего две черные семерки. Возьмите это соотношение и вычтите из предыдущей вероятности следующим образом:

      30/52 — 2/52 = 28/52

      Теперь у нас есть правильная вероятность.Шайенн имеет 28 из 52 шансов выбрать либо черную карту, либо семерку. Я бы сказал, это неплохие шансы!

      Практические задания

      Пример 1:

      Эбби принимает участие в своих первых соревнованиях по плаванию. В первом заезде участвуют семь девушек. Она должна занять первое или второе место, чтобы перейти на следующий уровень турнира. Предполагая, что ничьей нет, какова вероятность того, что Эбби получит первое или второе место?

      У Эбби шанс 2 из 7 или примерно 29% выйти на следующий уровень турнира.

      Это еще один пример взаимоисключающих событий. Эбби не может занять ни первое, ни второе место. Следовательно, нет совпадения событий. Поскольку это пример взаимоисключающих событий, мы можем использовать эту формулу из правила сложения вероятностей:

      Эбби имеет 1/7 шанс занять первое место и 1/7 шанс занять второе место. Мы можем сложить эти две вероятности вместе, чтобы найти вероятность того, что Эбби получит первое или второе место следующим образом:

      1/7 + 1/7 = 2/7

      Пример 2:

      Команда Эбби занимает первое место из другие команды по окончании соревнований по плаванию.После этого команда идет за пиццей и мороженым. В команде 20 человек; 8 человек заказывают пиццу, а 12 человек — мороженое. Из команды 5 человек заказали и пиццу, и мороженое. Какова вероятность того, что член команды закажет пиццу или мороженое, но не то и другое вместе?

      Вероятность того, что член команды закажет пиццу или мороженое, но не то и другое, составляет 15 из 20 или 75%.

      Это пример не исключающих друг друга мероприятий, поскольку некоторые члены команды смогли заказать и мороженое, и пиццу.Вероятность того, что член команды закажет пиццу, составляет 8/20, поскольку эта информация нам уже была предоставлена. Вероятность того, что член команды закажет мороженое, составляет 12/20. Сначала мы можем сложить эти две вероятности вместе:

      8/20 + 12/20 = 20/20

      Вы, вероятно, решили, что в этот момент что-то не так, поскольку в команде всего двадцать человек. Это потому, что в какой-то момент числа пересекаются. Помните, некоторые люди заказывали и пиццу, и мороженое.Мы знаем из проблемы, что 5 человек заказали и пиццу, и мороженое. Нам нужно вычесть вероятность 5/20 из нашей задачи следующим образом:

      20/20 — 5/20 = 15/20

      Помните, что в данном случае вероятность — это оценка или прогноз. Мы пытаемся предсказать, будет ли член команды на самом деле заказывать и то, и другое. Таким образом, мы можем только точно сказать, что заказали и то, и другое 5 человек. Мы можем сказать, что если товарищ по команде не закажет оба, то с вероятностью 75% он или она закажет то или другое.

      Краткое содержание урока

      Правило сложения вероятности — это правило для нахождения объединения двух событий: взаимоисключающих или не исключающих друг друга. Взаимоисключающие события — это события, которые не могут происходить одновременно. Не взаимоисключающие события — это события, которые могут происходить по отдельности или в одно и то же время.

      Чтобы найти объединение двух взаимоисключающих событий, используйте следующую формулу:

      Вероятность события A или B равна вероятности события A плюс вероятность события B.

      Чтобы найти объединение двух событий, которые не являются взаимоисключающими, используйте эту формулу:

      Вероятность события A или B равна вероятности события A плюс вероятность события B минус вероятность события A и B.

      Помните, правило сложения вероятностей помогает вам найти вероятность события A или события B, а не обоих событий. Чтобы найти пересечение двух событий, ознакомьтесь с нашим уроком о правиле вероятности умножения.

      Результаты обучения

      По завершении этого урока вы сможете:

      • Вспомнить правило сложения вероятности
      • Сравните / сопоставьте взаимоисключающее событие с неисключающим событием и приведите пример
      • Запомните формулы для расчета вероятности неисключающего или взаимоисключающего события
      • Рассчитать вероятность взаимоисключающего или не исключающего друг друга события

      Основная теория вероятностей: правила и формулы — видео и стенограмма урока

      Визуализация вероятностей

      Существует несколько способов визуализации вероятностей, но самый простой способ представить их — использовать метод дроби : превратить члены в дроби, разделив количество желаемых результатов на общее количество возможные исходы.Это всегда даст вам число от 0 до 1. Например, каковы шансы выпадения нечетного числа на 6-гранном кубике? Всего существует шесть чисел и три нечетных числа: 1, 3 и 5. Таким образом, вероятность выпадения нечетного числа равна 3/6 или 0,5. Вы можете использовать эту формулу при выполнении более сложных вычислений, как мы увидим позже в уроке.

      В этой формуле:

      • P (A) читается как «вероятность A », где A — это интересующее нас событие.
      • P (A | B) читается как «вероятность A при наличии B ».
      • P (не A) читается как «вероятность не A » или «вероятность того, что A не произойдет».

      Правила вероятности

      Есть три основных правила, связанных с базовой вероятностью: правило сложения, правило умножения и правило дополнения. Вы можете думать о правиле дополнения как о «правиле вычитания», если оно помогает вам его запомнить.

      1.) Правило сложения : P (A или B) = P (A) + P (B) — P (A и B)

      Если A и B являются взаимоисключающими событиями, , или те, которые не могут встречаться вместе, то третий член равен 0, и правило сводится к P (A или B) = P (A) + P (B) . Например, вы не можете подбросить монету, и она выпадет орлом и решкой за один бросок.

      2.) Правило умножения : P (A и B) = P (A) * P (B | A) или P (B) * P (A | B)

      Если A и B равны независимых событий , мы можем сократить формулу до P (A и B) = P (A) * P (B) .Термин «независимый» относится к любому событию, результат которого не зависит от результата другого события. Например, рассмотрим второй из двух подбрасываний монеты, для которого вероятность выпадения орла все еще составляет 0,50 (50%), независимо от того, что выпало при первом подбрасывании. Какова вероятность того, что во время двух подбрасываний монеты вы получите решку при первом подбрасывании и орел при втором подбрасывании?

      Проведем расчеты: P = P (хвосты) * P (головы) = (0,5) * (0,5) = 0,25

      3.) Правило дополнения : P (не A) = 1 — P (A)

      Понимаете ли вы, почему правило дополнения также можно рассматривать как правило вычитания? Это правило основывается на взаимоисключающем характере P (A) и P (не A) . Эти два события никогда не могут произойти вместе, но одно из них должно произойти всегда. Следовательно, P (A) + P (не A) = 1. Например, если метеоролог говорит, что вероятность дождя завтра составляет 0,3, какова вероятность того, что дождя не будет?

      Давайте посчитаем: P (без дождя) = 1 — P (дождь) = 1 — 0.3 = 0,7

      Закон полной вероятности

      Закон полной вероятности : P (A) = P (A | B) * P (B) + P (A | not B) * P (not B)

      Например, какова вероятность того, что любимый цвет человека будет синим, если вы знаете следующее:

      • У левшей синий цвет является любимым цветом в 30% случаев
      • Правши любят синий в 40% случаев
      • Левши составляют 10% населения

      Завершим уравнение:

      1.) P (синий) = P (левша) * P (как синий | левша) + P (не левша) * (P (как синий | не левша)

      2.) P ( Синий) = (0,1) (0,3) + (0,9) (0,4)

      3.) P (Синий) = 0,03 + 0,36 = 0,39

      Следовательно, вероятность того, что любимым цветом человека будет синий, составляет 39%.

      Теорема Байеса

      Теорема Байеса — это метод работы с условными свойствами. В нем говорится, что:

      P (A | B) = {P (B | A) * P (A)} / P (B)

      Использование закона полной вероятности для разложения P (B) Байеса ‘, мы также можем написать:

      P (A | B) = {P (B | A) * P (A)} / {P (A) * P (B | A) + P (not A) * P (B | not A)}

      Вы можете использовать теорему Байеса для вычисления P (A | B) , если у вас ограниченная информация о других величинах.Например, предположим, что случайно выбранный гонщик на Тур де Франс дал положительный результат на препараты, повышающие производительность. Тест имеет точность 95%. Насколько велика вероятность того, что этот спортсмен причастен к незаконной деятельности, если 1% спортсменов обманывают таким образом?

      Давайте составим наше уравнение и выполним вычисления:

      1.) P (Cheat | Positive) = {P (Positive | Cheat) * P (Cheat)} / {P (Positive | Cheat) * P ( Чит) + P (Положительный | не Чит) * P (Не Чит)}

      2.) P (Чит | Положительный) = (0.95) (0,01) / {(0,95) (. 01) + (. 05) (. 99)}

      3.) P (Чит | Положительный) = .0095 / .0095 + .0495

      4 .) P (Cheat | Positive) = 0,0095 / 0,059

      5.) P (Cheat | Positive) = 0,1610

      Затем мы следуем математическим правилам для преобразования десятичной дроби в дробь и завершаем операция:

      100% — 16,1% = 83%

      Несмотря на то, что тест достаточно точен, и этот гонщик дал положительный результат, наши результаты дают нам ответ, отличный от того, которого мы могли ожидать.После расчета вероятности появляется 83% вероятность, что этот гонщик не делает ничего противозаконного!

      Краткое содержание урока

      Вероятность относится к числу от 0 до 1 и включает взаимоисключающие или независимые события. Взаимоисключающие события не могут происходить одновременно, в то время как независимых событий не влияют на вероятность друг друга.

      Есть три основных правила, связанных с вероятностью: правила сложения, умножения и дополнения.

      Тангенс 46: Тангенс 46 равен чему? tg(46) = ?

      Таблица тангенсов.

      Таблица синусов Таблица косинусов Таблица тангенсов Таблица котангенсов Таблица Брадиса: синусы, косинусы, тангенсы и котангенсы

      Скачать таблицу тангенсов

      Таблица тангенсов — это записанные в таблицу посчитанные значения тангенсов углов от 0° до 360°. Используя таблицу тангенсов Вы сможете провести расчеты даже если под руками не окажется инженерного калькулятора. Чтобы узнать значение тангенса от нужного Вам угла достаточно найти его в таблице.

      Калькулятор — тангенс угла

      tg(°) = 0

      Калькулятор — арктангенс угла

      arctan() = 45°

      Таблица тангенсов в радианах

      α 0 π6 π4 π3 π2 π 3π2
      tg α 0 √33 1 √3 0 0

      Таблица тангенсов углов от 0° до 180°

      tg(0°) = 0
      tg(1°) = 0. 01746
      tg(2°) = 0.03492
      tg(3°) = 0.05241
      tg(4°) = 0.06993
      tg(5°) = 0.08749
      tg(6°) = 0.1051
      tg(7°) = 0.12278
      tg(8°) = 0.14054
      tg(9°) = 0.15838
      tg(10°) = 0.17633
      tg(11°) = 0.19438
      tg(12°) = 0.21256
      tg(13°) = 0.23087
      tg(14°) = 0.24933
      tg(15°) = 0.26795
      tg(16°) = 0.28675
      tg(17°) = 0.30573
      tg(18°) = 0.32492
      tg(19°) = 0.34433
      tg(20°) = 0.36397
      tg(21°) = 0.38386
      tg(22°) = 0.40403
      tg(23°) = 0.42447
      tg(24°) = 0.44523
      tg(25°) = 0.46631
      tg(26°) = 0.48773
      tg(27°) = 0.50953
      tg(28°) = 0.53171
      tg(29°) = 0.55431
      tg(30°) = 0.57735
      tg(31°) = 0.60086
      tg(32°) = 0.62487
      tg(33°) = 0.64941
      tg(34°) = 0.67451
      tg(35°) = 0.70021
      tg(36°) = 0.72654
      tg(37°) = 0.75355
      tg(38°) = 0.78129
      tg(39°) = 0.80978
      tg(40°) = 0.8391
      tg(41°) = 0.86929
      tg(42°) = 0.9004
      tg(43°) = 0.93252
      tg(44°) = 0.96569
      tg(45°) = 1
      tg(46°) = 1.03553
      tg(47°) = 1.07237
      tg(48°) = 1. 11061
      tg(49°) = 1.15037
      tg(50°) = 1.19175
      tg(51°) = 1.2349
      tg(52°) = 1.27994
      tg(53°) = 1.32704
      tg(54°) = 1.37638
      tg(55°) = 1.42815
      tg(56°) = 1.48256
      tg(57°) = 1.53986
      tg(58°) = 1.60033
      tg(59°) = 1.66428
      tg(60°) = 1.73205
      tg(61°) = 1.80405
      tg(62°) = 1.88073
      tg(63°) = 1.96261
      tg(64°) = 2.0503
      tg(65°) = 2.14451
      tg(66°) = 2.24604
      tg(67°) = 2.35585
      tg(68°) = 2.47509
      tg(69°) = 2.60509
      tg(70°) = 2.74748
      tg(71°) = 2.90421
      tg(72°) = 3.07768
      tg(73°) = 3.27085
      tg(74°) = 3.48741
      tg(75°) = 3.73205
      tg(76°) = 4.01078
      tg(77°) = 4.33148
      tg(78°) = 4.70463
      tg(79°) = 5.14455
      tg(80°) = 5.67128
      tg(81°) = 6.31375
      tg(82°) = 7.11537
      tg(83°) = 8.14435
      tg(84°) = 9.51436
      tg(85°) = 11.43005
      tg(86°) = 14.30067
      tg(87°) = 19.08114
      tg(88°) = 28.63625
      tg(89°) = 57.28996
      tg(90°) = ∞
      tg(91°) = -57.28996
      tg(92°) = -28.63625
      tg(93°) = -19. 08114
      tg(94°) = -14.30067
      tg(95°) = -11.43005
      tg(96°) = -9.51436
      tg(97°) = -8.14435
      tg(98°) = -7.11537
      tg(99°) = -6.31375
      tg(100°) = -5.67128
      tg(101°) = -5.14455
      tg(102°) = -4.70463
      tg(103°) = -4.33148
      tg(104°) = -4.01078
      tg(105°) = -3.73205
      tg(106°) = -3.48741
      tg(107°) = -3.27085
      tg(108°) = -3.07768
      tg(109°) = -2.90421
      tg(110°) = -2.74748
      tg(111°) = -2.60509
      tg(112°) = -2.47509
      tg(113°) = -2.35585
      tg(114°) = -2.24604
      tg(115°) = -2.14451
      tg(116°) = -2.0503
      tg(117°) = -1.96261
      tg(118°) = -1.88073
      tg(119°) = -1.80405
      tg(120°) = -1.73205
      tg(121°) = -1.66428
      tg(122°) = -1.60033
      tg(123°) = -1.53986
      tg(124°) = -1.48256
      tg(125°) = -1.42815
      tg(126°) = -1.37638
      tg(127°) = -1.32704
      tg(128°) = -1.27994
      tg(129°) = -1.2349
      tg(130°) = -1.19175
      tg(131°) = -1.15037
      tg(132°) = -1.11061
      tg(133°) = -1.07237
      tg(134°) = -1.03553
      tg(135°) = -1
      tg(136°) = -0. 96569
      tg(137°) = -0.93252
      tg(138°) = -0.9004
      tg(139°) = -0.86929
      tg(140°) = -0.8391
      tg(141°) = -0.80978
      tg(142°) = -0.78129
      tg(143°) = -0.75355
      tg(144°) = -0.72654
      tg(145°) = -0.70021
      tg(146°) = -0.67451
      tg(147°) = -0.64941
      tg(148°) = -0.62487
      tg(149°) = -0.60086
      tg(150°) = -0.57735
      tg(151°) = -0.55431
      tg(152°) = -0.53171
      tg(153°) = -0.50953
      tg(154°) = -0.48773
      tg(155°) = -0.46631
      tg(156°) = -0.44523
      tg(157°) = -0.42447
      tg(158°) = -0.40403
      tg(159°) = -0.38386
      tg(160°) = -0.36397
      tg(161°) = -0.34433
      tg(162°) = -0.32492
      tg(163°) = -0.30573
      tg(164°) = -0.28675
      tg(165°) = -0.26795
      tg(166°) = -0.24933
      tg(167°) = -0.23087
      tg(168°) = -0.21256
      tg(169°) = -0.19438
      tg(170°) = -0.17633
      tg(171°) = -0.15838
      tg(172°) = -0.14054
      tg(173°) = -0.12278
      tg(174°) = -0.1051
      tg(175°) = -0.08749
      tg(176°) = -0.06993
      tg(177°) = -0.05241
      tg(178°) = -0. 03492
      tg(179°) = -0.01746
      tg(180°) = 0

      Таблица тангенсов углов от 181° до 360°

      tg(181°) = 0.01746
      tg(182°) = 0.03492
      tg(183°) = 0.05241
      tg(184°) = 0.06993
      tg(185°) = 0.08749
      tg(186°) = 0.1051
      tg(187°) = 0.12278
      tg(188°) = 0.14054
      tg(189°) = 0.15838
      tg(190°) = 0.17633
      tg(191°) = 0.19438
      tg(192°) = 0.21256
      tg(193°) = 0.23087
      tg(194°) = 0.24933
      tg(195°) = 0.26795
      tg(196°) = 0.28675
      tg(197°) = 0.30573
      tg(198°) = 0.32492
      tg(199°) = 0.34433
      tg(200°) = 0.36397
      tg(201°) = 0.38386
      tg(202°) = 0.40403
      tg(203°) = 0.42447
      tg(204°) = 0.44523
      tg(205°) = 0.46631
      tg(206°) = 0.48773
      tg(207°) = 0.50953
      tg(208°) = 0.53171
      tg(209°) = 0.55431
      tg(210°) = 0.57735
      tg(211°) = 0.60086
      tg(212°) = 0.62487
      tg(213°) = 0.64941
      tg(214°) = 0.67451
      tg(215°) = 0.70021
      tg(216°) = 0.72654
      tg(217°) = 0. 75355
      tg(218°) = 0.78129
      tg(219°) = 0.80978
      tg(220°) = 0.8391
      tg(221°) = 0.86929
      tg(222°) = 0.9004
      tg(223°) = 0.93252
      tg(224°) = 0.96569
      tg(225°) = 1
      tg(226°) = 1.03553
      tg(227°) = 1.07237
      tg(228°) = 1.11061
      tg(229°) = 1.15037
      tg(230°) = 1.19175
      tg(231°) = 1.2349
      tg(232°) = 1.27994
      tg(233°) = 1.32704
      tg(234°) = 1.37638
      tg(235°) = 1.42815
      tg(236°) = 1.48256
      tg(237°) = 1.53986
      tg(238°) = 1.60033
      tg(239°) = 1.66428
      tg(240°) = 1.73205
      tg(241°) = 1.80405
      tg(242°) = 1.88073
      tg(243°) = 1.96261
      tg(244°) = 2.0503
      tg(245°) = 2.14451
      tg(246°) = 2.24604
      tg(247°) = 2.35585
      tg(248°) = 2.47509
      tg(249°) = 2.60509
      tg(250°) = 2.74748
      tg(251°) = 2.90421
      tg(252°) = 3.07768
      tg(253°) = 3.27085
      tg(254°) = 3.48741
      tg(255°) = 3.73205
      tg(256°) = 4.01078
      tg(257°) = 4.33148
      tg(258°) = 4.70463
      tg(259°) = 5.14455
      tg(260°) = 5.67128
      tg(261°) = 6. 31375
      tg(262°) = 7.11537
      tg(263°) = 8.14435
      tg(264°) = 9.51436
      tg(265°) = 11.43005
      tg(266°) = 14.30067
      tg(267°) = 19.08114
      tg(268°) = 28.63625
      tg(269°) = 57.28996
      tg(270°) = ∞
      tg(271°) = -57.28996
      tg(272°) = -28.63625
      tg(273°) = -19.08114
      tg(274°) = -14.30067
      tg(275°) = -11.43005
      tg(276°) = -9.51436
      tg(277°) = -8.14435
      tg(278°) = -7.11537
      tg(279°) = -6.31375
      tg(280°) = -5.67128
      tg(281°) = -5.14455
      tg(282°) = -4.70463
      tg(283°) = -4.33148
      tg(284°) = -4.01078
      tg(285°) = -3.73205
      tg(286°) = -3.48741
      tg(287°) = -3.27085
      tg(288°) = -3.07768
      tg(289°) = -2.90421
      tg(290°) = -2.74748
      tg(291°) = -2.60509
      tg(292°) = -2.47509
      tg(293°) = -2.35585
      tg(294°) = -2.24604
      tg(295°) = -2.14451
      tg(296°) = -2.0503
      tg(297°) = -1.96261
      tg(298°) = -1.88073
      tg(299°) = -1.80405
      tg(300°) = -1.73205
      tg(301°) = -1.66428
      tg(302°) = -1.60033
      tg(303°) = -1. 53986
      tg(304°) = -1.48256
      tg(305°) = -1.42815
      tg(306°) = -1.37638
      tg(307°) = -1.32704
      tg(308°) = -1.27994
      tg(309°) = -1.2349
      tg(310°) = -1.19175
      tg(311°) = -1.15037
      tg(312°) = -1.11061
      tg(313°) = -1.07237
      tg(314°) = -1.03553
      tg(315°) = -1
      tg(316°) = -0.96569
      tg(317°) = -0.93252
      tg(318°) = -0.9004
      tg(319°) = -0.86929
      tg(320°) = -0.8391
      tg(321°) = -0.80978
      tg(322°) = -0.78129
      tg(323°) = -0.75355
      tg(324°) = -0.72654
      tg(325°) = -0.70021
      tg(326°) = -0.67451
      tg(327°) = -0.64941
      tg(328°) = -0.62487
      tg(329°) = -0.60086
      tg(330°) = -0.57735
      tg(331°) = -0.55431
      tg(332°) = -0.53171
      tg(333°) = -0.50953
      tg(334°) = -0.48773
      tg(335°) = -0.46631
      tg(336°) = -0.44523
      tg(337°) = -0.42447
      tg(338°) = -0.40403
      tg(339°) = -0.38386
      tg(340°) = -0.36397
      tg(341°) = -0.34433
      tg(342°) = -0.32492
      tg(343°) = -0.30573
      tg(344°) = -0.28675
      tg(345°) = -0.26795
      tg(346°) = -0. 24933
      tg(347°) = -0.23087
      tg(348°) = -0.21256
      tg(349°) = -0.19438
      tg(350°) = -0.17633
      tg(351°) = -0.15838
      tg(352°) = -0.14054
      tg(353°) = -0.12278
      tg(354°) = -0.1051
      tg(355°) = -0.08749
      tg(356°) = -0.06993
      tg(357°) = -0.05241
      tg(358°) = -0.03492
      tg(359°) = -0.01746
      tg(360°) = 0

      Таблицы значений тригонометрических функций Таблица Брадиса: синусы, косинусы, тангенсы и котангенсы Таблица синусов Таблица косинусов Таблица тангенсов Таблица котангенсов Сводная таблица тригонометрических функций

      Тригонометрические формулы

      Все таблицы и формулы

      TG 46

      Р Е Н О К Р И Л   TG 46  Futur, Кристалл грунт, для нанесения погружением, белый

       

      Области применения:

      РЕНОКРИЛ TG 46 применяется как грунтовка для любых пород древесины, для строительных конструкций с высокими требованиями к сохранению размеров. Для древесины с особыми внутренними веществами должна быть применена изолирующая грунтовка IL 48.

      Продукт предназначен для применения на промышленных предприятиях.

       

      Характеристики:

      РЕНОКРИЛ TG 46 – шелковисто-матовая, содержащая белые пигменты, не препятствует диффузии водных паров, может наноситься кистью или окунанием. Экологична, имеет нейтральный запах. Не заливать в ёмкости, которые могут ржаветь, оказывает фунгицидное действие, защитное действие согласно DIN EN 152, раздел 1. если после нанесения грунтовки погружением на поверхности проявляются пятна, то необходимо дополнительно обработать грунтом IL 48 L.

       

      Указания по обработке:

      Способ нанесения    : погружение, облив, нанесение кистью, напыление.

       нанесение кистью   : Ренокрил TG 46 Futur можно наносить кистью в

              неразбавленном виде. Поверхность равномерно

              покрыть тонким слоем.

       напыление    : Ренокрил TG 46 Futur можно наносить в не разведённом виде напылением. Для правильного

              высыхания необходимо нанесение ровным слоем.

       нанесение погружением  : наносится в неразбавленном виде, при этом продукт

              имеет вязкость от 11 – до 12 сек. (DIN метал.

              стаканчик 4 мм). Провести пробное погружение, в

              зависимости от стекаемости разбавить с 5% воды.

      Очистка     : водой или универсальным растворителем.

      Температура обработки   : 20 °С относит. влажность воздуха – в зависимости от

              толщины наносимого слоя, через 30-60 минут

              можно шлифовать.

      Расход      : в зависимости от всасывающей способности

              древесины  ок. 100-150 мл/м².

      Хранение     : в прохладном месте, но без замерзания, вскрытые

      ёмкости держать плотно закрытыми. для предупреждения образования плёнки на поверхности, впрыснуть в ёмкость немного воды.

      В оригинальной упаковке срок хранения 1 год.

      Общие указания  : перед применением и после длительных пауз

                                                           перемешать, не смешивать с продуктами на основе

              растворителей.

              Не смешивать с другими водорастворимыми

              продуктами !!

      Точка воспламенения    : отсутствует.

      Класс опасности    : без обозначения.

              VbF: отсутствует.

              Gef.Stoff V: не требует обозначения.

              GGVS / ADR: не требует обозначения.

       

      Создание покрытия для деревянных окон и входных дверей из лиственной и хвойной

      древесины:

      Предварительные работы : деревянные поверхности тщательно очистить от пыли, смолистые и

            загрязнённые поверхности протереть нитрорастворителем.

       

       

      Rhenocryl TG 46

       

      Грунтовка   : нанести Rhenocryl TG 46 Futur (погружением или обливом) на древесину

            с особыми внутренними веществами.

            Дополнительно нанести (напылением) Rhenocryl IL 48 L как

            промежуточное покрытие (толщина мокрой плёнки ок. 175 µ).

      Окончательное покрытие : нанести Rhenocryl DL 90 Futur Airless-аппаратом, толщина мокрой

            плёнки 300 µ.

            (в случае нанесения промежуточного слоя: ок. 125 µ)

       

      В соответствии с директивами о лакокрасочных покрытиях деревянных окон, изданных федеральным комитетом красок и защиты ценностей, строительных норм для открытых строительных элементов (VOB, часть 1), DIN 18363 и рекомендациями IFT (Института оконной техники в Розенхайме) деревянные строительные элементы перед монтажом должны покрываться со всех сторон не менее чем в один слой грунтом и промежуточным грунтом.

      Примите во внимание при создании лакокрасочного покрытия так же таблицу «Группы лакокрасочных покрытий для окон и входных дверей», изданную Институтом техники (IFT), Розенхайм, в мае 1983г.

       

      Указания по мерам предосторожности:

      Хранить под замком, в местах недоступных для детей. Не допускать попадания в глаза. При попадании промыть большим количеством воды. При работе с малоопасными лаками необходимо соблюдать обычные меры предосторожности.

       

       

      Утилизация:

      Не затвердевшие или не высохшие остатки продукта утилизировать как специальные отходы по согласованию с комитетами по охране окружающей среды (код № 55508 согласно ТА-Abfall). Затвердевшие или высохшие остатки могут быть утилизированы по коду №55513 согласно ТА-Abfall.

      Пустые ёмкости должны быть подвергнуты вторичной переработке.

       

      Экология:

      WGK (Класс водоопасности): 1 (собственная классификация).

      Не допускать попадания в водоёмы, сточные воды и в почву.

       

      Сервис

      Наша техническая-информационная служба всегда в Вашем распоряжении.

      Тел.: Конкен, +49 — 6384/99 38 — 0, факс: +49 — 6384/99 38 — 126

       

      Эти данные без обязательства, основываются на опыте из практики и на результатах проведённых нами испытаний. Рекомендуется в любом случае провести собственные испытания, так как мы не можем оказывать влияния на многообразие окрашиваемых материалов и возможных способов применения данного материала. За возможные последствия вследствие неправомерного использования материала (не по назначению) изготовитель ответственности не несёт. Содержание технических инструкций не является основанием для ответственности продавца. Данные, не содержащиеся в технической инструкции или не совпадающие с ними, требуют письменного подтверждения заводом.

      В любом случае действуют наши общие договорные условия и условия поставок.

       

       

       

       

       

      Таблица тангенсов углов (углы, значения)

      В таблице значения тангенсов от 0° до 360°. Таблица тангенсов нужна, когда у вас под рукой нет калькулятора. Чтобы узнать, чему равен тангенс угла, просто найдите его в таблице. Для начала короткая версия таблицы:

      https://uchim.org/matematika/tablica-tangensov — uchim.org

      Таблица тангенсов для 0°-180°

      tg(1°)0.0175
      tg(2°)0.0349
      tg(3°)0.0524
      tg(4°)0.0699
      tg(5°)0.0875
      tg(6°)0.1051
      tg(7°)0.1228
      tg(8°)0.1405
      tg(9°)0.1584
      tg(10°)0.1763
      tg(11°)0.1944
      tg(12°)0.2126
      tg(13°)0.2309
      tg(14°)0.2493
      tg(15°)0.2679
      tg(16°)0.2867
      tg(17°)0.3057
      tg(18°)0.3249
      tg(19°)0.3443
      tg(20°)0. 364
      tg(21°)0.3839
      tg(22°)0.404
      tg(23°)0.4245
      tg(24°)0.4452
      tg(25°)0.4663
      tg(26°)0.4877
      tg(27°)0.5095
      tg(28°)0.5317
      tg(29°)0.5543
      tg(30°)0.5774
      tg(31°)0.6009
      tg(32°)0.6249
      tg(33°)0.6494
      tg(34°)0.6745
      tg(35°)0.7002
      tg(36°)0.7265
      tg(37°)0.7536
      tg(38°)0.7813
      tg(39°)0.8098
      tg(40°)0.8391
      tg(41°)0.8693
      tg(42°)0.9004
      tg(43°)0.9325
      tg(44°)0.9657
      tg(45°)1
      tg(46°)1. 0355
      tg(47°)1.0724
      tg(48°)1.1106
      tg(49°)1.1504
      tg(50°)1.1918
      tg(51°)1.2349
      tg(52°)1.2799
      tg(53°)1.327
      tg(54°)1.3764
      tg(55°)1.4281
      tg(56°)1.4826
      tg(57°)1.5399
      tg(58°)1.6003
      tg(59°)1.6643
      tg(60°)1.7321
      tg(61°)1.804
      tg(62°)1.8807
      tg(63°)1.9626
      tg(64°)2.0503
      tg(65°)2.1445
      tg(66°)2.246
      tg(67°)2.3559
      tg(68°)2.4751
      tg(69°)2.6051
      tg(70°)2.7475
      tg(71°)2. 9042
      tg(72°)3.0777
      tg(73°)3.2709
      tg(74°)3.4874
      tg(75°)3.7321
      tg(76°)4.0108
      tg(77°)4.3315
      tg(78°)4.7046
      tg(79°)5.1446
      tg(80°)5.6713
      tg(81°)6.3138
      tg(82°)7.1154
      tg(83°)8.1443
      tg(84°)9.5144
      tg(85°)11.4301
      tg(86°)14.3007
      tg(87°)19.0811
      tg(88°)28.6363
      tg(89°)57.29
      tg(90°)
      tg(91°)-57.29
      tg(92°)-28.6363
      tg(93°)-19.0811
      tg(94°)-14.3007
      tg(95°)-11.4301
      tg(96°)-9. 5144
      tg(97°)-8.1443
      tg(98°)-7.1154
      tg(99°)-6.3138
      tg(100°)-5.6713
      tg(101°)-5.1446
      tg(102°)-4.7046
      tg(103°)-4.3315
      tg(104°)-4.0108
      tg(105°)-3.7321
      tg(106°)-3.4874
      tg(107°)-3.2709
      tg(108°)-3.0777
      tg(109°)-2.9042
      tg(110°)-2.7475
      tg(111°)-2.6051
      tg(112°)-2.4751
      tg(113°)-2.3559
      tg(114°)-2.246
      tg(115°)-2.1445
      tg(116°)-2.0503
      tg(117°)-1.9626
      tg(118°)-1.8807
      tg(119°)-1.804
      tg(120°)-1. 7321
      tg(121°)-1.6643
      tg(122°)-1.6003
      tg(123°)-1.5399
      tg(124°)-1.4826
      tg(125°)-1.4281
      tg(126°)-1.3764
      tg(127°)-1.327
      tg(128°)-1.2799
      tg(129°)-1.2349
      tg(130°)-1.1918
      tg(131°)-1.1504
      tg(132°)-1.1106
      tg(133°)-1.0724
      tg(134°)-1.0355
      tg(135°)-1
      tg(136°)-0.9657
      tg(137°)-0.9325
      tg(138°)-0.9004
      tg(139°)-0.8693
      tg(140°)-0.8391
      tg(141°)-0.8098
      tg(142°)-0.7813
      tg(143°)-0.7536
      tg(144°)-0. 7265
      tg(145°)-0.7002
      tg(146°)-0.6745
      tg(147°)-0.6494
      tg(148°)-0.6249
      tg(149°)-0.6009
      tg(150°)-0.5774
      tg(151°)-0.5543
      tg(152°)-0.5317
      tg(153°)-0.5095
      tg(154°)-0.4877
      tg(155°)-0.4663
      tg(156°)-0.4452
      tg(157°)-0.4245
      tg(158°)-0.404
      tg(159°)-0.3839
      tg(160°)-0.364
      tg(161°)-0.3443
      tg(162°)-0.3249
      tg(163°)-0.3057
      tg(164°)-0.2867
      tg(165°)-0.2679
      tg(166°)-0.2493
      tg(167°)-0.2309
      tg(168°)-0. 2126
      tg(169°)-0.1944
      tg(170°)-0.1763
      tg(171°)-0.1584
      tg(172°)-0.1405
      tg(173°)-0.1228
      tg(174°)-0.1051
      tg(175°)-0.0875
      tg(176°)-0.0699
      tg(177°)-0.0524
      tg(178°)-0.0349
      tg(179°)-0.0175
      tg(180°)-0

      Таблица тангенсов для 180° — 360°

      tg(181°)0.0175
      tg(182°)0.0349
      tg(183°)0.0524
      tg(184°)0.0699
      tg(185°)0.0875
      tg(186°)0.1051
      tg(187°)0.1228
      tg(188°)0.1405
      tg(189°)0.1584
      tg(190°)0. 1763
      tg(191°)0.1944
      tg(192°)0.2126
      tg(193°)0.2309
      tg(194°)0.2493
      tg(195°)0.2679
      tg(196°)0.2867
      tg(197°)0.3057
      tg(198°)0.3249
      tg(199°)0.3443
      tg(200°)0.364
      tg(201°)0.3839
      tg(202°)0.404
      tg(203°)0.4245
      tg(204°)0.4452
      tg(205°)0.4663
      tg(206°)0.4877
      tg(207°)0.5095
      tg(208°)0.5317
      tg(209°)0.5543
      tg(210°)0.5774
      tg(211°)0.6009
      tg(212°)0.6249
      tg(213°)0.6494
      tg(214°)0.6745
      tg(215°)0. 7002
      tg(216°)0.7265
      tg(217°)0.7536
      tg(218°)0.7813
      tg(219°)0.8098
      tg(220°)0.8391
      tg(221°)0.8693
      tg(222°)0.9004
      tg(223°)0.9325
      tg(224°)0.9657
      tg(225°)1
      tg(226°)1.0355
      tg(227°)1.0724
      tg(228°)1.1106
      tg(229°)1.1504
      tg(230°)1.1918
      tg(231°)1.2349
      tg(232°)1.2799
      tg(233°)1.327
      tg(234°)1.3764
      tg(235°)1.4281
      tg(236°)1.4826
      tg(237°)1.5399
      tg(238°)1.6003
      tg(239°)1.6643
      tg(240°)1. 7321
      tg(241°)1.804
      tg(242°)1.8807
      tg(243°)1.9626
      tg(244°)2.0503
      tg(245°)2.1445
      tg(246°)2.246
      tg(247°)2.3559
      tg(248°)2.4751
      tg(249°)2.6051
      tg(250°)2.7475
      tg(251°)2.9042
      tg(252°)3.0777
      tg(253°)3.2709
      tg(254°)3.4874
      tg(255°)3.7321
      tg(256°)4.0108
      tg(257°)4.3315
      tg(258°)4.7046
      tg(259°)5.1446
      tg(260°)5.6713
      tg(261°)6.3138
      tg(262°)7.1154
      tg(263°)8.1443
      tg(264°)9. 5144
      tg(265°)11.4301
      tg(266°)14.3007
      tg(267°)19.0811
      tg(268°)28.6363
      tg(269°)57.29
      tg(270°)— ∞
      tg(271°)-57.29
      tg(272°)-28.6363
      tg(273°)-19.0811
      tg(274°)-14.3007
      tg(275°)-11.4301
      tg(276°)-9.5144
      tg(277°)-8.1443
      tg(278°)-7.1154
      tg(279°)-6.3138
      tg(280°)-5.6713
      tg(281°)-5.1446
      tg(282°)-4.7046
      tg(283°)-4.3315
      tg(284°)-4.0108
      tg(285°)-3.7321
      tg(286°)-3.4874
      tg(287°)-3.2709
      tg(288°)-3. 0777
      tg(289°)-2.9042
      tg(290°)-2.7475
      tg(291°)-2.6051
      tg(292°)-2.4751
      tg(293°)-2.3559
      tg(294°)-2.246
      tg(295°)-2.1445
      tg(296°)-2.0503
      tg(297°)-1.9626
      tg(298°)-1.8807
      tg(299°)-1.804
      tg(300°)-1.7321
      tg(301°)-1.6643
      tg(302°)-1.6003
      tg(303°)-1.5399
      tg(304°)-1.4826
      tg(305°)-1.4281
      tg(306°)-1.3764
      tg(307°)-1.327
      tg(308°)-1.2799
      tg(309°)-1.2349
      tg(310°)-1.1918
      tg(311°)-1.1504
      tg(312°)-1. 1106
      tg(313°)-1.0724
      tg(314°)-1.0355
      tg(315°)-1
      tg(316°)-0.9657
      tg(317°)-0.9325
      tg(318°)-0.9004
      tg(319°)-0.8693
      tg(320°)-0.8391
      tg(321°)-0.8098
      tg(322°)-0.7813
      tg(323°)-0.7536
      tg(324°)-0.7265
      tg(325°)-0.7002
      tg(326°)-0.6745
      tg(327°)-0.6494
      tg(328°)-0.6249
      tg(329°)-0.6009
      tg(330°)-0.5774
      tg(331°)-0.5543
      tg(332°)-0.5317
      tg(333°)-0.5095
      tg(334°)-0.4877
      tg(335°)-0.4663
      tg(336°)-0. 4452
      tg(337°)-0.4245
      tg(338°)-0.404
      tg(339°)-0.3839
      tg(340°)-0.364
      tg(341°)-0.3443
      tg(342°)-0.3249
      tg(343°)-0.3057
      tg(344°)-0.2867
      tg(345°)-0.2679
      tg(346°)-0.2493
      tg(347°)-0.2309
      tg(348°)-0.2126
      tg(349°)-0.1944
      tg(350°)-0.1763
      tg(351°)-0.1584
      tg(352°)-0.1405
      tg(353°)-0.1228
      tg(354°)-0.1051
      tg(355°)-0.0875
      tg(356°)-0.0699
      tg(357°)-0.0524
      tg(358°)-0.0349
      tg(359°)-0.0175
      tg(360°)-0

      Существуют также следующие таблицы тригонометрических функций по геометрии: таблица синусов, таблица косинусов и таблица котангенсов.

      Всё для учебы » Математика в школе » Таблица тангенсов углов (углы, значения)

      Если страница помогла, сохраните её и поделитесь ссылкой с друзьями:

      Ссылка: https://uchim.org/matematika/tablica-tangensov


      Таблица тангенсов | Онлайн калькуляторы, расчеты и формулы на GELEOT.RU

      Тангенс, как отношение катетов в прямоугольном треугольнике, представляет собой функцию которая выглядит как дуга окружности внутри данного треугольника с центром в вершине угла и прилежащим катетом в качестве радиуса.

      Значение тангенса показывает не только раскрытие угла α, но и насколько один катет больше другого. При тангенсе угла α, равном 1, катеты равны друг другу и треугольник считается равнобедренным. Значения всех тангенсов и соответствующих им углов можно найти в таблице, приведенной ниже.

      Найти тангенс угла tg(α), зная угол

      Угол α

      Таблица тангенсов от 0° до 180°


      tg(1°)0. 0175
      tg(2°)0.0349
      tg(3°)0.0524
      tg(4°)0.0699
      tg(5°)0.0875
      tg(6°)0.1051
      tg(7°)0.1228
      tg(8°)0.1405
      tg(9°)0.1584
      tg(10°)0.1763
      tg(11°)0.1944
      tg(12°)0.2126
      tg(13°)0.2309
      tg(14°)0.2493
      tg(15°)0.2679
      tg(16°)0.2867
      tg(17°)0.3057
      tg(18°)0.3249
      tg(19°)0.3443
      tg(20°)0.364
      tg(21°)0.3839
      tg(22°)0.404
      tg(23°)0.4245
      tg(24°)0.4452
      tg(25°)0.4663
      tg(26°)0.4877
      tg(27°)0. 5095
      tg(28°)0.5317
      tg(29°)0.5543
      tg(30°)0.5774
      tg(31°)0.6009
      tg(32°)0.6249
      tg(33°)0.6494
      tg(34°)0.6745
      tg(35°)0.7002
      tg(36°)0.7265
      tg(37°)0.7536
      tg(38°)0.7813
      tg(39°)0.8098
      tg(40°)0.8391
      tg(41°)0.8693
      tg(42°)0.9004
      tg(43°)0.9325
      tg(44°)0.9657
      tg(45°)1
      tg(46°)1.0355
      tg(47°)1.0724
      tg(48°)1.1106
      tg(49°)1.1504
      tg(50°)1.1918
      tg(51°)1.2349
      tg(52°)1. 2799
      tg(53°)1.327
      tg(54°)1.3764
      tg(55°)1.4281
      tg(56°)1.4826
      tg(57°)1.5399
      tg(58°)1.6003
      tg(59°)1.6643
      tg(60°)1.7321
      tg(61°)1.804
      tg(62°)1.8807
      tg(63°)1.9626
      tg(64°)2.0503
      tg(65°)2.1445
      tg(66°)2.246
      tg(67°)2.3559
      tg(68°)2.4751
      tg(69°)2.6051
      tg(70°)2.7475
      tg(71°)2.9042
      tg(72°)3.0777
      tg(73°)3.2709
      tg(74°)3.4874
      tg(75°)3.7321
      tg(76°)4.0108
      tg(77°)4. 3315
      tg(78°)4.7046
      tg(79°)5.1446
      tg(80°)5.6713
      tg(81°)6.3138
      tg(82°)7.1154
      tg(83°)8.1443
      tg(84°)9.5144
      tg(85°)11.4301
      tg(86°)14.3007
      tg(87°)19.0811
      tg(88°)28.6363
      tg(89°)57.29
      tg(90°)
      tg(91°)-57.29
      tg(92°)-28.6363
      tg(93°)-19.0811
      tg(94°)-14.3007
      tg(95°)-11.4301
      tg(96°)-9.5144
      tg(97°)-8.1443
      tg(98°)-7.1154
      tg(99°)-6.3138
      tg(100°)-5.6713
      tg(101°)-5.1446
      tg(102°)-4. 7046
      tg(103°)-4.3315
      tg(104°)-4.0108
      tg(105°)-3.7321
      tg(106°)-3.4874
      tg(107°)-3.2709
      tg(108°)-3.0777
      tg(109°)-2.9042
      tg(110°)-2.7475
      tg(111°)-2.6051
      tg(112°)-2.4751
      tg(113°)-2.3559
      tg(114°)-2.246
      tg(115°)-2.1445
      tg(116°)-2.0503
      tg(117°)-1.9626
      tg(118°)-1.8807
      tg(119°)-1.804
      tg(120°)-1.7321
      tg(121°)-1.6643
      tg(122°)-1.6003
      tg(123°)-1.5399
      tg(124°)-1.4826
      tg(125°)-1. 4281
      tg(126°)-1.3764
      tg(127°)-1.327
      tg(128°)-1.2799
      tg(129°)-1.2349
      tg(130°)-1.1918
      tg(131°)-1.1504
      tg(132°)-1.1106
      tg(133°)-1.0724
      tg(134°)-1.0355
      tg(135°)-1
      tg(136°)-0.9657
      tg(137°)-0.9325
      tg(138°)-0.9004
      tg(139°)-0.8693
      tg(140°)-0.8391
      tg(141°)-0.8098
      tg(142°)-0.7813
      tg(143°)-0.7536
      tg(144°)-0.7265
      tg(145°)-0.7002
      tg(146°)-0.6745
      tg(147°)-0.6494
      tg(148°)-0. 6249
      tg(149°)-0.6009
      tg(150°)-0.5774
      tg(151°)-0.5543
      tg(152°)-0.5317
      tg(153°)-0.5095
      tg(154°)-0.4877
      tg(155°)-0.4663
      tg(156°)-0.4452
      tg(157°)-0.4245
      tg(158°)-0.404
      tg(159°)-0.3839
      tg(160°)-0.364
      tg(161°)-0.3443
      tg(162°)-0.3249
      tg(163°)-0.3057
      tg(164°)-0.2867
      tg(165°)-0.2679
      tg(166°)-0.2493
      tg(167°)-0.2309
      tg(168°)-0.2126
      tg(169°)-0.1944
      tg(170°)-0.1763
      tg(171°)-0.1584
      tg(172°)-0. 1405
      tg(173°)-0.1228
      tg(174°)-0.1051
      tg(175°)-0.0875
      tg(176°)-0.0699
      tg(177°)-0.0524
      tg(178°)-0.0349
      tg(179°)-0.0175
      tg(180°)-0

      Таблица тангенсов от 181° до 360°


      tg(181°)0.0175
      tg(182°)0.0349
      tg(183°)0.0524
      tg(184°)0.0699
      tg(185°)0.0875
      tg(186°)0.1051
      tg(187°)0.1228
      tg(188°)0.1405
      tg(189°)0.1584
      tg(190°)0.1763
      tg(191°)0.1944
      tg(192°)0.2126
      tg(193°)0. 2309
      tg(194°)0.2493
      tg(195°)0.2679
      tg(196°)0.2867
      tg(197°)0.3057
      tg(198°)0.3249
      tg(199°)0.3443
      tg(200°)0.364
      tg(201°)0.3839
      tg(202°)0.404
      tg(203°)0.4245
      tg(204°)0.4452
      tg(205°)0.4663
      tg(206°)0.4877
      tg(207°)0.5095
      tg(208°)0.5317
      tg(209°)0.5543
      tg(210°)0.5774
      tg(211°)0.6009
      tg(212°)0.6249
      tg(213°)0.6494
      tg(214°)0.6745
      tg(215°)0.7002
      tg(216°)0.7265
      tg(217°)0. 7536
      tg(218°)0.7813
      tg(219°)0.8098
      tg(220°)0.8391
      tg(221°)0.8693
      tg(222°)0.9004
      tg(223°)0.9325
      tg(224°)0.9657
      tg(225°)1
      tg(226°)1.0355
      tg(227°)1.0724
      tg(228°)1.1106
      tg(229°)1.1504
      tg(230°)1.1918
      tg(231°)1.2349
      tg(232°)1.2799
      tg(233°)1.327
      tg(234°)1.3764
      tg(235°)1.4281
      tg(236°)1.4826
      tg(237°)1.5399
      tg(238°)1.6003
      tg(239°)1.6643
      tg(240°)1.7321
      tg(241°)1.804
      tg(242°)1. 8807
      tg(243°)1.9626
      tg(244°)2.0503
      tg(245°)2.1445
      tg(246°)2.246
      tg(247°)2.3559
      tg(248°)2.4751
      tg(249°)2.6051
      tg(250°)2.7475
      tg(251°)2.9042
      tg(252°)3.0777
      tg(253°)3.2709
      tg(254°)3.4874
      tg(255°)3.7321
      tg(256°)4.0108
      tg(257°)4.3315
      tg(258°)4.7046
      tg(259°)5.1446
      tg(260°)5.6713
      tg(261°)6.3138
      tg(262°)7.1154
      tg(263°)8.1443
      tg(264°)9.5144
      tg(265°)11.4301
      tg(266°)14. 3007
      tg(267°)19.0811
      tg(268°)28.6363
      tg(269°)57.29
      tg(270°)— ∞
      tg(271°)-57.29
      tg(272°)-28.6363
      tg(273°)-19.0811
      tg(274°)-14.3007
      tg(275°)-11.4301
      tg(276°)-9.5144
      tg(277°)-8.1443
      tg(278°)-7.1154
      tg(279°)-6.3138
      tg(280°)-5.6713
      tg(281°)-5.1446
      tg(282°)-4.7046
      tg(283°)-4.3315
      tg(284°)-4.0108
      tg(285°)-3.7321
      tg(286°)-3.4874
      tg(287°)-3.2709
      tg(288°)-3.0777
      tg(289°)-2. 9042
      tg(290°)-2.7475
      tg(291°)-2.6051
      tg(292°)-2.4751
      tg(293°)-2.3559
      tg(294°)-2.246
      tg(295°)-2.1445
      tg(296°)-2.0503
      tg(297°)-1.9626
      tg(298°)-1.8807
      tg(299°)-1.804
      tg(300°)-1.7321
      tg(301°)-1.6643
      tg(302°)-1.6003
      tg(303°)-1.5399
      tg(304°)-1.4826
      tg(305°)-1.4281
      tg(306°)-1.3764
      tg(307°)-1.327
      tg(308°)-1.2799
      tg(309°)-1.2349
      tg(310°)-1.1918
      tg(311°)-1.1504
      tg(312°)-1.1106
      tg(313°)-1. 0724
      tg(314°)-1.0355
      tg(315°)-1
      tg(316°)-0.9657
      tg(317°)-0.9325
      tg(318°)-0.9004
      tg(319°)-0.8693
      tg(320°)-0.8391
      tg(321°)-0.8098
      tg(322°)-0.7813
      tg(323°)-0.7536
      tg(324°)-0.7265
      tg(325°)-0.7002
      tg(326°)-0.6745
      tg(327°)-0.6494
      tg(328°)-0.6249
      tg(329°)-0.6009
      tg(330°)-0.5774
      tg(331°)-0.5543
      tg(332°)-0.5317
      tg(333°)-0.5095
      tg(334°)-0.4877
      tg(335°)-0.4663
      tg(336°)-0. 4452
      tg(337°)-0.4245
      tg(338°)-0.404
      tg(339°)-0.3839
      tg(340°)-0.364
      tg(341°)-0.3443
      tg(342°)-0.3249
      tg(343°)-0.3057
      tg(344°)-0.2867
      tg(345°)-0.2679
      tg(346°)-0.2493
      tg(347°)-0.2309
      tg(348°)-0.2126
      tg(349°)-0.1944
      tg(350°)-0.1763
      tg(351°)-0.1584
      tg(352°)-0.1405
      tg(353°)-0.1228
      tg(354°)-0.1051
      tg(355°)-0.0875
      tg(356°)-0.0699
      tg(357°)-0.0524
      tg(358°)-0.0349
      tg(359°)-0.0175
      tg(360°)-0

      Таблица тангенсов углов углов от 0° — 360°.

      Углы с шагом в 1°. Таблица значений тангенса, tg

      ГОСТы, СНиПы

      Карта сайта TehTab.ru

      Поиск по сайту TehTab.ru

      Навигация по справочнику TehTab.ru:  главная страница / / Техническая информация/ / Математический справочник/ / Таблицы численных значений. (Таблица квадратов, кубов, синусов ….) + Таблицы Брадиса / / Таблица тангенсов углов углов от 0° — 360°. Углы с шагом в 1°. Таблица значений тангенса, tg

      Подробная таблица тангенсов. Шаг — 1 градус.

      Таблица тангенсов углов углов от 0° — 360°. Углы с шагом в 1°.

      tg(0°)=tg(360°)=0 точная, но чуть более сложная таблица ( с точностью до 1″) здесь.

      Углы
      1° — 90°

      Углы
      91 ° — 180°

      Углы
      181° — 270°

      Углы
      271 ° — 360°

      Угол

      tg

      tg= 0. 0174
      tg= 0.0349
      tg= 0.0524
      tg= 0.0699
      tg= 0.0874
      tg= 0.1051
      tg= 0.1227
      tg= 0.1405
      tg= 0.1583
      10° tg= 0.1763
      11° tg= 0.1943
      12° tg= 0.2125
      13° tg= 0. 2308
      14° tg= 0.2493
      15° tg= 0.2679
      16° tg= 0.2867
      17° tg= 0.3057
      18° tg= 0.3249
      19° tg= 0.3443
      20° tg= 0.364
      21° tg= 0.3839
      22° tg= 0.404
      23° tg= 0.4245
      24° tg= 0.4452
      25° tg= 0. 4663
      26° tg= 0.4877
      27° tg= 0.5095
      28° tg= 0.5317
      29° tg= 0.5543
      30° tg= 0.5774
      31° tg= 0.6009
      32° tg= 0.6249
      33° tg= 0.6494
      34° tg= 0.6745
      35° tg= 0.7002
      36° tg= 0.7265
      37° tg= 0. 7535
      38° tg= 0.7813
      39° tg= 0.8098
      40° tg= 0.8390
      41° tg= 0.8693
      42° tg= 0.9004
      43° tg= 0.9325
      44° tg= 0.9657
      45° tg= 1
      46° tg= 1.0355
      47° tg= 1.0724
      48° tg= 1.1106
      49° tg= 1. 1504
      50° tg= 1.1918
      51° tg= 1.2349
      52° tg= 1.2799
      53° tg= 1.327
      54° tg= 1.3764
      55° tg= 1.4281
      56° tg= 1.4826
      57° tg= 1.5399
      58° tg= 1.6003
      59° tg= 1.6643
      60° tg= 1.7321
      61° tg= 1. 804
      62° tg= 1.8807
      63° tg= 1.9626
      64° tg= 2.0503
      65° tg= 2.1445
      66° tg= 2.2460
      67° tg= 2.3559
      68° tg= 2.475
      69° tg= 2.605
      70° tg= 2.7475
      71° tg= 2.9042
      72° tg= 3.0777
      73° tg= 3. 2709
      74° tg= 3.4874
      75° tg= 3.732
      76° tg= 4.0108
      77° tg= 4.3315
      78° tg= 4.7046
      79° tg= 5.1446
      80° tg= 5.6713
      81° tg= 6.3138
      82° tg= 7.1154
      83° tg= 8.1443
      84° tg= 9.5144
      85° tg= 11. 4301
      86° tg= 14.3007
      87° tg= 19.0811
      88° tg= 28.6363
      89° tg= 57.29
      90° tg не определен

      Угол

      tg

      91° tg= -57.29
      92° tg= -28.6363
      93° tg= -19.0811
      94° tg= -14. 3007
      95° tg= -11.4301
      96° tg= -9.5144
      97° tg= -8.1443
      98° tg= -7.1154
      99° tg= -6.3138
      100° tg= -5.6713
      101° tg= -5.1446
      102° tg= -4.7046
      103° tg= -4.3315
      104° tg= -4.0108
      105° tg= -3.732
      106° tg= -3. 4874
      107° tg= -3.2709
      108° tg= -3.0777
      109° tg= -2.9042
      110° tg= -2.7475
      111° tg= -2.605
      112° tg= -2.475
      113° tg= -2.3559
      114° tg= -2.2460
      115° tg= -2.1445
      116° tg= -2.0503
      117° tg= -1.9626
      118° tg= -1. 8807
      119° tg= -1.804
      120° tg= -1.7321
      121° tg= -1.6643
      122° tg= -1.6003
      123° tg= -1.5399
      124° tg= -1.4826
      125° tg= -1.4281
      126° tg= -1.3764
      127° tg= -1.327
      128° tg= -1.2799
      129° tg= -1.2349
      130° tg= -1. 1918
      131° tg= -1.1504
      132° tg= -1.1106
      133° tg= -1.0724
      134° tg= -1.0355
      135° tg= -1
      136° tg= -0.9657
      137° tg= -0.9325
      138° tg= -0.9004
      139° tg= -0.8693
      140° tg= -0.8390
      141° tg= -0.8098
      142° tg= -0. 7813
      143° tg= -0.7535
      144° tg= -0.7265
      145° tg= -0.7002
      146° tg= -0.6745
      147° tg= -0.6494
      148° tg= -0.6249
      149° tg= -0.6009
      150° tg= -0.5774
      151° tg= -0.5543
      152° tg= -0.5317
      153° tg= -0.5095
      154° tg= -0. 4877
      155° tg= -0.4663
      156° tg= -0.4452
      157° tg= -0.4245
      158° tg= -0.404
      159° tg= -0.3839
      160° tg= -0.364
      161° tg= -0.3443
      162° tg= -0.3249
      163° tg= -0.3057
      164° tg= -0.2867
      165° tg= -0.2679
      166° tg= -0. 2493
      167° tg= -0.2308
      168° tg= -0.2125
      169° tg= -0.1943
      170° tg= -0.1763
      171° tg= -0.1583
      172° tg= -0.1405
      173° tg= -0.1227
      174° tg= -0.1051
      175° tg= -0.0874
      176° tg= -0.0699
      177° tg= -0.0524
      178° tg= -0. 0349
      179° tg= -0.0174
      180° tg= 0

      Угол

      tg

      181° tg= 0.0174
      182° tg= 0.0349
      183° tg= 0.0524
      184° tg= 0.0699
      185° tg= 0.0874
      186° tg= 0.1051
      187° tg= 0. 1227
      188° tg= 0.1405
      189° tg= 0.1583
      190° tg= 0.1763
      191° tg= 0.1943
      192° tg= 0.2125
      193° tg= 0.2308
      194° tg= 0.2493
      195° tg= 0.2679
      196° tg= 0.2867
      197° tg= 0.3057
      198° tg= 0. 3249
      199° tg= 0.3443
      200° tg= 0.364
      201° tg= 0.3839
      202° tg= 0.404
      203° tg= 0.4245
      204° tg= 0.4452
      205° tg= 0.4663
      206° tg= 0.4877
      207° tg= 0.5095
      208° tg= 0.5317
      209° tg= 0. 5543
      210° tg= 0.5774
      211° tg= 0.6009
      212° tg= 0.6249
      213° tg= 0.6494
      214° tg= 0.6745
      215° tg= 0.7002
      216° tg= 0.7265
      217° tg= 0.7535
      218° tg= 0.7813
      219° tg= 0.8098
      220° tg= 0. 8390
      221° tg= 0.8693
      222° tg= 0.9004
      223° tg= 0.9325
      224° tg= 0.9657
      225° tg= 1
      226° tg= 1.0355
      227° tg= 1.0724
      228° tg= 1.1106
      229° tg= 1.1504
      230° tg= 1.1918
      231° tg= 1. 2349
      232° tg= 1.2799
      233° tg= 1.327
      234° tg= 1.3764
      235° tg= 1.4281
      236° tg= 1.4826
      237° tg= 1.5399
      238° tg= 1.6003
      239° tg= 1.6643
      240° tg= 1.7321
      241° tg= 1.804
      242° tg= 1. 8807
      243° tg= 1.9626
      244° tg= 2.0503
      245° tg= 2.1445
      246° tg= 2.2460
      247° tg= 2.3559
      248° tg= 2.475
      249° tg= 2.605
      250° tg= 2.7475
      251° tg= 2.9042
      252° tg= 3.0777
      253° tg= 3. 2709
      254° tg= 3.4874
      255° tg= 3.732
      256° tg= 4.0108
      257° tg= 4.3315
      258° tg= 4.7046
      259° tg= 5.1446
      260° tg= 5.6713
      261° tg= 6.3138
      262° tg= 7.1154
      263° tg= 8.1443
      264° tg= 9. 5144
      265° tg= 11.4301
      266° tg= 14.3007
      267° tg= 19.0811
      268° tg= 28.6363
      269° tg= 57.29
      270° tg не определен

      Угол

      tg

      271° tg= -57.29
      272° tg= -28. 6363
      273° tg= -19.0811
      274° tg= -14.3007
      275° tg= -11.4301
      276° tg= -9.5144
      277° tg= -8.1443
      278° tg= -7.1154
      279° tg= -6.3138
      280° tg= -5.6713
      281° tg= -5.1446
      282° tg= -4. 7046
      283° tg= -4.3315
      284° tg= -4.0108
      285° tg= -3.732
      286° tg= -3.4874
      287° tg= -3.2709
      288° tg= -3.0777
      289° tg= -2.9042
      290° tg= -2.7475
      291° tg= -2.605
      292° tg= -2.475
      293° tg= -2. 3559
      294° tg= -2.2460
      295° tg= -2.1445
      296° tg= -2.0503
      297° tg= -1.9626
      298° tg= -1.8807
      299° tg= -1.804
      300° tg= -1.7321
      301° tg= -1.6643
      302° tg= -1.6003
      303° tg= -1.5399
      304° tg= -1. 4826
      305° tg= -1.4281
      306° tg= -1.3764
      307° tg= -1.327
      308° tg= -1.2799
      309° tg= -1.2349
      310° tg= -1.1918
      311° tg= -1.1504
      312° tg= -1.1106
      313° tg= -1.0724
      314° tg= -1.0355
      315° tg= -1
      316° tg= -0. 9657
      317° tg= -0.9325
      318° tg= -0.9004
      319° tg= -0.8693
      320° tg= -0.8390
      321° tg= -0.8098
      322° tg= -0.7813
      323° tg= -0.7535
      324° tg= -0.7265
      325° tg= -0.7002
      326° tg= -0.6745
      327° tg= -0. 6494
      328° tg= -0.6249
      329° tg= -0.6009
      330° tg= -0.5774
      331° tg= -0.5543
      332° tg= -0.5317
      333° tg= -0.5095
      334° tg= -0.4877
      335° tg= -0.4663
      336° tg= -0.4452
      337° tg= -0.4245
      338° tg= -0. 404
      339° tg= -0.3839
      340° tg= -0.364
      341° tg= -0.3443
      342° tg= -0.3249
      343° tg= -0.3057
      344° tg= -0.2867
      345° tg= -0.2679
      346° tg= -0.2493
      347° tg= -0.2308
      348° tg= -0.2125
      349° tg= -0. 1943
      350° tg= -0.1763
      351° tg= -0.1583
      352° tg= -0.1405
      353° tg= -0.1227
      354° tg= -0.1051
      355° tg= -0.0874
      356° tg= -0.0699
      357° tg= -0.0524
      358° tg= -0.0349
      359° tg= -0.0174
      360° tg= 0

      таблица тангенсов, таблица тангенсов и синусов, таблица тангенсов косинусов, таблица синусов косинусов тангенсов котангенсов

      Дополнительная информация от TehTab. ru:


      Нашли ошибку? Есть дополнения? Напишите нам об этом, указав ссылку на страницу.

      TehTab.ru

      Реклама, сотрудничество: [email protected]

      Обращаем ваше внимание на то, что данный интернет-сайт носит исключительно информационный характер. Информация, представленная на сайте, не является официальной и предоставлена только в целях ознакомления. Все риски за использование информаци с сайта посетители берут на себя. Проект TehTab.ru является некоммерческим, не поддерживается никакими политическими партиями и иностранными организациями.

      Таблица тангенсов, найти тангенс угла

    • Все калькуляторы
    • /
    • org/ListItem»>Учеба и наука
    • /
    • Математика
    • /   Таблица тангенсов, найти тангенс угла

      Тангенс угла – одна из основных тригонометрических функций. Представляет собой соотношение катетов прямоугольного треугольника. То есть, tg(А)=ВС/АС, где ВС – противолежащий к углу (А) катет, АС – прилежащий катет.

      Зачем необходимо знать тангенс угла? Такие данные имеют вполне практическое применение: в геодезии, мореходстве, авиации. Зная одну из сторон треугольника и угол, можно легко получить все остальные данные, используя тригонометрические тождества. Все расчеты легко производить с помощью онлайн-калькулятора на нашем сайте. Данные указаны в таблице тангенсов.

      Для практического использования подходят не только таблицы Брадиса. Все тригонометрические функции вычисляются посредством калькулятора. Найдите красивое решение для вашей задачи.


      tg(1°)0. 0175
      tg(2°)0.0349
      tg(3°)0.0524
      tg(4°)0.0699
      tg(5°)0.0875
      tg(6°)0.1051
      tg(7°)0.1228
      tg(8°)0.1405
      tg(9°)0.1584
      tg(10°)0.1763
      tg(11°)0.1944
      tg(12°)0.2126
      tg(13°)0.2309
      tg(14°)0.2493
      tg(15°)0.2679
      tg(16°)0.2867
      tg(17°)0.3057
      tg(18°)0.3249
      tg(19°)0.3443
      tg(20°)0.364
      tg(21°)0.3839
      tg(22°)0.404
      tg(23°)0.4245
      tg(24°)0.4452
      tg(25°)0.4663
      tg(26°)0.4877
      tg(27°)0. 5095
      tg(28°)0.5317
      tg(29°)0.5543
      tg(30°)0.5774
      tg(31°)0.6009
      tg(32°)0.6249
      tg(33°)0.6494
      tg(34°)0.6745
      tg(35°)0.7002
      tg(36°)0.7265
      tg(37°)0.7536
      tg(38°)0.7813
      tg(39°)0.8098
      tg(40°)0.8391
      tg(41°)0.8693
      tg(42°)0.9004
      tg(43°)0.9325
      tg(44°)0.9657
      tg(45°)1
      tg(46°)1.0355
      tg(47°)1.0724
      tg(48°)1.1106
      tg(49°)1.1504
      tg(50°)1.1918
      tg(51°)1.2349
      tg(52°)1. 2799
      tg(53°)1.327
      tg(54°)1.3764
      tg(55°)1.4281
      tg(56°)1.4826
      tg(57°)1.5399
      tg(58°)1.6003
      tg(59°)1.6643
      tg(60°)1.7321
      tg(61°)1.804
      tg(62°)1.8807
      tg(63°)1.9626
      tg(64°)2.0503
      tg(65°)2.1445
      tg(66°)2.246
      tg(67°)2.3559
      tg(68°)2.4751
      tg(69°)2.6051
      tg(70°)2.7475
      tg(71°)2.9042
      tg(72°)3.0777
      tg(73°)3.2709
      tg(74°)3.4874
      tg(75°)3.7321
      tg(76°)4.0108
      tg(77°)4. 3315
      tg(78°)4.7046
      tg(79°)5.1446
      tg(80°)5.6713
      tg(81°)6.3138
      tg(82°)7.1154
      tg(83°)8.1443
      tg(84°)9.5144
      tg(85°)11.4301
      tg(86°)14.3007
      tg(87°)19.0811
      tg(88°)28.6363
      tg(89°)57.29
      tg(90°)
      tg(91°)-57.29
      tg(92°)-28.6363
      tg(93°)-19.0811
      tg(94°)-14.3007
      tg(95°)-11.4301
      tg(96°)-9.5144
      tg(97°)-8.1443
      tg(98°)-7.1154
      tg(99°)-6.3138
      tg(100°)-5.6713
      tg(101°)-5.1446
      tg(102°)-4. 7046
      tg(103°)-4.3315
      tg(104°)-4.0108
      tg(105°)-3.7321
      tg(106°)-3.4874
      tg(107°)-3.2709
      tg(108°)-3.0777
      tg(109°)-2.9042
      tg(110°)-2.7475
      tg(111°)-2.6051
      tg(112°)-2.4751
      tg(113°)-2.3559
      tg(114°)-2.246
      tg(115°)-2.1445
      tg(116°)-2.0503
      tg(117°)-1.9626
      tg(118°)-1.8807
      tg(119°)-1.804
      tg(120°)-1.7321
      tg(121°)-1.6643
      tg(122°)-1.6003
      tg(123°)-1.5399
      tg(124°)-1.4826
      tg(125°)-1. 4281
      tg(126°)-1.3764
      tg(127°)-1.327
      tg(128°)-1.2799
      tg(129°)-1.2349
      tg(130°)-1.1918
      tg(131°)-1.1504
      tg(132°)-1.1106
      tg(133°)-1.0724
      tg(134°)-1.0355
      tg(135°)-1
      tg(136°)-0.9657
      tg(137°)-0.9325
      tg(138°)-0.9004
      tg(139°)-0.8693
      tg(140°)-0.8391
      tg(141°)-0.8098
      tg(142°)-0.7813
      tg(143°)-0.7536
      tg(144°)-0.7265
      tg(145°)-0.7002
      tg(146°)-0.6745
      tg(147°)-0.6494
      tg(148°)-0. 6249
      tg(149°)-0.6009
      tg(150°)-0.5774
      tg(151°)-0.5543
      tg(152°)-0.5317
      tg(153°)-0.5095
      tg(154°)-0.4877
      tg(155°)-0.4663
      tg(156°)-0.4452
      tg(157°)-0.4245
      tg(158°)-0.404
      tg(159°)-0.3839
      tg(160°)-0.364
      tg(161°)-0.3443
      tg(162°)-0.3249
      tg(163°)-0.3057
      tg(164°)-0.2867
      tg(165°)-0.2679
      tg(166°)-0.2493
      tg(167°)-0.2309
      tg(168°)-0.2126
      tg(169°)-0.1944
      tg(170°)-0.1763
      tg(171°)-0.1584
      tg(172°)-0. 1405
      tg(173°)-0.1228
      tg(174°)-0.1051
      tg(175°)-0.0875
      tg(176°)-0.0699
      tg(177°)-0.0524
      tg(178°)-0.0349
      tg(179°)-0.0175
      tg(180°)-0

      tg(181°)0.0175
      tg(182°)0.0349
      tg(183°)0.0524
      tg(184°)0.0699
      tg(185°)0.0875
      tg(186°)0.1051
      tg(187°)0.1228
      tg(188°)0.1405
      tg(189°)0.1584
      tg(190°)0.1763
      tg(191°)0.1944
      tg(192°)0.2126
      tg(193°)0.2309
      tg(194°)0.2493
      tg(195°)0. 2679
      tg(196°)0.2867
      tg(197°)0.3057
      tg(198°)0.3249
      tg(199°)0.3443
      tg(200°)0.364
      tg(201°)0.3839
      tg(202°)0.404
      tg(203°)0.4245
      tg(204°)0.4452
      tg(205°)0.4663
      tg(206°)0.4877
      tg(207°)0.5095
      tg(208°)0.5317
      tg(209°)0.5543
      tg(210°)0.5774
      tg(211°)0.6009
      tg(212°)0.6249
      tg(213°)0.6494
      tg(214°)0.6745
      tg(215°)0.7002
      tg(216°)0.7265
      tg(217°)0.7536
      tg(218°)0.7813
      tg(219°)0. 8098
      tg(220°)0.8391
      tg(221°)0.8693
      tg(222°)0.9004
      tg(223°)0.9325
      tg(224°)0.9657
      tg(225°)1
      tg(226°)1.0355
      tg(227°)1.0724
      tg(228°)1.1106
      tg(229°)1.1504
      tg(230°)1.1918
      tg(231°)1.2349
      tg(232°)1.2799
      tg(233°)1.327
      tg(234°)1.3764
      tg(235°)1.4281
      tg(236°)1.4826
      tg(237°)1.5399
      tg(238°)1.6003
      tg(239°)1.6643
      tg(240°)1.7321
      tg(241°)1.804
      tg(242°)1.8807
      tg(243°)1.9626
      tg(244°)2. 0503
      tg(245°)2.1445
      tg(246°)2.246
      tg(247°)2.3559
      tg(248°)2.4751
      tg(249°)2.6051
      tg(250°)2.7475
      tg(251°)2.9042
      tg(252°)3.0777
      tg(253°)3.2709
      tg(254°)3.4874
      tg(255°)3.7321
      tg(256°)4.0108
      tg(257°)4.3315
      tg(258°)4.7046
      tg(259°)5.1446
      tg(260°)5.6713
      tg(261°)6.3138
      tg(262°)7.1154
      tg(263°)8.1443
      tg(264°)9.5144
      tg(265°)11.4301
      tg(266°)14.3007
      tg(267°)19.0811
      tg(268°)28. 6363
      tg(269°)57.29
      tg(270°)— ∞
      tg(271°)-57.29
      tg(272°)-28.6363
      tg(273°)-19.0811
      tg(274°)-14.3007
      tg(275°)-11.4301
      tg(276°)-9.5144
      tg(277°)-8.1443
      tg(278°)-7.1154
      tg(279°)-6.3138
      tg(280°)-5.6713
      tg(281°)-5.1446
      tg(282°)-4.7046
      tg(283°)-4.3315
      tg(284°)-4.0108
      tg(285°)-3.7321
      tg(286°)-3.4874
      tg(287°)-3.2709
      tg(288°)-3.0777
      tg(289°)-2.9042
      tg(290°)-2.7475
      tg(291°)-2. 6051
      tg(292°)-2.4751
      tg(293°)-2.3559
      tg(294°)-2.246
      tg(295°)-2.1445
      tg(296°)-2.0503
      tg(297°)-1.9626
      tg(298°)-1.8807
      tg(299°)-1.804
      tg(300°)-1.7321
      tg(301°)-1.6643
      tg(302°)-1.6003
      tg(303°)-1.5399
      tg(304°)-1.4826
      tg(305°)-1.4281
      tg(306°)-1.3764
      tg(307°)-1.327
      tg(308°)-1.2799
      tg(309°)-1.2349
      tg(310°)-1.1918
      tg(311°)-1.1504
      tg(312°)-1.1106
      tg(313°)-1.0724
      tg(314°)-1.0355
      tg(315°)-1
      tg(316°)-0. 9657
      tg(317°)-0.9325
      tg(318°)-0.9004
      tg(319°)-0.8693
      tg(320°)-0.8391
      tg(321°)-0.8098
      tg(322°)-0.7813
      tg(323°)-0.7536
      tg(324°)-0.7265
      tg(325°)-0.7002
      tg(326°)-0.6745
      tg(327°)-0.6494
      tg(328°)-0.6249
      tg(329°)-0.6009
      tg(330°)-0.5774
      tg(331°)-0.5543
      tg(332°)-0.5317
      tg(333°)-0.5095
      tg(334°)-0.4877
      tg(335°)-0.4663
      tg(336°)-0.4452
      tg(337°)-0.4245
      tg(338°)-0.404
      tg(339°)-0. 3839
      tg(340°)-0.364
      tg(341°)-0.3443
      tg(342°)-0.3249
      tg(343°)-0.3057
      tg(344°)-0.2867
      tg(345°)-0.2679
      tg(346°)-0.2493
      tg(347°)-0.2309
      tg(348°)-0.2126
      tg(349°)-0.1944
      tg(350°)-0.1763
      tg(351°)-0.1584
      tg(352°)-0.1405
      tg(353°)-0.1228
      tg(354°)-0.1051
      tg(355°)-0.0875
      tg(356°)-0.0699
      tg(357°)-0.0524
      tg(358°)-0.0349
      tg(359°)-0.0175
      tg(360°)-0

      Select rating12345

      Рейтинг: 3.1 (Голосов 42)

      Сообщить об ошибке

      Смотрите также

      Мэтуэй | Популярные задачи

      92
      1 Найти точное значение грех(30)
      2 Найти точное значение грех(45)
      3 Найти точное значение грех(30 градусов)
      4 Найти точное значение грех(60 градусов)
      5 Найти точное значение загар (30 градусов)
      6 Найти точное значение угловой синус(-1)
      7 Найти точное значение грех(пи/6)
      8 Найти точное значение cos(pi/4)
      9 Найти точное значение грех(45 градусов)
      10 Найти точное значение грех(пи/3)
      11 Найти точное значение арктан(-1)
      12 Найти точное значение cos(45 градусов)
      13 Найти точное значение cos(30 градусов)
      14 Найти точное значение желтовато-коричневый(60)
      15 Найти точное значение csc(45 градусов)
      16 Найти точное значение загар (60 градусов)
      17 Найти точное значение сек(30 градусов)
      18 Найти точное значение cos(60 градусов)
      19 Найти точное значение cos(150)
      20 Найти точное значение грех(60)
      21 Найти точное значение cos(pi/2)
      22 Найти точное значение загар (45 градусов)
      23 Найти точное значение arctan(- квадратный корень из 3)
      24 Найти точное значение csc(60 градусов)
      25 Найти точное значение сек(45 градусов)
      26 Найти точное значение csc(30 градусов)
      27 Найти точное значение грех(0)
      28 Найти точное значение грех(120)
      29 Найти точное значение соз(90)
      30 Преобразовать из радианов в градусы пи/3
      31 Найти точное значение желтовато-коричневый(30)
      32
      35 Преобразовать из радианов в градусы пи/6
      36 Найти точное значение детская кроватка(30 градусов)
      37 Найти точное значение арккос(-1)
      38 Найти точное значение арктан(0)
      39 Найти точное значение детская кроватка(60 градусов)
      40 Преобразование градусов в радианы 30
      41 Преобразовать из радианов в градусы (2 шт. )/3
      42 Найти точное значение sin((5pi)/3)
      43 Найти точное значение sin((3pi)/4)
      44 Найти точное значение тан(пи/2)
      45 Найти точное значение грех(300)
      46 Найти точное значение соз(30)
      47 Найти точное значение соз(60)
      48 Найти точное значение соз(0)
      49 Найти точное значение соз(135)
      50 Найти точное значение cos((5pi)/3)
      51 Найти точное значение cos(210)
      52 Найти точное значение сек(60 градусов)
      53 Найти точное значение грех(300 градусов)
      54 Преобразование градусов в радианы 135
      55 Преобразование градусов в радианы 150
      56 Преобразовать из радианов в градусы (5 дюймов)/6
      57 Преобразовать из радианов в градусы (5 дюймов)/3
      58 Преобразование градусов в радианы 89 градусов
      59 Преобразование градусов в радианы 60
      60 Найти точное значение грех(135 градусов)
      61 Найти точное значение грех(150)
      62 Найти точное значение грех(240 градусов)
      63 Найти точное значение детская кроватка(45 градусов)
      64 Преобразовать из радианов в градусы (5 дюймов)/4
      65 Найти точное значение грех(225)
      66 Найти точное значение грех(240)
      67 Найти точное значение cos(150 градусов)
      68 Найти точное значение желтовато-коричневый(45)
      69 Оценить грех(30 градусов)
      70 Найти точное значение сек(0)
      71 Найти точное значение cos((5pi)/6)
      72 Найти точное значение КСК(30)
      73 Найти точное значение arcsin(( квадратный корень из 2)/2)
      74 Найти точное значение загар((5pi)/3)
      75 Найти точное значение желтовато-коричневый(0)
      76 Оценить грех(60 градусов)
      77 Найти точное значение arctan(-( квадратный корень из 3)/3)
      78 Преобразовать из радианов в градусы (3 пи)/4 
      79 Найти точное значение sin((7pi)/4)
      80 Найти точное значение угловой синус(-1/2)
      81 Найти точное значение sin((4pi)/3)
      82 Найти точное значение КСК(45)
      83 Упростить арктан(квадратный корень из 3)
      84 Найти точное значение грех(135)
      85 Найти точное значение грех(105)
      86 Найти точное значение грех(150 градусов)
      87 Найти точное значение sin((2pi)/3)
      88 Найти точное значение загар((2pi)/3)
      89 Преобразовать из радианов в градусы пи/4
      90 Найти точное значение грех(пи/2)
      91 Найти точное значение сек(45)
      92 Найти точное значение cos((5pi)/4)
      93 Найти точное значение cos((7pi)/6)
      94 Найти точное значение угловой синус(0)
      95 Найти точное значение грех(120 градусов)
      96 Найти точное значение желтовато-коричневый ((7pi)/6)
      97 Найти точное значение соз(270)
      98 Найти точное значение sin((7pi)/6)
      99 Найти точное значение arcsin(-( квадратный корень из 2)/2)
      100 Преобразование градусов в радианы 88 градусов

      Tan 46 градусов — Найти значение Tan 46 градусов

      LearnPracticeDownload

      Значение tan 46 градусов равно 1,0355303. . . . Тангенс 46 градусов в радианах записывается как тангенс (46° × π/180°), то есть тангенс (23π/90) или тангенс (0,802851…). В этой статье мы обсудим способы нахождения значения тангенса 46 градусов на примерах.

      • Тан 46° в десятичном формате: 1.0355303. . .
      • Желто-коричневый (-46 градусов): -1,0355303. . .
      • Tan 46° в радианах: tan (23π/90) или желтовато-коричневый (0,8028514 . . .)

      Сколько стоит Тан 46 градусов?

      Значение тангенса 46 градусов в десятичной системе равно 1,035530313. . .. Tan 46 градусов также можно выразить, используя эквивалент данного угла (46 градусов) в радианах (0,80285 . . .)

      Мы знаем, используя преобразование градусов в радианы, θ в радианах = θ в градусах × (пи/ 180°)
      ⇒ 46 градусов = 46° × (π/180°) рад = 23π/90 или 0,8028. . .
      ∴ тангенс 46° = тангенс (0,8028) = 1,0355303. . .

      Объяснение:

      Для тангенса 46 градусов угол 46° лежит между 0° и 90° (первый квадрант). Поскольку функция тангенса положительна в первом квадранте, значение tan 46° = 1,0355303. . .
      Поскольку функция тангенса является периодической функцией, мы можем представить тангенс 46° как тангенс 46 градусов = тангенс (46° + n × 180°), n ∈ Z.
      ⇒ тангенс 46° = тангенс 226° = тангенс 406° и так далее.
      Примечание: Поскольку тангенс является нечетной функцией, значение тангенса (-46°) = -тангенса (46°).

      Методы определения значения Tan 46 градусов

      Функция тангенса положительна в 1-м квадранте. Значение тангенса 46° составляет 1,03553. . .. Мы можем найти значение тангенса 46 градусов по:

      • Используя единичный круг
      • Использование тригонометрических функций

      Tan 46 градусов с помощью единичной окружности

      Чтобы найти значение tan 46 градусов с помощью единичной окружности:

      • Поверните ‘r’ против часовой стрелки, чтобы образовать угол 46° с положительной осью x.
      • Тангенс 46 градусов равен координате y (0,7193), деленной на координату x (0,6947) точки пересечения (0,6947, 0,7193) единичной окружности и r.

      Следовательно, значение тангенса 46° = y/x = 1,0355 (приблизительно).

      Тангенс 46° в терминах тригонометрических функций

      Используя формулы тригонометрии, мы можем представить тангенс 46° как:

      • sin(46°)/cos(46°)
      • ± sin 46°/√(1 — sin²(46°))
      • ± √(1 — cos²(46°))/cos 46°
      • ± 1/√(косек²(46°) — 1)
      • ± √(сек²(46°) — 1)
      • 1/кроватка 46°

      Примечание. Поскольку 46° лежит в 1-м квадранте, окончательное значение тангенса 46° будет положительным.

      Мы можем использовать тригонометрические тождества для представления tan 46° как

      • cot(90° — 46°) = cot 44°
      • -кроватка(90° + 46°) = -кроватка 136°
      • -тангенс (180° — 46°) = -тангенс 134°

      ☛ Также проверьте:

      • загар 50 градусов
      • загар 120 градусов
      • загар 225 градусов
      • загар 4 градуса
      • загар 7 градусов
      • загар 14 градусов

      Примеры использования Tan 46 градусов

      1. Пример 1. Найдите значение тангенса 8 (46°)/тангажа 9 (134°).

        Решение:

        Используя тригонометрические тождества, мы знаем, что tan(46°) = -tan(180° — 46°) = -tan 134°.
        ⇒ тангенс (46°) = -тангенс (134°)
        ⇒ Значение 8 тангенса (46°)/9 тангенса (134°) = -8/9

      2. Пример 2: Используя значение тангенса 46°, найдите: (sec²(46°) — 1).

        Решение:

        Мы знаем, (сек²(46°) — 1) = (tan²(46°)) = 1,0723
        ⇒ (сек²(46°) — 1) = 1,0723

      3. Пример 3. Найдите значение tan 46°, если cot 46° равно 0,9656.

        Решение:

        Так как tan 46° = 1/cot 46°
        ⇒ тангенс 46° = 1/0,9656 = 1,0355

      перейти к слайдуперейти к слайдуперейти к слайду

       

      Готовы посмотреть на мир глазами математика?

      Математика лежит в основе всего, что мы делаем. Наслаждайтесь решением реальных математических задач на живых уроках и станьте экспертом во всем.

      Запишитесь на бесплатный пробный урок

      Часто задаваемые вопросы о Tan 46 Degrees

      Что такое Tan 46 Degrees?

      Тангенс 46 градусов — значение тангенса тригонометрической функции для угла, равного 46 градусам. Значение тангенса 46° составляет 1,0355 (приблизительно).

      Каково значение Tan 46° с точки зрения Sec 46°?

      Мы можем представить функцию тангенса в терминах функции секущей, используя тригонометрические тождества, тангенс 46° можно записать как √(sec²(46°) — 1). Здесь значение sec 46° равно 1,4395.

      Каково значение Tan 46 градусов по отношению к Cot 46°?

      Поскольку функция тангенса является обратной функцией котангенса, мы можем записать тангенс 46° как 1/cot(46°). Значение cot 46° равно 0,96568.

      Как найти тангенс 46° с точки зрения других тригонометрических функций?

      Используя формулу тригонометрии, значение тангенса 46° можно выразить через другие тригонометрические функции следующим образом:

      • sin(46°)/cos(46°)
      • ± sin 46°/√(1 — sin²(46°))
      • ± √(1 — cos²(46°))/cos 46°
      • ± 1/√(cosec²(46°) — 1)
      • ± √(сек²(46°) — 1)
      • 1/кроватка 46°

      ☛ Также проверьте: тригонометрическую таблицу

      Как найти значение Тан 46 градусов?

      Значение тангенса 46 градусов можно рассчитать, построив угол 46° с осью x и затем найдя координаты соответствующей точки (0,6947, 0,7193) на единичной окружности. Значение tan 46° равно координате y (0,7193), деленной на координату x (0,69).47). ∴ tan 46° = 1,0355

       

      Скачать БЕСПЛАТНЫЕ учебные материалы

      Тригонометрия

      Рабочие листы по математике и
      наглядный учебный план

      Портленд — Тангенс — 4 способа добраться на поезде, автобусе и автомобиле

      6

      Найти транспорт к касательной

      Путешествие из

      Путешествие из

      К

      Поиск жилья с Booking.com

      Нужен номер в отеле в Тангенте?

      Забронировать

      Есть 4 способа добраться из Портленда в Тангент: автобус, поезд или автомобиль

      Выберите один из вариантов ниже, чтобы просмотреть пошаговые инструкции и сравнить цены на билеты и время в пути в планировщике путешествий Rome2rio.

      Тренироваться

      1. Сядьте на поезд из Портленда до станции Albany Amtrak.

      Автобус

      1. Сядьте на автобус от станции Portland Amtrak до станции Albany Amtrak.

      Водить машину

      1. Поездка из Портленда в Тангент

      Самый быстрый способ добраться туда Самый дешевый вариант Дистанция между

      Портленд в Тангент на автобусе

      Время в пути на автобусе между Портлендом и Тангентом составляет около 2 часов 46 минут, а расстояние составляет около 146 км. Это включает в себя среднее время ожидания около 32 минут. Управляемый Greyhound USA и Linn-Benton Loop, автобус Портленд-Тангент отправляется из Портленда и прибывает на шоссе 9.9 в Портерс. Обычно еженедельно курсируют семь автобусов, хотя расписание выходных и праздничных дней может меняться, поэтому уточняйте заранее.

      7 Еженедельные автобусы

      2ч 46м Средняя продолжительность

      €18 Самая низкая цена

      Посмотреть расписание Поделиться

      Сохраните эту ссылку, чтобы быть в курсе ограничений, связанных с COVID-19

      Путешествуйте безопасно во время COVID-19

      Правила, которым необходимо следовать в США

      Обязательно

      Обязательно

      Путешествуйте по США

      Наблюдайте за COVID-19правила безопасности

      Помощь при COVID-19 в США

      Если вам нужна помощь, посетите национальный веб-сайт COVID-19 или позвоните по телефону доверия COVID-19 800-232-4636

      Часто задаваемые вопросы


      Могу ли я путешествовать из Портленда в Тангент?

      Каковы ограничения на поездки в Тангенте?

      Внутренние поездки не ограничены, но могут применяться некоторые условия

      • Маски обязательны
      • Необходимо соблюдать социальную дистанцию ​​2 метра
      • Соблюдать правила безопасности COVID-19

      Исследуйте варианты путешествий

      Что такое национальный COVID-19номер горячей линии в Tangent?

      Национальный номер горячей линии COVID-19 в Тангенте: 800-232-4636.

      Должен ли я носить маску для лица в общественном транспорте в Тангенте?

      Ношение маски в общественном транспорте в Тангенсе обязательно.

      Что мне делать, если у меня появятся симптомы COVID-19 по прибытии в Тангент?

      Сообщите о себе официальному сотруднику и/или позвоните по телефону горячей линии по коронавирусу 800-232-4636.

      Последнее обновление: 13 сентября 2022 г.
      Исключения могут применяться, для получения полной информации: Центры по контролю и профилактике заболеваний (CDC).

      Мы работаем круглосуточно, чтобы предоставить вам последние новости о поездках в связи с COVID-19.
      Информация собрана из официальных источников. Насколько нам известно, это правильно на момент последнего обновления. 909:31 Посетите путеводитель Rome2rio, чтобы получить общую помощь.

      Вопросы и ответы

      Как дешевле всего добраться из Портленда в Тангент?

      Самый дешевый способ добраться из Портленда в Тангент — это проехать на автомобиле, который стоит 14–21 евро и занимает 1 час 25 минут.

      Подробнее

      Как быстрее всего добраться из Портленда в Тангент?

      Самый быстрый способ добраться из Портленда в Тангент — это проехать на автомобиле, который стоит 14–21 евро и занимает 1 час 25 минут.

      Подробнее

      Есть ли прямой автобус между Портлендом и Тангентом?

      Нет, прямого автобуса из Портленда в Тангент нет. Однако есть рейсы, вылетающие из Портленда и прибывающие на шоссе 9.9 в Porters через Downtown Transit Center. Время в пути, включая пересадки, примерно 2 часа 46 минут.

      Подробнее

      На каком расстоянии Портленд от Тангента?

      Расстояние от Портленда до Тангента составляет 114 км. Ехать примерно 124 км.

      Получить маршрут проезда

      Как добраться из Портленда в Тангент без машины?

      Лучший способ добраться из Портленда в Тангент без машины — это сесть на автобус через Центр транзитных перевозок в центре города, который занимает 2 часа 46 минут и стоит 17–24 евро.

      Подробнее

      Сколько времени нужно, чтобы добраться из Портленда в Тангент?

      Дорога из Портленда в Тангент занимает примерно 2 часа 46 минут, включая пересадки.

      Подробнее

      Поездом или автобусом из Портленда в Тангент?

      Лучший способ добраться из Портленда в Танджент — это сесть на автобус через центр транзитных перевозок в центре города, который занимает 2 часа 46 минут и стоит 17–24 евро. Кроме того, вы можете поехать на поезде, который стоит 28–40 евро и занимает 2 часа 54 минуты.

      Детали режима

      Могу ли я проехать из Портленда в Тангент?

      Да, расстояние между Портлендом и Тангенсом составляет 124 км. Дорога от Портленда до Тангента занимает примерно 1 час 25 минут.

      Получить маршрут проезда

      Какие компании осуществляют перевозки между Портлендом, штат Орегон, США, и Тангентом, штат Орегон, США?

      Greyhound USA обслуживает автобус из Портленда в Корваллис один раз в день. Билеты стоят 15–22 евро, а время в пути — 1 час 55 минут. Кроме того, Amtrak ходит поездом из Портленда до станции Albany Amtrak 3 раза в день. Билеты стоят 26–40 евро, а время в пути — 1 час 41 минуту.

      Амтрак

      Amtrak — это железнодорожная служба, которая соединяет США и три провинции Канады. Покрывая 21 000 миль маршрута (34 000 км), Amtrak ежедневно обслуживает более 300 поездов. Эти междугородние перевозки на средние и дальние расстояния осуществляются со скоростью до 240 км/ч по более чем 500 направлениям. Основан в 1971, он базируется в Вашингтоне, округ Колумбия, и предлагает четыре класса обслуживания: первый класс, спальный, бизнес-класс и туристический автобус. Тарифы на билеты делятся на пять подклассов: Saver, Value, Flexible, Business и Premium. Поезда Amtrak известны своими широкими сиденьями, подключаемым питанием, большими окнами и возможностями хранения.

      Телефон
      +1 800-872-7245
      Веб-сайт
      amtrak.com
      Продолжительность
      1ч 41м
      Частота
      3 раза в день
      Ориентировочная цена
      €26 — €40
      Веб-сайт
      Амтрак
      Сиденье тренера
      €26 — €40
      Премиум
      €180 — 260 €
      Сиденье Flexi Coach
      €40 — 60 €
      Бизнес-место
      35 — 55 евро

      Грейхаунд США

      Greyhound — ведущая автобусная компания, базирующаяся в Далласе, штат Техас, и обслуживающая более 3800 пунктов назначения в Северной Америке, Мексике и Канаде. Greyhound перевозит около 18 миллионов пассажиров в год, которые проезжают 5,4 миллиарда миль (8,6 миллиарда км) в год на своем парке из около 1700 автомобилей. Для гибкости путешествия вы можете сесть или выйти из автобуса Greyhound на официальных станциях Greyhound, станциях-партнерах и остановках у обочины. В США есть около 230 станций Greyhound, где вы можете сесть на автобус и купить билеты, которые также доступны на официальном сайте и через мобильное приложение.

      Телефон
      +1 214-849-8100/1-800-231-2222
      Электронная почта
      [email protected]
      Веб-сайт
      greyhound.com
      Продолжительность
      1ч 55м
      Частота
      Один раз в день
      Ориентировочная цена
      €15 – 22 €
      Веб-сайт
      greyhound. com
      Эконом
      €15 — 22 €
      Гибкий
      30 — 50 евро

      Точка

      Телефон
      +1 888-846-4183
      Веб-сайт
      oregon-point.com
      Продолжительность
      1ч 55м
      Частота
      4 раза в день
      Ориентировочная цена
      €17 — 24 €
      Веб-сайт
      Точка

      Хотите узнать больше о путешествиях по Соединенным Штатам

      Серия путеводителей Rome2rio содержит жизненно важную информацию для путешественников со всего мира. Наполненные полезной и своевременной информацией о путешествии, гиды отвечают на все сложные вопросы, такие как «Как купить билет?», «Должен ли я бронировать билеты онлайн перед поездкой?» ‘, ‘Сколько я должен заплатить?’, ‘Есть ли в поездах и автобусах Wi-Fi?’ — чтобы помочь вам получить максимальную отдачу от вашей следующей поездки.

      Другие вопросы и ответы

      Где остановиться рядом с Тангентом?

      В Тангенте доступно 34+ отелей. Цены начинаются от 100 евро за ночь.

      Подробнее

      Включение внутренних лимфатических узлов молочной железы в стандартные касательные поля молочной железы: влияние габитуса тела

      . 2001 март-апрель;7(2):111-6.

      doi: 10. 1046/j.1524-4741.2001.007002111.x.

      Г М Пру 1 , RJ Lee, PC Stomper

      принадлежность

      • 1 Отделение радиационной онкологии, Онкологический институт Розуэлл Парк, Школа медицины и биомедицинских наук, Университет штата Нью-Йорк в Буффало, 14263, США. [email protected]
      • PMID: 11328318
      • DOI: 10.1046/j.1524-4741.2001.007002111.x

      GM Proulx et al. Грудь Дж. 2001 март-апрель.

      . 2001 март-апрель;7(2):111-6.

      doi: 10. 1046/j.1524-4741.2001.007002111.x.

      Авторы

      Г М Пру 1 , Р. Дж. Ли, ПК Стомпер

      принадлежность

      • 1 Отделение радиационной онкологии, Онкологический институт Розуэлл Парк, Школа медицины и биомедицинских наук, Университет штата Нью-Йорк в Буффало, 14263, США. [email protected]
      • PMID: 11328318
      • DOI: 10.1046/j.1524-4741.2001.007002111.x

      Абстрактный

      Цель этого исследования состояла в том, чтобы определить вариабельность покрытия внутреннего узла молочной железы (IMN) со стандартными касательными полями молочной железы с использованием поверхностной анатомии, определенной с помощью компьютерной томографии (КТ) планирования для пациентов, получавших либо органосохраняющее лечение, либо постмастэктомию, и оценить влияние габитуса и формы тела на покрытие ИМН стандартными касательными полями. В это проспективное исследование были включены последовательные женщины с раком молочной железы, которые подверглись либо локальному иссечению, либо мастэктомии и имели стандартные касательные поля, предназначенные для покрытия груди, плюс край, смоделированный с использованием поверхностной анатомии. При планировании КТ определялось расположение ИМН относительно касательных полей, рассчитанных на основании анатомии поверхности. Внутренние сосуды молочной железы использовались в качестве заменителей IMN. КТ-измерения толщины престернального жира и переднезаднего (AP) и поперечного диаметров скелета были выполнены для определения их связи с включением IMN в касательные поля. Только у семи пациентов (14%) IMN были полностью в пределах касательных полей. Двадцать пациентов (40%) имели частичное покрытие своих IMN, а 23 (46%) имели свои IMN полностью за пределами поля. Включение IMN было обратно пропорционально толщине престернального жира. Грудная форма скелета не была связана с включением IMN. Стандартные тангенциальные поля обычно не покрывают IMN полностью, но могут покрывать их, по крайней мере, частично у большинства пациентов. Толщина престернального жира обратно пропорциональна включению IMN в касательные поля.

      Похожие статьи

      • Покрытие внутренних лимфатических узлов молочной железы (ВМН) стандартными тангенциальными полями облучения у пациентов с дренированием ВМН на лимфосцинтиграфии: терапевтические последствия.

        Заяц GB, Proulx GM, Lamonica DM, Stomper PC. Харе ГБ и др. Am J Clin Oncol. 2004 июнь; 27 (3): 274-8. doi: 10.1097/01.coc.00000

        .03967.80. Am J Clin Oncol. 2004. PMID: 15170147

      • Использование ПЭТ-КТ 18 F-FDG для определения расположения внутренних лимфатических узлов молочной железы при планировании лучевой терапии у пациентов с раком молочной железы.

        Дэвидсон Т., Бен-Дэвид М., Гальпер С., Хаскин Т., Хоус М., Скейф Р., Канана Н., Амит У., Вейцман Н., Чикман Б., Гошен Э., Бен-Хаим С., Саймон З., Гольдштейн Дж. Дэвидсон Т. и др. Практика Radiat Oncol. 2017 ноябрь-декабрь;7(6):373-381. doi: 10.1016/j.prro.2016.11.001. Epub 2016 5 ноября. Практика Radiat Oncol. 2017. PMID: 28989000

      • Стандартные поля тангенциального излучения не обеспечивают случайного покрытия внутренних узлов молочной железы.

        Loganadane G, Kassick M, Kann BH, Young MR, Knowlton CA, Evans SB, Higgins SA, Belkacemi Y, Potenziani M, Saltmarsh N, Wilson LD, Moran MS. Логанадан Г. и соавт. Практика Radiat Oncol. 2020 янв-февраль;10(1):21-28. doi: 10.1016/j.prro.2019.07.014. Epub 2019 5 августа. Практика Radiat Oncol. 2020. PMID: 31394256

      • Картирование сигнальных лимфатических узлов при раке молочной железы: критическая переоценка проблемы внутренней цепи молочной железы.

        Манка Г., Вольтеррани Д., Маццарри С., Дуче В., Свириденко А., Джулиано А., Мариани Г. Манка Г. и др. Q J Nucl Med Mol Imaging. 2014 июнь;58(2):114-26. Q J Nucl Med Mol Imaging. 2014. PMID: 24835288 Обзор.

      • Должны ли внутренние молочные лимфатические узлы при раке молочной железы быть мишенью для онколога-радиолога?

        Фридман Г.М., Фаубл Б.Л., Николау Н., Сигурдсон Э.Р., Торосян М.Х., Бораас М.С., Хоффман Д.П. Фридман Г.М. и соавт. Int J Radiat Oncol Biol Phys. 2000 март 1;46(4):805-14. doi: 10.1016/s0360-3016(99)00481-2. Int J Radiat Oncol Biol Phys. 2000. PMID: 10705000 Обзор.

      Посмотреть все похожие статьи

      Цитируется

      • Дозиметрическая оценка случайного облучения внутренней цепи молочной железы после операции у больных раком молочной железы.

        Ван В, Сунь Т, Мэн И, Сюй М, Чжан И, Шао Ц, Сун И, Ли Дж. Ван В и др. Фронт Онкол. 2022 2 марта; 12:839831. doi: 10.3389/fonc.2022.839831. Электронная коллекция 2022. Фронт Онкол. 2022. PMID: 35311065 Бесплатная статья ЧВК.

      • Факторы, влияющие на случайное распределение дозы во внутренних узлах молочной железы: сравнительное исследование.

        Ван В, Ван Дж, Цю П, Сунь Т, Чжан Ю, Шао Ц, Сю М, Лю С, Ли Дж. Ван В и др. Фронт Онкол. 2020 9 апр; 10:456. doi: 10.3389/fonc.2020.00456. Электронная коллекция 2020. Фронт Онкол. 2020. PMID: 32328459 Бесплатная статья ЧВК.

      • Дозиметрическое сравнение случайного облучения внутренних узлов молочной железы после органосохраняющей операции с использованием 3 методов: лучевая терапия с обратной модуляцией интенсивности, лучевая терапия с модулированной интенсивностью поле-в-поле и трехмерная конформная лучевая терапия: ретроспективное клиническое исследование.

        Сун Ю, Ю Т, Ван В, Ли Дж, Сунь Т, Цю П, Сюй М, Шао К. Сонг Ю и др. Медицина (Балтимор). 2019 окт;98(41):e17549. doi: 10.1097/MD.0000000000017549. Медицина (Балтимор). 2019. PMID: 31593136 Бесплатная статья ЧВК. Клиническое испытание.

      • Внеплановое облучение внутренних молочных лимфатических узлов при раке молочной железы.

        Канилмаз Г., Актан М., Коч М., Демир Х., Демир Л.С. Каньилмаз Г. и др. Радиол Мед. 2017 июнь; 122(6):405-411. doi: 10.1007/s11547-017-0747-5. Epub 2017 3 марта. Радиол Мед. 2017. PMID: 28255809

      • Случайное облучение внутренних молочных лимфатических узлов при раке молочной железы: обычная двухмерная лучевая терапия по сравнению с конформной трехмерной лучевой терапией.

        Лейте Э.Т., Угино Р.Т., Сантана М.А., Феррейра Д.В., Лопес М.Р., Пелоси Э.Л., да Силва Х.Л., Карвалью Хде А. Лейте Э.Т. и др. Радиол Бюстгальтеры. 2016 май-июнь;49(3):170-5. дои: 10.1590/0100-3984.2015.0003. Радиол Бюстгальтеры. 2016. PMID: 27403017 Бесплатная статья ЧВК.

      Просмотреть все статьи «Цитируется по»

      термины MeSH

      Калькулятор кофункций

      Создано Maciej Kowalski, кандидатом наук

      Отзыв Стивена Вудинга

      Последнее обновление: 05 сентября 2022 г.

      Содержание:
      • Тригонометрические функции
      • Графики кофункций: sin и cos, tan и cot, sec и csc
      • Тождества кофункций
      • Пример: использование калькулятора кофункций тождества кофункций и как их использовать. По сути, в тригонометрии существует шесть функций, полностью описывающих отношения между углами и сторонами треугольника. Таким образом, они связаны друг с другом, поэтому мы часто думаем о них как о парах: sin и cos, tan и cot, sec и csc . Сегодня мы рассмотрим эти отношения и узнаем, как перейти от одной карты к ее паре, то есть к ее кофункции .

        Так что расслабьтесь, расслабьтесь и насладитесь хорошей математикой !

        Тригонометрические функции

        Прежде чем мы узнаем, что такое кофункция, нам нужно начать с основ . А в геометрии мы не можем пойти дальше, чем треугольники: три стороны, три вершины, три внутренних угла. В каком-то смысле проще полигона быть не может.

        Нас, однако, больше всего интересует конкретный тип треугольников: прямоугольных треугольников (вы знаете, о которых говорит теорема Пифагора). Один из их углов всегда равен 90 градусов (отсюда и название), поэтому у нас уже есть некоторая информация о нашей фигуре еще до того, как мы ее нарисуем.

        Кроме того, мы можем наблюдать некоторые другие зависимости , которые заставляют треугольник выглядеть так, как он есть. Ведь если мы увеличим один из острых углов, то легко увидим, что противоположная сторона тоже должна стать длиннее. Это наблюдение является более или менее идеей тригонометрии: каким-то образом соотнести внутренние углы треугольника с его сторонами .

        Определим тригонометрических функций как отношение сторон прямоугольного треугольника. Ниже вы можете найти картинку с формулами для всех шести из них. (Обратите внимание, что в калькуляторе кофункций для каждой из них есть по одному идентификатору.)

        🔎 Все тригонометрические функции (sin, cos, tan) являются отношениями. Таким образом, вы можете найти недостающие члены, используя только наш калькулятор отношений!

        Например, мы видим, что синус равен катету, противоположному углу, деленному на гипотенузу. Обратите внимание, что мы никогда не упоминаем, насколько велик треугольник. На самом деле там важнейшее свойство тригонометрии заключается в следующем: даже если мы удвоим размер треугольника, если мы сохраним углы нетронутыми, значения тригонометрических функций не изменятся .

        Однако приведенные выше формулы, при всем их изяществе, имеют существенный недостаток. Мы определили их с помощью прямоугольного треугольника, поэтому угол может быть только между 0 и 90 градусов (или 0 и π/2 в радианах). Но не дуйся! К счастью для нас, для математики и всего мира, есть способ это исправить ! Единственное, что нам нужно сделать, это перенести рассуждения на двумерное евклидово пространство, т. е. на плоскость.

        Пусть A = (x,y) — точка на плоскости, и определим α как угол, идущий против часовой стрелки от положительной половины горизонтальной оси к отрезку, конечные точки которого равны (0 ,0) и А . (Обратите внимание, как мы сказали, что α проходит от одной линии к другой, а не то, что это просто угол между ними. Вот почему мы часто называем α направленный угол .)

        Очевидно, что α теперь может быть больше, чем 90 градусов. На самом деле, может даже выйти за пределы 360 градусов . Для таких углов мы просто считаем первые 360 градусов первым кругом вокруг (0,0) , и от этого значения мы продолжаем со вторым (и, если нужно, с третьим, четвертым, так далее.). На самом деле даже больше — α тоже может быть отрицательным . В конце концов, мы сказали, что это направленный угол, поэтому для отрицательных значений мы просто меняем направление на противоположное, то есть идем по часовой стрелке, а не против часовой стрелки.

        Теперь, когда мы понимаем углы всех размеров, мы можем определить для них тригонометрические функции и точку A = (x,y) . Правило здесь довольно простое: повторяем формулы с рисунка выше , но подставляем b вместо x , a вместо y , и 92}}{x}sec(α)=xx2+y2

        Итак, мы познакомились с тригонометрическими функциями с точки зрения их определений, так что мы готовы копнуть глубже . В конце концов, имя « калькулятор кофункций » привело нас сюда, и мы до сих пор не знаем, как найти кофункцию. Для этого лучше всего использовать графики функций .

        Графики кофункций: sin и cos, tan и cot, sec и csc

        Шесть тригонометрических функций равны синус и косинус (обозначаются sin и cos), тангенс и котангенс (tan и cot) и секанс и косеканс (sec и csc).

        Вы, наверное, уже понимаете, почему мы объединили их так, как , только по именам. В каждой паре у нас есть «базовая» функция и карта с таким же названием, но с дополнительным префиксом « co-». Сходство со словом « кофункция » далеко не случайно. В каждой из пар мы говорим, что один является кофункцией другого .

        » Но что значит быть кофункцией? » Что ж, мы рады, что вы спросили! Мы подробно рассмотрим тождества кофункций в следующем разделе. Однако сначала попробуем мотивировать их, посмотрев на графики функций в парах и выявив сходство.

        Начнем с sin и cos .

        Они выглядят почти одинаково, не так ли? Синус — это просто косинус, сдвинутый на 90 градусов (или π/2 в радианах) вправо.

        Для наших целей сосредоточимся на области от 0 до 90 градусов и представим, что вы проходите это расстояние по наклону обеих кривых, но в разных направлениях: синус от 0 до 90 и косинус от 90 до 0 . Видите ли вы, что пути тогда одинаковы?

        Попробуем сделать нечто подобное для графиков tan и cot .

        Опять же, если мы проследим касательную от 0 до 90 и котангенс от 90 до 0 , мы заметим, что мы идем по тому же пути .

        Наконец, у нас есть функции sec и csc .

        Как вы могли догадаться, история повторяется для секанса и косеканса.

        По существу, это то, что характеризует кофункции . Можно было бы сказать, что их графики являются взаимными отражениями, если мы поместим зеркало в середину интервала (0°,90°) , т. е. на 45 градусов. Это, в свою очередь, означает, что значение тригонометрической функции в точке x должно быть таким же, как значение кофункции в точке 90° - x . И это именно то, что утверждают тождества кофункций.

        Кофункциональные тождества

        Как упоминалось в предыдущем разделе, мы делим тригонометрические функции на пары . В каждом из них одно является кофункцией другого. Это означает, что их графики (а значит, и значения) являются взаимными отражениями в интервале (0°,90°) . Следовательно, значение первого в точке х совпадает с другим в 90° - х для х из интервала.

        Если вы хотите порадовать нескольких высокомерных ученых, мы можем написать приведенный выше абзац эквивалентно, используя математические обозначения. Это будет выглядеть так:

        Однако помните, что работают только для углов между 0 и 90 градусов . Приведенные выше формулы основаны на том факте, что углы по обе стороны от знака 91 626 = 91 627 дополняют друг друга, т. е. в сумме дают 91 626 90 91 627 градусов.

        На самом деле, есть способ рассмотреть и другие углы . Однако это сложно. Здесь мы имели то удобство, что все функции принимают положительные значения в интервале (0°,90°) . За его пределами все может стать негативным. Также возникает вопрос, куда поставить « зеркало », что отражает кофункции.

        Несмотря на препятствия, в результате многочасовой напряженной работы математики смогли вывести формул, обобщающих тождества кофункций . Они называются формулами тригонометрического приведения (заметьте, не формулы приведения в степени). Хотя мы не будем приводить их здесь, мы с радостью рекомендуем вам найти их и наслаждайтесь дополнительными математическими знаниями . Если вы спросите нас, это, безусловно, лучше, чем пролистывание социальных сетей.

        И на этом мы объявляем конец теории на сегодня ! Возможно, мы потратили довольно много времени на изучение определений и формул. Почему бы не взять несколько примеров и не использовать тождества кофункций с пользой для ?

        Пример: использование калькулятора кофункций

        Скажите, что вы решили сделать ремонт в своей гостиной . В конце концов, у карантина из-за коронавируса есть как минимум один положительный момент — у вас много свободного времени .

        Вы планируете починить пол, может быть, заменить плитку на деревянную? Проблема в том, что вам нужны измерения , а комната не является идеальным прямоугольником; есть пара наклонных стен. К счастью, вы еще кое-что помните из колледжа и, немного почесав голову, понимаете, что для вам понадобится косинус 45 градусов и котангенс 30 градусов для дальнейших расчетов.

        Однако есть проблема. Прошло несколько лет с вашего последнего урока тригонометрии, а вы не можете вспомнить формулы для косинуса или котангенса. Что вы помните, так это синус и тангенс. О, как хорошо вписывается в этот сценарий калькулятор тождеств кофункций!

        (Ладно, признаем, что подробности немного притянуты за уши 909:10, но, пожалуйста, дайте нам перерыв. Воображение разработчиков контента Omni может только простираться.)

        Прежде всего, давайте посмотрим насколько легко задача, когда у нас есть калькулятор кофункций под рукой . Там мы начинаем с , выбирая функцию, которая у нас есть . Во-первых, мы выбираем косинус, то есть cos(x) из списка. Получив это, мы переходим к переменному полю ниже, которое содержит угол. Мы вводим 45° из нашей задачи, и в тот момент, когда мы это делаем, калькулятор кофункций выдает ответ внизу: кофункция вместе со значением . Аналогично, для второго случая мы выбираем котангенс ( cot(x) ) из списка и вводим 30° .

        Обратите внимание, как каждый раз, когда инструмент дает нам точное значение (т. е. в виде дроби с квадратными корнями), кроме округленного в большую сторону. Мы объясним, почему это так, через секунду. Кроме того, хотя калькулятор кофункций стремится к точности , вы можете уменьшить количество значащих цифр в ответе для любых дальнейших вычислений.

        Теперь оставим в стороне инструмент Omni и посмотрим как найти ответ самостоятельно . Мы выполним следующие шаги:

        1. Нарисуйте прямоугольный треугольник с заданным углом;
        2. Используйте тождества кофункций , чтобы преобразовать искомую функцию в ее кофункцию; и
        3. Рассчитайте значение кофункции, взяв отношения сторон треугольника.

        Начнем с угла 45° .

        Заметим, что это пример совершенно особого треугольника, в котором мы знаем отношения между сторонами , то есть мы знаем, что если катет имеет длину x , то гипотенуза должна быть x√2 . Это потому, что наша фигура на самом деле является половиной квадрата, где длинная сторона является диагональю квадрата.

        Теперь мы вспоминаем тождества кофункций из предыдущего раздела и используем их для преобразования cos(45°) в синус:

        cos(45°) = sin(90° - 45°) = sin(45°) .

        Таким образом, мы можем использовать формулу синуса , чтобы найти ответ. В первом разделе мы сказали, что это катет, противоположный углу, деленному на гипотенузу. Это дает:

        cos(45°) = sin(45°) = x / x√2 = 1 / √2 = √2 / 2 .

        Перейдем к случаю 30 градусов. Снова начинаем с чертежа .

        Как и прежде, нам посчастливилось знать отношения между сторонами . На этот раз это потому, что наша фигура на самом деле является половиной равностороннего треугольника.

        Мы используем тождества кофункций для преобразования cot(30°) в тангенс:

        cot(30°) = tan(90° - 30°) = tan(60°) .

        Обратите внимание, что хотя рассматриваемый угол изменился, мы все еще можем использовать ту же картинку . Это всегда будет иметь место с тождествами кофункций, поскольку мы всегда имеем дело с дополнительными углами , то есть с углами, образующими острые углы одного и того же прямоугольного треугольника.

        Наконец, вспоминаем формулу тангенса из первого раздела: функция возвращает катет, противоположный углу, деленному на другой. В нашем случае это:

        cot(30°) = tan(60°) = x√3 / x = √3 .

        Готово! Мы нашли нужные нам тригонометрические функции; мы готовы позаботиться об этой плитке и обновить гостиную. Конечно, с установкой фанеры будет намного уютнее. И когда вы закончите, почему бы не пойти дальше и придумать что-нибудь для спальни ?

        Часто задаваемые вопросы

        Как использовать тождества кофункций?

        Тождества кофункций позволяют определить значение тригонометрической функции по значению кофункции (вашей функции) под углом, дополнительным к вашему углу. Например, вы можете легко найти cos(20°) , если знаете sin(70°) , потому что синус и косинус являются кофункциями, а 70° + 20° = 90° .

        Для какого x выполняется sin(x)=cos(15°)?

        Ответ: x = 75° . Мы знаем, что синус и косинус являются кофункциями, т. е. их значения совпадают на дополнительных углах. Итак, х и 15° должны быть дополнительными: х + 15° = 90° . Следовательно, x = 75° , как и утверждалось.

        Как найти синус с косинусом?

        Чтобы определить sin(x) по cos(x) , выполните следующие действия:

        1. Вычислите квадрат cos(x) .
        2. Вычесть cos²(x) из 1 .
        3. Извлеките квадратный корень из результата шага 2. Помните, что есть два возможных результата: один положительный и один отрицательный.
        4. У вас есть два возможных значения sin(x) . Если вы знаете, например, что ваш угол x острый, то его синус положителен.
        5. Полная форма формулы, которую мы применили: sin²(x) + cos²(x) = 1 .

        Как найти синус с помощью косинуса и тангенса?

        Чтобы определить синус угла по его косинусу и тангенсу, нужно умножить косинус на тангенс . Вам нужна формула
        sin(α) = tan(α) × cos(α) .

        Maciej Kowalski, кандидат в PhD

        Функция

        Угол (x)

        . Для визуального сравнения COFUNCTIONS Будьте проверены на их графиках:

        .

        AC Приближение касательной линии

        Мотивирующие вопросы

        • Какова формула общей аппроксимации касательной к дифференцируемой функции \(y = f(x)\) в точке \((a,f(a))\text{?}\)

        • Что такое принцип локальной линейности и что такое локальная линеаризация дифференцируемой функции \(f\) в точке \((a,f(a))\text{?}\)

        • Каким образом знание только аппроксимации касательной дает нам информацию о поведении самой исходной функции вблизи точки аппроксимации? Каким образом знание значения второй производной в этот момент дает нам дополнительные сведения о поведении исходной функции?

        Среди всех функций линейные функции самые простые. Одно из важных следствий дифференцируемости функции \(y = f(x)\) в точке \((a,f(a))\) заключается в том, что вблизи функция \(y = f(x )\) локально линейна и имеет вид своей касательной в этой точке. В определенных обстоятельствах это позволяет нам аппроксимировать исходную функцию \(f\) более простой функцией \(L\), которая является линейной: это может быть выгодно, когда у нас есть ограниченная информация о \(f\) или когда \(f \) является вычислительно или алгебраически сложным. Далее мы рассмотрим все эти ситуации. 92+3x+2\текст{.}\)

        1. Использовать предельное определение производной для вычисления формулы для \(y = g'(x)\text{.}\)

        2. Определить наклон касательной к \(y = g(x)\) при значении \(x = 2\text{.}\)

        3. Вычислить \(g(2)\text{.}\)

        4. Найдите уравнение для касательной к \(y = g(x)\) в точке \((2,g(2))\text{.}\) Запишите результат в виде точка-наклон.

        5. На осях, показанных на рисунке 1.8.1, нарисуйте точный помеченный график \(y = g(x)\) вместе с его касательной в точке \((2,g(2))\text{ . }\)

        Рисунок 1.8.1. Оси для построения \(y = g(x)\) и его касательной к точке \((2,g(2))\text{.}\)

        Подраздел 1.8.1 Касательная линия

        Для данной функции \(f\), которая дифференцируема в точке \(x = a\text{,}\), мы знаем, что можем определить наклон касательной к \(y = f(x)\) в точке \ ((a,f(a))\) путем вычисления \(f'(a)\text{.}\) Уравнение полученной касательной линии задается в форме точка-наклон как

        \begin{уравнение*} y — f(a) = f'(a)(x-a) \ \ \text{or} \ \ y = f'(a)(x-a) + f(a)\text{.} \end{уравнение*}

        Обратите внимание: в этом контексте между \(f(a)\) и \(f(x)\) есть большая разница. Первая — это константа, полученная в результате использования заданного фиксированного значения \(a\text{,}\), а вторая — это общее выражение для правила, определяющего функцию. То же верно и для \(f'(a)\) и \(f'(x)\text{:}\), мы должны тщательно различать эти выражения. Каждый раз, когда мы находим касательную, нам нужно вычислять функцию и ее производную при фиксированном \(a\)-значении.

        На рисунке 1.8.2 мы видим график функции \(f\) и ее касательную в точке \((a,f(a))\text{.}\). Обратите внимание, что при увеличении масштаба мы см. более четко выделенную локальную линейность \(f\). Функция и ее касательная почти неразличимы вблизи. Локальная линейность также может наблюдаться динамически в этом апплете  2  .

        Рисунок 1.8.2. Функция \(y = f(x)\) и ее касательная в точке \((a,f(a))\text{:}\) слева на расстоянии и справа вблизи. Справа мы обозначаем функцию касательной через \(y = L(x)\) и замечаем, что для \(x\) вблизи \(a\text{,}\) \(f(x) \ приблизительно L( х)\текст{.}\)

        Подраздел 1.8.2 Локальная линеаризация

        Небольшое изменение перспективы и обозначений позволит нам более точно обсуждать, как касательная аппроксимирует \(f\) вблизи \(x = a\text{.}\). Решая для \(y\text{, }\) мы можем написать уравнение для касательной как

        \begin{уравнение*} у = f'(а)(х-а) + f(а) \end{уравнение*}

        Эта строка сама является функцией \(x\text{. }\) Заменив переменную \(y\) выражением \(L(x)\text{,}\), мы назовем

        \begin{уравнение*} L(x) = f'(a)(x-a) + f(a) \end{уравнение*}

        локальная линеаризация \(f\) в точке \((a,f(a))\text{.}\) В этих обозначениях \(L(x)\) есть не что иное, как новая имя касательной линии. Как мы видели выше, для \(x\) близких к \(a\text{,}\) \(f(x) \ приблизительно L(x)\text{.}\)

        Пример 1.8.3.

        Предположим, что функция \(y = f(x)\) имеет аппроксимацию касательной, заданную выражением \(L(x) = 3 — 2(x-1)\) в точке \((1,3)\ text{,}\), но мы ничего не знаем о функции \(f\text{.}\) Чтобы оценить значение \(f(x)\) для \(x\) около 1, например \(f(1.2)\text{,}\) мы можем использовать тот факт, что \(f(1.2) \ приблизительно L(1.2)\) и, следовательно,

        \begin{уравнение*} f(1.2) \приблизительно L(1.2) = 3 — 2(1.2-1) = 3 — 2(0.2) = 2.6\text{.} \end{equation*}

        Подчеркнем, что \(y = L(x)\) — это просто новое название функции касательной. Используя это новое обозначение и наше наблюдение, что \(L(x) \ приблизительно f(x)\) для \(x\) вблизи \(a\text{,}\), следует, что мы можем написать

        \begin{уравнение*} f(x) \ приблизительно f(a) + f'(a)(x-a) \ \text{for} \ x \ \text{рядом} \ a\text{. } \end{уравнение*}

        Мероприятие 1.8.2.

        . Предположим, что известно, что для данной дифференцируемой функции \(y = g(x)\text{,}\) ее локальная линеаризация в точке, где \(a = —1\), определяется выражением \(L(x) = -2 + 3(х+1)\текст{.}\)

        1. Вычислить значения \(L(-1)\) и \(L'(-1)\text{.}\)

        2. Какими должны быть значения \(g(-1)\) и \(g'(-1)\text{?}\) Почему?

        3. Ожидаете ли вы, что значение \(g(-1.03)\) будет больше или меньше значения \(g(-1)\text{?}\) Почему?

        4. Используйте локальную линеаризацию для оценки значения \(g(-1.03)\text{.}\)

        5. Предположим, что вы также знаете, что \(g»(-1) = 2\text{.}\) Что это говорит вам о графике \(y = g(x)\) в точке \(a = -1\текст{?}\)

        6. Для \(x\) вблизи \(-1\text{,}\) нарисуйте график локальной линеаризации \(y = L(x)\), а также возможный график \(y = g( x)\) по осям, указанным на рисунке 1.8.4.

        Рисунок 1.8. 4. Оси построения \(y = L(x)\) и \(y = g(x)\text{.}\)

        Из упражнения 1.8.2 мы видим, что локальная линеаризация \(y = L(x)\) является линейной функцией, которая имеет два общих значения с функцией \(y = f(x)\), полученной из . В частности,

        • , так как \(L(x) = f(a) + f'(a)(x-a)\text{,}\) следует, что \(L(a) = f(a)\text{;} \) и

        • , поскольку \(L\) — линейная функция, ее производная — это наклон.

        Следовательно, \(L'(x) = f'(a)\) для любого значения \(x\text{,}\) и, в частности, \(L'(a) = f'(a)\text {.}\) Таким образом, мы видим, что \(L\) является линейной функцией, которая имеет и то же значение, и тот же наклон, что и функция \(f\) в точке \((a,f(a)) \текст{.}\)

        Таким образом, если мы знаем линейную аппроксимацию \(y = L(x)\) для функции, мы знаем исходное значение функции и ее наклон в точке касания. Однако остается неизвестным форма функции \(f\) в точке касания. По сути, есть четыре возможности, как показано на рисунке 1. 8.5.

        Рисунок 1.8.5. Четыре возможных графика нелинейной дифференцируемой функции и то, как она может располагаться относительно своей касательной в точке.

        Эти возможные формы являются результатом того, что есть три варианта значения второй производной: либо \(f»(a) \lt 0\text{,}\) \(f»(a) = 0 \text{,}\) или \(f»(a) \gt 0\text{.}\)

        • Если \(f»(a) \gt 0\text{,}\), то мы знаем, что график \(f\) вогнут вверх, и мы видим первую возможность слева, где касательная линия лежит полностью ниже кривой.

        • Если \(f»(a) \lt 0\text{,}\), то \(f\) вогнута вниз и касательная лежит выше кривой, как показано на втором рисунке.

        • Если \(f»(a) = 0\) и \(f»\) меняет знак при \(x = a\text{,}\), то вогнутость графика изменится, и мы увидим либо третья, либо четвертая фигура.  3  .

        • Пятый вариант (который не очень интересен) может иметь место, если сама функция \(f\) является линейной, так что \(f(x) = L(x)\) для всех значений \(x\text {. }\)

        Графики на рис. 1.8.5 подчеркивают еще одну важную вещь, которую мы можем узнать из вогнутости графика вблизи точки касания: лежит ли касательная выше или ниже самой кривой. Это ключевой момент, потому что он говорит нам, будут ли значения аппроксимации касательной слишком большими или слишком маленькими по сравнению с истинным значением \(f\text{.}\). Например, в первой ситуации на крайнем левом графике на рисунке 1.8.5, где \(f»(a) > 0\text{,}\), поскольку касательная опускается ниже кривой, мы знаем, что \(L(x) \le f(x)\) для все значения \(x\) рядом с \(a\text{.}\)

        Мероприятие 1.8.3.

        Это действие касается функции \(f(x)\), о которой известна следующая информация:

        • \(f\) — дифференцируемая функция, определенная для каждого действительного числа \(x\)

        • \(\displaystyle f(2) = -1\)

        • \(y = f'(x)\) имеет график, показанный на рисунке 1.8.6

        Рисунок 1.8.6. В центре график \(y = f'(x)\text{;}\) слева, оси для построения \(y = f(x)\text{;}\) справа, оси для построения \ (y = f»(x)\text{. }\)

        Ваша задача — найти как можно больше информации о \(f\) (особенно вблизи значения \(a = 2\)) с помощью ответов на приведенные ниже вопросы.

        1. Найдите формулу аппроксимации касательной от \(L(x)\text{,}\) до \(f\) в точке \((2,-1)\text{.}\)

        2. Используйте аппроксимацию касательной для оценки значения \(f(2.07)\text{.}\) Внимательно и ясно покажите свою работу.

        3. Нарисуйте график \(y = f»(x)\) в правой сетке на рисунке 1.8.6; обозначьте его соответствующим образом.

        4. Является ли наклон касательной к \(y = f(x)\) возрастающим, убывающим или ни тем, ни другим, когда \(x = 2\text{?}\) Объясните.

        5. Нарисуйте возможный график \(y = f(x)\) вблизи \(x = 2\) в левой сетке на рисунке 1.8.6. Включите набросок \(y=L(x)\) (найден в части (a)). Объясните, откуда вы знаете, что график \(y = f(x)\) выглядит так, как будто вы его нарисовали.

        6. Ваша оценка в (b) завышает или занижает истинное значение \(f(2. 07)\text{?}\) Почему?

        Идея о том, что дифференцируемая функция выглядит линейной и может быть хорошо аппроксимирована линейной функцией, является важной и находит широкое применение в исчислении. Например, аппроксимируя функцию ее локальной линеаризацией, можно разработать эффективный алгоритм оценки нулей функции. Локальная линейность также помогает нам лучше понять некоторые сложные ограничения. Например, мы видели, что предел

        \begin{уравнение*} \lim_{x \to 0} \frac{\sin(x)}{x} \end{уравнение*}

        является неопределенным, потому что его числитель и знаменатель стремятся к 0. Хотя нет никакой алгебры, которую мы могли бы сделать, чтобы упростить \(\frac{\sin(x)}{x}\text{,}\), это просто покажите, что линеаризация \(f(x) = \sin(x)\) в точке \((0,0)\) определяется выражением \(L(x) = x\text{.}\) Следовательно , для значений \(x\) около 0, \(\sin(x) \ приблизительно x\text{,}\) и, следовательно,

        \begin{уравнение*} \ frac {\ sin (x)} {x} \ приблизительно \ frac {x} {x} = 1 \ text {,} \end{уравнение*}

        , что делает правдоподобным тот факт, что

        \begin{уравнение*} \lim_{x \to 0} \frac{\sin(x)}{x} = 1\text{. } \end{уравнение*}

        Подраздел 1.8.3 Резюме

        • Касательная к дифференцируемой функции \(y = f(x)\) в точке \((a,f(a))\) задается в виде точки-наклона уравнением

          \begin{уравнение*} y — f(a) = f'(a)(x-a)\text{.} \end{уравнение*}

        • Принцип локальной линейности говорит нам, что если мы увеличим масштаб точки, где функция \(y = f(x)\) дифференцируема, функция будет неотличима от своей касательной. То есть дифференцируемая функция выглядит линейной при ближайшем рассмотрении. Мы переименовываем касательную в функцию \(y = L(x)\text{,}\), где \(L(x) = f(a) + f'(a)(x-a)\text{.} \) Таким образом, \(f(x) \ приблизительно L(x)\) для всех \(x\) вблизи \(x = a\text{.}\)

        • Если мы знаем приближение касательной \(L(x) = f(a) + f'(a)(x-a)\) к функции \(y=f(x)\text{,}\), то поскольку \(L(a) = f(a)\) и \(L'(a) = f'(a)\text{,}\) мы также знаем значения как функции, так и ее производной в точке где \(x = a\text{.}\) Другими словами, линейная аппроксимация сообщает нам высоту и наклон исходной функции. Если, кроме того, мы знаем значение \(f»(a)\text{,}\), то мы знаем, лежит ли касательная выше или ниже графика \(y = f(x)\text{ ,}\) в зависимости от вогнутости \(f\text{.}\)

        Упражнения 1.8.4 Упражнения

        1. Приближение \(\sqrt{x}\).

        Используйте линейную аппроксимацию для аппроксимации \(\sqrt {36.1}\) следующим образом.

        Пусть \(f(x) = \sqrt x\text{.}\) Уравнение касательной к \(f(x)\) в точке \(x = 36\) можно записать в виде \ (y = mx+b\text{.}\) Вычислить \(m\) и \(b\text{.}\)

        \(m=\)

        \(b=\)

        Используя это найдите приближение для \(\sqrt {36.1}\text{.}\)

        Ответ:

        2. Локальная линеаризация графа.

        На рисунке ниже показано \(f(x)\) и его локальная линеаризация при \(x=a\text{,}\) \(y = 4 x — 4\text{.}\) (Локальная линеаризация показано синим цветом.)

        Каково значение \(a\text{?}\)

        \(a =\)

        Каково значение \(f(a)\text{?}\)

        \(f(a) =\)

        Используйте линеаризацию для аппроксимации значения \(f(3. 2)\text{.}\)

        \(f(3.2) =\)

        Является приближением заниженная или завышенная оценка?

        (Введите вместо или вместо .)

        3. Оценка с локальной линеаризацией.

        Предположим, что \(f(x)\) является функцией с \(f(130) = 46\) и \(f'(130) = 1\text{.}\). Оценка \(f(125,5)\ текст{.}\)

        \(f(125.5) =\)

        4. Прогнозирование поведения по локальной линеаризации.

        Температура, \(H\text{,}\) в градусах Цельсия, чашки кофе, стоящей на кухонном столе, определяется выражением \(H = f(t)\text{,}\), где \(t \) в минутах с момента подачи кофе на прилавок.

        (a) Является ли \(f'(t)\) положительным или отрицательным?

        • положительный

        • отрицательный

        (Убедитесь, что вы можете обосновать свой ответ.)

        (b) Каковы единицы измерения \(f'(30)\text{?}\)

        Предположим, что \(|f'(30)| = 0,9\) и \(f(30) = 51\text{.}\) Заполните пропуски (включая единицы измерения, где это необходимо) и выберите соответствующие термины, чтобы завершить следующее утверждение о температура кофе в данном случае.

        Через несколько минут после того, как кофе был поставлен на прилавок, его

        есть и будет

        • увеличение

        • уменьшение

        примерно за следующие 75 секунд.

        5.

        Некоторая функция \(y=p(x)\) имеет локальную линеаризацию в точке \(a = 3\), заданную выражением \(L(x) = -2x + 5\text{.}\)

        1. Каковы значения \(p(3)\) и \(p'(3)\text{?}\) Почему?

        2. Оценить значение \(p(2,79)\текст{.}\)

        3. Предположим, что \(p»(3) = 0\) и вы знаете, что \(p»(x) \lt 0\) для \(x \lt 3\text{.}\) Ваша оценка в (б) слишком большой или слишком маленький?

        4. Предположим, что \(p»(x) \gt 0\) для \(x \gt 3\text{.}\) Используйте этот факт и приведенную выше дополнительную информацию, чтобы нарисовать точный график \(y = p (x)\) рядом с \(x = 3\text{.}\) Включите набросок \(y = L(x)\) в свою работу.

        6.

        Картофель помещают в печь, измеряют температуру картофеля \(F\) (в градусах по Фаренгейту) в различные моменты времени и записывают в следующую таблицу. Время \(t\) измеряется в минутах.

        Таблица 1.8.7. Температурные данные для картофеля.

        \(т\) \(Ф(т)\)
        \(0\) \(70\)
        \(15\) \(180.5\)
        \(30\) \(251\)
        \(45\) \(296\)
        \(60\) \(324.5\)
        \(75\) \(342.8\)
        \(90\) \(354,5\)

        1. Используйте центральную разность для оценки \(F'(60)\text{.}\) При необходимости используйте эту оценку в последующих вопросах.

        2. Найти локальную линеаризацию \(y = L(t)\) функции \(y = F(t)\) в точке, где \(a = 60\text{.}\)

        3. Определите оценку для \(F(63)\), используя локальную линеаризацию.

        4. Как вы думаете, ваша оценка в (c) слишком велика или слишком мала? Почему?

        7.

        Объект, движущийся по прямолинейному пути, имеет дифференцируемую функцию положения \(y = s(t)\text{;}\) \(s(t)\) измеряет положение объекта относительно начала координат в момент времени \(t \text{.}\) Известно, что в момент времени \(t = 9\) секунд объект находится в \(s(9) = 4\) футах (т.е. на 4 фута правее начала координат). Кроме того, мгновенная скорость объекта при \(t = 9\) составляет \(-1,2\) фута в секунду, а его ускорение в тот же момент составляет \(0,08\) фута в секунду в секунду.

        1. Использовать локальную линейность для оценки положения объекта в точке \(t = 9,34\text{.}\)

        2. Возможно, ваша оценка слишком велика или слишком мала? Почему?

        3. Обычным языком опишите поведение движущегося объекта в точке \(t = 9\text{.}\) Он движется к началу координат или от него? Его скорость увеличивается или уменьшается?

        8.

      Отрицательное математическое ожидание: Please Wait… | Cloudflare

      Математическое ожидание в трейдинге. Риски и вероятность выигрыша :Blog Siwitpro

      В трейдинге достаточно много нюансов, которые, не являясь значительными в принципе, существенно влияют на конечный результат. К примеру, математическое ожидание. Примечательно, что, даже хорошо владея фундаментальным и техническим анализом, трейдер, чья торговая система показывает отрицательное математическое ожидание, не добьётся успеха и сольёт депозит в долгосрочной перспективе.  В этой статье мы постараемся максимально просто объяснить, что такое математическое ожидание в трейдинге, каким оно бывает и как сказывается на торговле. Также мы обсудим, что можно сделать, чтобы повысить мат. ожидание по сделкам.

      Математическое ожидание в трейдинге – простыми словами

      Если говорить просто, то математическое ожидание – это усреднённый статистический показатель, дающий представление о прибыльности торговой системы или стратегии. Расчёт математического ожидания позволяет трейдеру  увидеть, что превалирует в его торговле – убыток или прибыль.

      Казалось бы, чтобы это понять, достаточно просто подбить процент прибыльных и убыточных сделок по итогу какого-то периода – недели, месяца и т. п.  Но такая статистика не всегда будет объективна, ведь на прибыльность сделок в этот период могли влиять самые разные факторы, не имеющие отношения к эффективности торговой системы.

      Для расчёта же математического ожидания берётся как минимум, 100 сделок. Расчёт происходит по простой формуле: От процента успешных сделок торговой системы, умноженного на прибыль в средней прибыльной сделке, отнимается процент убыточных сделок, умноженный на средний убыток в такой сделке. Статистические данные для расчёта можно без труда выгрузить из торгового терминала.

      Каким бывает математическое ожидание и что это даёт?

      Математическое ожидание бывает положительным и отрицательным.  То есть, если после расчёта по вышеприведённой формуле у Вас получилась цифра от 0 и выше, мат. ожидание положительное. Если же получилась цифра со знаком «минус» — оно отрицательное. Что это даёт трейдеру?

      Положительное мат. ожидание означает, что доход от прибыльных сделок способен перекрыть потери от убыточных. Следовательно, торговая система работает хорошо, трейдер всегда в плюсе, даже несмотря на периодические неудачи. Поэтому, в долгосрочной перспективе можно рассчитывать на рост депозита.

      Отрицательное значение математического ожидания – плохая новость для трейдера. Это означает, что торговая система работает не так, как должна, а убытки превышают прибыль. Даже если на данном этапе процент прибыльных сделок превышает процент убыточных, но имеет место отрицательное математическое ожидание, в долгосрочной перспективе трейдер уйдёт в минус и неизбежно сольёт депозит. Как такое возможно?

      Тут всё достаточно просто. К примеру, у трейдера 70% прибыльных сделок. Это хороший показатель. Но при этом, математическое ожидание показывает минус. Это значит, что общая сумма прибыли от этих 70% не перекроет сумму убытков от оставшихся 30% убыточных.

      Поясним на примере. Допустим, трейдер заключил 100 сделок. Из них было 70 прибыльных и 30 убыточных. На прибыльных он заработал в сумме 1000 долларов, а на убыточных потерял 1200 долларов. В итоге, убытки на 200 долларов превысили доход, хотя прибыльных сделок и было больше. В чём причина? Скорее всего, прибыльными оказались более мелкие позиции, а убыточными оказались крупные.

      По сути, именно такую вероятность развития событий прогнозирует отрицательное математическое ожидание, даже если на момент расчёта убытки ещё не превышают прибыль.

      Итак, что даёт трейдеру расчёт мат. ожидания? По сути, возможность оценить эффективность своей торговой системы в перспективе. Либо по результатам расчётов он ещё раз убедится, что делает всё правильно, либо заметит риск слива депозита и поймёт, что необходимо пересмотреть систему и стратегию, и то-то поменять. В каком-то смысле, расчёт математического ожидания – как система раннего оповещения о потере депозита (если он отрицательный).

      Мат. ожидание в минусе. Всё плохо?

      Если говорить откровенно, то да, перспективы у трейдера с отрицательным математическим ожиданием не радужные. Но это лишь в том случае, если он не захочет ничего предпринять. А что можно сделать, чтобы повысить математическое ожидание?

      Один из самых эффективных вариантов – повысить соотношение между стоп-лоссом и тейк-профитом. Вероятнее всего, математическое ожидание показало минус, потому что соотношение между стопом и тейком сейчас 1:1 или 1:2. При соотношении 1:1 убытки почти гарантированы, поскольку на бирже взымают комиссионные, что уже лишает это соотношение равенства.   Соотношение 1:2 уже лучше, но если трейдеру предстоит пройти через череду неудач, этот показатель его не спасёт.

      Многие считают, что оптимальное соотношение стопа к тейку – 1:3 или 1:4. В этом действительно есть смысл, ведь при таких соотношениях прибыль сможет перекрыть убытки даже в трудные времена для трейдера.

      Однако стоит понимать, что чем больше это соотношение, тем больше риск, что цена попросту не дойдёт до отметки тейка. Тут нужно сохранять уравновешенность – вероятность, что цена пройдёт путь до тейка при соотношении 1:3 гораздо выше, чем, что она пройдёт этот путь при соотношении 1:10. Таковы уж рыночные условия – редко можно наблюдать такую волатильность  достаточно долго, чтобы она сорвала тейк.

      Итак, как видно, математическое ожидание в трейдинге – полезный показатель для оценки эффективности своей торговли в перспективе. Он позволяет вовремя заметить проблему и успеть предпринять меры для её решения до того, как трейдер окажется в минусе.

      Помочь создать эффективную торговую систему с положительным математическим ожиданием может обучение в Школе трейдинга Александра Пурнова у опытного наставника. А полезные материалы на тему трейдинга из нашего блога будут доступны Вам в полном объёме после подписки.

      Математическое ожидание трейдинг

      Всем привет!

      Математическое ожидание играет важную роль в трейдинге. Многие недооценивают это показатель. Можно отлично разбираться в фундаментальном и техническом анализе, но при торговле с отрицательным мат. ожиданием трейдер будет обречен на провал. Но в тоже время многие слишком усложняют себе задачу и пытаются рассчитать мат. ожидание там где это совершенно не нужно и при идеальных условиях. Здесь нужно понять одно, идеальных условий в трейдинге не бывает. В данной статье я не буду вас загружать нудными формулами, которые описаны на других сайтах. Я лишь расскажу о том, как, когда и в каких случаях, стоит учитывать мат. ожидание.

      Мат. ожидание в трейдинге

      Одну формулу в пример я все-таки приведу, чтобы можно было уловить суть. Это один из вариантов, в котором учитывают показатель мат. ожидания.

      При расчете мат. ожидания берется следующая формула: вероятность получения прибыли * на среднюю прибыль от одной сделки минус вероятность получения убытков * средний убыток от одной сделки. И если, к примеру, учесть тот факт, что положительных и отрицательных сделок у нас 50 на 50, при этом средняя прибыль 500 пунктов, а средний убыток 250, то получится формула вида: (0,5*500) – (0,5*250) = 250 – 125 = 125.

      В данном идеальном варианте мат. ожидание положительное. И на самом деле, очень странно, когда пытаются взять идеальные условия и доказать что нужно делать так-то и так. Например, что обязательно каждая сделка должна быть не меньше чем 1 к 2 (убыток к прибыли). Или средний профит обязательно выше среднего убытка. Мы никогда не сможем точно определить вероятность прибыльной/убыточной сделки. Все необходимые значения мы сможем оценить лишь постфактум на условии статистики. Торговля не сможет вам гарантировать той или иной вероятности по сделке и по профиту.

      Все это я рассказываю к тому, что пытаться рассчитать положительное или отрицательное мат. ожидание постфактум, учитывая только вышеуказанные показатели, не совсем верно. На положительные результаты в торговле влияет очень много факторов. Важнее просто грамотно вести статистику, записывать подробный результат и пытаться выяснить почему получился тот или иной итог. Возможно по текущей торговой формации слишком мало положительных сделок. Либо при увеличении показателя риск к прибыли результат был бы положительным. В этом случае важно учесть тот факт, что нужный нам показатель профита действительно будет оправданным и сделка будет срабатывать. Так как вроде бы с точки зрения мат. ожидания все сошлось, но на деле в реальной торговле инструмент не будет доходить до нашего профита, так как он оказался завышенным, либо мы не учли других факторов.

      Также я могу сказать следующее, что даже если совершать сделки 1 к 1, то в некоторых случаях они могут быть абсолютно оправданными, если положительных сделок будет больше чем отрицательных. В некоторых моих формациях есть сделки 1 к 1, при этом результат по данным формациям положительный. Поэтому, в некоторых случаях не нужно доверять всему что написано. И когда я вижу утверждение, что можно зарабатывать на рынке лишь тогда, когда риск к прибыли будет не меньше чем 1 к 2, то для меня это звучит странно.

      А теперь, еще один простой пример в каких случаях стоит учитывать мат. ожидание. Например, при использовании такого показателя как ATR. Допустим, инструмент превысил свой показатель ATR более чем на 100 %, то в таком случае глупо заходить в позицию, так как с точки зрения мат. ожидания вероятность разворота выше. Либо заходить в позицию в том случае, когда ATR не позволяет вам закрыть позицию, скажем, 1 к 3. Например, если вы понимаете что инструмент прошел 90 % своего ATR и вы явно не сможете забрать ту прибыль которую планировали, не нарушив мат. ожидание. Это обычная математика против которой идти глупо.

      Подробнее об ATR читайте здесь.

      В трейдинге нужно всегда стараться чтобы мат. ожидание было положительным. И когда будете анализировать ваши статистические данные, не забывайте про это и вносите коррективы в вашу торговлю верно.

      На этом буду заканчивать. Надеюсь, вы уловили суть из моих размышлений 🙂 Подписывайтесь на новости сайта, всем пока.

      С уважением, Станислав Станишевский.

      Мат.ожидание или «Теория казино»

      Принято считать, что основной товар в казино — это адреналин. Часто мы слышим, что казино предлагает вытянуть «счастливый билет», много реже говорят что казино продает сервис. На самом же деле, основной товар казино — это азарт от возможности выигрыша. В этой статье мы рассмотрим основные принципы, на которых организована работа игорных домов, обоснование прибыли заведения, и какую роль в ее деятельности играет «госпожа удача». 

      А начнем обзор с рассмотрения основных математических законов, на которых построены азартные игры. Как связаны математика и казино? Ведь все игры в казино были придуманы и разработаны именно математиками. Можно ли использовать их же оружие для получения преимущества в игорном доме? 

      Математика игр казино 


      Рассмотрим процессы, происходящие в азартных играх, с точки зрения теории вероятности, и попробуем определить, подчиняются ли игры казино математике. 

      Бросая монету, можно утверждать, что любая из ее сторон может выпасть с одинаковой вероятностью. Есть всего две возможности — выпадет либо орел, либо решка. Вероятность того, что при бросании монеты выпадет решка равна? (50%), то есть мы вправе ожидать, что в половине случаев будет выпадать решка. Часто говоря о вероятности употребляют слово шанс. Шанс на то, что при броске монеты она упадет решкой вверх, равен 50% 

      Вероятность показывает, как часто ожидаемый нами результат может быть достигнут, и может быть представлена как отношение ожидаемых исходов к общему количеству всех возможных исходов за достаточно продолжительный период времени при большом количестве повторений. 

      Математическое ожидание при игре в рулетку 


      Рассчитаем математическое ожидание при игре в рулетку (американская версия с двумя секторами «зеро» ноль и двойной ноль) при ставке 5$ на цвет (черное): 18\38 х (+5$) + 20\38 х (-5$) = -0,263 

      Как вы уже наверное заметили, в обоих приведенных примерах, величина математического ожидания имеет знак «-», что характерно для большинства ставок казино. Отрицательное математическое ожидание на практике означает, что, чем дольше длится игра, тем больше вероятность проигрыша для игрока. 

      Перевес казино (House Edge) [доля заведения] – величина, противоположная математическому ожиданию игрока и показывающая, какой процент от ставок, сделанных в процессе игры за определенный промежуток времени, удерживается в пользу казино.Сейчас мы будем рассматривать самый популярный вид игры в казино, знаете какой? Самая популярная игра казино во всем мире — это игра в рулетку.Перевес казино в европейской рулетке составляет 1 — 36/37 = 2,7%, в американской рулетке уже 1 — 36/38 = 5,26% (за счет двух зеро). Это означает, что, если вы, играя в рулетку, за определенное время поставили в общей сложности 1000 долларов, то велика вероятность, что в конечном итоге около 27$ (европейская рулетка) и 54$ (американская рулетка) пойдет в доход игорному заведению. В настольных играх перевес казино меньше (Баккара, Блэкджек или Крэпс), поэтому шансы выиграть в них выше. 

      В качестве примера посчитаем, каковы наши шансы в казино при игре в американскую версию рулетки, игровое колесо которой, напомню, насчитывает 38 секторов (1-36 цифры + 2 сектора зеро). Предположим, что мы поставили на число. Оплата выигрыша, в этом случае производится в соотношении 1 к 36 

      Вероятность выиграть в этом случае 1\38 или 2,63% 
      Возможный выигрыш игрока (в процентах к ставке): 1/38 х 36х100 = 94.74% 
      Процент казино: 100 – 94,7 = 5.26 % 
      Математическое ожидание: [(1\38) х 36 (+1)] + [(37\38) x (-1)] = -0,0263 
      То есть, с каждого поставленного вами доллара, игорный дом надеется заработать 2,63 цента. Другими словами математическое ожидание выигрыша игрока при игре в американскую рулетку в казино составляет -2.6% от каждой вашей ставки. 

      Выводы: 


      Не надо быть великим математиком, чтобы играть в казино. Можно даже не считать математическое ожидание и дисперсию — это сделали до вас и можно пользоваться готовыми результатами. Главное понимать, что игры, имеющие большую величину математического ожидания, выгоднее для игрока, так как в них преимущество казино перед вами меньше и, соответственно, время вашей игры и возможная сумма выигрыша увеличивается. Ищите игры, в которых реализовано преимущество игрока, только в этом случае вы можете рассчитывать на выигрыш в достаточно долгой игре. 

      При выборе рулетки отдавайте предпочтение европейскому варианту (с одним «зеро») так как в ней преимущество казино будет 2,7%, в отличии от американской версии (с двумя «зеро»), в котором перевес игорного заведения равен уже 5,26%. 

      Но, рассуждая о положительных и отрицательных математических ожиданиях, вы не должны забывать и о том, что существует дисперсия. И чем она выше, тем больше вас будет «лихорадить» в игре. Вы будете проигрывать в играх с преимуществом игрока, и, в то же время, можете выиграть там, где казино имеет значительный перевес математического ожидания. Помните, что вся математика азартных игр казино корректно работает только в случае, когда число попыток велико и, поэтому, достигнуть на практике расчетных ожидаемых величин достаточно сложно из-за ограниченности бюджета игрока, величины ставок или времени игры.

      Источник[1];
      Отдельная благодарность Алексею Маркову и его книге «Хулиномика».
      Именно из за него и его творения побудилась идея создания данной статьи.

      Математическое ожидание в трейдинге | Азбука трейдера

      olegas

      Июн 12, 2015 / 150 Views

      Помимо фундаментального и технического анализа в трейдинге большую роль играет математика. Для успешной работы в качестве трейдера вы должны иметь четкую систему управления капиталом, важным параметром которой является такое понятие как математическое ожидание.

      Казалось бы, чего тут заморачиваться, если количество прибыльных сделок превышает количество убыточных, то всё, что называется, “на мази” и можно спокойно работать и дальше. Однако не всё так просто, ведь количество не всегда означает качество. И даже в том случае, когда прибыльных сделок по факту получается больше чем убыточных, трейдер всё равно может остаться в минусе. И причиной тому будет ни что иное, как отрицательное математическое ожидание.

      Трейдер может в совершенстве знать технический и фундаментальный анализ, но при торговле с отрицательным математическим ожиданием он будет обречен на неудачу. Даже если благодаря использованию, какого либо из указанных выше видов анализа в отдельности или вместе взятых, трейдер совершает 8 прибыльных сделок из 10, он все равно может оказаться в минусе. Если, например, его прибыль по каждой прибыльной сделке составила 10 пунктов, а по каждой убыточной 50 пунктов, то в результате он имеет:

      Прибыль: 8х10=80 пунктов;

      Убыток: 2х50=100 пунктов;

      Итого: 80-100=-20 пунктов убытка.

      Математическое ожидание вычисляется по следующей формуле:

      Математическое ожидание=вероятность получения прибыли х средняя прибыль от одной сделки – вероятность получения убытков х средний убыток от одной сделки.

      Так в приведенном выше примере математическое ожидание отрицательное:

      8х10-2х50=-20<0

      А если бы, например, трейдер заключал прибыльные и убыточные сделки с вероятностью 50/50 (то есть, вероятность прибыльной сделки составляет 50% и вероятность убыточной сделки составляет 50%). И если бы каждая прибыльная сделка приносила ему 20 пунктов прибыли, а каждая убыточная 10 пунктов убытка, то математическое ожидание было бы положительным:

      0,5х20-0,5х10=5>0

      Математическое ожидание при тестировании торговых стратегий

      Такой показатель как математическое ожидание очень важен при оценке эффективности торговой системы. Проводя тестирование торговых систем (на исторических данных) в тестере стратегий МТ4 (Metatrader 4), вы можете увидеть этот параметр в отчёте о результатах тестирования.

      Отчёт тестера стратегий МТ4

      Для корректного расчёта данного показателя следует брать достаточно глубокий срез статистики по совершённым сделкам. Как минимум необходимы данные о 100 – 150 закрытых сделках. В ином случае рассчитанный показатель не будет иметь должной объективности.

      Кстати в МТ4, математическое ожидание вычисляется по формуле:

      Мат.ожидание = (Общая прибыль + Общий убыток) / Кол-во сделок

      Положительное математическое ожидание говорит трейдеру о том, что тестируемая им торговая стратегия является потенциально прибыльной. А отрицательное, соответственно, о том, что стратегия убыточна.

      Что можно сделать для того, чтобы повысить математическое ожидание торговой стратегии? Самое очевидное, что можно для этого сделать, так это повысить соотношение Take Profit (TP) к Stop Loss (SL). Например, при соотношении TP/SL = 1 (размер профитов равен размеру убытков по каждой сделке), торговая стратегия показывает отрицательное матожидание, но стоит повысить это соотношение до TP/SL = 1,5…2, как стратегия сразу выходит в плюс.

      Однако, здесь важно не перестараться. Ведь, хотя большинство авторов и рекомендуют соотношение TP/SL в пределах 2…3, но следует учитывать тот факт, что чем больший размер профита относительно лосса вы установите, тем больше в вашей статистике появится убыточных сделок. Увеличивая разрыв между значениями Stop Loss и Take Profit, вы тем самым, уменьшаете и вероятность того, что цена в итоге достигнет профита, а не столкнётся с лоссом.

      Вы можете поделиться этой статьёй на своей странице в соцсетях:


      Математическое ожидание

      ]]]]]]]]>]]]]]]>]]]]>]]>]]]]]]]]>]]]]]]>]]]]>]]>]]]]]]]]>]]]]]]>]]]]>]]>]]]]]]]]>]]]]]]>]]]]>]]>]]]]]]]]]]>]]]]]]]]>]]]]]]>]]]]>]]>

      В своей работе, планируя размещение торгового капитала, вы должны уметь прогнозировать ситуацию. Особенно это касается «положительного/отрицательного ожидания».

      Проще говоря, распределяя капиталовложения, трейдер должен представлять себе перспективу положительного ожидания. Кроме то­го, он должен уметь рассчитывать размеры этого ожидания. «Положи­тельное/отрицательное ожидание» можно определить как математиче­ски доказанную вероятность прибылей/убытков. Допустим:

      Вероятность выигрышных сделок = 50% Вероятность проигрышных сделок = 50%

      Сумма каждого выигрыша = 2 доллара Сумма каждого проигрыша = 1 доллар

      Математическое выражение положительного ожидания будет сле­дующим:

      [1+(W/L)] х Р -1 (где Р — это вероятность выигрыша)

      Поэтому предыдущий пример будет иметь следующее математиче­ское ожидание:

      (1+2) х 0,5-1 = 3×0,5-1  = 1,5-1  =0,5

      Положительное ожидание определяется значением этого выраже­ния, превышающим ноль. Чем больше это число/тем сильнее статис­тическое ожидание. Если значение меньше нуля, то математическое ожидание также будет отрицательным. Чем больше модуль отрица­тельного значения, тем хуже ситуация. Если результат равен нулю, то ожидание  является безубыточным.

      Трейдеры могут использовать математические формулы в двух си­туациях. Первая ситуация, когда все суммы выигрышей равны так же, как и суммы проигрышей. Однако суммы выигрышей могут отличать­ся от сумм проигрышей так же, как и между собой. Другой случай, ког­да формулы могут быть полезны, — подсчет средних выигрышей и про­игрышей. Очевидно, что вероятностное выражение применяется к ис­торическим данным о проигрышах и выигрышах и не может использо­ваться в прогнозировании. Есть выражение, которое позволяет оце­нить ситуацию, когда суммы выигрышей и проигрышей могут прини­мать бесконечные количественные значения. Это выражение беспо­лезно для целей торговли, поскольку оно применяется к историческим данным о выигрышах/проигрышах. Вероятностное значение соотно­шения выигравших ставок к проигравшим в любой конкретной систе­ме (либо стратегии) является лишь оценочной величиной. А оценка при этом строится на статистических данных. Поэтому, прежде чем под­ставлять в выражение какие-либо данные, необходимо собрать стати­стику. В результате такого положения вещей мы будем использовать данное выражение и просто измерять силу и надежность статистичес­ких данных. При подбрасывании монет мы уже знаем вероятные в бу­дущем варианты, которые существуют вне зависимости от прошлых исходов любого количества падений монеты. В реальном мире торгов­ли мы не имеем подобной информации.

      В следующем примере используем это уравнение для известных статистических данных. Для вероятности выигрыша в 63%, при сред­ней сумме выигрышной сделки в 454 доллара, а проигрышной сделки в 458 долларов математическое ожидание будет следующим:

      [l+(W/L)]xP-l  = [1+(454/458)] х 0,63-1  =

      1,99×0,63-1  =0,2537

      Сравним это со стратегией, которая имеет следующую статистику:

      Средний выигрыш       = 2.025 долларов

      Средний проигрыш       = 1.235 долларов

      Процент выгоды       =0,52

      (1 + 1,64) х 0,52       =

      1.37-1 =0,37

      Эта система дает немного более высокий математический резуль­тат по сравнению с вышеприведенной статистикой. Следующая стати­стика имеет такие математические характеристики:

      Средний выигрыш =3.775 долларов Средний проигрыш = 1.150 долларов Вероятность выигрыша = 65% Математический результат =1,78

      Данный математический результат по своему характеру не подда­ется прогнозированию и может использоваться только для вычисле­ния мощности системы по достигнутым результатам в прошлом. В лю­бом случае — это единственная польза от статистических данных, полу­ченных путем записей истории сделок.

      Зная, что управление капиталом — это всего лишь числовая игра, которая требует использования положительных ожиданий, трейдер может прекратить поиски «священного Грааля» биржевой торговли. Вместо этого он может заняться проверкой своего торгового метода, выяснить, насколько этот метод логически обоснован, дает ли он поло­жительные ожидания. Правильные методы управления капиталом, применяемые по отношению к любым, даже весьма посредственным методам ведения торговли, сами сделают всю остальную работу.

      Читать «Математика покера от профессионала» — Склански Дэвид — Страница 3

      2. Ожидание и выигрыш в час

      Математическое ожидание

      Математическое ожидание показывает, насколько в среднем прибыльной или проигрышной окажется ставка. Данное понятие крайне важно для игроков, поскольку оно помогает оценить большинство игровых проблем. Использование математического ожидания также является лучшим способом для анализа большинства действий в покере.

      Допустим, вы ставите $1 на подбрасывание монеты. Каждый раз, когда выпадает орел, вы выигрываете, в противном случае – проигрываете. Шансы, что выпадет орел, равны 1 к 1, и вы ставите $1 против $1. Таким образом, математическое ожидание составляет в точности ноль, поскольку вы не можете математически ожидать оказаться впереди или позади после двух или двухсот подбрасываний.

      Ваше почасовое ожидание также в точности ноль. Почасовое ожидание – это размер денежной суммы, которую вы рассчитываете выиграть за час. Даже если вы способны подбросить монетку 500 раз за час, пока вы не получаете отличные от нейтральных шансы, вы не можете ни выигрывать, ни проигрывать деньги. С точки зрения серьезного игрока, это не самая плохая ситуация. Просто потеря времени.

      Однако допустим, что кто-то не особенно смышленый готов поставить $2 против вашего $1 на подбрасывание монеты. Внезапно у вас уже есть положительное ожидание в размере 50 центов за бросок. Почему 50 центов? В среднем вы выиграете столько же подбрасываний, как и проиграете. Вы ставите первый доллар и проигрываете, ставите второй – и выигрываете $2. Вы поставили $1 дважды и оказались в плюсе на $1. Каждый раз ставка в $1 выигрывает вам 50 центов. Если вы способны на 500 подбрасываний в час, ваше почасовое ожидание составляет $250, поскольку в среднем вы проиграете $1 250 раз и выиграете $2 250 раз. $500 минус $250 составляет в итоге $250. Еще раз обратите внимание, что ваше математическое ожидание, которое является размером среднего выигрыша за ставку, будет составлять 50 центов.

      Математическое ожидание не имеет ничего общего с результатами. Этот простофиля может выиграть первые 10 подбрасываний подряд, но, имея шансы 2 к 1 в ситуации, когда шансы на выигрыш равны, вы все равно зарабатываете 50 центов, ставя $1. Пока у вас достаточный банкрол, чтобы с легкостью покрыть потери, не имеет значения, выигрываете вы или проигрываете отдельно взятую последовательность ставок. Если вы продолжите, то начнете выигрывать, и на дистанции результат будет стремиться к совокупному ожиданию.

      Каждый раз, когда шансы в вашу пользу, вы зарабатываете что-то на этой ставке, выигрываете ли вы по факту или проигрываете. В той же мере когда вы ставите, имея шансы не в свою пользу, вы что-то теряете независимо от результата. Серьезные игроки принимают риск, только если шансы в их пользу, и пасуют в ином случае.

      Что означает иметь шансы в вашу пользу? Это значит в результате выигрывать больше, чем позволяют реальные шансы. Реальные шансы выпадения орла при подбрасывании монеты – 1 к 1, но вы получаете 2 к 1 за ваши деньги. Шансы в данном случае в вашу пользу. Вы впереди с положительным ожиданием в 50 центов за ставку.

      Вот также немного более сложный пример математического ожидания. Человек записывает номер от одного до пяти и ставит $5 против ваших $1, что вы не сможете угадать номер. Должны ли вы принять ставку? Какое ваше математическое ожидание?

      В среднем четыре попытки угадать будут неверными и одна верной. Таким образом, шансы ответить правильно – 4 к 1. Чаще всего в отдельной попытке вы проиграете доллар. Однако вы получаете $5 к $1, в то время как реальные шансы 4 к 1. То есть шансы в вашу пользу, вы впереди и должны принять ставку. Если вы сыграете пять раз, в среднем вы проиграете $1 в четырех случаях и выиграете $5 в одном. Вы заработали $1 за пять ставок, имея положительное ожидание в 20 центов за ставку.

      Если вы ставите $50 против $10, являясь фаворитом с шансами всего 4 к 1, ваше отрицательное ожидание составляет $2 за ставку, потому что в среднем вы четыре раза выиграете $10 и проиграете $50 один раз, что в сумме приведет к потере $10 после 5 ставок. С другой стороны, если вы ставите $30 против $10, являясь фаворитом с шансами 4 к 1, ваше положительное ожидание составляет $2, так как вы выиграете $10 четыре раза и проиграете $30 один раз, что в сумме даст прибыль в размере $10. Математическое ожидание демонстрирует, что первая ставка является плохой, а вторая – хорошей.

      Математическое ожидание лежит в основе любой игровой ситуации. Когда букмекер предлагает клиенту поставить $11, чтобы выиграть $10, он имеет положительное ожидание в размере 40 центов за $10 ставку. Когда казино выплачивает деньги, равные ставке, за столом в крэпс, оно имеет положительное ожидание в размере около $1,40 за ставку $100, поскольку игра сконструирована таким образом, что участник в среднем проиграет в 50,7 % случаев и выиграет в 49,3 %. Действительно, это, казалось бы, мизерное положительное ожидание приносит казино по всему миру их внушительные прибыли. Как сказал владелец казино Vegas World Боб Ступак: «Одна тысячная процента отрицательной вероятности на достаточно длинной дистанции разорит богатейшего человека в мире».

      В большинстве игровых ситуаций, таких как крэпс или рулетка в казино, любые предоставляемые шансы фиксированы. В других же случаях они меняются, и математическое ожидание может помочь вам в оценке отдельно взятой ситуации. Например, в блек-джеке, с целью найти правильную стратегию, ученые вычислили математическое ожидание от разных стилей игры. Розыгрыш, дающий вам более высокое ожидание, является верным. Например, когда у вас 16 против 10 дилера, вы – фаворит на проигрыш. Однако, когда эти 16 представляют собой две восьмерки, вашей лучшей игрой будет их разделить, удвоив ставку. Разделив восьмерки против десятки дилера, вы по-прежнему ожидаете потерять деньги, однако отрицательное ожидание будет ниже, нежели если бы вы тянули еще карту, имея две восьмерки против десятки.

      Математическое ожидание в покере

      Покерные действия могут быть проанализированы с точки зрения математического ожидания. Вы можете думать, что определенный розыгрыш является прибыльным, однако иногда он может оказаться отнюдь не лучшим, поскольку существует более прибыльный вариант. Допустим, у вас фулл хаус в 5-карточном дро. Игрок перед вами делает ставку. Вы знаете, что если вы повысите, ваш противник сделает колл. Следовательно, повышение выглядит лучшей игрой. Однако в таком случае два человека за вами сбросят карты. С другой стороны, если вы уравняете ставку первого игрока, то очень вероятно, что и два игрока за вами сделают колл. Играя через рейз, вы заработаете одну ставку, а через колл – две. В итоге получается, что колл имеет более положительное математическое ожидание, а значит, является лучшей игрой.

      Вот аналогичная, но немного более сложная ситуация. На последней улице в 7-карточный стад вы собрали флеш. Оппонент перед вами, которого вы кладете на две пары, ставит, и, кроме того, в раздаче присутствует игрок за вами, – вы уверены, что тоже бьете его. Если вы повысите, противник, сидящий после вас, сбросит. Более того, игрок, первоначально сделавший ставку, вероятно, также сбросит, если он действительно имеет две пары; но если он собрал фулл хаус, то он сделает ререйз. В данной ситуации у игры через рейз не положительное математическое ожидание, а отрицательное. В случае, когда первый игрок собрал фулл хаус и сделает ререйз, такая игра будет стоить вам две ставки, если вы сделаете колл его ререйза, и одну ставку, если сбросите.

      Пойдем в этом примере еще дальше. Если вы последней картой не соберете флеш и игрок перед вами сделает ставку, вы можете сделать рейз против определенных оппонентов! Следуя логике ситуации, когда вы не собрали флеш, соперник позади вас сбросит, и если игрок, первоначально сделавший ставку, имел только две пары, он тоже может сбросить. Имеет ли розыгрыш положительное ожидание (или менее негативное ожидание, нежели пас), зависит от шансов, предоставляемых вам за ваши деньги: то есть размер банка и ваши предполагаемые шансы на то, что оппонент, сделавший первоначальную ставку, не имеет фулл хауса и сбросит, имея две пары. Последнее предположение требует, конечно, умения читать руки и оппонентов, о чем я поговорю в более поздних главах. На таком уровне игры расчет математического ожидания становится намного запутаннее, нежели когда вы просто подбрасываете монетку.

      Мифы. Отрицательное математическое ожидание в Бинарных Опционах

      Существует миф, что отрицательное математическое ожидание является фундаментальным для бинарных опционов. Те, кто не умеют вовремя остановиться с профитом в руке, намекают, что система построение так, чтобы трейдеры теряли свои деньги. Ведь по закону отрицательного математического ожидания доходность торговой системы или стратегии неизбежно приведет к полной потере средств. Например, идиоты приводят пример того, что сделав такое же количество ставок, как и оппонент (в данном случае речь о рынке), Вы неизбежно окажетесь в минусе.

      Соответствует ли это действительности? Действительно ли математика ставит крест на профите?

      На мгновение представим, что этот усреднённый статистический показатель создан гениями и действительно работает. Кто заставляет Вас ставить слишком много раз? Почему Вы входите в позицию сотни раз? Ловите волны графика, а не пытайтесь шортить или лонговать каждую минуту. Уменьшив количество сделок до минимума и увеличив суммы до максимума, у Вас больше шансов на профит даже в том случае, если верите этим бестолковым математическим выводам.

      Почему математика здесь не играет никакой роли в Бинарных Опционах?

      Рынок настолько волатильный, что не поддается никакому микроанализу. Только макроанализ может позволить учесть тренд и другие нюансы, с которыми Вам еще предстоит познакомиться. Например, Вы торгуете евро к доллару. Думаете, что успеете отреагировать на тотальный слив после подтверждения того, что Deutsche Bank банкрот? Нет, крупные фонды начнут избавляться от евро, покупая золото и доллары США. И Вы ничего не успеете сделать, сидя в позиции и торгуя на повышение евро. Здесь работает только тренд и фундаментал, который может толкать рынок в ту или иную сторону. Какая к черту математика? Откройте глаза. Вы должны научиться забирать сливки с этого рынка. Помните, когда Вы остаетесь в профите — брокеру это на руку. Никто здесь не хочет Вас слить. Вы платите комиссию по каждой сделке. Больше сделок — больше комиссионных.

      Пошлите математику к черту и отдайте ее тем, кто играет в покер. Там это работает. Здесь — настоящая война на рынке, где нужно быть готовым ко всему. Вооружайтесь!

      отрицательных ожиданий в предложении

      Эти примеры взяты из корпусов и из источников в Интернете. Любые мнения в примерах не отражают мнение редакторов Cambridge Dictionary, Cambridge University Press или его лицензиаров.

      отрицательное ожидание от женщин нашло дальнейшее усиление в стереотипах как работодателей, так и профсоюзов о женщинах как о «послушных» и управляемых работницах.

      Эти дети, вероятно, попадут в новые ситуации с негативными ожиданиями в отношении своей компетентности и того, как другие будут с ними взаимодействовать.

      Они предположили, что неприятные телесные ощущения от субтоксичных концентраций химических веществ являются результатом негативных ожиданий и страхов (эффект ноцебо).

      Предположительно, зависимость может частично объяснить умеренные или отрицательные ожидания и большее беспокойство относительно этического качества помощи пациентам.

      Безнадежность определяется как когнитивная система негативных ожиданий в отношении себя, социальной среды и будущей жизни.

      Когда и начальник, и подчиненный замечают низкую производительность, отрицательные ожидания подтверждаются, а вера укрепляется.

      Из

      Википедия